Sie sind auf Seite 1von 304

GOVERNMENT OF TAMIL NADU

HIGHER SECONDARY SECOND YEAR

PHYSICS
VOLUME - II

A publication under Free Textbook Programme of Government of Tamil Nadu

Department of School Education


Untouchability is Inhuman and a Crime

XII_Physics_First 8 pages Higher.indd 1 7/31/2019 7:13:46 PM


Government of Tamil Nadu

First Edition - 2019


(Published Under New Syllabus)

NOT FOR SALE

Content Creation

The wise
possess all

State Council of Educational


Research and Training
© SCERT 2019

Printing & Publishing

Tamil NaduTextbook and Educational


Services Corporation
www.textbooksonline.tn.nic.in

II

XII_Physics_First 8 pages Higher.indd 2 7/31/2019 7:13:46 PM


CONTENTS

PHYSICS

UNIT 6 Optics 01

UNIT 7 Dual Nature of Radiation and Matter 105

UNIT 8 Atomic and Nuclear physics 138

UNIT 9 Semiconductor Electronics 192

UNIT 10 Communication Systems 238

UNIT 11 Recent Developments in Physics 256

Glossary 288

E-book Assessment DIGI links

Let’s use the QR code in the text books!


• Download DIKSHA app from the Google Play Store.
• Tap the QR code icon to scan QR codes in the textbook.
• Point the device and focus on the QR code.
• On successful scan, content linked to the QR code gets listed.
Note: For ICT corner, Digi Links QR codes use any other QR scanner.

III

XII_Physics_First 8 pages Higher.indd 3 7/31/2019 7:13:46 PM


HOW TO USE THE BOOK
• Awareness on higher learning - courses, institutions and required
Scope of Physics competitive exams
• Financial assistance possible to help students to climb academic ladder
• Gender initiatives by the Government of India.

Learning Objectives: • Overview of the unit


• Gives clarity on the goals and objective of the topics

• Additional facts related to the topics covered to facilitate


curiosity driven learning

• To ensure understanding, problems/illustrations are given at every stage


Example problems before advancing to next level

• Visual representation of concepts with illustrations


• Videos, animations, and tutorials

ICT • To harness the digital skills to class room learning and experimenting

Summary • Recap of salient points of the lesson

Concept Map • Schematic outline of salient learning of the unit

• Evaluate students’ understanding and get them acquainted with the


Evaluation application of physical concepts to numerical and conceptual questions

Books for Reference • List of relevant books for further reading

• Solutions to exercise problems are accessible here. In addition, a few solved


Solved examples examples are given to facilitate students to apply the concepts learnt.
Competitive • Model Questions - To motivate students aspiring to take up competitive
Exam corner examinations such as NEET, JEE, Physics Olympiad, JIPMER etc

Glossary • Scientific terms frequently used with their Tamil equivalents

Content Focus • Covers ray and wave optics extensively, salient concepts in dual nature of
radiation and matter, atomic and nuclear physics. Topics in semiconductors
and communication are optimised. An exclusive unit on ‘Recent
developments in physics’ highlights that physics is the basic building block
for sciences, engineering, technology and medicine. With this, students are
motivated to pursue higher education confidently.

Back Wrapper
Richard Philip Feynman, (1918–1988) a theoretical physicist who received noble prize in physics in
1965 for his contributions to the development of quantum electrodynamics. He is the first person to discuss the
possiblity of manipulation of atoms that seeded nanotechnology. His lectures on various topics in physics are
very popular among physicists.
Illustration of Gravitational waves from two merging black holes.
Front Wrapper
Actual photograph of a super massive black hole M87*

IV

XII_Physics_First 8 pages Higher.indd 4 7/31/2019 7:13:47 PM


Scope of Physics - Higher Education

XII_Physics_First 8 pages Higher.indd 5


Entrance Examinations
After +2

Physics Olympiad Exam Education Career


NEET-National Eligibility cum Entrance Test
IIT JEE-Joint Entrance Examination (Mains & Advanced)
NEST- National Entrance Screening Test
KVPY-Kishore Vaigyanik Protsahan Yojana
After Completing +2 After B.Sc Physics
JEE Mains Paper II for B.Arch
AIIMS - All Indian Institute of Medical Science’s Examination

V
Chennai Mathematical Institute Entrance Examination Integrated M.Sc. Physics
BITSAT- Birla Institute of Science And Technology Admission Test Central Uiversities through CUCET
AIEEE – All India Engineering Entrance Exam Central Research Institutes like IISER using KVPY, JEE
CUCET – Central Universities Common Entrance Test Advanced M.Sc Physics in IIT’s and NIT’s through JAM
JIPMER - Jawaharlal Institute of Postgraduate Medical Education & IISER aptitude test Integrated Ph.D in IISER’s and IISc through JAM and JEST
Research
Top 1% students in State board are eligible for IISER Aptitude M.Sc Physics in Central Universities throught CUCET
CLAT – Common Law Admission Test Test
HSEE- Humanities and Social Sciences Entrance Examination M.Sc in Energy Physics, Applied Physics in IIT’s through JAM
Admission in NISER through NEST
AIPVT -All India Pre-Veterinary Test Integrated Ph.D in IMSc, TIFR, JNCASR through JEST score
B.Sc Photonics
NDA – National Defence Academy Examination Integrated Ph.D in TIFR through JEST and TIFR exam
B.Sc Hons. in Mathematics and Physics in CMI
M.Sc Photonics, Reactor physics, Nuclear Engineering ,
B.Sc Hons. in Mathematics and Computer Science in CMI
After Graduation M.Sc Medical Physics
Five-Year Dual degree In IIST ( B.Tech + Master of Science)
M.Sc Biophyiscs
JAM- Joint Admission Test Master of Science (Astronomy and Astrophysics, Solid State
Physics) Research Institutes abroad like CERN, NASA, LIGO offer
JEST – Joint Entrance Screening Test Summer internship programmes for motivated Indian students
GATE- Graduate Aptitude Test in Engineering Note
pursuing Undergraduate course in physics
CAT – Common Admission Test (for MBA) Students admitted to IISc, IIT’s, NIT’s Indian Academy of Science & various other research institutes
Exams conducted by Respective Universities IISER’s, IIST, will get a Scholorship equivalent to INSPIRE offer paid Summer Internship for science students to get an
Assured placement in ISRO and other divisions for hands on experience in research.
the students of IIST
After Post Graduation
Institutes and their ranking can be found in www.nirfindia.org

CSIR NET - National Eligibility Test for JRF and Lectureship

7/31/2019 7:13:50 PM
Opportunities after B.Sc. Physics

XII_Physics_First 8 pages Higher.indd 6


Scholarships
Jobs in Government Sector
• INSPIRE Scholarship - Scholarship for Higher Education (SHE) - 80000 per
annum, for B.Sc/B.S/Int M.Sc/Int M.S
• Scientific Officer and Scientific Assistant Jobs Eligibility Criteria:- Top 1% students in their +2 board exam
Top 10000 rank holders in JEE or NEET
• CSIR Labs • Students studying at NIISER, IISER, Department of Atomic Energy Centre for Basic Science Gen
NTSE, KVPY, JBNSTS Scholars Astr

VI
• DRDO – Defence Research and Development Organisation • International Olympiad Medalists Qua
• Indira Gandhi Scholarship for single girl child for full time regular Master's Degree Plas
• DAE -Department of Atomic Energy • Post Graduate Merit Scholarship for University rank holders in UG level Mete
• Mathematics Training and Talent Search (MTTS) Programme Strin
• DoS - Department of Science Eligibility Criteria:- Students who studied Maths at UG or PG level Opti
• Dr. K S Krishnan Research Associateship (KSKRA) Con
• IMD- Indian Meteorological Department Eligibility Criteria:- Students who posses Master's Degree or Ph.D in science or engineering Cryp
• IGCAR JRF Math
• ONGC -Oil and Natural Gas Corporation Eligibility Criteria:- Passing JEST, GATE, NET Exams Crys
• Promotion of Science Education (POSE) Scholarship Scheme Ato
• ATC – Air Traffic Controller • Dhirubhai Ambani Scholarship Programme Biop
• Foundation for Academic Excellence and Access Scholarship (FAEA) Nucl
• Teaching faculty in schools and colleges through SET, NET,TET • Central Sector Scheme of National Fellowship and Scholarship for Higher Education of ST Ener
students
Geo
• Scientist post in various research institutes in India • Pre - Matric and Post - Matric Scholarship for students belonging to
Qua
minority communities to pursue their School and Collegiate education by
the Ministry of Minority affairs, Government of India.
• Pre Matric and Post Matric Scholarship for students with Disabilities to
pursue their School and Collegiate Education by the Department of
Empowerment of Persons with Disabilities, Government of India.
• Maulana Azad scholarship for minorities.

7/31/2019 7:13:51 PM
Institutes in india to pursue research in physics

XII_Physics_First 8 pages Higher.indd 7


Famous Research Institutes for Physics in India
Name of the Institution Website
Institute of Mathematical Sciences,Chennai(IMSc) www.imsc.res.in
Saha Institute of Nuclear Physics, Kolkata www.saha.ac.in
International Centre for Theoretical Sciences, Bangalore www.icts.res.in
Harish chandra Research Institute, Allahabad www.hri.res.in
Aryabhatta Research Institute of Observational Sciences, Nainital www.aries.res.in
Jawaharlal Nehru Centre for Advanced Scientific Research (JNCASR) www.jncasr.ac.in
Institute of Physics (IOP), Bhubaneshwar www.iopb.res.in
Indian Association for the Cultivation of Sciences (IACS), Kolkata www.iacs.res.in
Research Areas Vikram Sarabhai Space Centre (VSSC), Thiruvananthapuram www.vssc.gov.in
National Physical Laboratory (NPL), Delhi www.nplindia.in
National Institute of Science Education and Research (NISER), Bhubaneshwar www.niser.ac.in
General Relativity and Cosmology
Indian Institute of Science(IISc), Bangalore www.iisc.ac.in
Astronomy and Astrophysics
Quantum Optics and Information theory Raman Research Institute(RRI), Bangalore www.rri.res.in

VII
Plasma physics Tata Institute of Fundamental Research (TIFR), Mumbai www.tifr.res.in
Meteorology and Atmospheric science Bhaba Atomic Research Centre (BARC), Mumbai www.barc.gov.in
String Theory, Quantum Gravity www.igcar.gov.in
Indira Gandhi Centre for Atomic Research (IGCAR), Kalpakkam
Optics and Photonics
Inter University Centre for Astronomy and Astrophysics (IUCAA),Pune www.iucaa.in
Condensed Matter Theory, Material Science and Spintronics
Indian Institute of Space Science and Technology(IIST), Trivandrum www.iist.ac.in
Cryptography
Mathematical Physics,Statistical Physics Institute of Plasma Research (IPR), Gujarat www.ipr.res.in
Crystal Growth and Crystallography Physical Research Laboratory (PRL),Ahmedabad www.prl.res.in
Atomic and Molecular Physics Inter-University Accelerator Center (IUAC), Delhi www.iuac.res.in
Biophysics, Medical Physics
Indian Institute of Astrophysics (IIA), Bangalore www.iiap.res.in
Nuclear and High energy Particle Physics
Chennai Mathematical Institute (CMI), Chennai www.cmi.ac.in
Energy and Environmental Studies
Geophysics Liquid Propulsion Systems Centre www.lpsc.gov.in
Quantum Biology and Quantum Thermodynamics and Cymatics S.N.Bose Centre for Basic Sciences www.bose.res.in
CSIR National laboratories
Indian Institute of Technology (IIT) in various places
IISER’s in various places
National Institute of Technology (NIT) in various places
Indian Institute of Information Technology (IIITs) at various places
Central and State Universities

7/31/2019 7:13:52 PM
Gender Initiatives by the Government of India

XII_Physics_First 8 pages Higher.indd 8


Global STEM (Science, Technology, Engineering
Women Scientist Scheme by the Department of Sci- and Mathematics) Scholarships for Indian
ence and Technology (DST) Women in Science

Under this scheme, women scientists are being encouraged to A list of some STEM scholarships offered to Indian
Women if they would like to pursue their higher educa-
pursue research in frontier areas of science and engineering, on tion abroad.
problems of societal relevance and to take up S&T-based intern-
ship followed by self-employment. Following three categories of
fellowships, with research grants, are available for Indian citizen: 1. Society of Women Engineers (SWE) Scholarship
1. Women Scientist Scheme-A(WOS-A): Research in Basic/Applied 2. The Google Anita Borg Memorial Scholarship Program
Science
2. Women Scientist Scheme-B (WOS-B): S&T interventions for 3. Women In Aviation International Scholarships
Societal Benefit
3. Women Scientist Scheme-C (WOS-C): Internship in Intellectual 4. Amelia Earhart Fellowship by Zonta International
Property Rights (IPRs) for the Self-Employment
5. The Graduate Women In Science (GWIS) National Fel-
Eligibility: lowships Program
The scheme is meant to encourage women in S&T domain,

VIII
preferably those having a break in career and not having regular https://feminisminindia.com/2017/06/14/glob-
employment, to explore possibility of re-entry into the profession. al-stem-scholarships/
Qualifications:
1. Minimum Post Graduate degree, equivalent to M.Sc. in Basic
or Applied Sciences or B.Tech. or MBBS or other equivalent
professional qualifications
2. M.Phil/M.Tech/M.Pharm/M.VSc or equivalent qualifications
3. Ph.D. in Basic or Applied Sciences

http://www.dst.gov.in/ scientific-programmes/ scientific-engi-


neering- research/women-scientists- programs

KIRAN (Knowledge Involvement in Research Advancement


through Nurturing) Scheme of DST

https://www.ugc.ac.in/pdfw/

7/31/2019 7:13:52 PM
UNIT

6 OPTICS

An age is called dark, not because the light fails to shine, but because people refuse to see.
— James Albert Michener

LEARNING OBJECTIVES

In this unit, the students are exposed to,


• The two aspects of treating light as a ray and a wave.
• The behaviour and propagation of light.
• The concepts related to mirrors, lenses prisms etc.
• The different optical instruments like microscope, telescope etc.
• The terms like magnification and resolving power etc.
• The various phenomena that support the wave nature of light.

6.1
INTRODUCTION A ray of light gives information about
only the direction of light. It does not give
Light is mystical. Yet, its behaviour is information about the other characteristics
so fascinating. It is difficult to comprehend of light like intensity and colour. However, a
light to a single entity. In this unit we learn it ray is a sensible representation of light in ray
in two different scientific aspects called ray optics. The path of the light is called a ray
optics and wave optics. Ray optics deals with of light and a bundle of such rays is called a
light that is represented as a ray travelling in beam of light. In this chapter, we can explain
straight lines. The geometrical constructs get the phenomena of reflection, refraction and
the permanence to understand the various dispersion and scattering of light, using the
characteristics of light. In wave optics, we ray depiction of light.
study about the phenomenon associated
with the propagation of light as a wave.
6.1.2 Reflection
First, let us learn the ray optics followed by
the wave optics. The bouncing back of light into the
same medium when it encounters a
reflecting surface is called reflection of
6.1.1 Ray optics light. Polished surfaces can reflect light.
Light travels in a straight line in a medium. Mirrors which are silver coated at their back
Light may deviate in its path only when it can reflect almost 90% of the light falling on
encounters another medium or an obstacle. them. The angle of incidence i and the angle

UNIT-6(XII-Physics_Vol-2).indd 1 7/31/2019 7:36:36 PM


of reflection r are measured with respect to
the normal drawn to the surface at the point
of incidence of light. According to law of
reflection,
(a) The incident ray, reflected ray and normal
to the reflecting surface all are coplanar
(ie. lie in the same plane).
(b) The angle of incidence i is equal to the
angle of reflection r.

i = r(6.1)

The law of reflection is shown in


Figure 6.1.
Normal
Incident ray Reflected ray

i r

Mirror Figure 6.2  Regular and irregular


reflections

A N B 6.1.3  Angle of deviation due


to reflection
The angle between the incident
i and deviated light ray is called angle of
r
deviation of the light ray. In reflection, it
X Y is calculated by a simple geometry as shown
in Figure 6.3(a). The incident light is AO.
O Silvered
The reflected light is OB. The un-deviated
light is OC which is the continuation of the
Figure 6.1  Reflection of light incident light. The angle between OB and
OC is the angle of deviation d. From the
The law of reflection is valid at each geometry, it is written as, d = 180 – (i+r).
point for any reflecting surface whether the As, i = r in reflection, we can write angle of
surface is plane or curved. If the reflecting deviation in reflection at plane surface as,
surface is flat, then incident parallel rays
after reflection come out parallel as per the d = 180 – 2i(6.2)
law of reflection. If the reflecting surface is
irregular, then the incident parallel rays after The angle of deviation can also be
reflection come out irregular (not parallel) measured in terms of the glancing angle α
rays. Still law of reflection is valid at every which is measured between the incident ray
point of incidence as shown in Figure 6.2. AO and the reflecting plane surface XY as

2 Unit 6  OPTICS

UNIT-6(XII-Physics_Vol-2).indd 2 7/31/2019 7:36:37 PM


shown in Figure 6.3(b). By geometry, the in the tilted system the angle of incidence is
angles ∠AOX = α, ∠BOY = α and ∠YOC = α now i+θ and the angle of reflection is also
(are all same). The angle of deviation (d) is i+θ. Now, OR2 is the reflected ray. The angle
the angle ∠BOC. Therefore, between OR2 and OR1 is,
d = 2α(6.3) ∠R1OR 2 = ∠N ′OR 2 −∠NOR1
(i+θ) – (i–θ) = 2θ.

A N B N N' R1
I
i+θ
i r d=180-2i R2
θ i–θ i+θ
X Y i
i 2θ
O
θ
(a) C O

A N B

d=2 6.1.4  Image formation in


  plane mirror
X Y
O  Let us consider a point object A is placed
in front of a plane mirror and the point of
(b)
C incidence is O on the mirror as shown in the
Figure 6.3  Angle of deviation due to Figure. 6.4. A light ray AO from the point
reflection object is incident on the mirror and it is
reflected along OB. The normal is ON.
The angle of incidence ∠AON = angle of
EXA MP LE 6 .1 reflection ∠BON
Another ray AD incident normally on
Prove that when a reflecting surface of the mirror at D is reflected back along
light is tilted by an angle θ, the reflected DA. When BO and AD are extended
light will be tilted by an angle 2θ. backwards, they meet at a point A¢ . Thus,
the rays appear to come from a point A¢
Solution
which is behind the plane mirror. The
For the reflecting surface AB, the incident ray object and its image in a plane mirror are
IO and the reflected ray OR1 subtend angle at equal perpendicular distances from the
i with the normal N as angle of incidence is plane mirror which can be shown by the
equal to angle of reflection as shown in figure . following explanation.
When the surface AB is tilted to A¢B ¢ by an In Figure 6.4, Angle ∠AON = angle
angle θ, the normal N ′ is also is tilted by ∠DAO [Since they are alternate angles]
the same angle θ. Remember the position Angle ∠BON = angle ∠O A¢ D [Since they
of incident ray IO remains unaltered. But, are corresponding angles]

Unit 6  OPTICS 3

UNIT-6(XII-Physics_Vol-2).indd 3 7/31/2019 7:37:00 PM


This shows that the image distance
Plane mirror inside the plane mirror is equal to the object
distance in front of the plane mirror.
B

i 6.1.5  Characteristics of the


O
N i b image formed by plane mirror
b
A A (i) The image formed by a plane mirror is
D virtual, erect, and laterally inverted.
(ii) The size of the image is equal to the size
of the object.
Plane mirror
(iii) The image distance far behind the
Eye mirror is equal to the object distance in
front of it.
Ray appears to
originate from
(iv) If an object is placed between two plane
i behind the mirror mirrors inclined at an angle θ, then the
i number of images n formed is as,
 360   360 
)) If   is even then, n =  −1
 q   q 
Object for objects placed symmetrically or
unsymmetrically,

 360  360 
Plane mirror )) If   is odd then, n =  −1
 q   q 
for objects placed symmetrically,
 360   360 
Eye )) If  is odd then, n =  
 q   q 
Virtual for objects placed unsymmetrically.
image

6.1.6  Real and virtual


images by a plane mirror

do di When a real object is placed at a point


Image O in front of a plane mirror it produces
Object
distance distance divergent rays in all directions as shown in
Figure 6.5(a). After reflection from the plane
Figure 6.4  Formation of image in plane
mirror they appear to come out from a point
mirror
I behind the mirror. This image cannot be
formed on a screen as the image is behind the
Hence, it follows that angle, ∠DAO =  mirror. This type of image which cannot be
∠OA¢D formed on the screen but can only be seen
The triangles ∆ODA and ∆OD A¢ are with the eyes is called virtual image.
congruent On the other hand, if convergent rays
∴ AD = A¢D are incident on a plane mirror, the rays after

4 Unit 6  OPTICS

UNIT-6(XII-Physics_Vol-2).indd 4 7/31/2019 7:37:28 PM


reflection pass through a point I in front of Table 6.1 Conditions for nature of
the mirror and form an image as shown in
objects and images
Figure 6.5(b). This image can be formed on
Nature of
a screen as the image is in front of the mirror. Condition
This type of image which canbe formed on object/image
a screen and can also be seen with the eyes Real Image Rays actually converge at
is called real image. the image

Virtual Image Rays appear to diverge


from the image

Real Object Rays actually diverge


from the object
O I
Real Virtual Virtual Object Rays appear to converge
object image
at the object

EX AM P L E 6 . 2

What is the height of the mirror needed to


see the image of a person fully on the mirror?

Solution
I O Let us assume a person of height h is
Real Virtual
image standing in front of a vertical plane mirror.
object
The person could see his/her head when
light from the head falls on the mirror and
gets reflected to the eyes. Same way, light
from the feet falls on the mirror and gets
Figure 6.5 Real and virtual images by
reflected to the eyes.
plane mirror
Mirror

H H'
h1/2
h i1
1
It is generally known that a E
h1/2
i1

Note plane mirror can only form (h1+h2)/2


h2/2
a virtual image. But, now we
Image
Object

i2
h
have understood that a plane mirror can 2 i2

form a real image when converging rays h2/2

fall on it. F
F'

The above discussion is consolidated in If the distance between his head H and eye
Table 6.1. These concepts will be very much E is h1 and distance between his feet F and
useful in deciding about the nature of object eye E is h2. The person’s total height h is,
and image in ray optics. h = h1 + h2

Unit 6 OPTICS 5

UNIT-6(XII-Physics_Vol-2).indd 5 7/31/2019 7:37:28 PM


By the law of reflection, the angle of surface of the glass is silvered. The reflection
incidence and angle of reflection are the takes place at the other polished surface. If
same in the two extreme reflections. The the reflection takes place at the convex
normals are now the bisectors of angles surface, it is called a convex mirror and if
between incident and reflected rays in the the reflection takes place at the concave
two reflections. By geometry, the height of surface, it is called a concave mirror. These
the mirror needed is only half of the height are shown in Figure 6.6.
h1 + h2 h We shall now become familiar with some
of the person. =
2 2 of the terminologies pertaining to spherical
Does the height depend on the distance mirrors.
between the person and the mirror? Centre of curvature: The centre of the
sphere of which the mirror is a part is called
the center of curvature (C) of the mirror.
6.2 Radius of curvature: The radius of the
SPHERICAL MIRRORS sphere of which the spherical mirror is a
part is called the radius of curvature (R) of
We shall now study about the reflections the mirror.
that take place in spherical surfaces. Pole: The middle point on the spherical
surface of the mirror (or) the geometrical
center of the mirror is called pole (P) of the
A A mirror.
Principal axis: The line joining the pole
and the centre of curvature is called the
C P principal axis of the mirror. The light ray
P
Hollow sphere travelling along the principal axis towards the
mirror after reflection travels back along the
Reflecting
surface same principal axis. It is also called optical axis
B Focus (or) Focal point: Light rays
B
travelling parallel and close to the principal
(a) Spherical mirrors
axis when incident on a spherical mirror,
converge at a point for concave mirror or
appear to diverge from a point for convex
Inner reflective Outer reflective mirror on the principal axis. This point is
called the focus or focal point (F) of the mirror.
Focal length: The distance between the
pole and the focus is called the focal length
(f) of the mirror.
(b) A concave mirrors (c) A convex mirrors
Focal plane: The plane through the focus
and perpendicular to the principal axis is
Figure 6.6  Spherical mirrors
called the focal plane of the mirror.
All the above mentioned terms are
A spherical surface is a part cut from
shown in Figure 6.7 for both concave and
a hollow sphere. Spherical mirrors are
convex mirrors.
generally constructed from glass. One
6 Unit 6  OPTICS

UNIT-6(XII-Physics_Vol-2).indd 6 7/31/2019 7:37:34 PM


us to make some approximations with the
angles in ray optics.

Marginal rays
C F P
Paraxial rays

Principal axis F
f C P
R

Figure 6.8  Paraxial and marginal rays

P F C 6.2.2  Relation between


f and R
Let C be the centre of curvature of the
mirror. Consider a light ray parallel to the
f principal axis is incident on the mirror at
R M and passes through the principal focus F
after reflection. The geometry of reflection
Figure 6.7  Focal length of concave and of the incident ray is shown in Figure 6.9(a).
convex mirrors
The line CM is the normal to the mirror at
M. Let i be the angle of incidence and the
6.2.1  Paraxial Rays and same will be the angle of reflection.
If MP is the perpendicular from M on
Marginal Rays
the principal axis, then from the geometry,
The rays travelling very close to the The angles ∠MCP = i and ∠MFP = 2i
principal axis and make small angles with From right angle triangles ∆MCP and
it are called paraxial rays. The paraxial ∆MFP,
rays fall on the mirror very close to the pole
of the mirror. On the other hand, the rays PM PM
tani = and tan2i =
travelling far away from the principal axis PC PF
and fall on the mirror far away from the As the angles are small, tan i ≈ i ,
pole are called as marginal rays. These
two rays behave differently (get focused at PM PM
i= and 2i =
different points) as shown in Figure 6.8. In PC PF
this chapter, we shall restrict our studies Simplifying further,
only to paraxial rays. As the angles made by PM PM
the paraxial rays are very small, this helps 2 = ; 2PF = PC
PC PF
Unit 6  OPTICS 7

UNIT-6(XII-Physics_Vol-2).indd 7 7/31/2019 7:38:17 PM


the following rays to locate the image point
as shown in Figure 6.10.
M
i B
i B

i 2i A
C F P
A
C F P
C F P
(a) (b)

B B

A A
C F P C F P
(a) Concave mirror
(c) (d)

i Figure 6.10  Image tracing


i M

(i) A ray parallel to the principal axis after


2i i reflection will pass through or appear
P F C to pass through the principle focus.
(Figure 6.10(a))
(ii) A ray passing through or appear to
pass through the principal focus, after
reflection will travel parallel to the
(b) Convex Mirror principal axis. (Figure 6.10(b))
(iii) A ray passing through the centre
Figure 6.9  relation between R and f of curvature retraces its path after
reflection as it is a case of normal
PF is focal length f and PC is the radius incidence.(Figure 6.10(c))
of curvature R. (iv) A ray falling on the pole will get reflected
R as per law of reflection keeping principal
2f =R (or) f = (6.4) axis as the normal. (Figure 6.10(d))
2
Equation (6.4) is the relation between f
and R. The construction is shown for convex 6.2.4  Cartesian sign
mirror in figure 6.9(b) convention
While tracing the image, we would
normally come across the object distance
6.2.3  Image formation in u, the image distance v, the object height h,
spherical mirrors the image height ( h ′ ), the focal length f and
The image can be located by graphical the radius of curvature R. A system of signs
construction. To locate the point of an for these quantities must be followed so that
image, a minimum of two rays must meet the relations connecting them are consistent
at that point. We can use at least any two of in all types of physical situations. We shall
8 Unit 6  OPTICS

UNIT-6(XII-Physics_Vol-2).indd 8 7/31/2019 7:38:26 PM


follow the Cartesian sign convention which at the pole P is reflected along PB´. The third
is now widely used as given below and also paraxial ray BC passing through centre of
shown in Figure 6.11. curvature C, falls normally on the mirror at
(i) The Incident light is taken from left to E is reflected back along the same path. The
right (i.e. object on the left of mirror). three reflected rays intersect at the point
(ii) All the distances are measured from the A′ B ′ . A perpendicular drawn as A′ B ′ to the
pole of the mirror (pole is taken as origin). principal axis is the real, inverted image of
(iii) The distances measured to the right of the object AB.
pole along the principal axis are taken
D
as positive. B

(iv) The distances measured to the left of A A'


C P
pole along the principal axis are taken B'
F
as negative. E
(v) Heights measured in the upward
perpendicular direction to the principal Figure 6.12  Mirror equation
axis are taken as positive.
(vi) Heights measured in the downward As per law of reflection, the angle of
perpendicular direction to the principal incidence ∠BPA is equal to the angle of
axis, are taken as negative. reflection ∠ B ′PA′ .
The triangles ∆BPA and ∆ B ′PA′ are
Height upwords
Incident light
similar. Thus, from the rule of similar triangles,
(Positive)
A′B ′ PA′
= (6.5)
Distance to the left Distance to the right AB PA
(Negative) (Positive)
Height downwards
The other set of similar triangles are,
(Negative) ∆DPF and ∆ B ′A′ F . (PD is almost a straight
Figure 6.11  Cartesian sign convention vertical line)
A ′B ′ A ′F
=
PD PF
6.2.5  The mirror equation
As, the distances PD = AB the above
The mirror equation establishes a relation equation becomes,
among object distance u, image distance v
and focal length f for a spherical mirror. A ′B ′ A ′F
= (6.6)
An object AB is considered on the AB PF
principal axis of a concave mirror beyond the From equations (6.5) and (6.6) we can
center of curvature C. The image formation write,
is shown in the Figure 6.12. Let us consider PA′ A′F
three paraxial rays from point B on the =
PA PF
object. The first paraxial ray BD travelling
As, A′ F = PA′ - PF  , the above equation
parallel to principal axis is incident on the
becomes,
concave mirror at D, close to the pole P.
After reflection the ray passes through the PA′ PA′ − PF
=  (6.7)
focus F. The second paraxial ray BP incident PA PF

Unit 6  OPTICS 9

UNIT-6(XII-Physics_Vol-2).indd 9 7/31/2019 7:38:57 PM


We can apply the sign conventions for
the various distances in the above equation.
PA = −u, PA′ = −v , PF = − f
All the three distances are negative as per
sign convention, because they are measured
to the left of the pole. Now, the equation
(6.7) becomes,

−v −v − (− f )
=
−u −f
On further simplification,

v v− f v v
= ; = −1
u f u f
Dividing either side with v,
Mirror
1 1 1 v
= −
u f v
After rearranging,
1 1 1
+ = (6.8)
v u f
The above equation (6.8) is called mirror Identify the type of mirror used in
equation. Although this equation is derived each of the application shown above.
for a special situation shown in Figure
(6.12), it is also valid for all other situations
with any spherical mirror. This is because Applying proper sign conventions for
proper sign convention is followed for u, v equation (6.5),
and f in equation (6.7).
A′B ′ PA′
=
AB PA
6.2.6  Lateral magnification
in spherical mirrors A′B ′ = −h,AB = h, PA′ = −v , PA = −u

The lateral or transverse magnification −h ′ −v


=
is defined as the ratio of the height of the h −u
image to the height of the object.The height On simplifying we get,
of the object and image are measured
h′ v
perpendicular to the principal axis. m= = − (6.10)
h u
height of theimage (h ′) Using mirror equation, we can further
magnification (m) =
height of theobject (h) write the magnification as,
h′ h′ f − v f
m= (6.9) m= = = (6.11)
h h f f −u

10 Unit 6  OPTICS

UNIT-6(XII-Physics_Vol-2).indd 10 7/31/2019 7:40:25 PM


As the magnitude of magnification is 3, the
The students are advised image is enlarged three times.
Note to refresh themselves with
As the image is formed to the left of the
the image tracing for the
concave mirror, the image is real.
concave and convex mirrors for various
predetermined positions of the object
and the position of image, nature EX AM P L E 6 . 4
of image etc. studied in 9th standard
(Science, Unit 6. Light). A thin rod of length f/3 is placed along
the optical axis of a concave mirror of
focal length f such that its image which is
EXA MP LE 6 .3 real and elongated just touches the rod.
Calculate the longitudinal.
An object is placed at a distance of 20.0
cm from a concave mirror of focal length Solution
15.0 cm.
longitudinal (m) = length of image (l´)
(a) What distance from the mirror a screen magnification length of object (l)
should be placed to get a sharp image? f
Given: length of object, l =
(b) What is the nature of the image? 3

Solution
2f
Given, f = –15 cm, u = –20 cm f/3
A' A
1 1 1 Image Rod
(a) Mirror equation, + =
v u f C F P
u
v
1 1 1
Rewriting to find v, = −
v f u l' u' = 2f

1 11
Substituting for f and u, =
− Let, l be the length of the image, then,
−15 −20v
l′ l′ mf
1 (−20) − (−15) −5 −1 m= = (or) l =
= = = l f /3 3
v 300 300 60
Image of one end coincides with the object.
v = −60.0 cm
Thus, the coinciding end must be at center
As the image is formed at 60.0 cm to the of curvature.
left of the concave mirror, the screen is to
Hence, u ′ = R = 2f
be placed at distance 60.0 cm to the left of
the concave mirror. f
u′ = u +
h′ v 3
(b) Magnification, m = =−
h u f f 5f
h′ (−60) u = u′ − =2f − =
m= =− = −3 3 3 3
h (−20)
f mf 5 f f mf f (6 + m)
As the sign of magnification is negative, v =u+ + = + + =
3 3 3 3 3 3
the image is inverted.

Unit 6 OPTICS 11

UNIT-6(XII-Physics_Vol-2).indd 11 7/31/2019 7:41:55 PM


1 1 1 in the wheel will get reflected by a mirror
Mirror equation, + = M kept at a long distance d, about 8 km
v u f
from the toothed wheel. If the toothed
1 1 1 wheel was not rotating, the reflected light
+ =
 f (6 + m)   5 f  − f from the mirror would again pass through
−  − 
 3  3 the same cut and reach the eyes of the
After simplifying, observer through the partially silvered
3 3 1 3 2 glass plate.
+ = ; = Working: The angular speed of rotation
f (6 + m) 5 f f (6 + m) 5
of the toothed wheel was increased from
15 15 zero to a value ω until light passing through
6+m = ;m = −6
2 2 one cut would completely be blocked by
3 the adjacent tooth. This is ensured by
m = = 1. 5
2 the disappearance of light while looking
through the partially silvered glass plate.
Expression for speed of light:  The speed
6.3
of light in air v is equal to the ratio of the
SPEED OF LIGHT distance the light travelled from the toothed
wheel to the mirror and back 2d to the time
Light travels with the highest speed in taken t.
vacuum. The speed of light in vacuum is
denoted as c and its value is, c = 3×108 m s-1. 2d
It is a very high value. Several attempts were v= (6.12)
t
made by scientists to determine the speed
of light. The earliest attempt was made by The distance d is a known value from the
a French scientist Hippolyte Fizeau (1819– arrangement. The time taken t for the light
1896). That paved way for the other scientists to travel the distance to and fro is calculated
too to determine the speed of light. from the angular speed ω of the toothed
wheel.
6.3.1  Fizeau’s method to The angular speed ω of the toothed
determine speed of light wheel  when the light disappeared for the
first time is,
Apparatus:  The apparatus used by
Fizeau for determining speed of light in q
air is shown in Figure 6.13. The light from w = (6.13)
t
the source S was first allowed to fall on a
partially silvered glass plate G kept at an Here, θ is the angle between the tooth
angle of 45o to the incident light from the and the slot which is rotated by the toothed
source. The light then was allowed to pass wheel within that time t.
through a rotating toothed-wheel with N total angle of the circle in radian
teeth and N cuts of equal widths whose q=
number of teeth+number off cuts
speed of rotation could be varied through
an external mechanism (not shown in the 2p p
q= =
Figure). The light passing through one cut 2N N

12 Unit 6  OPTICS

UNIT-6(XII-Physics_Vol-2).indd 12 7/31/2019 7:42:51 PM


M
Light
S source
d

G
45o Tooth

Slot
Partially
silvered Toothed
Observer glass plate wheel

Figure 6.13 Speed of light by Fizeau’s method

Substituting for θ in the equation 6.13.


for ω, After the disappearance
Note
of light for the first time
p /N p while increasing the speed
w= =
t Nt of rotation of the toothed-wheel from
Rewriting the above equation for t, zero to ω, on further increase of speed
of rotation of the wheel to 2ω, the light
p would appear again due to the passing
t= (6.14)
Nw of reflected light through the next slot.
So, for every odd value of ω, light will
Substituting t from equation (6.14) in
disappear (stopped by tooth) and for
equation (6.12),
every even value of ω light will appear
2d (allowed by slot).
v=
p / Nw
After rearranging,
6.3.2 Speed of light through
2dNw
v= (6.15) different media
p
Different transparent media like glass,
Fizeau had some difficulty to visually water etc. were introduced in the path of
estimate the minimum intensity of the light light by scientists like Foucault (1819–
when blocked by the adjacent tooth, and 1868) and Michelson (1852–1931) to find
his value for speed of light was very close the speed of light in different media. Even
to the actual value. Later on, with the same evacuated glass tubes were also introduced
idea of Fizeau and with much sophisticated in the path of light to find the speed of light
instruments, the speed of light in air was in vacuum. It was found that light travels
determined as, v = 2.99792×108 m s–1. with lesser speed in any medium than its

Unit 6 OPTICS 13

UNIT-6(XII-Physics_Vol-2).indd 13 7/31/2019 7:43:26 PM


speed in vacuum. The speed of light in should not be confused with mass density
vacuum was determined as, c = 3×108 m s-1. of the material of the medium. They two
We could notice that the speed of light in are different entities]. The Table 6.2 shows
vacuum and in air are almost the same. the refractive index of different transparent
media.

6.3.3  Refractive index Table 6.2  Refractive index of


Refractive index of a transparent different media
medium is defined as the ratio of speed Media Refractive index
of light in vacuum (or air) to the speed of Vacuum 1.00
light in that medium.
Air 1.0003
refractive speed of light in vacuum (c)
index n of a = Carbon dioxide gass 1.0005
speed of light inmedium (v )
medium Ice 1.31
c Pure water 1.33
n= (6.16)
v Ethyl alcohol 1.36
Refractive index of a transparent medium Quartz 1.46
gives an idea about the speed of light in that Vegetable oil 1.47
medium. Olive oil 1.48

EXA MP LE 6 .5 Acrylic 1.49


Table salt 1.51
One type of transparent glass has refractive
Glass 1.52
index 1.5. What is the speed of light through
this glass? Sapphire 1.77
Zircon 1.92
Solution
Qubic zirconia 2.16
c c
n= ; v= Diamond 2.42
v n
Gallium phosphide 3.50
3×108
v= = 2×108 m s-1
1. 5
Light travels with a speed of 2 × 108 m s-1
6.3.4  Optical path
through this glass.
Optical path of a medium is defined as
Refractive index does not have unit. the distance d' light travels in vacuum in
The smallest value of refractive index is for the same time it travels a distance d in the
vacuum, which is 1. For any other medium medium.
refractive index is greater than 1. Refractive Let us consider a medium of refractive
index is also called as optical density of index n and thickness d. Light travels with
the medium. Higher the refractive index a speed v through the medium in a time t.
of a medium, greater is its optical density Then we can write,
and speed of light through the medium is d d
lesser and vice versa. [Note: optical density v = ; rewrittenas, t =
t v
14 Unit 6  OPTICS

UNIT-6(XII-Physics_Vol-2).indd 14 7/31/2019 7:44:06 PM


In the same time, light can cover a (iii) What is the optical path of the glass
greater distance d' in vacuum as it travels slab?
with greater speed c in vacuum as shown in
Figure 6.14. Then we have, Solution
Given, thickness of glass slab, d = 50 cm =
0.5 m, refractive index, n = 1.5
d c
refractive index, n =
v
Medium of refractive index n speed of light in glass is,
c 3×108
v= = = 2×108 m s−1
n 1. 5
d' = n d
Time taken by light to travel through glass
Vacuum of refractive index 1 slab is,
d 0. 5
Figure 6.14  Optical path t= = 8
= 2.5×10−7 s
v 2×10
Optical path,
d ′ = nd = 1.5×0.5 = 0.75 m = 75 cm
d′ d′ d′ d′
rewritten
c = c =; rewritten )as,
for (tfor
; rewritten as(t, )tas=, t =
t t c c Light would have travelled 25 cm more
As the time taken in both the cases is the (75  cm – 50 cm) in vacuum by the same
same, we can equate the time t as, time had there not been a glass slab.
d′ d′ d d
==
c c v v 6.4
c c
rewritten
rewrittenfor thetheoptical
forfor
rewritten opticalpath
theoptical path(dd'
path ) as,
′(d ′) ,asd,′ d=′ =d d
as REFRACTION
v v
c
As, = n;The Theoptical pathd ′d'is,is,
opticalpath Refraction is passing through of light
v
from one optical medium to another optical
d ′ = nd (6.17) medium through a boundary. In refraction,
the angle of incidence i in one medium
As n is always greater than 1, the optical and the angle of reflection r in the other
path d' of the medium is always greater medium are measured with respect to the
than d. normal drawn to the surface at the point of
incidence of light. Law of refraction is called
Snell’s law.
EXA M P LE 6 .6 Snell’s law states that,
Light travels from air in to glass slab of a) The incident ray, refracted ray and
thickness 50 cm and refractive index 1.5. normal to the refracting surface are
all coplanar (ie. lie in the same plane).
(i) What is the speed of light in glass? b) The ratio of angle of incident i in the
(ii) What is the time taken by the light to first medium to the angle of reflection
travel through the glass slab? r in the second medium is equal to the

Unit 6  OPTICS 15

UNIT-6(XII-Physics_Vol-2).indd 15 7/31/2019 7:45:12 PM


ratio of refractive index of the second from normal as shown in Figure 6.17. The
medium n2 to that of the refractive angle of deviation in this case is,
index of the first medium n1.
d=r–i (6.21)
sin i n2
= (6.18)
sin r n1

The above equation is in the ratio form. It n1<n2


i
can also be written in a much useful product n1
form as, n2

r
n1 sin i = n2 sin r (6.19)
d=i-r

The refraction at a boundary is shown in


Figure 6.15. Figure 6.16 Angle of deviation due to
refraction from rarer to denser medium

i i n1>n2
n1
n2 n1
n2
n2
r
r d=i-r

Figure 6.15 Refraction of light


Figure 6.17 Angle of deviation due to
refraction from denser to rarer medium
For normal incidence of light
Note
on a surface, the angle of
6.4.2 Characteristics of
incidence is zero.
refraction
a) When light passes from rarer medium
6.4.1 Angle of deviation due to denser medium it deviates towards
to refraction normal in the denser medium.
b) When light passes from denser medium
We know that the angle between the to rarer medium it deviates away from
incident and deviated light is called angle of normal in the rarer medium.
deviation. When light travels from rarer to c) In any refracting surface there will also
denser medium it deviates towards normal be some reflection taking place. Thus, the
as shown in Figure 6.16. The angle of intensity of refracted light will be lesser
deviation in this case is, than the incident light. The phenomenon
d=i–r (6.20) in which a part of light from a source
undergoing reflection and the other
On the other hand, if light travels from part of light from the same source
denser to rarer medium it deviates away undergoing refraction at the same

16 Unit 6 OPTICS

UNIT-6(XII-Physics_Vol-2).indd 16 7/31/2019 7:45:30 PM


surface is called simultaneous reflection 6.4.3  Principle of
or simultaneous refraction. This is
reversibility
shown in Figure 6.18. Such surfaces are
available as partially silvered glasses. The  principle of reversibility  states
that light will follow exactly the same path
Normal if its direction of travel is reversed. This
(1) is true for both reflection and refraction as
Incident ray Reflected ray shown in Figure 6.15.

i i

i r i r
Reflecting surface (a)
r
(2) i
Rarer (1)
r
Rarer (1)
Denser (2) Denser (2)
r i
(b)
Refracted ray Plane Mirror

Figure 6.18  Simultaneous reflection and Figure 6.19  Principle of reversibility in


refraction (a) reflection and (b) refraction

Production of optical surfaces


capable of refracting as well as 20%
LIGHT RATIO
80%

reflecting is possible by properly Reflective Silver


20 Grade Flim
coating the surfaces with suitable materials.
CAN SEE
Thus, a glass can be made partially see CAN’T SEE

through and partially reflecting by varying


the amount of coating on its surface. It is
commercially called as two way mirror,
half-silverd or semi-silvered mirror etc.
This gives a perception of regular mirror
if the other side is made dark. But, still
hidden cameras can be kept behind such
mirrors. We need to be cautious when we
stand in front of mirrors kept in unknown places.
There is a method to test the two way mirror. Place
the finger nail against the mirror surface. If there is
a gap between nail and its image, then it is a regular
mirror. If the fingernail directly touches its image,
then it is a two way mirror.
Normal mirror Two way mirror

Unit 6  OPTICS 17

UNIT-6(XII-Physics_Vol-2).indd 17 7/31/2019 7:45:31 PM


6.4.4  Relative refractive 6.4.5  Apparent depth
index It is a common observation that the bottom
In the equation for Snell’s law, the term of a tank filled with water appears raised as
 n  shown in Figure 6.19(a). An equation could
 2  is called relative refractive index of
 n  be derived for the apparent depth for viewing
1

second medium with respect to the first in the near normal direction. The ray diagram
medium which is denoted as (n21). is shown in Figure 6.19(b) and (c).

n2
n21 = (6.22)
n1

The concept of relative refractive index


gives rise to other useful relation such as,
a) Inverse rule:

1 n 1
n12 = (or ) 1 = (6.23)
n21 n2 n2 / n1 (a)

b) Chain rule:
(b)

n3 n3 n1
  n32 = n31 ×n12 (or ) = × (6.24)
n2 n1 n2
Air
X A D B Y

EXA MP LE 6 .7 Water
Apparent
depth
Real
Light travelling through transparent oil depth
P' Q'
I
enters in to glass of refractive index 1.5. If
O
the refractive index of glass with respect to P Q

the oil is 1.25, what is the refractive index (c)


C
of the oil?
N

Solution X D
r
Air Y
B Water
Given, ngo =1.25 and ng =1.5 d'
r
i
I
Refractive index of glass with respect to oil, N'
i
ng d
ngo = O
no
Rewriting for refractive index of oil, Figure 6.20  Apparent depth
ng1. 5
no = = = 1.2
ngo 1.25 Light from the object O at the bottom
The refractive index of oil is, no = 1.2 of the tank passes from denser medium
(water) to rarer medium (air) to reach our
18 Unit 6  OPTICS

UNIT-6(XII-Physics_Vol-2).indd 18 7/31/2019 7:46:14 PM


eyes. It deviates away from the normal in the In that case, the equation for apparent depth
rarer medium at the point of incidence B. becomes,
The refractive index of the denser medium
d
is n1 and rarer medium is n2. Here, n1 > n2. d ′ = (6.27)
n
The angle of incidence in the denser
medium is i and the angle of refraction in The bottom appears to be elevated by
the rarer medium is r. The lines NN ′ and d-d',
OD are parallel. Thus angle ∠DIB is also d  1
r. The angles i and r are very small as the   d − d ′ = d − or d − d ′ = d 1 − (6.28)
n  n
diverging light from O entering the eye is
very narrow. The Snell’s law in product form
for this refraction is, EX AM P L E 6 . 8

n1 sin i = n2 sin r (6.19) A coin is at the bottom of a trough


containing three immiscible liquids of
As the angles i and r are small, we can refractive indices 1.3, 1.4 and 1.5 poured
approximate, sin i ≈ tan i; one above the other of heights 30 cm, 16
cm, and 20 cm respectively. What is the
n1 tan i = n2 tani apparent depth at which the coin appears
to be when seen from air medium outside?
In triangles ∆DOB and ∆DIB,
In which medium the coin will be seen?
DB DB
tan(i) = and tan (r ) = Solution
DO DI
When seen from top, the coin will still
DB DB appear to be at the bottom with each
n1 = n2
DO DI medium appearing to have shrunk with
DB is cancelled on both sides, DO is respect to the air medium outside. This
the  actual depth d and DI is the apparent situation is illustrated below.
depth d'.

1 1
n1 = n2
d d′
d'= 47.8 cm

d ′ n2 n3=1.5 d3'=13.3 cm
= (6.25) n1=1.3 d3=20cm
n2=1.4 d2'=11.4 cm
d= 66 cm

d n1 n2=1.4 d2=16cm n1=1.3 d1'=23.1 cm


Coin
n3=1.3
Rearranging the above equation for the Coin
d1=30 cm

apparent depth d',

n2 The equations for apparent depth for each


d′ = d (6.26) medium is,
n1
d1 d d3
As the rarer medium is air and its d1′ = ; d2′ = 2 ; d3′ =
n1 n2 n3
refractive index n2 can be taken as 1, (n2=1).
And the refractive index n1 of denser d1 d2 d3
d ′ = d1′ + d2′ + d3′ = + +
medium could then be taken as n, (n1=n). n1 n2 n3

Unit 6  OPTICS 19

UNIT-6(XII-Physics_Vol-2).indd 19 7/31/2019 7:47:54 PM


Atmospheric refraction: Due to refraction of light through different
layers of atmosphere which vary in refractive index, the path of light deviates
continuously when it passes through atmosphere. For example, the Sun is
visible a little before the actual sunrise and also until a little after the actual sunset due to
refraction of light through the atmosphere. By actual sunrise what we mean is the actual
crossing of the sun at the horizon. Figure shows the actual and apparent positions of the
sun with respect to the horizon. The figure is highly exaggerated to show the effect. The
apparent shift in the direction of the sun is around half a degree and the corresponding
time difference between actual and apparent positions is about 2 minutes. Sun appears
flattened (oval shaped) during sun rise and sunset due to the same phenomenon.
The same is also applicable for the positions of stars as shown in Figure. The stars
actually do not twinkle. They appear twinkling because of the movement of the atmospheric
layers with varying refractive indices which is clearly seen in the night sky.

Apparent
Possition of sun

Actual
Possition of sun

Atmosphere Horizon

30 16 30 incidence in the denser medium for which


d′ = + + = 23.1 + 11.4 + 13.3 the refracted ray graces the boundary is
1. 3 1. 4 1. 5
d ′ = 47.8 cm called critical angle ic.
If the angle of incidence in the denser
medium is increased beyond the critical
angle, there is no refraction possible in to the
6.4.6  Critical angle and total
rarer medium. The entire light is reflected
internal reflection
back into the denser medium itself. This
When a ray passes from an optically
phenomenon is called total internal
denser medium to an optically rarer medium,
reflection. These are shown in Figure 6.21.
it bends away from normal. Because of this,
The two conditions for total internal
the angle of refraction r on the rarer medium
reflection are,
is greater than the corresponding angle of
incidence i in the denser medium. As angle (a) light must travel from denser to rarer
of incidence i is gradually increased, r rapidly medium,
increases and at a certain stage it becomes (b) angle of incidence in the denser medium
90° or gracing the boundary. The angle of must be greater than critical angle (i>ic).

20 Unit 6  OPTICS

UNIT-6(XII-Physics_Vol-2).indd 20 7/31/2019 7:48:10 PM


Air

ay
dr
cte
fra
Critical angle Total internal
r

Re
n2 r = 90 reflection

n1
i r=i
ray

i
ent

(i>ic)
d
Inci

Water

Figure 6.21  Critical angle and total internal reflection

Snell’s law in the product form, equation Table 6.3  Refractive index and
(6.19) for critical angle incidence becomes,
critical angle of different media
n1 sin ic = n2 sin 90(6.29) Material Refractive Critical
index Angle
n1 sin ic = n2 Q sin 90o = 1 Ice 1.310 49.8°
Water 1.333 48.6°
n2
sinic = (6.30) Fused Quartz (SiO2) 1.458 43.3°
n1
Crown Glass 1.541 40.5°
Here, n1 > n2 Flint Glass 1.890 31.9°

If the rarer medium is air, then its Calcite (CaCO2) 1.658 37.0°
refractive index is 1 and can be taken as n Diamond 2.417 24.4°
itself. i.e. (n2=1) and (n1=n). Strontium Titanate 2.417 24.4°
(SrTiO3)
1 1
sinic = (or) ic = sin−1  (6.31) Rutile 2.621 22.4°
n  n 

For example the refractive index of


glass is about 1.5. The critical angle for 6.4.7  Effects due to total
 1  internal reflection
glass-air interface is, ic = sin−1  = 41.8.
1.5  6.4.7.1  Glittering of diamond
The critical angle for water-air interface is, Diamond appears dazzling because the
1 total internal reflection of light happens
ic = sin−1   = 48.6.
1.3 
inside the diamond. The refractive index
The critical angle ic depends on the of only diamond is about 2.417. It is much
refractive index of the medium. Table 6.3 larger than that for ordinary glass which is
shows the refractive index and the critical about only 1.5. The critical angle of diamond
angle for different materials. is about 24.4°. It is much less than that of
Unit 6  OPTICS 21

UNIT-6(XII-Physics_Vol-2).indd 21 7/31/2019 7:49:36 PM


glass. A skilled diamond cutter makes use of at a height. In the cold regions like glaciers
this larger range of angle of incidence (24.4° and frozen lakes and seas, the reverse effect
to 90° inside the diamond), to ensure that of mirage will happen. Hence, an inverted
light entering the diamond is total internally image is formed little above the surface as
reflected from the many cut faces before shown in Figure 6.23(b). This phenomenon
getting out as shown in Figure 6.22. This is called looming.
gives a sparkling effect for diamond.
Critical angle
o

E
Denser
Air Diamond
Total
Reflection i>ic

Rare
Figure 6.22  Total internal reflection in
diamond
Earth
6.4.7.2  Mirage and looming
The refractive index of air increases with
its density. In hot places, air near the ground (a) Mirage
is hotter than air at a height. Hot air is less
dense. Hence, in still air the refractive index
of air increases with height. Because of this,
light from tall objects like a tree, passes
Rarer
through a medium whose refractive index Sky
decreases towards the ground. Hence, a
ray of light successively deviates away from
the normal at different layers of air and Denser
undergoes total internal reflection when the
angle of incidence near the ground exceeds
the critical angle. This gives an illusion as if Earth
the light comes from somewhere below the
(b)Looming
ground. For of the shaky nature of the layers
of air, the observer feels as if the object is Figure 6.23  Mirage and looming
getting reflected by a pool of water or wet
surface beneath the object as shown in 6.4.7.3 Prisms making using of total
Figure 6.23(a). This phenomenon is called internal reflection
mirage. Prisms can be designed to reflect light
In the cold places the refractive index by 90o or by 180o by making use of total
increases towards the ground because the internal reflection as shown in Figure
temperature of air close to the ground is 6.24(a) and (b). In the first two cases, the
lesser than the temperature above the surface critical angle ic for the material of the prism
of earth. Thus, the density and refractive must be less than 45o. We see from Table 6.3
index of air near the ground is greater than that this is true for both crown glass and

22 Unit 6  OPTICS

UNIT-6(XII-Physics_Vol-2).indd 22 7/31/2019 7:49:38 PM


flint glass. Prisms are also used to invert undergo total internal reflection.The light
images without changing their size as shown falling particularly at critical angle graces
in Figure. 6.24(c). the surface.Thus, the entire surface of water
appears illuminated when seen from outside
45°
B
45° as shown in Figure 6.25.
B On the other hand, when light entering
A
45° 90° the water from outside is seen from
A
B'
inside the water, the view is restricted to a
90° 45°
A' particular angle equal to the critical angle
45°

B' A'
ic. The restricted illuminated circular area
is called Snell’s window as shown in Figure
B A' 6.26(a).The Figure 6.26(b) shows the angle
of view for water animals.
A B'

Figure 6.24  Prisms making use of total


internal reflection

6.4.7.4  Radius of illumination (Snell’s


window)

(a)

ic ic

Figure 6.25  Light source inside water tank


(b)
When a light source like electric bulb Figure 6.26  (a) Snell’s window and
is kept inside a water tank, the light from (b) angle of view for water animals
the source travels in all direction inside the
water. The light that is incident on the water The angle of view for water animals is
surface at an angle less than the critical restricted to twice the critical angle 2ic. The
angle will undergo refraction and emerge critical angle for water is 48.6o. Thus the angle
out from the water. The light incident at of view is 97.2o. The radius R of the circular
an angle greater than critical angle will area depends on the depth d from which it
Unit 6  OPTICS 23

UNIT-6(XII-Physics_Vol-2).indd 23 7/31/2019 7:49:39 PM


is seen and also the refractive indices of the 2 2
d2  n  d 2  n1 
media. The radius of Snell’s window can be 1 + 2 =  1  ; =   −1;
R  n2  R 2  n2 
deduced with the illustration as shown in 2
Figure 6.27. d 2 n1 n12 − n22
= −1 =
R 2 n22 n22
Again taking reciprocal and rearranging,
r=90° n2
R R2 n22  n2 
C = ; R 2 = d 2  2 2 2 
B d 2 n12 − n22  n1 − n2 
The radius of illumination is,
ic n1
d ic
n22
R=d (6.35)
(n
2
1 − n22 )

A If the rarer medium outside is air, then,


Figure 6.27  Radius of Snell’s window n2 = 1, and we can take n1=n

Light is seen from a point A at a depth  1  d


R = d   (or ) R = (6.36)
d. The Snell’s law in product form, equation  n −1 
2  2
n −1
(6.19) for the refraction happening at the
point B on the boundary between the two
media is, EX AM P L E 6 . 9

n1 sin ic = n2 sin 900 (6.32) What is the radius of the illumination


when seen above from inside a swimming
pool from a depth of 10 m on a sunny day?
n1 sin ic = n2  sin 900 = 1 What is the total angle of view? [Given,
refractive index of water is 4/3]
n2
sinic = (6.33) Solution
n1
Given, n = 4/3, d = 10 m.
From the right angle triangle ∆ABC, d
Radius of illumination, R =
CB R n2 − 1
sinic = = (6.34)
AB d 2 + R2 10 10×3
R= =
Equating the above two equation 6.34 (4 / 3)2 −1 16 − 9

R n2 30
and equation 6.35, = R= = 11.32m
2
d +R 2 n1 7
 n 
2 To find the angle of the view of the cone,
R2  2 
Squaring on both sides, 2 =
R + d 2  n1 
1
ic = sin−1  
2
 n 
R 2 + d 2  n1 
Taking reciprocal, =    1  3
R2  n2  ic = sin−1  = sin−1   = 48.6


 4 / 3   4 
On further simplifying,
24 Unit 6  OPTICS

UNIT-6(XII-Physics_Vol-2).indd 24 7/31/2019 7:51:48 PM


The total angle of view is, for total internal reflection is ensured at
2ic = 2× 48.6 = 97.2 every reflection as shown in Figure 6.28(b).
6.4.7.6 Acceptance angle in optical fibre
6.4.7.5 Optical fiber To ensure the critical angle incidence in
the core-cladding boundary inside the optical
fibre, the light should be incident at a certain
angle at the end of the optical fiber while
entering in to it. This angle is called acceptance
angle. It depends on the refractive indices of
the core n1, cladding n2 and the outer medium
n3. Assume the light is incident at an angle
called acceptance angle ia at the outer medium
and core boundary at A.
(a) (a) n2 r=90°
B
n11 i2i2
Total internal reflection n3 A r1
i1 C
Cladding has lower
refractive index
n2

(b) Acceptance Cone


Core has higher
(b) refractive index

Figure 6.28  Optical fibre

Transmitting signals through optical


fibres is possible due to the phenomenon
of total internal reflection. Optical fibres
consists of inner part called core and outer Figure 6.29  (a) acceptance angle and
(b) acceptance cone.
part called cladding (or) sleeving. The
refractive index of the material of the core The Snell’s law in the product form,
must be higher than that of the cladding for equation (6.19) for this refraction at the
total internal reflection to happen. Signal in point A is as shown in the Figure 6.29(a),
the form of light is made to incident inside
the core-cladding boundary at an angle n3 sin ia = n1 sin ra (6.37)
greater than the critical angle. Hence, it To have the total internal reflection
undergoes repeated total internal reflections inside optical fibre, the angle of incidence
along the length of the fibre without at the core-cladding interface at B should
undergoing any refraction. The light travels be atleast critical angle ic. Snell’s law in
inside the core with no appreciable loss in the product form, equation (6.19) for the
the intensity of the light as shown in Figure refraction at point B is,
6.28(a). Even while bending the optic fiber,
it is done in such a way that the condition n1 sin ic = n2 sin 90o (6.38)

Unit 6  OPTICS 25

UNIT-6(XII-Physics_Vol-2).indd 25 7/31/2019 7:52:12 PM


n1 sin ic = n2   sin90° = 1 called numerical aperture NA of the optical
fibre.
n2
∴ sini c = (6.39)
n1 NA = n3 sin ia = n12 − n22 (6.46)
From the right angle triangle ∆ABC,
If outer medium is air, then n3 = 1. The
ic = 90°− ra numerical aperture NA becomes,
Now, equation (6.39) becomes,
NA = sin ia = n12 − n22 (6.47)
n2
sin (90 − ra ) =
n1
EX AM P L E 6 . 1 0
n2 A optical fibre is made up of a core material
  Using trigonometry, cosra =  (6.40)
n1 with refractive index 1.68 and a cladding
material of refractive index 1.44. What is
sin ra = 1 − cos2 ra
the acceptance angle of the fibre kept in air
Substituting for cos ra medium? What is the answer if there is no
cladding?
  n 
2
n2 − n2
sinra = 1 −  2  = 1 2 2 (6.41) Solution
 n1  n1
Given, n1 = 1.68,  n2 = 1.44,  n1 = 1
Substituting this in equation (6.37).
Acceptance angle, ia = sin−1 ( n12 − n22 )
n12 − n22
  n3 sin ia = n1 2
n1
= n12 − n22 (6.42) ia = sin−1 ( )
(1.68)2 − (1.44)2 = sin−1 (0.865)

On further simplification, ia ≈ 60°


If there is no cladding then, n2 = 1
n12 − n22 2
n −n 2
sin ia =
n3
(or ) sin ia = 1

n23
(6.43)
2
Acceptance angle, ia = sin−1 ( n12 −1 )
 n2 − n2 
ia = sin−1 ( )
(1.68)2 −1 = sin−1 (1.35)

ia = sin−1  1 2 2 (6.44) sin−1 (more than 1) is not possible. But, this
 n3 
includes the range 0o to 90o. Hence, all the
If outer medium is air, then n3 = 1. The rays entering the core from flat surface will
acceptance angle ia becomes, undergo total internal reflection.
Note: If there is no cladding then there is
ia = sin −1
( n − n (6.45)
2
1
2
2 ) a condition on the refractive index (n1) of
the core.
Light can have any angle of incidence
from 0 to ia with the normal at the end of the ia = sin−1 ( n12 −1 )
optical fibre forming a conical shape called Here, as per mathematical rule,
acceptance cone as shown in Figure 6.29(b). (n2
−1) ≤ 1 or (n ) ≤ 2
2
1 1
In the equation (6.42), the term (n3 sin ia) is
or n1 £ 2
26 Unit 6  OPTICS

UNIT-6(XII-Physics_Vol-2).indd 26 7/31/2019 7:54:54 PM


Hence, in air (no cladding) the refractive leaves the slab it travels from denser medium
index n1 of the core should be, n1 £ 1.414 to rarer medium resulting in deviation of
ray away from the normal. After the two
refractions, the emerging ray has the same
An endoscope is an direction as that of the incident ray on the
instrument used by doctors slab with a lateral displacement or shift L.
which has a bundle of optical i.e. There is no change in the direction of ray
fibres that are used to see but the path of the incident ray and refracted
inside a patient’s body. Endoscopes ray are different and parallel to each other.
work on the phenomenon of total To calculate the lateral displacement, a
internal reflection. The optical fibres perpendicular is drawn in between the paths
are inserted in to the body through of incident ray and refracted ray as shown in
mouth, nose or a special hole made in Figure 6.30.
the body. Even operations could be
N1
carried out with the endoscope cable A

which has the necessary instruments i


attached at their ends. B air

i-r
r glass

Endoscope

r
E

C
i L

D
N2

Figure 6.30  Refraction in glass slab

Consider a glass slab of thickness t and


refractive index n is kept in air medium. The
path of the light is ABCD and the refractions
occur at two points B and C in the glass slab.
The angles of incidence i and refraction r are
measured with respect to the normal N1 and
N2 at the two points B and C respectively. The
6.4.9  Refraction in glass slab
lateral displacement L is the perpendicular
When a ray of light passes through distance CE drawn between the path of light
a glass slab it refracts at two refracting and the undeviated path of light at point C.
surfaces. When the light ray enters the slab In the right angle triangle ∆BCE,
it travels from rarer medium (air) to denser
L L
medium (glass). This results in deviation of   sin(i − r ) = ; BC = (6.48)
BC sin (i − r )
ray towards the normal. When the light ray
Unit 6  OPTICS 27

UNIT-6(XII-Physics_Vol-2).indd 27 7/31/2019 7:55:09 PM


In the right angle triangle ∆BCF, 6.5
t t REFRACTION AT SINGLE
cos (r ) = ; BC = (6.49) SPHERICAL SURFACE
BC cos (r )

Equating equations (6.48) and (6.49), We have so far studied only the refraction
L t at a plane surfaces. The refractions also do
=
sin (i − r ) cos (r ) take place at spherical surface between two
transparent media. The laws of refraction
After rearranging, hold good at every point on the spherical
 sin (i − r ) surface. The normal at the point of incidence
L = t   (6.50) is perpendicular to the tangent plane to the
 cos (r ) 
spherical surface at that point. Therefore,
Lateral displacement depends upon the normal always passes through its center
the thickness of the slab. Thicker the slab, of curvature. The study of refraction at
greater will be the lateral displacement. single spherical surface paves way to the
Greater the angle of incident, larger will be understanding of thin lenses which consist
the lateral displacement. of two surfaces of which one or both must
be spherical.
The following assumptions are made
EXA MP LE 6 .1 1 while considering refraction at spherical
surfaces.
The thickness of a glass slab is 0.25 m. it
(a) The incident light is assumed to be
has a refractive index of 1.5. A ray of light
monochromatic (single colour)
is incident on the surface of the slab at an
(b) The incident ray of light is very close to
angle of 60o. Find the lateral displacement
the principal axis (paraxial rays).
of the light when it emerges from the other
The sign conventions are similar to that
side of the mirror.
of the spherical mirrors.
Solution
Given, thickness of the lab, t = 0.25 m, 6.5.1  Equation for refraction
refractive index, n = 1.5, angle of incidence, at single spherical surface
i = 60o.
Let us consider two transparent media
Using Snell’s law, 1 × sin i = n sin r having refractive indices n1 and n2 are
sini sin 60 separated by a spherical surface as shown in
sin r = = = 0.58
n 1. 5 Figure 6.31. Let C be the centre of curvature
r = sin−1 0.58 = 35.25 of the spherical surface. Let a point object O
 sin (i − r ) be in the medium n1. The line OC cuts the
Lateral displacement is, L = t  
 cos (r )  spherical surface at the pole P of the surface.
 sin (60 − 35.25) As the rays considered are paraxial rays,
L = (0.25)×  = 0.1281 m
the perpendicular dropped for the point of
 cos (35.25) 
incidence to the principal axis is very close
The lateral displacement is, L = 12.81 cm
to the pole or passes through the pole itself.

28 Unit 6  OPTICS

UNIT-6(XII-Physics_Vol-2).indd 28 7/31/2019 7:56:10 PM


Substituting for i and r from equations
N (6.53) and (6.54) in the equation (6.51).
n1 i n2
r n1 (a + b ) = n2 (b − g )
O    I
P R C Rearranging,

u v n1a + n2g = (n2 − n1 ) b

Substituting for α, β and γ from equation


Figure 6.31  Refraction at single spherical (6.52),
surface
 PN   PN   PN 
n1   + n   = (n − n )
Light from O falls on the refracting

 PO  2  PI  2  PC 
1 

surface at N. The normal drawn at the point Further simplifying by cancelling PN,
of incidence passes through the centre of
n1 n n − n1
curvature C. As n2>n1, light in the denser + 2 = 2 (6.55)
PO PI PC
medium deviates towards the normal and
meets the principal axis at I where the image Following sign conventions, PO = –u,
is formed. PI = +v and PC = +R in equation (6.58),
Snell’s law in product form for the
refraction at the point N could be written as, n1 n2 (n2 − n1 )
+ =
−u v R
n1 sin i = n2 sin r (6.19) After rearranging, finally we get,
As the angles are small, sin of the angle n2 n1 (n2 − n1 )
could be approximated to the angle itself. − = (6.56)
v u R
n1i = n2r (6.51) Equation (6.56) gives the relation among
the object distance u, image distance v,
Let the angles,
refractive indices of the two media (n1 and
∠NOP = α, ∠NCP = β, ∠NIP = γ n2 ) and the radius of curvature R of the
PN PN PN spherical surface. It holds for any spherical
tan a = ; tan b = ; tan g =
PO PC PI surface.
As these angles are small, tan of the angle If the first medium is air then, n1 = 1 and
could be approximated to the angle itself. the second medium is taken just as n2 = n,
then the equation is reduced to,
PN PN PN
a= ; b= ;g= (6.52)
PO PC PI n 1 (n −1)
− = (6.57)
v u R
For the triangle, ∆ONC,
i = α + β (6.53)
EX AM P L E 6 . 1 2
For the triangle, ∆INC,
β = r + γ (or) r = β – γ(6.54) Locate the image of the point object O in
the situation shown. The point C denotes

Unit 6  OPTICS 29

UNIT-6(XII-Physics_Vol-2).indd 29 7/31/2019 7:57:29 PM


the centre of curvature of the separating goes undeviated in the second medium.The
surface. position of image may be located using the
Equation (6.60).

n1 = 1 n2 = 1.5
O'
n1 n2
o P c h1 I
15 cm 30 cm O P C h2
I'

Solution
Figure 6.32  Lateral magnification in
Given, u = –15 cm, R = 30 cm, n1 = 1 and single spherical surface
n2 = 1.5
Equation for single spherical surface is, The lateral or transverse magnification
n2 n1 (n2 − n1 ) m is defined as the ratio of height of the
− =
v u R image to the height of the object.
Substituting the values, II ′
m= (6.58)
1. 5 1 (1.5 −1) 1.5 1 (0.5) OO ′
− = ; + =
v −15 30 v 15 30 From the two similar triangles ∆ COO′
1. 5 1 1 1. 5 1 1 1.5 and
1 − 4∆ CII
−′3, we can
1 write,
+ = ; = − ; = = ;= −
v 15 60 v 60 15 v 60 II ′ 60 CI 20
=
1. 5 1 1 1 . 5 1 − 4 −3 1 OO ′ CO
= − ; = = ;= −
v 60 15 v 60 60 20 From the geometry,
v = –30 cm CI PI − PC
=
CO PC + PO
The image is a virtual image formed 30 cm
Hence,
to the left of the spherical surface.
II ′ PI − PC
m= =  (6.59)
OO ′ PC + PO
6.5.2  Lateral magnification Applying sign conventions in the above
in single spherical surface equation (6.59),
Let us, consider an extended object OO′ II ′ = −h2 , OO′ = h1 , PI = +v ,
is kept perpendicular to the principal axis PC = +R, PO = −u
to the left of the single spherical surface as
shown in Figure 6.32. The image formed on Where, h1 is the height of the object and
the other side of the surface is II ′ . Consider h2 is the height of the image.
a ray from O ′ in the first medium towards
−h2 v−R h  v − R 
C in the second medium. As this ray is m= = ; m = 2 = −
h1 R + (−u) h1  R − u 
incident normal to the spherical surface, it
30 Unit 6  OPTICS

UNIT-6(XII-Physics_Vol-2).indd 30 7/31/2019 7:58:26 PM


After rearranging, Solution
h2 R − v Given, u = –40 cm, R = –20 cm, n1 = 1 and
m= = (6.60)
h1 R − u n2 = 1.33
We can also arrive at an equation for Equation for single spherical surface is,
lateral magnification involving the refractive n2 n1 (n2 − n1 )
indices of the two media. − =
v u R
Let us consider the equation for single Substituting the values,
spherical surface as,
1.33 1 (1.33 −1) 1.33 1 (0.33)
− = ; + =
n2 n1 (n2 − n1 ) v −40 −20 v 40 −20
− =
v u R 1.33 (0.33) 1
=− − ;
v 20 40
n2u − n1v (n2 − n1 ) 1.33 −0.66 −1 1.66
Further simplifying, = = =−
vu R v 40 40
(n2 − n1 )vu v = −40×
1.33
= −32.0 cm
Rewriting for R, R=
n2u − n1v 1.66
h2 n1v
Rearranging, Equation for magnification is, m = =
h1 n2u
n2u (v − u) h2 (1.0)×(−32)
R −u = (6.61) = = 0.6 cm (or ) h2 = 0.6 cm
n2u − n1v 1.0 (1.33)×(−40)
The erect virtual image of height 0.6 cm is
formed at 32.0 cm to the left of the single
n1v (v − u)
R −v = (6.62) spherical surface.
n2u − n1v

Substituting equations (6.61) and (6.62) 6.6


in equation (6.60) we get the equation for
THIN LENS
lateral magnification as,
h2 n1v A lens is formed by a transparent material
m= = (6.63)
h1 n2u bounded between two spherical surfaces or
one plane and another spherical surface. In a
EXA MP LE 6 .1 3 thin lens, the distance between the surfaces
is very small. If there are two spherical
Find the size of the image formed in the
surfaces, then there will be two centres of
given figure.
curvature C1 and C2 and correspondingly two
radii of curvature R1 and R2. A plane surface
n =1 n = 1.33
1 2 has its center of curvature C at infinity and
1.0 cm C its radius of curvature R is infinity (R = ∞).
O 20 cm The terminologies of spherical mirrors also
hold good very much for thin lens except for
40 cm
focal length.

Unit 6  OPTICS 31

UNIT-6(XII-Physics_Vol-2).indd 31 7/31/2019 8:00:04 PM


6.6.1  Primary and secondary 6.6.2  Sign conventions for
focal points lens on focal length
As the thin lens is formed by two spherical The sign conventions for thin lenses
surfaces, the lens may separate two different differ only in the signs followed for focal
media. i.e. the media to the left and right of lengths.
the lens may be different. Hence, we have (a) The sign of focal length is not decided
two focal lengths. on the direction of measurement of the
focal length from the pole of the lens as
they have two focal lengths, one to the
F1 P P F1
left and another to the right (primary
and secondary focal lengths on either
side of the lens).
(a) Primary focal point
(b) The focal length of the thin lens is taken
as positive for a converging lens and
negative for a diverging lens.
F2 P
P F2
The other sign conventions for object
distance, image distance, radius of curvature,
(b) Secondary focal point object height and image height (except
Figure 6.33  Focal length of convex for the focal lengths as mentioned above)
and concave lenses remain the same for thin lenses as that of
spherical mirrors.
The primary focus F1 is defined as a
point where an object should be placed
to give parallel emergent rays to the 6.6.3  Lens maker’s formula
principal axis as shown in Figure 6.33(a). and lens equation
For a convergent lens, such an object is a real
object and for a divergent lens, it is a virtual n1 n2 n1
object. The distance PF1 is the primary focal
length f1.
1 2
The secondary focus F2 is defined as a
point where all the parallel rays travelling O P I I'
close to the principal axis converge to form
an image on the principal axis as shown in
Figure 6.33(b). For a convergent lens, such v
an image is a real image and for a divergent
u v
lens, it is a virtual image. The distance PF2 is
the secondary focal length f2. Figure 6.34  Refraction through thin lens
If the media on the two sides of a thin
lens have same refractive index, then the two Let us consider a thin lens made up of a
focal lengths are equal. We will mostly be medium of refractive index n2 is placed in
using the secondary focus F2 in our further a medium of refractive index n1. Let R1 and
discussions. R2 be the radii of curvature of two spherical
32 Unit 6  OPTICS

UNIT-6(XII-Physics_Vol-2).indd 32 7/31/2019 8:00:04 PM


surfaces  and  respectively and P be If the refractive index of the lens is n2 and
the pole as shown in figure 6.34. Consider a it is placed in air, then n2 = n and n1 = 1. So
point object O on the principal axis. The ray the equation (6.67) becomes,
which falls very close to P, after refraction 1
1 1
at the surface  forms image at I'. Before = (n −1) − (6.68)
f  R1 R2 
it does so, it is again refracted by the surface
. Therefore the final image is formed at I. The above equation is called the lens
The general equation for the refraction maker’s formula, because it tells the lens
at a spherical surface is given from Equation manufactures what curvature is needed to
(6.59), make a lens of desired focal length with a
material of particular refractive index. This
n2 n1 (n2 − n1 ) formula holds good also for a concave lens.
− =
v u R By comparing the equations (6.66) and
For the refracting surface , the light (6.67) we can write,
goes from n1 to n2. 1 1 1
- = (6.69)
n2 n1 (n2 − n1 ) v u f
− = (6.64)
v′ u R1 This equation is known as lens equation
For the refracting surface , the light which relates the object distance u and
goes from medium n2 to n1. image distance v with the focal length f of
the lens. This formula holds good for a any
n1 n2 (n1 − n2 )
− = (6.65) type of lens.
v v′ R2
Adding the above two equations (6.64) 6.6.4  Lateral magnification
and (6.65) in thins lens
n1 n1 1 1 Let us consider an object OO' of height
− = (n2 − n1 ) −  h1 placed on the principal axis with its
v u  R1 R2 
height perpendicular to the principal
Further simplifying and rearranging,
axis as shown in Figure 6.35. The ray OP
1 1  n2 − n1  1 1 passing through the pole of the lens goes
− =   − 
v u  n1  R1 R2  undeviated. The inverted real image II ′
formed has a height h2.
1 1  n2  1 1
− =  −1 − (6.66)
v u  n1  R1 R2  O
If the object is at infinity, the image is
formed at the focus of the lens. Thus, for
F I
u = ∞, v = f. Then the equation becomes. O P

1 1  n2  1 1 I
− =  −1 − 
f ∞  n1  R1 R2 
Figure 6.35  Lateral magnification in thin
1  n2  1 1 lens
=  −1 − (6.67)
f  n1  R1 R2 

Unit 6  OPTICS 33

UNIT-6(XII-Physics_Vol-2).indd 33 7/31/2019 8:01:16 PM


The lateral or transverse magnification Solution
m is defined as the ratio of the height of the For a biconvex lens, radius of curvature of
image to that of the object. the first surface is positive and that of the
II ′ second surface is negative side as shown in
m= (6.70)
OO ′ the figure.
From the two similar triangles ∆ POO′ and
∆ PII ′ , we can write,
1 2
II ′ PI
= (6.71)
OO ′ PO
R2 = cm
Applying sign convention,
C2 R1 = 20cm C1
−h2 v
=
h1 −u
Substituting this in the equation (6.70)
for magnification,

−h2 v
m= = Given, n = 1.5, R1=20 cm and R2= –15 cm
h1 −u
1 1 1
After rearranging, Lens maker’s formula, = (n −1) − 
f  R1 R2 
h2 v
m= = (6.72) Substituting the values,
h1 u
1 1 1 
= (1.5 −1) −
The magnification is negative for real f  20 −15 
image and positive for virtual image. In the
case of a concave lens, the magnification is 1 1 1  3 + 4   1 7  7
= (0.5) +  = (0.5) 
 =  ×  =
f  20 15   60   2 60  120
always positive and less than one.
We can also have the equations for 120
f= = 17.14 cm
magnification by combining the lens equation 7
with the formula for magnification as, As the focal length is positive the lens is a
converging lens.
h2 f h f −v
m= = (or) m = 2 = (6.73)
h1 f +u h1 f If the lens is flipped back to front,

EXA MP LE 6 .1 4
1 2

A biconvex lens has radii of curvature 20


cm and 15 cm each. The refractive index R2 = cm
of the material of the lens is 1.5. What is its C2 C1
focal length? Will the focal length change if R1 = 15cm
the lens is flipped by the side?

34 Unit 6  OPTICS

UNIT-6(XII-Physics_Vol-2).indd 34 7/31/2019 8:02:34 PM


Now, R1= 15 cm and R2= –20 cm, n = 1.5 1  2 −1  1  0.52
= (0.52)  = (0 . 52)  =
Substituting the values, f  20   20  20
1 1 1  20
= (1.5 −1) − f= = 38.46 cm
f 15 −20  0.52
1 1 1 As the focal length is positive, the lens is a
= (1.5 −1) + 
f 15 20  converging lens.
This will also result in, f = 17.14 cm
Thus, it is concluded that the focal length
6.6.5  Power of a lens
of the lens will not change if it is flipped
side wise. This is true for any lens. Students Power of lens is the measurement of
can verify this for any lens. deviating strength of a lens i.e. when a ray
is incident on a lens then the degree with
which the lens deviates the ray is determined
EXA MP LE 6 .1 5 by the power of the lens. Power of the lens
is inversely proportional to focal length i.e.
Determine the focal length of the lens greater the power of lens, greater will be
made up of a material of refractive index the deviation of ray and smaller will be the
1.52 as shown in the diagram. (Points C1 focal length. In Figure 6.36, the lens (b) has
and C2 are the centers of curvature of the greater deviating strength than that of (a).
first and second surface.) As (b) has greater deviating strength, its
focal length is less and vice versa.

1
C1 C2
F

F1
10 cm
(a)
20 cm

Solution
2
This lens is called convexo-concave lens
F
Given, n = 1.52, R1= 10 cm and R2= 20 cm
1 F2
1 1
Lens makers formula, = (n −1) − 
f  R1 R2  (b)
Substituting the values,
Figure 6.36  Power of lens
1 1 1
= (1.52 −1) − 
f 10 20 
In other words, the power of a lens is a
measure of the degree of convergence or
Unit 6  OPTICS 35

UNIT-6(XII-Physics_Vol-2).indd 35 7/31/2019 8:03:09 PM


divergence of light falling on it. The power contact with each other so that they have a
of a lens P is defined as the reciprocal of its common principal axis. For an object placed
focal length. at O beyond the focus of the first lens   on
the principal axis, an image is formed by it
1
P= (6.74) at I'. This image I' acts as an object for the
f
second lens   and the final image is formed
The unit of power is diopter D. 1 D = 1 m–1. at I as shown in Figure. 6.37. As these two
Power is positive for converging lens and lenses are thin, the measurements are done
negative for diverging lens. with respect to the common optical centre P
From the lens makers formula, equation in the middle of the two lenses.
(6.68), the equation for power can be written
as, 1 2

1 1 1
P= = (n −1) − (6.75)
f  R1 R2 
The outcome of this equation of power O P I I
is that larger the value of refractive index,
greater is the power of lens and vice versa. v
Also for lenses with small radius of curvature
(bulky) the power is large and for lenses u v
with large the radius of curvature (skinny), Figure. 6.37  Lenses in contact
the power is small.
Let, PO be object distance u and PI' be
EXA MP LE 6 .1 6 the image distance (v′) for the first lens 
and object distance for the second lens 
If the focal length is 150 cm for a glass lens,
and PI = v be the image distance for the
what is the power of the lens?
second lens  .
Solution Writing the lens equation for first lens ,
Given, focal length, f = 150 cm (or) f = 1.5 m 1 1 1
− = (6.76)
1 v ′ u f1
Equation for power of lens is, P =
f Writing the lens equation for second
Substituting the values, lens ,
1 1 1 1
P= = 0.67diopter − = (6.77)
1. 5 v v ′ f2
As the power is positive, it is a converging Adding the above two equations (6.76)
lens. and (6.77),
1 1 1 1
− = + (6.78)
6.6.6  Focal length of lenses v u f1 f 2
in contact If the combination acts as a single lens
Let us consider two lenses  and   of of focal length f so that for an object at the
focal length f1 and f2 are placed coaxially in position O it forms the image at I then,

36 Unit 6  OPTICS

UNIT-6(XII-Physics_Vol-2).indd 36 7/31/2019 8:03:58 PM


1 1 1 Equation for focal length of lenses in
− = (6.79) 1 1 1
v u F contact, = +
F f1 f 2
Comparing equations (6.78) and (6.79) Substituting the values,
we can write,
1 1 1 1 1
= + =− +
1 1 1 F −70 150 70 150
= + (6.80)
F f1 f 2
1 −150 + 70 −80 80
The above equation can be extended for = = =−
F 70×150 70×150 10500
any number of lenses in contact as,
−1050
1 1 1 1 1 F= = −131.25 cm
= + + + +......... (6.81) 8
F f1 f 2 f 3 f 4 As the focal length is negative, the
The above equation can be written as combination of two lenses is a diverging
power of the lenses as, system of lenses.
P = P1 + P2 + P3 + P4 + ......... (6.82) The power of combination is,
1 1
Where, P is the net power of the lens P= = = −0.76 diopter
F −1.3125 m
combination of lenses in contact. One
should note that the sum in equation
(6.82) is an algebraic sum. The powers 6.6.7  Focal length of lenses
of individual lenses may be positive (for in out of contact
convex lenses) or negative (for concave When two thin lenses are separated by
lenses). Combination of lenses helps to a distance d common optical center cannot
obtain diverging or converging lenses of be chosen for them. Hence, they cannot be
desired magnification. Also, combination treated as a single thin lens. Actually, such
of lenses enhances the sharpness of the a combination should be treated as a thick
images. As the image formed by the first lens for which the theory is more involved
lens becomes the object for the second (beyond the scope of present study).
and so on, the total magnification m of the However, as a special case, only when the
combination is a product of magnification object is placed at infinity, the combination
of individual lenses. We can write, can be replaced by a single thin lens. The
m = m1 × m2 × m3 . . . . (without proof). focal length and position of the equivalent
lens can be derived by considering the
EXA MP LE 6 .1 7 concept of angle of deviation.
Let O be a point object on the principal
What is the focal length of the combination axis of a lens as shown in Figure 6.38. OA is
if a lens of focal length –70 cm is brought in the incident ray on the lens at a point A at a
contact with a lens of focal length 150 cm? height h above the optical centre. The ray is
What is the power of the combination? deviated through an angle δ and forms the
image at I on the principal axis.
Solution
The incident and refracted rays subtend
Given, focal length of first lens, f1 = –70 cm, the angles, ∠AOP = α and ∠AIP = β with
focal length of second lens, f2 = 150 cm. the principal axis respectively.
Unit 6  OPTICS 37

UNIT-6(XII-Physics_Vol-2).indd 37 7/31/2019 8:05:00 PM


The above equation tells that the angle
A
of deviation is the ratio of height to the
b focal length. Now, the case of two lenses of
h
  focal length f1 and f2 arranged coaxially but
O P I separated by a distance d can be considered
as shown in Figure 6.39.
For a parallel ray that falls on the
Figure 6.38  Angle of deviation in lens arrangement, the two lenses produce
deviations δ1 and δ2 respectively and The net
deviation δ is.
In the triangle ∆OAI, the angle of
deviation δ can be written as, δ = δ1 + δ2 (6.88)
δ = α + β (6.83)
From Equation (6.87),
If the height h is small as compared to
PO and PI, the angles α, β and δ are also h1 h h
d1 = ; d 2 = 2 and d = 1 (6.89)
small. Then, f1 f2 f

PA PA
a ≈ tan a = ; and b ≈ tan b = (6.84) The equation (6.88) becomes,
PO PI
h1 h1 h2
PA PA = + (6.90)
 Then, d = + (6.85) f f1 f 2
PO PI
Here, PA = h, PO = –u and PI = v From the geometry,
h2 – h1 = P2G – P2C = CG
h h  1 1
d= + = h  +  (6.86)
−u v 
 −u v  h2 – h1 = BG tan d1 » BG d1

After rearranging h1
h2 − h1 = d
f1
1 1  h
d = h  −  =
 v u  f
h1
h2 = h1 + d (6.91)
h f1
d= (6.87)
f
Substituting the above equation in
Equation (6.90)
d
B G
h1 h1 h1 h1d
A  
= + +
h1

C f f1 f 2 f1 f 2
h2 

P
On further simplification,
P1 P D

1 1 1 d
= + + (6.92)
f f1 f 2 f1 f 2

Figure 6.39  Lens in out of contact The above equation could be used to
find the equivalent focal length. To find
38 Unit 6  OPTICS

UNIT-6(XII-Physics_Vol-2).indd 38 7/31/2019 8:06:25 PM


the position of the equivalent lens, we can The above equation (6.93) is the position
further write from the geometry, of the equivalent single lens from the second
lens. Its position from the first lens is,
GC
PP2 = EG =  f
tand PP1 = d − d 
 f1 
h1 − h2 h1 − h2
PP2 = EG = =  f
tand d PP1 = d 1 −  (6.94)
From equations (6.89) and (6.91)
 f1 

h1 h1
h2 − h1 = d and δ=
f1 f The Equation (6.92), (6.93)
Note
and (6.94) hold good only
 h f
PP2 = d 1 ×  for the special case of parallel
 f1   h1  incident rays or object at infinity. We
cannot use these equations if the object
 f
PP2 = d  (6.93) is at a finite distance. For finite distance
 f1  of the object, the image positions must
be calculated separately using the lens
Points to Ponder equation for the two lenses.

EX AM P L E 6 . 1 8
15 cm
1 2

5 mm

40 cm

An object of 5 mm height is placed at a


distance of 15 cm from a convex lens of
focal length 10 cm. A second lens of focal
length 5 cm is placed 40 cm from the first
System of combination of lenses is lens and 55 cm from the object. Find (a) the
commonly used in designing lenses position of the final image, (b) its nature
for cameras, microscopes, telescopes and (c) its size.
and other optical instruments. They
Solution
produce better magnification as well as
Given, h1 = 5 mm = 0.5 cm, u1 = –15 cm,
sharpness of the images.
f1 = 10 cm, f2 = 5 cm, d = 40 cm
Unit 6 OPTICS 39

UNIT-6(XII-Physics_Vol-2).indd 39 7/31/2019 8:07:18 PM


For the first lens, the lens equation is,
h1′ is the image height of the first lens,
1 1 1 h1′ = h2′
− =
v1 u1 f1 h′ v
Equation for magnification is, m = 2 = 2
Substituting the values, h1′ u2
h2′ 10
1 1 1 1 1 1 Substituting the values, =
− = ; + = −1 −10
v1 −15 10 v1 15 10
 10 
1 1 1 15 −10 5 1 h2′ = (−1)× = 1 cm = 10 mm
= − = = =  −10 
v1 10 15 150 150 30
As the height of the image is positive, the
v1= 30 cm image is erect, and it is real.
First lens forms image 30 cm to the right of
first lens.
6.7
Let us find the height of this image. PRISM
h2 v
Equation for magnification is, m = =
h1 u A prism is a triangular block of glass
h 30 or plastic. It is bounded by the three plane
Substituting the values, 2 =
0.5 −15 faces not parallel to each other. Its one face is
30 grounded which is called base of the prism.
h2 = 0.5× = −1 cm
−15 The other two faces are polished which are
As the height of the lens is negative, the called refracting faces of the prism. The
image is inverted, real image. angle between the two refracting faces is
Object is at 10 cm to the left of the second called angle of prism (or) refracting angle
lens (40–30=10 cm). Hence, u2 = –10 cm (or) apex angle of the prism represented as
A as shown in Figure 6.40.
For the second, the lens equation is,
1 1 1
− =
v2 u2 f2
Substituting the values,
1 1 1 1 1 1 A
− = ; + =
v2 −10 5 v2 10 5
1 1 1 10 − 5 5 1
= − = = =
v2 5 10 50 50 10

v2 = 10 cm
Figure 6.40  Prism
The image is formed 10 cm to the right of
the second lens.
6.7.1  Angle of deviation
Let us find the height of the final image.
produced by prism
Assume, the final height of the image
formed by the second lens is h2′ and the Let light ray PQ is incident on one of
height of the object for the second lens the refracting faces of the prism as shown
40 Unit 6  OPTICS

UNIT-6(XII-Physics_Vol-2).indd 40 7/31/2019 8:08:52 PM


in Figure 6.41. The angles of incidence and In the quadrilateral AQNR, two of the
refraction at the first face AB are i1 and r1. angles (at the vertices Q and R) are right
The path of the light inside the prism is QR. angles. Therefore, the sum of the other
The angle of incidence and refraction at the angles of the quadrilateral is 180o.
second face AC is r2 and i2 respectively. RS
∠A + ∠QNR = 180º (6.99)
is the ray emerging from the second face.
Angle i2 is also called angle of emergence. From the triangle ∆QNR,
The angle between the direction of the
r1+ r2+ ∠QNR = 180o(6.100)
incident ray PQ and the emergent ray RS
is called the angle of deviation d. The two Comparing these two equations (6.99)
normals drawn at the point of incidence and (6.100) we get,
Q and emergence R are QN and RN. They
meet at point N. The incident ray and the r1 + r2 = A (6.101)
emergent ray meet at a point M.
Substituting this in equation (6.98) for
angle of deviation,
A
d = i1 + i2 − A (6.102)
M
d Thus, the angle of deviation depends on
i1 i2
Q
r1 r2
R
the angle of incidence angle of emergence and
N the angle for the prism. For a given angle of
P S incidence the angle of emergence is decided
by the refractive index of the material of the
B C prism. Hence, the angle of deviation depends
on these following factors.
Figure 6.41  Refraction through prism
(i) the angle of incidence
(ii) the angle of the prism
The deviation d1 at the surface AB is, (iii) the material of the prism
(iv) the wave length of the light
angle ∠RQM = d1 = i1 − r1 (6.95)

The deviation d2 at the surface AC is, EX AM P L E 6 . 1 9


A monochromatic light is incident on
angle ∠QRM = d2 = i2 − r2 (6.96)
an equilateral prism at an angle 30o and
Total angle of deviation d produced is, emerges at an angle of 75o. What is the
angle of deviation produced by the prism?
d = d1 + d2 (6.97) Solution

Substituting for d1 and d2, Given, as the prism is equilateral,


A = 60°; i1 = 30°; i2 = 75°
d = (i1 − r1 ) + (i2 − r2 )
Equation for angle of deviation, d = i1 + i2 – A
After rearranging, Substituting the values, d = 30°+75°–60°=45°
d = (i1 − r1 ) + (i2 − r2 ) (6.98) The angle of deviation produced is, d = 45°

Unit 6  OPTICS 41

UNIT-6(XII-Physics_Vol-2).indd 41 7/31/2019 8:09:55 PM


EXA MP LE 6 .2 0 6.7.2  Angle of minimum
Light ray falls at normal incidence on the deviation
first face of an equilateral prism and emerges A graph plotted between the angle of
gracing the second face. What is the angle incidence and angle of deviation is shown
of deviation? What is the refractive index in Figure 6.42. One could observe that the
of the material of the prism? angle of deviation decreases with increase in
angle of incidence and reaches a minimum
Solution value and then continues to increase.

60°

30°

ic
d

The given situation is shown in the figure.


Figure 6.42  Graph between i and d
Here, A = 60°; i1 = 0°; i2 = 90°

Equation for angle of deviation,


The minimum value of angle of deviation
d = i1 + i2 − A is called angle of minimum deviation D. At
minimum deviation,
Substituting the values,
(a) the angle of incidence is equal to the
d = 0° + 90°− 60° = 30° angle of emergence, i1=i2.
(b) the angle of refraction at the face one
The angle of deviation produced is, d = 30°
and face two are equal, r1=r2).
The light inside the prism must be falling
on the second face at critical angle as it
graces the boundary.
1
Equation for critical angle is, sinic =
n
i r r i
1 1 1
n= ; n= = =2
sin ic sin 30 
1/ 2

The refractive index of the material of the


prism is, n = 2
Figure 6.43  Angle of minimum
deviation

42 Unit 6  OPTICS

UNIT-6(XII-Physics_Vol-2).indd 42 7/31/2019 8:10:50 PM


(c) the incident ray and emergent ray are  A + D 
sin 
symmetrical with respect to the prism.  2 
(d) the refracted ray inside the prism is n=
 A
parallel to its base of the prism. sin  
 2 
The case of angle of minimum deviation Substituting the values,
is shown in Figure 6.43.  600 + 370 
sin  
 2  sin (48.50 ) 0.75
6.7.3  Refractive index of the n= = = = 1. 5
 600 
 sin (300 ) 0. 5
material of the prism sin  
 2 
At minimum deviation, i1=i2=i and The refractive index of the material of the
r1=r2=r prism is, n = 1.5
Now, the equation (6.102) becomes,
( A + D) 6.7.4  Dispersion of white
D = i1 + i2 − A = 2i − A (or) i=
2
light through prism
The equation (6.101) becomes,
So far the angle of deviation produced
A by a prism is discussed for monochromatic
r1 + r2 = A ⇒ 2r = A (or) r =
2 light (i.e. light of single colour). When
Substituting i and r in Snell’s law, white light enter in to a prism, the effect
called dispersion takes place. Dispersion is
sin i
n= splitting of white light into its constituent
sin r
colours. This band of colours of light is
 A + D  called its spectrum. When a narrow beam
sin 
 2  of parallel rays of white light is incident
n= (6.103) on the face of a prism and the refracted
 A 
sin   beam is received on a white screen, a
 2 
band of colours is obtained in the order,
The above equation is used to find recollected by the word: VIBGYOR i.e.,
the refractive index of the material of the Violet, Indigo, Blue, Green, Yellow, Orange
prism. The angles A and D can be measured and Red. Violet is the most deviated and
experimentally. red is the least deviated colour as shown in
Figure 6.44.
EXA MP LE 6 .2 1

The angle of minimum deviation for a


prism is 37o. If the angle of prism is 60o,
find the refractive index of the material of
the prism.
Solution
Given, A=60°; D=37° Figure 6.44  dispersion of white light in
Equation for refractive index is, to its constituent colours

Unit 6  OPTICS 43

UNIT-6(XII-Physics_Vol-2).indd 43 7/31/2019 8:11:35 PM


The colours obtained in a spectrum Table 6.4  Refractive indices for
depend on the nature of the source of the different wavelengths
light used. Each colour of light is associated Colour Wavelength Crown Flint
with a definite wavelength. Red light is at (nm) glass glass
the longer wavelength end (700 nm) while Violet 396.9 1.533 1.663
the violet light is at the shorter wavelength Blue 486.1 1.523 1.639
end (400 nm). Therefore the violet ray Yellow 589.3 1.517 1.627
travels with a smaller velocity in glass
Red 656.3 1.515 1.622
prism than red ray.

Points to Ponder
6.7.5  Dispersive Power
Sir Isaac Newton has demonstrated Consider a beam of white light passes
through a classic experiment to through a prism; it gets dispersed into its
produce white light when all the constituent colours as shown in Figure
colours of VIBGYOR are recombined. 6.45. Let δV, δR are the angles of deviation
He used a prism to produce dispersion for violet and red light. Let nV and nR are the
and made all the colours to incident on refractive indices for the violet and red light
another inverted prism to combine all respectively.
the colours to get white light as shown
in figure.

Figure 6.45  Angle of deviation for


different colours
Dispersion takes place because light of
The refractive index of the material of a
different wave lengths travel with different
prism is given by the equation (6.103),
speeds inside the prism. In other words, the
refractive index of the material of the prism  A + D 
sin 
is different for different colours. For violet,  2 
n=
the refractive index is high and for red the  A
sin  
refractive index is the low. In Vacuum, all  2 
the colours travel with the same speed. The Here A is the angle of the prism and D
refractive index of two different glasses for is the angle of minimum deviation. If the
different colours is shown in Table 6.4. angle of prism is small of the order of 10o,
The speed of light is independent of the prism is said to be a small angle prism.
wavelength in vacuum. Therefore, vacuum When rays of light pass through such prisms,
is a non-dispersive medium in which all the angle of deviation also becomes small. If
colours travel with the same speed. A be the angle of a small angle prism and

44 Unit 6  OPTICS

UNIT-6(XII-Physics_Vol-2).indd 44 7/31/2019 8:11:43 PM


δ the angle of deviation then the prism is called the angular dispersion. Clearly,
formula becomes. the angular dispersion produced by a prism
depends upon.
 A + δ 
sin  (i) Angle of the prism
 2 
n=  (6.104) (ii) Nature of the material of the prism.
 A 
sin   If we take δ is the angle of deviation for
 2 
any middly ray (green or yellow) and n the
For small angles of A and δm, corresponding refractive index. Then,

 A + d   A + d  δ = (n – 1)A(6.111)
sin  ≈  (6.105)
 2   2 
Dispersive power (ω) is the ability of the
material of the prism to cause dispersion.
 A  A
sin   ≈  (6.106) It is defined as the ratio of the angular
 2   2 
dispersion for the extreme colours to the
deviation for any mean colour.
 A + d 
 Dispersive power (ω),
 2  A + d d
∴n= = = 1+
 A 
  A A angular dispersion d V − d R

 2  w= = (6.112)
mean deviation d
d Substituting for (d V − d R )and (δ),
Further simplifying, = n −1
A (nV − nR )
w= (6.113)
d = (n −1) A(6.107) (n −1)

When white light enters the prism, the Dispersive power is a dimensionless
deviation is different for different colours. quality. It has no unit. Dispersive power
Thus, the refractive index is also different is always positive. The dispersive power
for different colours. of a prism depends only on the nature of
material of the prism and it is independent
  For Violet light, d V = (nV −1) A(6.108) of the angle of the prism.

  For Red light, d R = (nR −1) A(6.109) EX AM P L E 6 . 2 2

As, angle of deviation for violet colour Find the dispersive power of flint glass if
δV is greater than angle of deviation for red the refractive indices of flint glass for red,
colour δR, the refractive index for violet green and violet light are 1.613, 1.620 and
colour nV is greater than the refractive index 1.632 respectively.
for red colour nR.
Subtracting δV from δR we get, Solution
Given, nV=1.632; nR=1.613; nG=1.620
δV– δR = (nV–nR)A(6.110)
Equation for dispersive power is,
The term (δV–δR) is the angular
(nV − nR )
separation between the two extreme w=
(nG −1)
colours (violet and red) in the spectrum
Unit 6  OPTICS 45

UNIT-6(XII-Physics_Vol-2).indd 45 7/31/2019 8:13:08 PM


Substituting the values, scattering is called Rayleigh’s scattering.
The intensity of Rayleigh’s scattering is
1.632 −1.613 0.019
w= = = 0.0306 inversely proportional to fourth power of
1.620 −1 0.620
wavelength.
The dispersive power of flint glass is,
w = 0.0306 1
Iµ (6.114)
l4
According to equation 6.114, violet colour
which has the shortest wavelength gets
6.7.6  Scattering of sunlight
much scattered during day time. The next
When sunlight enters the scattered colour is blue. As our eyes are more
atmosphere of the earth, the sensitive to blue colour than violet colour the
atmospheric particles present sky appears blue during day time as shown
in the atmosphere change the in Figure 6.46(a). But, during sunrise and
direction of the light. This process sunset, the light from sun travels a greater
is known as scattering of light. distance through the atmosphere. Hence,
If the scattering of light is by atoms and the blue light which has shorter wavelength
molecules which have size a very less than is scattered away and the less-scattered red
that of the wave length λ of light a<<λ, the light of longer wavelength manages to reach

Rainbow is an example of dispersion of sunlight through droplets of water


during rainy days. Rainbow is observed during a rainfall or after the rainfall
or when we look at a water fountain provided the sun is at the back of the
observer. When sunlight falls on the water drop suspended in air, it splits (or
dispersed) into its constituent seven colours. Thus, water drop suspended in air behaves
as a glass prism. Primary rain bow is formed when light entering the drop undergoes one
total internal reflection inside the drop before coming out from the drop as shown in
figure. The angle of view for violet to red in primary rainbow is 40o to 42o. A secondary
rainbow appears outside of a primary rainbow and develops when light entering a raindrop
undergoes two internal reflections. The angle of view for red to violet in a secondary
rainbow is, 52o to 54o.
White Primary rainbow
Light

Whit
e Lig
ht

42
40

Secondary rainbow
Whit
e Lig
ht

Whit
e Lig
ht

54
52

46 Unit 6  OPTICS

UNIT-6(XII-Physics_Vol-2).indd 46 7/31/2019 8:13:19 PM


our eye. This is the reason for the reddish of wavelength. This is the reason for the
appearance of sky during sunrise and sunset whitish appearance of cloud as shown in
as shown in Figure 6.46(b). Figure 6.46(c). But, the rain clouds appear
a) dark because of the condensation of water
droplets on dust particles that makes the
Sunrise
cloud become opaque.
Sun
If earth has no atmosphere there would
not have been any scattering and the sky
would appear dark. That is why sky appears
Earth dark for the astronauts who could see the
sky from above the atmosphere.

b)
6.8
THEORIES ON LIGHT

Day Sun
S Light is a form of energy that is
transferred from one place to another. A
Earth
glance at the evolution of various theories of
light put forward by scientists will give not
only an over view of the nature of light but
also its propagation and some phenomenon
demonstrated by it.

6.8.1  Corpuscular theory


Sir Isaac Newton (1672) gave the
corpuscular theory of light which was also
suggested earlier by Descartes (1637) to
explain the laws of reflection and refraction.
According this theory, light is emitted as
tiny, massless (negligibly small mass) and
Figure 6.46.  Scattering of different types perfectly elastic particles called corpuscles.
As the corpuscles are very small, the source
If light is scattered by large particles of light does not suffer appreciable loss of
like dust and water droplets present in mass even if it emits light for a long time. On
the atmosphere which have size a greater account of high speed, they are unaffected
than the wavelength λ of light, a >> λ, the by the force of gravity and their path is
intensity of scattering is equal for all the a straight line in a medium of uniform
wavelengths. It is happening in clouds refractive index. The energy of light is the
which contains large amount of dust and kinetic energy of these corpuscles. When
water droplets. Thus, in clouds all the these corpuscles impinge on the retina
colours get equally scattered irrespective of the eye, the vision is produced. The

Unit 6  OPTICS 47

UNIT-6(XII-Physics_Vol-2).indd 47 7/31/2019 8:13:20 PM


different size of the corpuscles is the reason Nevertheless, the interaction
for different colours of light. When the phenomenon of light with matter like
corpuscles approach a surface between two photoelectric effect, Compton effect could
media, they are either attracted or repelled. not be explained by this theory.
The reflection of light is due to the repulsion
of the corpuscles by the medium and
refraction of light is due to the attraction of 6.8.4  Quantum theory
the corpuscles by the medium.
Albert Einstein (1905), endorsing the
This theory could not explain the reason
views of Max Plank (1900), was able to
why the speed of light is lesser in denser
explain photoelectric effect (discussed in
medium than in rarer medium and also the
Unit 7) in which light interacts with matter
phenomena like interference, diffraction
as photons to eject the electrons. A photon is
and polarisation.
a discrete packet of energy. Each photon has
energy E of,
6.8.2  Wave theory
Christian Huygens (1678) proposed the E = hv(6.115)
wave theory to explain the propagation
of light through a medium. According to Where, h is Plank’s constant (h = 6.625 ×
him, light is a disturbance from a source 10 J s) and ν is frequency of electromagnetic
–34

that travels as longitudinal mechanical wave.


waves through the ether medium that was As light has both wave as well as particle
presumed to pervade all space as mechanical nature it is said to have dual nature. Thus,
wave requires medium for its propagation. it is concluded that light propagates as
The wave theory could successfully explain a wave and interacts with matter as a
phenomena of reflection, refraction, particle.
interference and diffraction of light.
Later, the existence of ether in all space was 6.9
proved to be wrong. Hence, this theory could
WAVE NATURE OF LIGHT
not explain the propagation of light through
vacuum. The phenomenon of polarisation
Light is a transverse, electromagnetic
could not be explained by this theory as it is
wave. The wave nature of light was first
the property of only transverse waves.
illustrated through experiments on
interference and diffraction. Like all
6.8.3  Electromagnetic wave electromagnetic waves, light can travel
theory through vacuum. The transverse nature of
Maxwell (1864) proved that light is an light is demonstrated in polarization.
electromagnetic wave which is transverse in
nature carrying electromagnetic energy. He
6.9.1  Wave optics
could also show that no medium is necessary
for the propagation of electromagnetic Wave optics deals with the wave
waves. All the phenomenon of light could characteristics of light. With the help of
be successfully explained by this theory. wave optics, we are going to learn in details

48 Unit 6  OPTICS

UNIT-6(XII-Physics_Vol-2).indd 48 7/31/2019 8:13:20 PM


the phenomena of interference, diffraction The shape of a wavefront observed at a
and polarization. Even the law of reflection point depends on the shape of the source
and refraction are proved only with the help and also the distance at which the source
of wave optics. Though light propagates as is located. A point source located at a finite
a wave, its direction of propagation is still distance gives spherical wavefronts. An
represented as a ray. extended (or) line source at finite distance
gives cylindrical wavefronts. The plane
wavefronts are received from any source
that is located at infinity as shown in
Figure 6.48.

(a)
O
B

Point source Line source Source at infinity

Figure 6.48  Wavefronts

(b)
6.9.2  Huygens’ Principle
Figure 6.47  (a) Ripples on water surface
(b) Wavefront and ray Huygens principle is a geometrical
construction which gives the shape of the
An example for wave propagation is the wavefront at any time if we know its shape
spreading of circular ripples on the surface at t = 0. According to Huygens principle,
of still water from a point at which a stone each point of the wavefront is the source
is dropped. The molecules or particles of secondary wavelets emanating from
of water are moving only up and down these points spreading out in all directions
(oscillate) when a ripple passes out that part. with the speed of the wave. These are
All these particles on the circular ripple are called as secondary wavelets. The common
in the same phase of vibration as they are tangent, in other words the envelope to all
all at the same distance from the center. The these wavelets gives the position and shape
ripple represents a wavefront as shown in of the new wavefront at a later time. Thus,
Figure 6.47(a). A wavefront is the locus of Huygens’ principle explains the propagation
points which are in the same state or phase of a wavefront.
of vibration. When a wave propagates it is The propagation of a spherical and plane
treated as the propagation of wavefront. The wavefront is explained in using Huygens’
wavefront is always perpendicular to the principle. Let, AB be the wavefront at a time,
direction of the propagation of the wave. t = 0. According to Huygens’ principle, every
As the direction of ray is in the direction of point on AB acts as a source of secondary
propagation of the wave, the wavefront is wavelet which travels with the speed of the
always perpendicular to the ray as shown in wave (speed of light c). To find the position
Figure 6.47(b). of the wavefront after a time t, circles of

Unit 6  OPTICS 49

UNIT-6(XII-Physics_Vol-2).indd 49 7/31/2019 8:13:20 PM


radius equal to ct are drawn with points P, the time point A of the incident wavefront
Q, R ... etc., as centers on AB. The tangent touches the reflecting surface, the point B
or forward envelope A′ B ′ of the small is yet to travel a distance BB′ to touch
circles is the new wavefront at that instant. the reflecting surface a B ′ . When the
The wavefront A′ B ′ will be a spherical point B falls on the reflecting surface at
wavefront from a point object which is at a B ′ , the point A would have reached A′ .
finite distance as shown in Figure 49(a) and This is applicable to all the points on the
it is a plane wavefront if the source of light wavefront. Thus, the reflected wavefront
is at a large distance (infinity) as shown in A′ B ′ emanates as a plane wavefront. The
Figure 6.49(b). two normals N and N ′ are considered
A' A A'
at the points where the rays L and M fall
A
on the reflecting surface. As reflection
P
P
happens in the same medium, the speed of
light is same before and after the reflection.
Q Q
Hence, the time taken for the ray to travel
from B to B ′ is the same as the time taken
R R
for the ray to travel from A to A′ . Thus, the
B B' B B' distance BB′ is equal to the distance AA′ ;
(a) Spherical wavefront (b) Plane wavefront
(AA′ = BB ′) .
Figure 6.49  Huygens’ Principle
L'
M

There is one shortcoming in the above N


B A'
N'
Huygens’ construction for propagation of a
wavefront. It could not explain the absence L M'

i r
of backwave which also arises in the above i
r
A B'
construction. According to electromagnetic X Y

wave theory, the backwave is ruled out Figure 6.50  Laws of reflection
inherently. However, Huygens’ construction
diagrammatically explains the propagation (i) The incident rays, the reflected rays and
of the wavefront. the normal are in the same plane.
(ii) Angle of incidence,
6.9.3  Proof for laws of ∠i = ∠NAL = 90o – ∠NAB = ∠ BAB′
reflection using Huygens’ Angle of reflection,
Principle ∠r = ∠ N ′ B ′ M ′ = 90 o–∠ N ′ B ′ A′ = ∠ A′ B ′A
Let us consider a parallel beam of light, For the two right angle triangles, ∆ ABB′
incident on a reflecting plane surface and ∆ B ′A′ A , the right angles, ∠B and
such as a plane mirror XY as shown in ∠ A′ are equal, (∠B and ∠ A′ = 90o); the two
Figure 6.50. The incident wavefront is AB sides, AA′ and BB′ are equal, ( AA′ = BB ′ ) ;
and the reflected wavefront is A′ B ′ in the side AB′ is the common. Thus, the two
the same medium. These wavefronts are triangles are congruent. As per the property
perpendicular to the incident rays L, M of congruency, the two angles, ∠ BAB′ and
and reflected rays L ′ , M ′ respectively. By ∠ A′ B ′A must also be equal.

50 Unit 6  OPTICS

UNIT-6(XII-Physics_Vol-2).indd 50 7/31/2019 8:13:26 PM


i = r(6.1) v1 and v2 before and after refraction and v1is
greater than v2 (v1>v2). But, the time taken t
Hence, the laws of reflection are proved. for the ray to travel from B to B ′ is the same
as the time taken for the ray to travel from
A to A′ .
6.9.4  Proof for laws of
refraction using Huygens’ BB ′ AA′ BB ′ v1
t= = (or) =
Principle v1 v2 AA′ v2

M (i) The incident rays, the refracted rays and


the normal are in the same plane.
B Medium(1)
(ii) Angle of incidence,
L
N i = ∠NAL = 90o – ∠NAB = ∠ BAB′
i Angle of refraction,
i
X A r
B'
Y
r = ∠ N ′ B ′ M ′ = 90 o – ∠ N ′ B ′ A′ = ∠ A′ B ′ A
r r

N'
For the two right angle triangles ∆ ABB′
M'

A'
and ∆ B ′A′ A ,
Medium(2)
L'
sin i BB ′ / AB ′ BB ′ v1 c / v2 n2
= = = = =
Figure 6.51  Law of refraction sin r AA′ / AB ′ AA′ v2 c / v1 n1

Here, c is speed of light in vacuum. The


Let us consider a parallel beam of light ratio c/v is the constant, called refractive
is incident on a refracting plane surface index of the medium. The refractive index
XY such as a glass surface as shown in of medium (1) is, c/v1 = n1 and that of
Figure 6.51. The incident wavefront AB medium (2) is, c/v2 = n2.
is in rarer medium  (1) and the refracted
wavefront A′ B ′ is in denser medium (2). sin i n2
= (6.18)
These wavefronts are perpendicular to the sin r n1
incident rays L, M and refracted rays L ′,M ′
respectively. By the time the point A of the In product form,
incident wavefront touches the refracting n1sin i = n2 sin r  (6.19)
surface, the point B is yet to travel a distance
BB′ to touch the refracting surface at B ′. Hence, the laws of refraction are proved.
When the point B falls on the refracting In the same way the laws of refraction
surface at B ′, the point A would have reached can also be proved for wavefront travelling
A′ in the other medium. This is applicable from denser to rarer medium.
to all the points on the wavefront. Thus, Light travels with greater speed in rarer
the refracted wavefront A′ B ′ emanates as a medium and lesser speed in denser medium.
plane wavefront. The two normals N and N ′ Hence, the wavelength of the light is longer
are considered at the points where the rays in rarer medium and shorter in denser
L and M fall on the refracting surface. As medium.
refraction happens from rarer medium (1) λ1 n2
to denser medium (2), the speed of light is = (6.116)
λ 2 n1

Unit 6  OPTICS 51

UNIT-6(XII-Physics_Vol-2).indd 51 7/31/2019 8:14:26 PM


If light of a particular n1
Rewriting, v2 = ×v1
Note n2
frequency travels through
different media, then, its Substituting the values,
frequency remains unchanged in 1
v2 = ×3×108 = 2.256×108
all the media. Only the wavelength 1.33
changes according to speed of light in v2 = 2.256×108 ms−1
that medium.
(c) Frequency of light in vacuum is,
c
v1 =
EXA MP LE 6 .2 3 l1

The wavelength of light from sodium Substituting the values,


source in vacuum is 5893Å.What are its 3×108
v1 = = 5.091×1014 Hz
(a) wavelength, (b) speed and (c) frequency 5893×10 −10

when this light travels in water which has a v


Frequency of light in water is, v2 =
refractive index of 1.33. l2
Substituting the values,
Solution
The refractive index of vacuum, n1 = 1 2.256×108 ms−1
v2 = −10
= 5.091×1014 Hz
4431×10
The wavelength in vacuum, λ1 = 5893 Å.
The results show that the frequency
The speed in vacuum, c = 3 × 108 m s–1 remains same in all media.
The refractive index of water, n2 = 1.33
The wavelength of light in water, λ2 6.10
The speed of light in water, v2 INTERFERENCE
(a) The equation relating the wavelength
The phenomenon of addition or
and refractive index is,
superposition of two light waves which
l1 n2 produces increase in intensity at some
=
l2 n1 points and decrease in intensity at some
n1 other points is called interference of light.
Rewriting, l2 = × l1
n2 Superposition of waves refers to addition
of waves. The concept of superposition of
Substituting the values,
mechanical waves is studied in (XI Physics
1 o o 11.7). When two waves simultaneously pass
l2 = ×5893 A = 4431 A
1.33 through a particle in a medium, the resultant
o displacement of that particle is the vector
l2 = 4431 A
addition of the displacements due to the
(b) The equation relating the speed and individual waves. The resultant displacement
refractive index is, will be maximum or minimum depending
v1 n2 upon the phase difference between the two
=
v2 n1 superimposing waves. These concepts hold
good for light as well.
52 Unit 6 OPTICS

UNIT-6(XII-Physics_Vol-2).indd 52 7/31/2019 8:15:37 PM


Let us consider two light waves from the The resultant amplitude is minimum,
two sources S1 and S2 meeting at a point P as
2
shown in Figure 6.52. Amin = (a1 − a2 ) ; when ϕ= ±π, ±3π, ±5π...,
(6.124)
P
S1P
The intensity of light is proportional to
square of amplitude,
S1
I ∝ A2(6.125)

Now, equation 6.121 becomes,


S2P
I ∝ I1 + I 2 + 2 I1 I 2 cosf (6.126)
S2
In equation 6.126 if the phase difference,
Figure 6.52  Superposition principle ϕ = 0, ±2π, ±4π. . . , it corresponds to the
condition for maximum intensity of light
The wave from S1 at an instant t at P is, called as constructive interference.
y1 = a1sin ωt(6.117) The resultant maximum intensity is,
2
The wave form S2 at an instant t at P is,   I max ∝ (a1 + a2 ) ∝ I1 + I 2 + 2 I1I 2 (6.127)

y2 = a2sin (ωt+ϕ)(6.118) In equation 6.126 if the phase difference,


ϕ = ±π, ±3π, ±5π. . . , it corresponds to the
The two waves have different amplitudes condition for minimum intensity of light
a1 and a2, same angular frequency ω, and called destructive interference.
a phase difference of ϕ between them. The The resultant minimum intensity is,
resultant displacement will be given by,
2
I min ∝ (a1 − a2 ) ∝ I1 + I 2 − 2 I1 I 2 (6.128)
y = y1 + y2 = a1sin ωt + a1sin2 (ωt+ϕ) (6.119)

The simplification of the above equation As a special case, if a1 = a2 = a, then


by using trigonometric identities as done in equation 6.121. becomes,
(XI Physics 11.7) gives the equation,
A = 2a 2 + 2a 2c os f = 2a 2 (1 + c os f )
y = A sin (ωt+θ)(6.120) = 2a 2 2 cos2 (f / 2)

Where, A = a12 + a22 + 2a1a2 cosf (6.121) A = 2a cos(f / 2) (6.129)

  I ∝ 4a cos (f / 2)  I ∝ A  (6.130)


2 2 2
a2 sin f
q = tan −1
(6.122)
a1 + a2 cos f
The resultant amplitude is maximum,   I = 4 I0 cos2 (f / 2)  I 0 ∝ a2  (6.131)
2
Amax = (a1 + a2 ) ; when ϕ = 0, ±2π, ±4π. . . , I max = 4 I 0 when, f = 0, ±2p , 4p ..., (6.132)
(6.123)

Unit 6  OPTICS 53

UNIT-6(XII-Physics_Vol-2).indd 53 7/31/2019 8:17:08 PM


I min = 0 when, f = ±p , ±3p , ±5p ..., (6.133) EX AM P L E 6 . 2 5

We conclude that the phase difference ϕ, Two light sources of equal amplitudes
between the two waves decides the intensity interfere with each other. Calculate the ratio
of light at that point where the two waves of maximum and minimum intensities.
meet.
Solution
Let the amplitude be a.
EXA MP LE 6 .2 4
The intensity is, I µ 4a2 cos2 (f / 2)
Two light sources with amplitudes 5 units
and 3 units respectively interfere with each or I = 4 I 0 cos2 (f / 2)
other. Calculate the ratio of maximum and
Resultant intensity is maximum when,
minimum intensities.
f = 0, cos 0 = 1, Imax ∝ 4a 2
Solution
Resultant amplitude is minimum when,
Amplitudes, a1 = 5, a2 = 3
f = p , cos (p / 2) = 0, Imin = 0
Resultant amplitude,
Imax : I min = 4a 2 : 0
A = a12 + a22 + 2a1a2 cosf

Resultant amplitude is maximum when, EX AM P L E 6 . 2 6


f = 0, cos 0 = 1, Amax = a12 + a22 + 2a1a2 Two light sources have intensity of light as
I0. What is the resultant intensity at a point
2 2 2
Amax = (a1 + a2 ) = (5 + 3) = (8) where the two light waves have a phase
= 8 units difference of π/3?

Resultant amplitude is minimum when, Solution

f = p , cos p = −1, Amax = a12 + a22 − 2a1a2


Let the intensities be I0.
The resultant intensity is, I =4 I0cos2(ϕ/2)
2 2 2
Amin = (a1 − a2 ) = (5 − 3) = (2) Resultant intensity when, f = p / 3, is
= 2 units I = 4 I 0 cos2 (p / 6)

I ∝ A2
( )
2
I = 4 I0 3 / 2 = 3I 0
2
I max ( Amax )
=
I min ( Amin )2
6.10.1  Phase difference and
Substituting, path difference
2
I max (8) 64 Phase is the angular position of a
= 2 = = 16 (or )
I min (2) 4 vibration. As a wave is progressing, there is
I max : I min = 16 : 1 a relation between the phase of the vibration
and the path travelled by the wave. One

54 Unit 6  OPTICS

UNIT-6(XII-Physics_Vol-2).indd 54 7/31/2019 8:18:52 PM


can express the phase in terms of path and Relation between phase difference and
vice versa. In the path of the wave, one 2p
path difference is, f = ×d
wavelength λ corresponds to a phase of 2π l
as shown in Figure 6.53. A path difference δ Substituting,
corresponds to a phase difference ϕ as given 2p p
f= ×3×10−3 = ×106
by the equation, 450×10−9
75
p
l 2p f = ×106 rad
d= ×f (or) f = ×d  (6.134) 75
2p l

λ 6.10.2  Coherent sources


Two light sources are said to be
coherent if they produce waves which
0 2π have same phase or constant phase
difference, same frequency or wavelength
(monochromatic), same waveform and
Figure 6.53  Path difference and phase
preferably same amplitude. Coherence is
difference
a property of waves that enables to obtain
stationary interference patterns.
For constructive interference, the phase
Two independent monochromatic
difference should be, ϕ = 0, 2π, 4π . . . Hence,
sources can never be coherent, because
the path difference must be, δ = 0, λ, 2λ . . .
they may emit waves of same frequency and
In general, the integral multiples of λ.
same amplitude, but not with same phase.
  δ = nλ where, n = 0, 1, 2, 3 . . . (6.135) This is because, atoms while emitting light,
produce change in phase due to thermal
For destructive interference, phase vibrations. Hence, these sources are said to
difference should be, ϕ = π, 3π, 5π . . . Hence, be incoherent sources.
l 3l To obtain coherent light waves, we have
the path difference must be, d = , .....
2 2 three techniques. They are,
In general, the half integral multiples of λ. (i) Intensity or amplitude division
(ii) wavefront division
l (iii) source and images.
d = (2n −1) where, n = 1, 2, 3 . . . ( 6.136)
2
(i) Intensity or amplitude division: If we
allow light to pass through a partially
EXA MP LE 6 .2 7 silvered mirror (beam splitter),
The wavelength of a light is 450 nm. How both reflection and refraction take
much phase it will differ for a path of place simultaneously. As the two
3 mm? light beams are obtained from the
same light source, the two divided
Solution light beams will be coherent beams.
The wavelength is, l = 450 nm = 450×10−9 m They will be either in-phase or at
constant phase difference as shown
Path difference is, d = 3 mm = 3×10−3 m in Figure  6.54. Instruments like

Unit 6  OPTICS 55

UNIT-6(XII-Physics_Vol-2).indd 55 7/31/2019 8:19:27 PM


Michelson’s interferometer, Fabray- coherent source, because the source and
Perrot etalon work on this principle. its image will have waves in-phase or
constant phase difference as shown in
Mirror Figure 6.56. The Instrument, Fresnel’s
biprism uses two virtual sources as two
Coherent Light coherent sources and the instrument,
Beams
Lloyd’s mirror uses a source and its
virtual image as two coherent sources.
Half- Silvered Superposition region
Monochromatic Mirror Virtual Image
Light Source

Source
Mirror

Virtual Image Biprism


Figure 6.54  Intensity or amplitude
division Fresnel’s Biprism Screen

(ii) Wavefront division: This is the most


Superposition region
commonly used method for producing
Source
two coherent sources. We know a point S
source produces spherical wavefronts.
All the points on the wavefront are at the
same phase. If two points are chosen on S'
the wavefront by using a double slit, the Virtual Image
of Source
two points will act as coherent sources
Lloy’s Mirror
as shown in Figure 6.55.
Figure 6.56  Using virtual or reals images
of a source as coherent sources
S1
6.10.3  Double slit as
S
coherent sources
Double slit uses the principle of wavefront
division. Two slits S1 and S2 illuminated by a
single monochromatic source S act as a set
S2
of coherent sources. The waves from these
two coherent sources travel in the same
medium and superpose. The constructive
Figure 6.55  Wavefront division and destructive are shown in Figure 6.57(a).
The crests of the waves are shown by thick
(iii) Source and images: In this method a continuous lines and troughs are shown by
source and its image will act as a set of broken lines in Figure 6.57(b).
56 Unit 6  OPTICS

UNIT-6(XII-Physics_Vol-2).indd 56 7/31/2019 8:19:27 PM


Constructive Interference Brightness
bright and dark bands which are referred as
interference fringes. The constructive and
destructive are shown in Figure 6.57(a).
+ =

6.10.4.  Young’s double slit


Destructive Interference experiment

Darkness
Thomas Young, a British Physicist used
+ = an opaque screen with two small openings
called double slit S1 and S2 kept equidistance
(a) Superposition
from a source S as shown in Figure 6.62. The
Double slit width of each slit is about 0.03 mm and they
are separated by a distance of about 0.3 mm.
As S1 and S2 are equidistant from S, the light
waves from S reach S1 and S2 in-phase. So,
S1 and S2 act as coherent sources which are
the requirement of obtaining interference
S1
S pattern.
S2
Experimental setup
Double slit Screen
P
S1
Superpositon region
S O
(b) Double slit as a set of coherent sources

Figure 6.57  Interference due to double S2


slit
Figure 6.58  Young’s double slit
At points where the crest of one wave experiment
meets the crest of the other wave or the trough
of one wave meets the trough of the other Wavefronts from S1 and S2 spread out
wave, the waves are in-phase. Hence, the and overlapping takes place to the right side
displacement is maximum and these points of double slit. When a screen is placed at
appear bright. This type of interference is a distance of about 1 meter from the slits,
said to be constructive interference. alternate bright and dark fringes which are
At points where the crest of one wave equally spaced appear on the screen. These
meets the trough of the other wave and vice are called interference fringes or bands.
versa, the waves are out-of-phase. Hence, Using an eyepiece the fringes can be seen
the displacement is minimum and these directly. At the center point O on the screen,
points appear dark. This type of interference waves from S1 and S2travel equal distances
is said to be destructive interference. and arrive in-phase as shown in Figure 6.58.
On a screen the intensity of light will be These two waves constructively interfere
alternatively maximum and minimum i.e. and bright fringe is observed at O. This is
Unit 6  OPTICS 57

UNIT-6(XII-Physics_Vol-2).indd 57 7/31/2019 8:19:28 PM


called central bright fringe. The fringes ∠OCP = ∠S2S1M = θ.
disappear and there is uniform illumination In right angle triangle ∆S1S2M, the path
on the screen when one of the slits is covered. difference, S2M = d sin θ
This shows clearly that the bands are due to
interference. δ = d sin θ(6.138)

Equation for path difference If the angle θ is small, sin θ ≈ tan θ ≈ θ


The schematic diagram of the From the right angle triangle ∆OCP,
experimental set up is shown in Figure 6.59. y
tanq =
The Let d be the distance between the double D
slits S1 and S2 which act as coherent sources
of wavelength λ. A screen is placed parallel dy
  The path difference, d = (6.139)
to the double slit at a distance D from it. The D
mid-point of S1 and S2 is C and the mid- Based on the condition on the path
point of the screen O is equidistant from S1 difference, the point P may have a bright or
and S2. P is any point at a distance y from O. dark fringe.
The waves from S1 and S2 meet at P either in-
phase or out-of-phase depending upon the Condition for bright fringe (or) maxima
path difference between the two waves. The condition for the constructive
interference or the point P to be have a
P
bright fringe is,
Path difference, δ = nλ
S1 Y where, n = 0, 1, 2, . . .
θ dy
d C θ O ∴ = nλ
D
M
S2 б
D lD lD
y =n (or ) yn = n (6.140)
d d

Figure 6.59  Young’s double slit This is the condition for the point P to be
experimental setup a bright fringe. The distance is the distance
of the nth bright fringe from the point O.
The path difference δ between the light Condition for dark fringe (or) minima
waves from S1 and S2 to the point P is, The condition for the destructive
δ = S2P – S1P interference or the point P to be have a dark
A perpendicular is dropped from the fringe is,
point S1 to the line S2P at M to find the path
l
difference more precisely. Path difference, d = (2n −1)
2
δ = S2P – MP = S2M(6.137) where, n = 1, 2, 3 . . .
dy l
∴ = (2n −1)
The angular position of the point P from D 2
C is θ. ∠OCP = θ.
(2n −1) l D (2n −1) l D
From the geometry, the angles ∠OCP y= (or ) yn = (6.141)
2 d 2 d
and ∠S2S1M are equal.
58 Unit 6  OPTICS

UNIT-6(XII-Physics_Vol-2).indd 58 7/31/2019 8:20:15 PM


This is the condition for the point P to be lD
a dark fringe. The distance yn is the distance b= (6.142)
d
of the nth dark fringe from the point O.
Bright
Similarly, the distance between (n+1)th
Dark
(constructive
interference)
(destructive and nth consecutive dark fringes from O is
interference)
given by,
 (2(n + 1) −1) l D   (2n −1) l D 
b = y(n+1) − yn =   −  
θ θ  2 d   2 d 

  lD
b= (6.142)
d
Figure 6.60  Formation of bright and
dark fringes Equations (6.142) show that the bright
and dark fringes are of same width equally
The formation of bright and dark fringes spaced on either side of central bright
is shown in Figure 6.60. fringe.
This shows that on the screen, alternate
bright and dark bands are seen on either Conditions for obtaining clear and broad
side of the central bright fringe. The central interference bands
bright is referred as 0th bright followed by (i) The screen should be as far away from
1st dark and 1st bright and then 2nd dark and the source as possible.
2nd bright and so on, on either side of O (ii) The wavelength of light used must be
successively as shown in Figure 6.61. larger.
(iii) The two coherent sources (here S1 and
S2) must be as close as possible.
I
I0

Position
EX AM P L E 6 . 2 8
Y
of fringe - 3D/d - 2D/d - D/d 0 D/d 2D/d 3D/d
Phase
difference -6 -4 -2 0 2 4 6
 In Young’s double slit experiment, the two
Path
difference - 3 - 2 - 0  2 3
 slits are 0.15 mm apart. The light source
has a wavelength of 450 nm. The screen is
Figure 6.61  Interference fringe pattern
2 m away from the slits.
(need to make it vertical)
(i) Find the distance of the second bright
Equation for bandwidth fringe and also third dark fringe from the
The bandwidth (β) is defined as the central maximum.
distance between any two consecutive (ii) Find the fringe width.
bright or dark fringes.
(iii) How will the fringe pattern change if
The distance between (n+1)th and nth
the screen is moved away from the slits?
consecutive bright fringes from O is given
by, (iv) What will happen to the fringe width
if the whole setup is immersed in water of
 l D   l D 
b = y(n+1) − yn = (n + 1)  − n  refractive index 4/3.
 d   d 

Unit 6  OPTICS 59

UNIT-6(XII-Physics_Vol-2).indd 59 7/31/2019 8:20:42 PM


Solution 6.10.5  Interference with
d = 0.15 mm = 0.15× 10 m; D = 2 m; -3
polychromatic light
λ = 450 nm = 450 × 10-9 m; n = 4/3
When a polychromatic light (white light)
(i) Equation for n bright fringe is,
th
is used in interference experiment, coloured
lD fringes of varied thickness will be formed on
yn = n
d the screen. This is because, different colours
Distance of 2nd bright fringe is, have different wavelengths. However, the
450×10−9 ×2 central fringe or 0th fringe will always be
y 2 = 2×
0.15×10−3 bright and white in colour, because for all
y2 = 12×10−3 m = 12mm the colours falling at the point O will have
no path difference. Hence, only constructive
Equation for nth dark fringe is, interference is possible at O for all the
(2n −1) l D colours.
yn =
2 d
Distance of 3rd dark fringe is, EX AM P L E 6 . 2 9
−9
5 450×10 ×2 Two lights of wavelengths 560 nm and
y3 = ×
2 0.15×10−3 420 nm are used in Young’s double slit
y2 = 15×10−3 m = 15 mm experiment. Find the least distance from
the central fringe where the bright fringe
lD
(ii) Equation for fringe width is, b = of the two wavelengths coincides. Given
d
D = 1 m and d = 3 mm.
450×10−9 ×2
Substituting, b =
0.15×10−3 Solution
b = 6×10−3 m = 6 mm λ1 = 560 nm = 560×10−9 m;

(iii) The fringe width will increase as D is λλ22 = 420 nm = 420×10−9 m;


lD D = 1 m; d = 3 mm = 3×10−3 m
increased, b = (or) b ∝D
d
(iv) The fringe width will decrease as the For a given y, n and λ are inversely
setup is immersed in water of refractive proportional.
index 4/3 Let nth order bright fringe of λ1 coincides
lD with (n+1)th order bright fringe of λ2.
b= (or) b ∝l
d lD
Equation for nth bright fringe is, yn = n
The wavelength will decrease refractive d
index n times. Hence, b ∝ l and b ′ ∝ l ′ l1D lD
Here, n = (n + 1) 2 (as l1 > l2 )
l d d
We know that, l ′ =
n l1 (n+ 1) 1 560×1
nl1 = (n + 1)l2 (or ) = (or ) 1 + =
b′ l′ l / n 1 b 6×10 −3
l2 n n 420×1
= = = (or) b′ = =
b l l n n 4/3
l1 (n+ 1) 1 560×10−9 1 4
nl = (n + 1)l (or ) = (or ) 1 + = (or ) 1 + =
b ′ = 41.5×10−3 m =2 4.5 mm l2 n n 420×10−9 n 3

60 Unit 6  OPTICS

UNIT-6(XII-Physics_Vol-2).indd 60 7/31/2019 8:22:45 PM


Dazzling colours are exhibited by thin films of oil spread on the surface of
water and also by soap bubbles as shown in the figure. These colours are due
to interference of white light undergoing multiple reflections from the top and
the bottom surfaces of thin films. The colour depends upon the thickness of the
film, refractive index of the film and also the angle of incidence of the light.

Inner Outer Surface


surface

All Wave Lengths

Colour
Depends
Air on
Interference

SOAP FLIM

1 1 the lower surface into two parts; one is


= (or ) n = 3 transmitted out of the film and the other
n 3
is reflected back in to the film. Reflected as
Thus, the 3rd bright fringe of λ1 and 4th
well as refracted waves are sent by the film
bright fringe of λ2 coincide at the least
as multiple reflections take place inside the
distance y.
film. The interference is produced by both
The least distance from the central fringe the reflected and transmitted light.
where the bright fringes of the two
lD
wavelengths coincides is, yn = n
d i
−9
560×10 ×1
y n = 3× −3
= 560×10−6 m A C E
3×10
d
yn = 0.560×10−3 m = 0.560 mm r
B D F

6.10.6  Interference in thin


films
Figure 6.62  Interference in thin films
Let us consider a thin film of transparent
material of refractive index µ (not to confuse
with order of fringe n) and thickness d. A For transmitted light
parallel beam of light is incident on the The light transmitted may interfere to
film at an angle i as shown in Figure 6.62. produce a resultant intensity. Let us consider
The wave is divided into two parts at the the path difference between the two light
upper surface, one is reflected and the waves transmitted from B and D. The two
other is refracted. The refracted part, which waves moved together and remained in
enters into the film, again gets divided at phase up to B where splitting occurred. The

Unit 6  OPTICS 61

UNIT-6(XII-Physics_Vol-2).indd 61 7/31/2019 8:32:58 PM


extra path travelled by the wave transmitted The additional path difference λ/2 is due
from D is the path inside the film, BC + CD. to the phase change of π in rarer to denser
If we approximate the incidence to be nearly reflection taking place at A.
normal (i = 0), then the points B and D are The condition for destructive interference
very close to each other. The extra distance for reflected ray is,
travelled by the wave is approximately twice
l l
thickness of the film, BC + CD = 2d. As 2 md + = (2n + 1) (or ) 2 md = nl (6.141)
2 2
this extra path is traversed in a medium of
refractive index µ, the optical path difference
If the incidence is not
is, δ = 2µd. Note
nearly normal, but at an
The condition for constructive
angle of incidence i which
interference in transmitted ray is,
has an angle of refraction r, then the
2µd = nλ (6.143) expression for path difference 2µd
on the left hand side of the above
Similarly, the condition for destructive
equations are to be replaced with the
interference in transmitted ray is,
expression, 2µd cos r.
l
2 md = (2n −1) (6.144)
2
EX AM P L E 6 . 3 0
For reflected light
Find the minimum thickness of a film of
It is experimentally and theoretically
refractive index 1.25, which will strongly
proved that a wave while travelling in a rarer
reflect the light of wavelength 589 nm. Also
medium and getting reflected by a denser
find the minimum thickness of the film to
medium, undergoes a phase change of π.
be anti-reflecting.
Hence, an additional path difference of λ/2
should be considered. Solution
Let us consider the path difference λ = 589 nm = 589×10−9 m
between the light waves reflected by the
For the film to have strong reflection,
upper surface at A and the other wave
the reflected waves should interfere
coming out at C after passing through the
constructively. The least optical path
film. The additional path travelled by wave
difference introduced by the film should
coming out from C is the path inside the
be λ/2. The optical path difference between
film, AB + BC. For nearly normal incidence
the waves reflected from the two surfaces of
this distance could be approximated as,
the film is 2µd. Thus, for strong reflection,
AB + BC = 2d. As this extra path is travelled
2µd = λ/2 [As given in equation 6.145. with
in the medium of refractive index µ, the
n = 1]
optical path difference is, δ = 2µd.
l
The condition for constructive Rewriting, d =
4m
interference for reflected ray is,
589×109
l Substituting, d = = 117.8×10−9
2µd + l = nl (or) 2 md = (2n −1) 4 ×1.25
2 2
2 d = 117.8×10−9 = 117.8 nm
(6.145)

62 Unit 6 OPTICS

UNIT-6(XII-Physics_Vol-2).indd 62 7/31/2019 8:33:49 PM


For the film to be anti-reflecting, the is a violation to the rectilinear propagation
reflected rays should interfere destructively. of light, we have studied in ray optics,
The least optical path difference introduced which says light should travel in straight
by the film should be λ. The optical path line in a medium without bending. But,
difference between the waves reflected the diffraction is prominent only when the
from the two surfaces of the film is 2µd. size of the obstacle is comparable to the
For strong reflection, 2µd = λ [As given in wavelength of light. This is the reason why
equation 6.146. with n = 1]. sound waves get diffracted prominently by
l obstacles like doors, windows, buildings
Rewriting, d = etc. The wavelength of sound wave is large
2m
589×109 and comparable to the geometry of these
Substituting, d = = 235.6×10−9 obstacles. But the diffraction in light is more
2×1.25
d = 235.6×10−9 = 235.6 nm
pronounced when the obstacle size is of the
order of wavelength of light.

6.11 6.11.1  Fresnel and


DIFFRACTION Fraunhofer diffractions
Based on the type of wavefront which
Diffraction is a general characteristic undergoes diffraction, the diffraction could
of all types of waves, be it sound wave, be classified as Fresnel and Fraunhofer
light wave, water wave etc. Diffraction is diffractions. The differences between
bending of waves around sharp edges into Fresnel and Fraunhofer diffractions are
the geometrically shadowed region. This shown in Table 6.4.

Table 6.4  Difference between Fresnel and Fraunhofer diffractions


S.No. Fresnel diffraction Fraunhofer diffraction
1 Spherical or cylindrical wavefront Plane wavefront undergoes
undergoes diffraction diffraction

2 Light wave is from a source at finite distance Light wave is from a source at infinity

3 For laboratory conditions, convex lenses In laboratory conditions, convex


need not be used lenses are to be used

4 difficult to observe and analyse Easy to observe and analyse

5 Slit Screen Slit Screen

P
P

S θ

Unit 6  OPTICS 63

UNIT-6(XII-Physics_Vol-2).indd 63 7/31/2019 8:34:07 PM


As Fraunhofer diffraction is easy to The basic idea is to divide the slit
observe and analyse, let us take it up for into much smaller even number of parts.
further discussions. Then, add their contributions at P with
the proper path difference to show that
destructive interference takes place at that
6.11.2.  Diffraction at single
point to make it minimum. To explain
slit maximum, the slit is divided into odd
Let a parallel beam of light fall normally number of parts.
on a single slit AB of width a as shown
in Figure 6.63. The diffracted beam falls Condition for P to be first minimum
on a screen kept at a distance. The center Let us divide the slit AB into two half ’s
of the slit is C. A straight line through C AC and CB. Now the width of AC is (a/2).
perpendicular to the plane of slit meets We have different points on the slit which
the center of the screen at O. We would are separated by the same width (here a/2)
like to find the intensity at any point P called corresponding points as shown in
on the screen. The lines joining P to the Figure 6.64.
different points on the slit can be treated as The path difference of light waves from
parallel lines, making an angle θ with the different corresponding points meeting at
normal CO. point P and interfere destructively to make
All the waves start parallel to each it first minimum. The path difference δ
other from different points of the slit and between waves from these corresponding
a
interfere at point P and other points to give points is, d = sinq
the resultant intensities. The point P is in 2
The condition for P to be first minimum,
the geometrically shadowed region, up to a l
which the central maximum is spread due sinq =
2 2
to diffraction as shown Figure 6.63. We need
to give the condition for the point P to be of a sin θ = λ (first minimum)(6.147)
various minima.

Figure 6.63  Diffraction at single slit (to redraw diffraction pattern)

64 Unit 6  OPTICS

UNIT-6(XII-Physics_Vol-2).indd 64 7/31/2019 8:34:17 PM


Condition for maxima
For points of maxima, the slit is to be
divided in to odd number of equal parts so
that one part remains un-cancelled making
the point P appear bright.
The condition for first maximum is,

a l 3l
sin q = (or ) a sin q = (6.151)
3 2 2

The condition for second maximum is,


Figure 6.64 Corresponding points
a l 5l
sin q = (or ) a sin q = (6.152)
5 2 2
Condition for P to be second minimum
Let us divide the slit AB into four equal The condition for third maximum is,
parts. Now, the width of each part is a/4. We a l 7l
have several corresponding points on the slit sin q = (or ) a sinq = (6.153)
7 2 2
which are separated by the same width a/4.
The path difference δ between waves from In the same way, condition for nth
a maximum is,
these corresponding points is, d = sinq .
4
l th
The condition for P to be second a sin q = (2n + 1) (n maximum) (6.154)
a l 2
minimum, sinq =
4 2 where, n = 0, 1, 2, 3, . . . , is the order of
a sin θ = 2λ (second minimum) (6.148) diffraction maximum.
The central maximum is called 0th order
Condition for P to be third order minimum maximum. The points of the maximum
intensity lie nearly midway between the
The same way the slit is divided in to six
successive minima.
equal parts to explain the condition for P to
a l
be third minimum is, sinq =
6 2 Here, sinθ gives the
Note
angular spread of the
a sin θ = 3λ (third minimum) (6.149)
diffraction. The position of
Condition for P to be nthorder minimum the minimum or maximum in terms
of y may be expressed by replacing
Dividing the slit into 2n number of
(even number of) equal parts makes the sinθ approximated by tanθ, as θ is
y
light produced by one of the corresponding small, sinq = tanq
D
points to be cancelled by its counterpart.
Where, y is the position of the
Thus, the condition for nth order minimum
minimum from the center of the screen
a l
is, sinq = and D is the distance between single
2n 2
slit and the screen.
a sin θ = nλ (nth minimum) (6.150)

Unit 6 OPTICS 65

UNIT-6(XII-Physics_Vol-2).indd 65 7/31/2019 8:35:13 PM


EXA MP LE 6 . 31 To find the value of y2 for second minimum
Light of wavelength 500 nm passes through with (n = 2) is, a sin θ = 2λ
a slit of 0.2 mm wide. The diffraction y2 2l D
a = 2l   rewriting, y2 =
pattern is formed on a screen 60 cm away. D a
Determine the, Substituting,
(i) angular spread of central maximum 2×500×10−9 ×60×10−2
y2 = = 3×10−3 = 3 mm
(ii) the distance between the central 0.2×10−3
maximum and the second minimum. The distance between the central maximum
and second minimum is, y2 – y1
Solution
y2 – y1 = 3 mm – 1.5 mm = 1.5 mm
λ = 500 nm = 500×10-9  m; a = 0.2 mm =
0.2×10-3 m; D = 60 cm = 60×10-2 m
(i) Equation for diffraction minimum is,
a sin θ = nλ
The central maximum is spread up to the
first minimum. Hence, n = 1

Note: The above calculation shows that in


the diffraction pattern caused by single
slit, the width of each maximum is equal
with central maximum as the double that
l l  of others. But the bright and dark fringes
Rewriting, sinq = (or ) q = sin−1  
a  a 
are not of equal width.
Substituting,
 500×10−9 
q = sin−1   = sin−1 (2.5×10−3 ) EX AM P L E 6 . 3 2
 0.2×10−3 
A monochromatic light of wavelength
q = 0.0025 rad
5000  Å passes through a single slit
(ii) To find the value of y1 for central producing diffraction pattern for the
maximum, which is spread up to first central maximum as shown in the figure.
minimum with (n = 1) is, a sin θ = λ Determine the width of the slit.
y1
As θ is very small, sinq ≈ tanq =
D
y1 lD
a = l   rewriting, y1 =
D a
Substituting,
500×10−9 ×60×10−2
y1 = −3
= 1.5×10−3 = 1.5 mm
0.2×10

66 Unit 6  OPTICS

UNIT-6(XII-Physics_Vol-2).indd 66 7/31/2019 8:36:16 PM


Solution θ = 300. These are practical cases
λ = 5000 Å = 5000×10-10 m; sin 30o  = 0.5; where diffraction could be observed
n = 1; a =? effectively.

Equation for diffraction minimum is,


asin θ = nλ 6.11.4  Fresnel’s distance
The central maximum is spread up to the Fresnel’s distance is the distance up
first minimum. Hence, n = 1 to which the ray optics is valid in terms
l of rectilinear propagation of light. As
Rewriting, a = there is bending of light in diffraction, the
sin q
rectilinear propagation of light is violated.
5000×10−10 But, this bending is not significant till the
Substituting, a =
0. 5 diffracted ray crosses the central maximum
a = 1×10−6 m = 0.001×10−3 m = 0.001mm at a distance z as shown in Figure 6.65.
Hence, Fresnel’s distance is the distance
upto which ray optics is obeyed and
beyond which ray optics is not obeyed but,
6.11.3  Discussion on first wave optics becomes significant.
minimum
Let us consider the condition for first
minimum with (n = 1). a sin θ = λ
The first minimum has an angular spread

Wave optics
l θ
a Ray optics
of, sinq = 2θ
a
Now, we have special cases to discuss on
Fresnel’s distance
the above condition.
(i) When a < λ, the diffraction is not
possible, because sinθ can never be
Figure 6.65  Fresnel’s distance
greater than 1.
(ii) When a ≥ λ, the diffraction is possible. From the diffraction equation for first
 For a = λ, sinθ = 1 i.e, θ = 900. That l l
minimum, sinq = ;q=
means the first minimum is at a a
90o. Hence, the central maximum From the definition of Fresnel’s distance,
spreads fully in to the geometrically a a
sin2q = ; 2q =
shadowed region leading to bending z z
of the diffracted light to 90o. Equating the above two equation gives,
 For a >> λ, sinθ << 1 i.e, the first l a
=
minimum will fall within the width a 2z
of the slit itself. The diffraction will After rearranging, we get Fresnel’s distance
not be noticed at all. z as,
(iii) When a > λ and also comparable,
l l 1 a2
say a = 2λ, sinq = = = ; then z= (6.149)
a 2l 2 2l

Unit 6  OPTICS 67

UNIT-6(XII-Physics_Vol-2).indd 67 7/31/2019 8:37:20 PM


EXA MP LE 6 .3 3 6.11.6  Diffraction in grating

Calculate the distance for which ray optics Grating has multiple slits with equal
is good approximation for an aperture of widths of size comparable to the wavelength
5 mm and wavelength 500 nm. of diffracting light. Grating is a plane sheet
of transparent material on which opaque
Solution rulings are made with a fine diamond
a = 5 mm = 5 × 10-3 m; pointer. The modern commercial grating
contains about 6000 lines per centimetre.
5×10−3 m; l = 500nm = 500×10−9 m; z = ? The rulings act as obstacles having a
a2 definite width b and the transparent space
Equation for Fresnel’s distance, z = between the rulings act as slit of width a.
2l
Substituting, The combined width of a ruling and a
2 slit is called grating element (e = a + b).
(5×10 ) −3
25×10−6 Points on successive slits separated by a
z= = = 25 m
2×500×10−9 1×10−6 distance equal to the grating element are
z = 25 m called corresponding points.

(a+b)sin 
A
a P
6.11.5  Difference between b

interference and diffraction  O

It is very difficult to distinguish


between interference and diffraction as B
they both exhibit the wave nature of light. Grating
Screen
In both the phenomena the light reaches
the geometrically shadowed regions Figure 6.66  Diffraction grating
and also interferes to produce maxima experiment
and minima. Nevertheless, based on the
A plane transmission grating is
appearance, the differences are given in
represented by AB in Figure 6.66. Let a
Table 6.5.

Table 6.5  Difference between interference and diffraction


S.No. Interference Diffraction
1 Superposition of two waves Bending of waves around edges
2 Superposition of waves from two Superposition wavefronts emitted from
coherent sources. various points of the same wavefront.
3 Equally spaced fringes. Unequally spaced fringes
4 Intensity of all the bright fringes is Intensity falls rapidly for higher orders
almost same
5 Large number of fringes are obtained Less number of fringes are obtained

68 Unit 6  OPTICS

UNIT-6(XII-Physics_Vol-2).indd 68 7/31/2019 8:37:40 PM


plane wavefront of monochromatic light If we take,
with wave length λ be incident normally on 1
the grating. As the slits size is comparable N= (6.159)
a +b
to that of wavelength, the incident light
diffracts at the grating. Then, N gives the number of grating
A diffraction pattern is obtained on elements or rulings drawn per unit width
the screen when the diffracted waves are of the grating. Normally, this number N
focused on a screen using a convex lens. is specified on the grating itself. Now, the
Let us consider a point P at an angle θ with equation becomes,
the normal drawn from the center of the 1
grating to the screen. The path difference δ sin q = ml (or ) sin q = N m l (6.160)
N
between the diffracted waves from one pair
of corresponding points is,
The students should
Note
δ = (a + b) sinθ (6.156) remember that in a single
slit experiment the formula,
This path difference is the same for any a sinθ = nλ is condition for minimum
pair of corresponding points. The point P with n as order of minimum. But, the
will be bright, when formula in diffraction grating, sinθ = Nmλ
is condition for maxima with m as the
δ = m λ where m = 0, 1, 2, 3 (6.157)
order of diffraction.
Combining the above two equations,
we get,
(a + b) sinθ = m λ (6.158) EX AM P L E 6 . 3 4

Here, m is called order of diffraction. A  diffraction grating  consisting of 4000


Condition for zero order maximum, m = 0 slits per centimeter is illuminated with
For (a + b) sinθ = 0, the position, θ = 0. a monochromatic light that produces the
sinθ = 0 and m = 0. This is called zero order second order diffraction at an angle of
diffraction or central maximum. 30°. What is the  wavelength  of the light
used?
Condition for first order maximum, m = 1
If (a + b) sinθ1 = λ, the diffracted light Solution
meet at an angle θ1 to the incident direction Number of lines per cm = 4000; m = 2;
and the first order maximum is obtained.
θ = 30°; λ = ?
Condition for second order maximum, m = 2
Number of lines per unit length,
Similarly, (a + b) sinθ2 = 2λ forms the
second order maximum at the angular 4000
N= = 4 ×105
position θ2. 1×10−2
Equation for diffraction maximum in
Condition for higher order maximum
grating is, sinθ = Nmλ
On either side of central maxima different
higher orders of diffraction maxima are sin q
Rewriting, l =
formed at different angular positions. Nm

Unit 6 OPTICS 69

UNIT-6(XII-Physics_Vol-2).indd 69 7/31/2019 8:38:08 PM


Substituting, number of lines per centimeter =

sin 30 0.5 2.5×105 ×10−2 = 2500 lines per centimetre


l= =
4 ×10 ×2 4 ×105 ×2
5

1 1
= =
2× 4 ×10 ×2 16×105
5

6.11.7  Experiment to
o
l = 6250×10 −10
m = 6250 A determine the wavelength of
monochromatic light
The wavelength of a spectral line
EXA MP LE 6 .3 5
can be very accurately determined with
A  monochromatic  light of wavelength the help of a diffraction grating and a
of  500 nm strikes a grating and  produces spectrometer. Initially all the preliminary
fourth order bright line at an angle of 30°. adjustments of the spectrometer are made.
Find the number of slits per centimeter. The slit of collimator is illuminated by a
monochromatic light, whose wavelength is
Solution to be determined. The telescope is brought
λ = 500 nm = 500×10-9 m; m = 4; in line with collimator to view the image
of the slit. The given plane transmission
θ = 30°; number of lines per cm = ?
grating is then mounted on the prism table
Equation for diffraction maximum in with its plane perpendicular to the incident
grating is, sin θ = Nm λ beam of light coming from the collimator.
sinq The telescope is turned to one side until
Rewriting, N =
ml the first order diffraction image of the slit
Substituting, coincides with the vertical cross wire of the
0. 5 1 eye piece. The reading of the position of the
N= −9
= telescope is noted.
4 ×500×10 2× 4 ×500×10−9
= 2.5×105 lines per meter Similarly the first order diffraction image
on the other side is made to coincide with

You would have noticed the colourful appearance of the compact disc.
On the read/writable side which is polished, there are many narrow circular
tracks with widths comparable to the wavelength of visible light. Hence, the
diffraction takes place after reflection for incident white light to give colourful
appearance. The tracks act as reflecting grating.

β
ds
in

α
α

d sin β

70 Unit 6  OPTICS

UNIT-6(XII-Physics_Vol-2).indd 70 7/31/2019 8:38:32 PM


the vertical cross wire and corresponding diffraction of different orders for all colours
reading is noted. The difference between from violet to red. It produces a spectrum
two positions gives 2θ. Half of its value of diffraction pattern from violet to red on
gives θ, the diffraction angle for first order either side of central maximum as shown in
maximum as shown in Figure 6.66. The Figure 6.67. By measuring the angle at which
wavelength of light is calculated from the these colours appear for various orders of
equation, diffraction, the wavelength of different
colours could be calculated using the formula,
sin q
l= (6.161)
Nm sin q
l= (6.161)
Nm
Here, N is the number of rulings per metre
in the grating and m is the order of the Here, N is the number of rulings per
diffraction image. metre in the grating and m is the order of
the diffraction image.

m=2

Red

m=1
Diffraction
grating
Violet

m = 0 White

Violet
θ n=1
Telescope
Sodium m=1
Turntable
lamp Collimator θ
Angular Position
Red
θ n=0
Angular Position
Diffraction m=2
grating
θ n=1

Figure 6.67  Diffraction with white light


Figure 6.66  Determination of
wavelength using grating and
spectrometer 6.11.9  Resolution
The effect of diffraction has an adverse
impact in the image formation by the
6.11.8  Determination of
optical instruments such as microscope
wavelength of different
and telescope. For a single rectangular slit,
colours the half angle θ  subtended by the spread
When white light is used, the diffraction of central maximum (or position of first
pattern consists of a white central maximum minimum) is given by the relation,
and on both sides continuous coloured
a sinθ = λ(6.162)
diffraction patters are formed. The central
maximum is white as all the colours meet Similar to a rectangular slit, when a
here constructively with no path difference. circular aperture or opening (like a lens or
As θ increases, the path difference, (a+b)sinθ, the iris of our eye) forms an image of a point
passes through condition for maxima of object, the image formed will not be a point
Unit 6  OPTICS 71

UNIT-6(XII-Physics_Vol-2).indd 71 7/31/2019 8:39:12 PM


but a diffraction pattern of concentric circles minimum distance between their diffraction
that becomes fainter while moving away images must be in such a way that the
from the center as shown in Figure 6.68. central maximum of one coincides with the
These are known as Airy’s discs. The circle first minimum of the other and vice versa as
of central maximum has the half angular shown in Figure 6.69(b). Such an image is
spread given by the equation, said to be just resolved image of the object.
The Rayleigh’s criterion is said to be limit of
a sinθ = 1.22 λ(6.163)
resolution.
Here, the numerical value
1.22 comes for central maximum Rayleigh Criterion

formed by circular apertures. This


involves higher level mathematics
which is avoided in this discussion. (a) Unresolved (b) Just resolved (c) Well resolved

ro ro

a
Figure 6.69  Rayleigh’s criterion

f According to Rayleigh’s criterion the two


Figure 6.68  Airy’s discs point sources are said to be just resolved
when the distance between the two maxima
For small angles, sin θ ≈ θ is at least ro. The angular resolution has a
unit in radian (rad) and it is given by the
a θ =1.22 λ(6.164) equation,
Rewriting further, 1.22l
q= (6.166)
1.22l r 1.22l a
q= and 0 =
a f a It shows that the first order diffraction
angle must be as small as possible for greater
1.22l f
r0 = (6.165) resolution. This further shows that for better
a
resolution, the wavelength of light used
When two point sources close to each must be as small as possible and the size of
another form image on the screen, the the aperture of the instrument used must be
diffraction pattern of one point source as large as possible. The Equation 6.165 is
can overlap with another and produce a used to calculate spacial resolution.
blurred image as shown in Figure 6.69(a). The inverse of resolution is called
To obtain a good image of the two sources, resolving power. This implies, smaller the
the two point sources must be resolved i.e., resolution, greater is the resolving power
the point sources must be imaged in such of the instrument. The ability of an optical
a way that their images are sufficiently far instrument to separate or distinguish
apart that their diffraction patterns do not small or closely adjacent objects through
overlap. According to Rayleigh’s criterion, the image formation is said to be resolving
for two point objects to be just resolved, the power of the instrument. In general, the

72 Unit 6  OPTICS

UNIT-6(XII-Physics_Vol-2).indd 72 7/31/2019 8:39:38 PM


term resolution is pertaining to the quality particular direction perpendicular to the
of the image formed and the term resolving direction of wave propagation motion is
power is associated with the ability of the called polarization of light. In this lesson
optical instrument. only the electric field is considered for
discussion.
EXA MP LE 6 .3 6

The optical telescope in the Vainu Bappu 6.12.1  Plane polarised light
observatory at Kavalur has an objective A transverse wave which has vibrations
lens of diameter 2.3 m. What is its angular in all directions in a plane perpendicular
resolution if the wavelength of light used is to the direction of propagation is said to
589 nm? be unpolarised light as shown in Figure
Solution 6.70(a). All these vibrations could be
resolved into parallel and perpendicular
a = 2.3 m; λ = 589 nm = 589×10-9 m; θ = ? components as shown in Figure 6.70(b)
The equation for angular resolution is, which represents unpolarised light. If the
1.22l vibrations of a wave are present in only
q= one direction in a plane perpendicular to
a
the direction of propagation of the wave
Substituting,
is said to be polarised or plane polarised
1.22×589×10−9
q= = 321.4 ×10−9 light as shown in Figure 6.70(c) and
2. 3 6.70(d).
q = 321.4 ×10−7 rad
Unpolarised light Unpolarised light Polarised light
Note: The angular resolution of human eye
E
is approximately, 3×10−4 rad E

6.12
E Direction of
POLARISATION Direction of Direction of
propagation propagation
propagation
(a) (b) (c)

The phenomena of interference and


diffraction demonstrated thatUnpolarised
Unpolarised light light light
is Polarised light Polarised light
propagated E in the form of waves. They did
E
not specify whether the light waves are
transverse or longitudinal. The phenomena
of interference andE diffraction are possible
in both transverse and longitudinal waves.
The phenomenon E of polarization distinctly
Direction of Direction of Direction of Direction of
propagation propagation propagation
proves that
(a)
light waves are
propagation only transverse
(b) (c) (d)
in nature. Light is propagated in the form of
electromagnetic waves. The phenomenon Figure 6.70  Unpolarised and polarised
of restricting the vibrations of light light
(electric or magnetic field vector) to a
Unit 6  OPTICS 73

UNIT-6(XII-Physics_Vol-2).indd 73 7/31/2019 8:40:02 PM


The plane containing the vibrations of 6.12.3  Polarisation by
the electric field vector is known as the plane selective absorption
of vibration ABCD as shown in Figure 6.71.
The plane perpendicular to the plane of Selective absorption is the property of
vibration and containing the ray of light is a material which transmits waves whose
known as the plane of polarisation EFGH. electric fields vibrate in a plane parallel
to a certain direction of orientation and
Plane of absorbs all other waves. The polaroids
vibration
or polarisers are thin commercial
Incident
sheets which make use of the property
light of selective absorption to produce an
intense beam of plane polarised light.
Plane of
Polarisation Selective absorption is also called as
dichroism.
In 1932, an American scientist
Edwin Land developed polarisers in the
Figure 6.71  Plane of vibration and plane
form of sheets. Tourmaline is a natural
of polarisation
polarising material. Polaroids are also
made artificially. It was discovered
The Table 6.6 consolidates few
that small needle shaped crystals of
characteristics of polarised and unpolarised
quinine iodosulphate have the property
light.
of polarising light. A number of these
crystals with their axes parallel to
6.12.2  Polarisation
one another packed in between two
Techniques transparent plastic sheets serve as a
The unpolarised light can be polarised by good polaroid. Recently new types of
several techniques. Here, we are discussing polaroids are prepared in which thin
the following four methods, film of polyvinyl alcohol is used. These
(i) polarisation by selective absorption are colourless crystals which transmit
(ii) polarisation by reflection more light, and give better polarisation.
(iii) polarisation by double refraction Polaroids have many applications as the
(iv) polarisation by scattering. one shown in Figure 6.72.

Table 6.6  Characteristics of polarised light and unpolarised light


S.No Polarised light Unpolarised light
1 Consists of waves having their electric Consists of waves having their electric
field vibrations in a single plane field vibrations equally distributed in all
normal to the direction of ray. directions normal to the direction of ray.
2 Asymmetrical about the ray direction Symmetrical about the ray direction
3 It is obtained from unpolarised light Produced by conventional light sources.
with the help of polarisers

74 Unit 6  OPTICS

UNIT-6(XII-Physics_Vol-2).indd 74 7/31/2019 8:40:03 PM


which plane polarises the unpolarised
Unpolarised
light
Glare greatly light passing through it is called a
Glasses transmit reduced
only vertically
polarised light
polariser. The polaroid (here P2) which is
used to examine whether a beam of light
Direct light
not reduced
is polarised or not is called an analyser.
as much as
glare If the intensity of the unpolarised light is
Light partially polarised
I then the intensity of plane polarised light
in the horizontal plane I 
by reflection
will be  . The other half of intensity is
 2 
Figure 6.72  Polaroid sun glasses
restricted by the polariser.
6.12.3.2 Plane and partially polarised light
P1
Polarised
light P2 In plane polarised light the intensity
Unpolarised
light varies from maximum to zero for every
rotation of 90o of the analyser as shown in
Figure 6.74(a). This is because the vibrations
are allowed in one axis and completely
Source Polariser Analiser
restricted in the perpendicular axis. On the
Figure 6.73  Polariser and analyser other hand, if the intensity of light varies
between maximum and minimum for
every rotation of 90o of the analyser, the
6.12.3.1 Polariser and analyser
light is said to be partially polarised light as
Let us consider an unpolarised shown in Figure 6.74(b). This is because the
beam of light. The vibrations can be light is not fully restricted in that particular
in all possible directions all of them axis which shows a minimum intensity.
being perpendicular to the direction of
propagation as shown in Figure 6.73.
Max
When this light passes through polaroid
P1 the vibrations are restricted to only one
Intensity

plane. The emergent beam can be further


passed through another polaroid P2. If
the polaroid P2 is rotated about the ray of Zero
     
light as axis, for a particular position of (a) Plane polarised light
P2 the intensity is maximum. When the
polaroid P2 is rotated from this position
Max
the intensity starts decreasing. There is
Intensity

complete extinction of the light when P2 is


rotated through 90o. On further rotation
of P2 the light reappears and the intensity
increases and becomes a maximum for a Min

further rotation through 90o. The light     
(b) Partially polarised light
coming out from polaroid P1 is said to be
plane polarised. The Polaroid (here P1) Figure 6.74  Intensity variation in plane
and partially polarised light

Unit 6  OPTICS 75

UNIT-6(XII-Physics_Vol-2).indd 75 7/31/2019 8:40:07 PM


Y
Polariser Analyser

E
E Ey
θ
X
Ex
I0

Unpolarised I = I0 cos2 θ
Light

Figure 6.75  Malus’s law

6.12.3.3 Malus’ law Only the component (acosθ) will be


When a plane polarised light is seen transmitted by the analyser. The intensity
through an analyser, the intensity of of light transmitted from the analyser
transmitted light varies as the analyser is is proportional to the square of the
rotated through an angle perpendicular component of the amplitude transmitted
to the incident direction. In 1809, French by the analyser.
Physicist E.N Malus discovered that
I µ (a cos q )2
when a beam of plane polarised light of
intensity I0 is incident on an analyser, I = k(a cos q )2
the light transmitted of intensity I from
Where k is constant of proportionality.
the analyser varies directly as the square
of the cosine of the angle θ between I = ka 2 cos2 q
the transmission axis of polariser and
analyser as shown in Figure 6.75. This is I = I 0 cos2 q
known as Malus’ law. Where, I 0 = ka2 is the maximum intensity
of light transmitted from the analyser.
I = I0 cos2 θ(6.167)
Plane of
polariser
Plane of
The proof of Malus’s law is as follows. analyser
Let us consider the plane of polariser and
analyser are inclined to each other at an a
angle θ is as shown in Figure 6.76. Let I 0
be the intensity and a be the amplitude a cos θ
of the electric vector transmitted by θ
a sin θ
the polariser. The amplitude a of the
incident light has two rectangular
Figure 6.76  Malus’ law
components, (acosθ) and (asinθ) which
are the parallel and perpendicular
The following are few special cases.
components to the axis of transmission
of the analyser. Case (i) When θ = 0o, cos 0= 1, I = I 0

76 Unit 6  OPTICS

UNIT-6(XII-Physics_Vol-2).indd 76 7/31/2019 8:40:48 PM


When the transmission axis of polariser EX AM P L E 6 . 3 8
is along that of the analyser, the intensity of
light transmitted from the analyser is equal Two polaroids are kept crossed
to the incident light that falls on it from the (transmission axes at 90o) to each other.
polariser. (i) What will be the intensity of the light
Case (ii) When θ = 90 , cos 90 = 0, I = 0
o o
coming out from the second polaroid when
When the transmission axes of polariser an unpolarised light of intensity I falls on
and analyser are perpendicular to each the first polaroid?
other, the intensity of light transmitted from (ii) What will be the intensity of light
the analyser is zero. coming out from the second polaroid if a
third polaroid is kept at 45o inclination to
EXA MP LE 6 .3 7
both of them.
Two polaroids are kept with their
transmission axes inclined at 30o. Solution
Unpolarised light of intensity I falls on the (i) As the intensity of the unpolarised
first polaroid. Find out the intensity of light light falling on the first polaroid is I, the
emerging from the second polaroid. intensity of polarized light emerging from
I 
Solution it will be I0 =  . Let I ′ be the intensity of
 2 
As the intensity of the unpolarised light light emerging from the second polaroid.
falling on the first polaroid is I, the intensity
I  Malus’ law, I ′ = I 0 cos2 θ
of polarized light emerging will be, I0 =  .
 2  Here θ is 90o as the transmission axes are
Let I ′ be the intensity of light emerging
perpendicular to each other.
from the second polaroid.
Substituting,
Malus’ law, I ′ = I 0 cos2 q
 
I ′ =  I  cos2 (90 ) = 0 ∴ cos (90 ) = 0
Substituting, 2  
2
I  I  3  3 No light comes out from the second
I ′ =   cos2 (30 ) =    = I
 2   2  2  8 polaroid.
(ii) Let the first polaroid be P1 and the
3
I ′ =   I second polaroid be P2. They are oriented
 8 
at 90o. The third polaroid P3 is introduced
between them at 45o. Let I ′ be the intensity
of light emerging from P3.
Angle between P1 and P3 is 45o. The intensity
I
θ of light coming out from P3 is, I ′ = I 0 cos2 q

I
Substituting,
2 3I
2
8 I   I  1  I I
I ′ =   cos2 (45 ) =    = ; I ′ =
 2   2  2  4′ 4

Unit 6  OPTICS 77

UNIT-6(XII-Physics_Vol-2).indd 77 7/31/2019 8:41:33 PM


Angle between P3 and P2 is 45o. Let I″ is Vibrations in AB which are parallel to the
the intensity of light coming out from P2 plane of the diagram are shown by arrows.
I ′′ = I ′ cos2 q The vibrations which are perpendicular
to the plane of the diagram and parallel to
Here, the intensity of polarized light the reflecting surface are shown by dots in
I
existing between P3 and P2 is . Figure 6.77. A part of the light is reflected
4
Substituting, along BC, and the rest is refracted along
I
2
 I  1  I BD. On examining the reflected beam BC
I ′′ =   cos2 (45 ) =    = with an analyser, it is found that the ray is
 4   4  2  8
partially plane polarised. When the light is
I allowed to be incident at a particular angle
I ′′ =
8 the reflected beam is found to be plane
polarised. The angle of incidence at which
the reflected beam is plane polarised is
I
called polarising angle ip.
I
2
I´ I´´
Incident beam
Reflected beam C
A
ip
ip

6.12.3.4. Uses of polaroids X B Y


rp
1. Polaroids are used in goggles and
cameras to avoid glare of light.
Refracted beam
2. Polaroids are useful in three dimensional
motion pictures i.e., in holography. D

3. Polaroids are used to improve contrast in


old oil paintings. Figure 6.77  Polarisation by reflection
4. Polaroids are used in optical stress
analysis. 6.12.4.1  Brewster’s Law
5. Polaroids are used as window glasses to
In 1808, Malus discovered that when
control the intensity of incoming light.
ordinary light is incident on the surface of
6. Polarised laser beam acts as needle to
a transparent medium, the reflected light
read/write in compact discs (CDs).
is partially plane polarised. The extent
7. Polaroids produce polarised lights to be
of polarisation depends on the angle
used in liquid crystal display (LCD).
of incidence. For a particular angle of
incidence, the reflected light is found to be
6.12.4  Polarisation by plane polarised. The angle of incidence at
reflection which a beam of unpolarised light falling
on a transparent surface is reflected as
The simplest method of producing plane
a beam of plane polarised light is called
polarised light is by reflection. Consider a
polarising angle or Brewster’s angle. It is
beam of unpolarised light AB is incident at
denoted by ip
any angle on the reflecting glass surface XY.
78 Unit 6  OPTICS

UNIT-6(XII-Physics_Vol-2).indd 78 7/31/2019 8:41:52 PM


Further, the British Physicist, Sir. David For glass, tan i p = 1.5; i p = tan−1 1.5; i p = 56.3
Brewster found that at the incidence
of polarising angle, the reflected and Forwater,tan i p = 1.33; i p = tan−1 1.33; i p = 53.1
transmitted rays are perpendicular to each
other. Suppose, ip is the polarising angle and
rp is the corresponding angle of refraction. 6.12.4.2  Pile of plates
Then from Figure 6.82,

i p + 900 + rp = 1800(6.168)
56.3°

rp = 900 − i p (6.169)
33.7° 56.3°

From Snell’s law, the refractive index of


the transparent medium is,
sin i p
= n(6.170)
sinrp
where n is the refractive index of the
medium with respect to air. 56.3°

Substituting the value of rp from Equation 33.7°


6.163, we get,
sin i p sin i p Figure 6.78  Pile of plates
= =n
sin (90 − i p ) cosi p
The phenomenon of polarisation by
tani p = n(6.171) reflection is used in the construction of
pile of plates. It consists of a number of
This relation is known as Brewster’s
glass plates placed one over the other in a
law. The law states that the tangent of the
tube as shown in Figure 6.78. The plates are
polarising angle for a transparent medium
inclined at an angle of 33.7 (90 - 56.3 ) to
is equal to its refractive index. The value of
the axis of the tube. A beam of unpolarised
Brewster’s angle depends on the nature of
light is allowed to fall on the pile of plates
the transparent refracting medium and the
along the axis of the tube. So, the angle of
wavelength of light used.
incidence of light will be at 56.3o which is
the polarising angle for glass. The vibrations
EXA MP LE 6 .3 9 perpendicular to the plane of incidence are
reflected at each surface and those parallel
Find the polarizing angles for (i) glass to it are transmitted. The larger the number
of refractive index 1.5 and (ii) water of of surfaces, the greater is the intensity of the
refractive index 1.33. reflected plane polarised light. The pile of
plates is used as a polarizer and also as an
Solution
analyser.
Brewster’s law, tani p = n

Unit 6  OPTICS 79

UNIT-6(XII-Physics_Vol-2).indd 79 7/31/2019 8:42:58 PM


EXA MP LE 6 .4 0 laws of refraction, called as extraordinary
rays. The extraordinary ray is found to be
What is the angle at which a glass plate plane polarised. Inside a double refracting
of refractive index 1.65 is to be kept with crystal the ordinary ray travels with same
respect to the horizontal surface so that an velocity in all directions and the extra
unpolarised light travelling horizontal after ordinary ray travels with different velocities
reflection from the glass plate is found to along different directions. A point source
be plane polarised? inside a refracting crystal produces spherical
wavefront corresponding to ordinary ray
Solution
and elliptical wavefront corresponding to
n = 1.65 extraordinary ray. Inside the crystal there
is a particular direction in which both the
Brewster’s law, tani p = n
rays travel with same velocity. This direction
tan i p = 1.65; i p = tan−1 1.65; i p = 58.80 is called optic axis. Along the optic axis, the
refractive index is same for both the rays
The inclination with the horizontal surface and there is no double refraction along this
is, (900 − 58.80 ) = 31.20 direction.
Extraordinary
ray

6.12.5  Polarisation by
double refraction
Erasmus Bartholinus, a Danish
physicist discovered that when a ray Ordinary
of unpolarised light is incident on Unpolarised ray
light
a calcite crystal, two refracted rays
are produced. Hence, two images
of a single object are formed. This
phenomenon is called double refraction as E
E

shown in Figure 6.79. Double refraction is O


O

also called birefringence. This phenomenon


is also exhibited by several other crystals like
quartz, mica etc.
When an ink dot on a sheet of paper is Figure 6.79  Double refraction
viewed through a calcite crystal, two images
will be seen. On rotating the crystal, one 6.12.6  Types of optically
image remains stationary, while the other active crystals
rotates around the first. The stationary image
is known as the ordinary image O, produced Crystals like calcite, quartz, tourmaline
by the refracted rays which obey the laws of and ice having only one optic axis are called
refraction, called as ordinary rays. The other uniaxial crystals.
image is extraordinary image E, produced Crystals like mica, topaz, selenite and
by the refracted rays which do not obey the aragonite having two optic axes are called
biaxial crystals.
80 Unit 6  OPTICS

UNIT-6(XII-Physics_Vol-2).indd 80 7/31/2019 8:43:17 PM


6.12.7  Nicol prism other face. The extraordinary ray alone is
transmitted through the crystal which is
Nicol prism is an optical device plane polarised.
incorporated in optical instruments both
for producing and analysing plane polarised Uses of Nicol prism
light. The construction of a Nicol prism (i) It produces plane polarised light and
is based on the phenomenon of Double functions as a polariser
refraction and was designed by William (ii) It can also be used to analyse the plane
Nicol in 1828. polarised light i.e used at an analyser.
One of the most common forms of the Drawbacks of Nicol prism
Nicol prism is made by taking a calcite (i) Its cost is very high due to scarity of
crystal which is a double refracting crystal large and flawless calcite crystals
with its length three times its breadth. As (ii) Due to extraordinary ray passing
shown in Figure 6.80, ABCD represents obliquely through it, the emergent ray
the principal section of a calcite crystal. It is always displaced a little to one side.
is cut into two halves along the diagonal so (iii) The effective field of view is quite
that their face angles are 72o and 108o. The limited
two halves are joined together by a layer of (iv) Light emerging out of it is not uniformly
canada balsam, a transparent cement. plane polarised.
A Canada balsam
D

108 6.12.8  Polarisation by


scattering
72 Extraordinary The light from a clear blue portion of
C ray
B the sky shows a rise and fall of intensity
Ordinary
ray when viewed through a polaroid which is
Figure 6.80  Nicol Prism rotated. This is because of sunlight, which
has changed its direction (having been
Let us consider a ray of unpolarised scattered) on encountering the molecules
light from monochromatic source such as a of the earth’s atmosphere. As Figure 6.81
sodium vapour lamp is incident on the face shows, the incident sunlight is unpolarised.
AC of the Nicol prism. Double refraction The electric field of light interact with the
takes place and the ray is split into ordinary electrons present in the air molecules.
and extraordinary rays. They travel Under the influence of the electric field
with different velocities. The refractive of the incident wave the electrons in the
index of the crystal for the ordinary ray molecules acquire components of motion
(monochromatic sodium light) is 1.658 and in both these directions. We have shown
for extraordinary ray is 1.486. The refractive an observer looking at 90° to the direction
index of canada balsam is 1.523. Canada of the sun. Clearly, charges accelerating
balsam does not polarise light. parallel do not radiate energy towards
The ordinary ray is total internally this observer since their acceleration has
reflected at the layer of canada balsam no transverse component. The radiation
and is prevented from emerging from the scattered by the molecule is therefore

Unit 6  OPTICS 81

UNIT-6(XII-Physics_Vol-2).indd 81 7/31/2019 8:43:38 PM


polarized perpendicular to the plane of the (2) Normal focusing – The image is formed
Figure 6.81. This explains the reason for at infinity. In this position the eye is
polarisation of sunlight by scattering. most relaxed to view the image. This is
shown in Figure 6.83(b).
Unpolarised
Molecule
Sunlight Unpolarised 6.13.1.1 Magnification in near point
light
focusing
Partially The near point focusing is shown in
polarised
light
Figure 6.82. Object distance u is less than f.
The image distance is the near point D. The
magnification m is given by the relation,
v
Polarised m = (6.172)
light u

1 1 1
With the help of lens equation, − =
Figure 6.81  Polarisation by scattering v u f
the magnification can further be written as,
v
6.13 m = 1 − (6.173)
f
OPTICAL INSTRUMENTS Substituting for v with sign convention,
v = –D
There are plenty of optical instruments
we used in our daily life. We shall discuss D
m = 1+ (6.174)
here about microscopes, telescopes, f
spectrometer and of course human eye.
This is the magnification for near point
focusing.
6.13.1  Simple microscope
A simple microscope is a single Eye focussed
magnifying (converging) lens of small focal on near point

length. The idea is to get an erect, magnified


u
and virtual image of the object. For this the D
object is placed between F and P on one side
of the lens and viewed from other side of
the lens. There are two magnifications to be
discussed for two kinds of focussing. Figure 6.82  Near point focusing
(1) Near point focusing – The image is
formed at near point, i.e. 25 cm for 6.13.1.2 Magnification in normal focusing
normal eye. This distance is also called (angular magnification)
as least distance D of distinct vision. In The normal focusing is shown in Figure
this position, the eye feels comfortable 6.83(b). We will now find the magnification
but there is little strain on the eye. This for the image formed at infinity. If we take
is shown in Figure 6.82 the ratio of height of image to height of

82 Unit 6  OPTICS

UNIT-6(XII-Physics_Vol-2).indd 82 7/31/2019 8:44:06 PM


 h′  This is the magnification for normal
object  m =  to find the magnification, focusing.
h
we will not get a practical relation as the The magnification for normal focusing
image will also be of infinite size when the is one less than that for near point focusing.
image is formed at infinity. Hence, we can But, the viewing is more comfortable in
practically use the angular magnification. normal focusing than near point focusing.
The angular magnification is defined as For large values of D/f, the difference
the ratio of angle q i subtended by the image in magnification is usually small. In
with aided eye to the angle q 0 subtended by subsequent discussions, we shall only
the object with unaided eye. consider the normal focusing.

qi
m= (6.169) EX AM P L E 6 . 4 1
q0
A man with a near point of 25 cm reads a
book with small print using a magnifying
h θº glass, a convex lens of focal length 5 cm.
D
(a) What is the closest and the farthest
(a) with unaided eye
distance at which he should keep the lens
from the page so that he can read the book
when viewing through the magnifying
glass? (b) What is the maximum and
h
the minimum angular magnification
θi
f θi Eye focussed (magnifying power) possible using the
at infinity above simple microscope?
Solution
(b)with aided eye
D = 25 cm; f = 5 cm;
Figure 6.83  Normal focusing
For closest object distance, u; the image
For unaided eye shown in Figure 6.83(a), distance, v is, –25 cm. (near point
focusing)
h
tanq 0 ≈ q 0 = (6.176) For farthest object distance, ( u ′ ); the
D
corresponding image distance, ( v ′ ) is, v = ∞
For aided eye shown in Figure 6.83(b),
(normal focusing)
h
tanq i ≈ q i = (6.177) (a) To find closest image distance, lens
f 1 1 1
equation, − =
The angular magnification is, v u f
Rewriting for closest object distance,
qi h/ f
m= = 1 1 1
q0 h / D = −
u v f
D Substituting,
m= (6.178)
f 1 1 1 1 1  −1 − 5  6
= − = − =   = − ;

u −25 5 25 5  25  25
Unit 6  OPTICS 25 83
u = − = −4.167 cm
6

UNIT-6(XII-Physics_Vol-2).indd 83 7/31/2019 8:45:14 PM


1 1 1 1 1  −1 − 5  6
= − = − =  
 =− ;
u −25 5 25 5  25  25
25 The radius of central maxima is already
u = − = −4.167 cm
6 derived as equation 6.159,
The closest distance at which the person 1.22lv
should can keep the book is, u = –4.167 cm r0 =  (6.179)
a
To find farthest object distance, lens In the place of focal length f we have the
1 1 1 image distance v. If the difference between
equation is, − =
v ′ u′ f ′ the two points on the object to be resolved is
Rewriting for farthest object distance, dmin, then the magnification m is,
1 1 1
= − r
u′ v ′ f ′ m= (6.180)
dmin
1 1 1
Substituting, = − ; u = −5 cm
u′ ∞ 5 r 1.22lv 1.22lv 1.22lu
dmin = = = = [ ∴ m = v / u ]
The farthest distance at which the person m am a (v / u) a
cm
can keep the book is, u ′ = −5 cm
1.22l f
  dmin =
a
[ ∴ u ≈ f ] (6.181)
(b) To find magnification in near point
D 25
focusing, m = 1 + = 1+ = 6 On the object side,
f 5
To find magnification in normal focusing, a
2 tan b ≈ 2 sin b = ∴ [a = f 2 sin b ] (6.182)
f
D 25
m= = =5
f 5 1.22l
dmin = (6.183)
2 sin b
6.13.1.3. Resolving power of microscope
To further reduce the value of dmin the
The diagram related to the calculation
optical path of the light is increased by
of resolution of microscope is illustrated in
immersing the objective of the microscope
Figure 6.84. A microscope is used to see the
in to a bath containing oil of refractive
details of the object under observation. The
index n.
ability of microscope depends not only in
magnifying the object but also in resolving 1.22l
dmin = (6.184)
two points on the object separated by a 2n sin b
small distance dmin. Smaller the value of dmin Such an objective is called oil immersed
better will be the resolving power of the objective. The term n sin β is called
microscope. numerical aperture NA.
Object Image 1.22l
dmin = (6.185)
a
θ 2(NA)
β
1.22f λ
D
f
Object v
plane
Objective Image plane 6.13.2  Compound
lens microscope
Figure 6.84  Resolving power of
microscope The diagram of a compound microscope
is shown in Figure 6.85. The lens near the
84 Unit 6  OPTICS

UNIT-6(XII-Physics_Vol-2).indd 84 7/31/2019 8:47:07 PM


object, called the objective, forms a real, Here, the distance L is between the first
inverted, magnified image of the object. focal point of the eyepiece to the second
This serves as the object for the second lens focal point of the objective. This is called the
which is the eyepiece. Eyepiece serves as a tube length L of the microscope as fo and fe
simple microscope that produces finally are comparatively smaller than L.
an enlarged and virtual image. The first If the final image is formed at P (near
inverted image formed by the objective point focussing), the magnification me of
is to be adjusted close to, but within the the eyepiece is,
focal plane of the eyepiece so that the final
D
image is formed nearly at infinity or at me = 1 + (6.189)
fe
the near point. The final image is inverted
with respect to the original object. We can The total magnification m in near point
obtain the magnification for a compound focusing is,
microscope.  L  D
m = mme =  1 + (6.190)
 f   f e 
u fo fe
If the final image is formed at infinity
A
h β B' (normal focusing), the magnification me of
the eyepiece is,
β
B'' B O
h' E

Objective A' D
h'' me = (6.191)
fe
Eyepiece
The total magnification m in normal
D focusing is,
A''
 L  D 
Figure 6.85  Compound microscope m = mme =   (6.192)
 f   f e 

6.13.2.1. Magnification of compound


microscope EX AM P L E 6 . 4 2
From the ray diagram, the linear
magnification due to the objective is, A microscope has an objective and
eyepiece of focal lengths 5 cm and 50 cm
h′ respectively with tube length 30 cm. Find
m = (6.186)
h the magnification of the microscope in the
(i) near point and (ii) normal focusing.
h h′
From the Figure 6.85, tan b = = , then Solution
f L
f 0 = 5cm = 5×10−2 m; f e = 50cm = 50×10−2 m;
h′ L
= (6.187)
h f0 L = 30cm = 30×10−2 m; D = 25cm = 25×10−2 m

L
(i) The total magnification m in near point
m = (6.188)  L  D
f focusing is, m = mme =  1 + 
 f   f e 

Unit 6  OPTICS 85

UNIT-6(XII-Physics_Vol-2).indd 85 7/31/2019 8:48:48 PM


Substituting, image to the angle α which the object
 30×10−2  25×10−2  subtends at the lens or the eye.
m = mme =  −2 
1 + −2 

 5×10  50×10  b
m= (6.193)
= (6)(1.5) = 9 a
(ii) The total magnification m in normal
 L  D  h / fe
focusing is, m = mme =      From the diagram, m = (6.194)
 f   f e  h / f0
Substituting, f0
m= (6.195)
 30×10−2  25×10−2  fe
m = mme =  −2 
 −2 

 5×10  50×10  The length of the telescope is
= (6)(0.5) = 3 approximately, L = fo + fe

6.13.3.  Astronomical EX AM P L E 6 . 4 3
telescope
A small telescope has an objective lens of
An astronomical telescope is used to get focal length 125 cm and an eyepiece of focal
the magnification of distant astronomical length 2 cm. What is the magnification
objects like stars, planets, moon etc. the of the telescope? What is the separation
image formed by astronomical telescope between the objective and the eyepiece?
will be inverted. It has an objective of long Two stars separated by 1′ will appear at
focal length and a much larger aperture than what separation when viewed through the
the eyepiece as shown in Figure 6.86. Light telescope?
from a distant object enters the objective
and a real image is formed in the tube at its Solution
second focal point. The eyepiece magnifies fo = 125 cm; fe = 2 cm; m = ?; L = ?; θi = ?
this image producing a final inverted image.
Equation for magnification of telescope,
fo f0
m=
fe
fe 125
Substituting, m = = 62.5
α B' 2
α β
O h E Equation for approximate length of
A' telescope, L = fo+ fe

Objective Eyepiece Substituting, L = 125+2 = 127 cm = 1.27 m


qi
Equation for angular magnification, m =
Figure 6.86  Astronomical telescope q0
Rewriting, q i = m×q 0
6.13.3.1 Magnification of astronomical
Substituting,
telescope
62.5
The magnification m is the ratio of the q i = 62.5×1′ = 62.5′ = = 1.04
angle β subtended at the eye by the final 60

86 Unit 6  OPTICS

UNIT-6(XII-Physics_Vol-2).indd 86 7/31/2019 8:51:06 PM


6.13.4  Terrestrial telescope an eye piece inside obstructing some light.
This problem could also be overcome by
A terrestrial telescope is used to see object introducing a secondary mirror which
at long distance on the surface of earth. would take the light outside the tube for
Hence, image should be errect. A terrestrial view as shown in the Figure 6.88.
telescope has an additional erecting lens
to make the final image erect as shown in
Figure 6.87. 6.13.6  Spectrometer
The spectrometer is an optical
instrument used to study the spectra of
To ∞
A'' different sources of light and to measure the
α B'
C
α Fo C'
Fo
B'' C'' refractive indices of materials. It is shown
in Figure 6.89. It consists of basically three
A'

Objective
Erecting lens
Eyepiece
parts. They are (i)  collimator, (ii) prism
table and (iii) Telescope.
Figure 6.87  Terrestrial telescope

6.13.5  Reflecting telescope

Eyeplece
Secondary
mirror
(convex)
Figure 6.89  Spectrometer

Primary mirror (parabolic)


(i) Collimator
Figure 6.88  Reflecting telescope The collimator is an arrangement to
produce a parallel beam of light. It consists
Modern telescopes use a concave of a long cylindrical tube with a convex lens
mirror rather than a lens for the objective. at the inner end and a vertical slit at the
It is rather difficult and expensive to make outer end of the tube. The distance between
lenses of large size which form images that the slit and the lens can be adjusted such
are free from any optical defect. Telescopes that the slit is at the focus of the lens. The slit
with mirror objectives are called reflecting is kept facing the source of light. The width
telescopes. They have several advantages. of the slit can be adjusted. The collimator is
Only one surface it to be polished and rigidly fixed to the base of the instrument.
maintained. Support can be given from the
entire back of the mirror rather than only (ii) Prism table
at the rim for lens. Mirrors weigh much The prism table is used for mounting the
less compared to lens. But the one obvious prism, grating etc. It consists of two circular
problem with a reflecting telescope is that metal discs provided with three levelling
the objective mirror would focus the light screws. It can be rotated about a vertical axis
inside the telescope tube. One must have passing through its centre and its position

Unit 6  OPTICS 87

UNIT-6(XII-Physics_Vol-2).indd 87 7/31/2019 8:51:07 PM


can be read with verniers V1 and V2 . The telescope. Since the telescope is already
prism table can be raised or lowered and can adjusted for parallel rays, a well-defined
be fixed at any desired height. image of the slit can be formed, only when
the light rays emerging from the collimator
(iii) Telescope
are parallel.
The telescope is an astronomical type.
(d) Levelling the prism table The prism table
It consists of an eyepiece provided with
is adjusted or levelled to be in horizontal
cross wires at one end and an objective
position by means of levelling screws and a
lens at its other end. The distance between
spirit level.
the objective lens and the eyepiece can be
adjusted so that the telescope forms a clear 6.13.6.1 Determination of refractive index
image at the cross wires, when a parallel of material of the prism
beam from the collimator is incident on it. The preliminary adjustments of the
The telescope is attached to an arm telescope, collimator and the prism table of
which is capable of rotation about the same the spectrometer are made. The refractive
vertical axis as the prism table. A circular index of the prism can be determined by
scale graduated in half degree is attached knowing the angle of the prism and the
to it. Both the telescope and prism table are angle of minimum deviation.
provided with radial screws for fixing them (i) Angle of the prism (A)
in a desired position and tangential screws
for fine adjustments.
Adjustments of the spectrometer Collimator

The following adjustments must be


made before doing the experiment using A
spectrometer.
(a) Adjustment of the eyepiece The A
telescope is turned towards an
illuminated surface and the eyepiece is B C
T1 T2
moved to and fro until the cross wires PRISM TABLE

are clearly seen.


Telescope Telescope
2A
(b) Adjustment of the telescope The
telescope is adjusted to receive parallel Figure 6.90  Angle of prism
rays by turning it towards a distant object
and adjusting the distance between the The prism is placed on the prism table
objective lens and the eyepiece to get a with its refracting edge facing the collimator
clear image on the cross wire. as shown in Figure 6.90. The slit is illuminated
(c) Adjustment of the collimator The telescope by a sodium light (monochromotic
is brought along the axial line with the light). The parallel rays coming from the
collimator. The slit of the collimator is collimator fall on the two faces AB and AC.
illuminated by a source of light. The The telescope is rotated to the position T1
distance between the slit and the lens of the until the image of the slit formed by the
collimator is adjusted until a clear image reflection at the face AB is made to coincide
of the slit is seen at the cross wire of the with the vertical cross wire of the telescope.
88 Unit 6  OPTICS

UNIT-6(XII-Physics_Vol-2).indd 88 7/31/2019 8:51:07 PM


The readings of the verniers are noted. The are noted. The difference between the
telescope is then rotated to the position T2 two readings gives the angle of minimum
where the image of the slit formed by the deviation D. The refractive index of the
reflection at the face AC coincides withthe material of the prism n is calculated using
vertical cross wire. The readings are again the formula,
noted.
 A + D 
The difference between these two sin 
 2 
readings gives the angle rotated by the n= (6.103)
 A 
telescope, which is twice the angle of the sin  
 2 
prism. Half of this value gives the angle of
the prism A. The refractive index of a liquid may be
(ii) Angle of minimum deviation (D) determined in the same way using a hollow
glass prism filled with the given liquid.
The prism is placed on the prism table
Spectrometer experiments are discussed
so that the light from the collimator falls
in the Practicals given in Volume 1 of this
on a refracting face, and the refracted
book.
image is observed through the telescope as
shown in Figure 6.91. The prism table is
now rotated so that the angle of deviation 6.13.7  The eye
decreases. A stage comes when the image
Eye is a natural optical instrument given
stops for a moment and if we rotate the
by God to the human beings. The internal
prism table further in the same direction,
structure and the Physics aspect of the
the image is seen to recede and the angle of
functioning of different parts of human eye
deviation increases. The vertical cross wire
are discussed already in (X Physics Unit-2).
of thetelescope is made to coincide with the
As the eye lens is flexible, its focal length can
image of the slit where it turns back. This
be changed to some extent. When the eye is
gives the minimum deviation position.
fully relaxed, the focal length is maximum
and when it is strained the focal length is
Collimator Telescope position No.1 minimum. The image must be formed on
A the retina for a clear vision. The diameter
θ
PRISM
of eye for a normal adult is about 2.5 cm.
BASE D
Hence, the image-distance, in other words,
PRISM TABLE
the distance between eye lens and retina
is fixed always at 2.5 cm for a normal eye.
Telescope position No.2
We can just discuss the optical functioning
Figure 6.91  Angle of minimum of eye without giving importance to the
deviation refractive indices of the two liquids, aqueous
humor and virtuous humor present in
The readings of the verniers are noted. the eye A person with normal vision can
Now, the prism is removed and the telescope see objects kept at infinity in the relaxed
is turned to receive the direct ray and the condition with maximum focal length fmax of
vertical cross wire is made to coincide with the eye as shown in Figure 6.92(a). Also at a
the image. The readings of the verniers distance of 25 cm in the strained condition

Unit 6  OPTICS 89

UNIT-6(XII-Physics_Vol-2).indd 89 7/31/2019 8:51:16 PM


with minimum focal length fmin of the eye as See, the small variation of
shown in Figure 6.92(b). f max − f min = 0.23 cm of the focal length of
eye lens makes objects visible from infinity
to near point for a normal person. Now, we
can discuss some common defects of vision
in the eye.
6.13.7.125Nearsightedness
cm (myopia)
A person suffering from nearsightedness
(a) Relexed eye or myopia (b) Strained
cannot eye
see distant objects clearly.
This may result because the lens has too
short focal length due to thickening of
the lens or larger diameter of the eyeball
than usual. These people have difficulty in
25 cm relaxing their eye more than what is needed
to overcome this difficulty. Thus, they need
eye (b) Strained eye correcting lens.
Figure 6.92  Focusing of normal eye The parallel rays coming from the
distant object get focused before reaching
Let us find fmax and fmin of human eye the retina as shown in Figure 6.93(a). But,
from the lens equation given below. these persons can see objects which are
nearer. Let x be the maximum distance up
1 1 1 to which a person with nearsightedness
= − (6.69) can see as shown in Figure 6.93(b). To
f v u
overcome this difficulty, the virtual
When the object is at infinity, u = –∞, image of the object at infinity should be
and v = 2.5 cm (distance between eye lens formed at a distance x from the eye using
and retina), the eye can see the object in a correcting lens as shown in Figure
relaxed condition with fmax. Substituting 6.93(c).
these values in the lens equation gives, The focal length of the correcting lens
for a myopic eye can be calculated using the
1 1 1
= − lens equation.
f max 2.5 cm −∞
1 1 1
f max = 2.5 cm = − (6.69)
f v u
When the object is at near point, u = –25 Here, u = –∞, v = –x. Substituting these
cm, and v = 2.5 cm, the eye can see the object values in the lens equation gives,
in strained condition with fmin. Substituting
these values in the lens equation gives, 1 1 1
= −
f −x −∞
1 1 1
= −
f min 2.5 cm −25 cm Focal length f of the correcting lens is,

f min = 2.27 cm f = –x(6.196)

90 Unit 6  OPTICS

UNIT-6(XII-Physics_Vol-2).indd 90 7/31/2019 8:52:22 PM


x x
(a) (b) (c)

Figure 6.93  Myopic eye and correction

The negative sign in the above result The rays coming from the object at near
suggests that the lens should be a concave point get focused beyond the retina as shown
lens. Basically, the concave lens slightly in Figure 6.94(a). But, these persons can see
diverges the parallel rays from infinity and objects which are far say, more than 25 cm.
makes them focus now at the retina which Let y be the minimum distance from the eye
got earlier focused before reaching retina in beyond which a person with farsightedness
the unaided condition. can see as shown in Figure 6.94(b). To
6.13.7.2  Farsightedness (hypermetopia) overcome this difficulty, the virtual image of
the object at y should be formed at a distance
A person suffering from farsightedness
of 25 cm (near point) from the eye using a
or hypermetropia or hyperopia cannot
correcting lens as shown in Figure 6.94(c).
clearly see objects close to the eye. It
The focal length of the correcting lens
occurs when the eye lens has too long
for a hypermetropic eye can be calculated
focal length due to thining of eye lens or
using the lens equation.
shortening of the eyeball than normal.
The least distance for clear vision for 1 1 1
= − (6.69)
these people is appreciably more than 25 f v u
cm and the person has to keep the object Here, u = –y, v = –25 cm. Substituting
inconveniently away from the eye. Thus, these values in the lens equation gives,
reading or viewing smaller things held in
1 1 1
the hands is difficult for them. This kind of = −
f − y −25 cm
farsightedness arising due to aging is called
presbyopia as the aged people cannot strain Simplifying the above equation gives,
their eye more to reduce the focal length of 1 1 1 y − 25 cm
= − =
the eye lens. f 25 cm y y ×25 cm

y
25 cm 25 cm
y
(a) (b) (c)

Figure 6.94  Hypermetropic eye and correction

Unit 6  OPTICS 91

UNIT-6(XII-Physics_Vol-2).indd 91 7/31/2019 8:53:31 PM


y ×25 cm Solution
f= (6.197)
y − 25 cm The maximum distance the person could
The focal length calculated using above see is, x = 1.8 m.
formula will be positive as y is always The lens should have a focal length of,
greater than 25 cm. The positive sign of the f = –x m = –1.8 m.
focal length suggests that the lens should be
It is a concave or diverging lens.
a convex lens. In principle, the convex lens
1
slightly converges the rays coming from The power of the lens is, P = − 1.8 m = −0.56
beyond y and makes them focus now at the diopter
retina which got earlier focused beyond
retina for the unaided eye.
6.13.6.3 Astigmatism EX AM P L E 6 . 4 5
Astigmatism is the defect arising due to
A person has farsightedness with the
different curvatures along different planes
minimum distance he could see clearly
in the eye lens. Astigmatic person cannot see
is 75 cm. Calculate the power of the lens
all the directions equally well. The defect due
of the spectacles necessary to rectify the
to astigmatism is more serious than myopia
defect.
and hyperopia. The remedy to astigmatism
is using of lenses with different curvatures Solution
in different planes to rectify the defect. In
The minimum distance the person could
general, these specially made glasses with
see clearly is, y = 75 cm.
different curvature for different planes are
called as cylindrical lenses. The lens should have a focal length of,
Due to aging people may develop y ×25 cm
combination of more than one defect. If it f=
y − 25 cm
is the combination of nearsightedness and
farsightedness then, such persons may need 75 cm×25 cm
f= = 37.5 cm
a converging glass for reading purpose and 75 cm − 25 cm
a diverging glass for seeing at a distance. It is a convex or converging lens.
Bifocal lenses and progressive lenses provide 1
solution for these problems. The power of the lens is, P = 0.375 m = 2.67
diopter

EXA MP LE 6 .4 4
Calculate the power of the lens of the
spectacles necessary to rectify the defect of
nearsightedness for a person who could see
clearly only up to a distance of 1.8 m.

92 Unit 6  OPTICS

UNIT-6(XII-Physics_Vol-2).indd 92 7/31/2019 8:54:10 PM


SUMMARY

„„ In ray optics, light is treated as a ray in the direction of light.


„„ Light undergoes reflection at polished surfaces and it is governed by laws of reflection.
„„ In general, plane mirrors form virtual and laterally inverted images at equal distance
inside the mirror.
„„ The height of plane mirror needed to see a person fully in a mirror is half of the
height of person.
„„ Spherical mirrors form a part of a sphere.
„„ Paraxial rays are the rays travelling close to the principle axis of the mirror and make
small angles withit.
„„ There is a relation between f and R in spherical mirrors for paraxial rays.
„„ Image formation in spherical mirrors is based on mirror equation.
„„ There is a set of Cartesian sign conventions to be followed to trace image formed by
spherical mirrors.
„„ Light travels with lesser velocity in optically denser medium.
„„ Optical path is the equivalent path travelled in vacuum in the same time light travels
through a optically denser medium.
„„ The phenomenon of refraction is governed by laws of refraction (Snell’s law).
„„ The apparent depth is always lesser than actual depth.
„„ Refraction takes place in atmosphere due to different layers of air with varying
refractive indices.
„„ Total internal reflection takes place when light travels from denser to rarer medium
with the angle of incidence greater than critical angle.
„„ There are several applications of total internal reflection.
„„ A glass slab produces lateral displacement or shift of ray entering into it.
„„ Thin lenses are formed by two spherical refracting surfaces.
„„ The image tracing in thin lenses is done with the Cartesian sign conventions and
with the help of lens equation.
„„ Power and focal length are inverse to each other.
„„ There is effective focal length for lenses in contact and out of contact.
„„ Prism produces deviation on the incident ray.
„„ Angle of deviation depends on angle of prism, angle of incidents and refractive index
of material of prism.
„„ The refractive index of prism depends on angle of prism and angle of minimum
deviation.
„„ When white light travels through a medium, different colours travel with different
speeds leading to dispersion of light.
„„ Dispersive power is the measure of ability of the medium to disperse white light.
„„ Rainbow is formed by dispersion of light by droplets of water.

Unit 6  OPTICS 93

UNIT-6(XII-Physics_Vol-2).indd 93 7/31/2019 8:54:10 PM


„„ Light can be scattered by the particles present in atmosphere.
„„ The scattering of light by particles of size less than wavelength of light is called
Rayleigh scattering which is inversely proportional to fourth power of wavelength.
„„ If the scattering is by suspended dust particles whose size is greater than wavelength
of light, the scattering is independent of wavelength.
„„ There are four theories on light each explaining few aspects of light.
„„ Light has wave and particle nature.
„„ In wave optics we treat light propagating as a wavefront.
„„ Huygens’ principle explains the propagation of light as wavefront.
„„ Laws of reflection and refraction are proved by Huygens’ principle.
„„ In interference, two light waves are added to get varying intensities at different points.
„„ Coherent sources produce monochromatic light waves in phase or with constant
phase difference.
„„ Coherent sources are obtained by intensity division, wavefront division and real and
virtual images of light source.
„„ Young’s double slit uses wavefront division to obtain coherent sources.
„„ Interference with polychromatic (white) light produces coloured interference fringes.
„„ Thin films appear coloured due to interference of white light.
„„ Bending of light around sharp edges is called diffraction.
„„ There are two types of diffractions called Fresnel and Fraunhofer diffractions
„„ Diffraction takes place at single slit which has a width comparable to the wavelength
of light.
„„ Fresnel’s distance is the distance up to which ray optics is obeyed.
„„ Diffraction can also happen in grating which has multiple slits of thickness comparable
to wavelength of light used.
„„ Using diffraction grating and spectrometer wavelength of monochromatic light and
also different colours of polychromatic light can be determined.
„„ Resolution is the quality of image which is decided by diffraction effect and Rayleigh
criterion.
„„ Resolution is measured by the smallest distance which could be seen clearly without
the blur due to diffraction.
„„ Polarisation is restricting electric or magnetic field vibrations to one plane.
„„ Polarisation is obtained by selective absorption, reflection, double refraction and
scattering.
„„ Malus’ law gives the intensity of emerging light when a polarised light enters two
polaroids kept at an angle.
„„ Brewster’s law relates angle of polarisation and refractive index of the medium.
„„ Optically active crystals can be classified as uniaxial and biaxial crystals.
„„ When light enters in to optically active crystals, double refraction takes place.

94 Unit 6  OPTICS

UNIT-6(XII-Physics_Vol-2).indd 94 7/31/2019 8:54:10 PM


„„ In double refraction, ordinary ray obeys laws of refraction and extraordinary ray
does not obey laws of refraction.
„„ Nicol prism separates ordinary and extraordinary rays by transparent cement called
Canada balsam.
„„ Light scattered by molecules at perpendicular direction to the incident light is found
to be plane polarised.
„„ Single convex lens can act as a simple microscope when object is within the focal
length.
„„ Two focusing namely, near point focusing and normal focusing are possible.
„„ In near point focusing, the image is formed at 25 cm which is the distance of distinct
vision for normal eye. Whereas, in normal focusing the image is formed and infinity.
„„ To find magnification in near point focusing we use lateral magnification and for
normal focusing we use angular magnification.
„„ Resolution of microscope could be improved by using oil immersed eye piece.
„„ Compound microscope has improved magnification.
„„ Astronomical telescope has an eye piece of long focal length and eye piece of short
focal length.
„„ Terrestrial telescopes have one addition lens for producing erect image.
„„ Reflecting telescopes have advantages as well as disadvantages.
„„ Eye has three problems (i) nearsightedness, (ii) farsightedness, (iii) astigmatism.

Unit 6  OPTICS 95

UNIT-6(XII-Physics_Vol-2).indd 95 7/31/2019 8:54:11 PM


E V AL U A T I O N

Multiple choice questions 6. Stars twinkle due to,


1. The speed of light in an isotropic (a) reflection
medium depends on, (b) total internal reflection
(a) its intensity (c) refraction
(b)its wavelength (d) polarisation
(c) the nature of propagation 7. When a biconvex lens of glass having
(d) the motion of the source refractive index 1.47 is dipped in a
w.r.to medium liquid, it acts as a plane sheet of glass.
This implies that the liquid must have
2. A rod of length 10 cm lies along the
refractive index,
principal axis of a concave mirror of
focal length 10 cm in such a way that (a) less than one
its end closer to the pole is 20 cm away (b) less than that of glass
from the mirror. The length of the (c) greater than that of glass
image is, (AIPMT Main 2012) (d) equal to that of glass
(a) 2.5 cm (b) 5cm 8. The radius of curvature of curved
(c) 10 cm (d) 15cm surface at a thin planoconvex lens is 10
3. An object is placed in front of a convex cm and the refractive index is 1.5. If the
mirror of focal length of f and the plane surface is silvered, then the focal
maximum and minimum distance of length will be,
an object from the mirror such that the (a) 5 cm (b) 10 cm
image formed is real and magnified. (c) 15 cm (d) 20 cm
(IEE Main 2009)] 9. An air bubble in glass slab of refractive
(a) 2f and c (b) c and ∞ index 1.5 (near normal incidence) is 5
(c) f and O (d) None of these cm deep when viewed from one surface
4. For light incident from air onto a slab of and 3 cm deep when viewed from the
refractive index 2. Maximum possible opposite face. The thickness of the slab is,
angle of refraction is, (a) 8 cm (b) 10 cm
(a) 30o (b) 45o (c) 12 cm (d) 16 cm
(c) 60o (d) 90o 10. A ray of light travelling in a transparent
5. If the velocity and wavelength of light in medium of refractive index n falls, on a
air is Va and λa and that in water is Vw and surface separating the medium from air
λw, then the refractive index of water is, at an angle of incidents of 45o. The ray
V V can undergo total internal reflection
(a) w (b) a for the following n,
Va Vw
lw Va la
(a) n = 1.25 (b) n = 1.33
(c) (d) (c) n = 1.4 (d) n = 1.5
la Vw lw

96 Unit 6 OPTICS

UNIT-6(XII-Physics_Vol-2).indd 96 7/31/2019 8:54:27 PM


11. A plane glass is placed over a various (a) 400 Å (b) 500 Å
coloured letters (violet, green, yellow, (c) 600 Å (d) 700 Å
red) The letter which appears to be 17. A ray of light strikes a glass plate at an
raised more is, angle 60o. If the reflected and refracted
(a) red (b) yellow rays are perpendicular to each other,
(c) green (d) violet the refractive index of the glass is,
3
12. Two point white dots are 1 mm apart on (a) 3 (b) 2
a black paper. They are viewed by eye
3
of pupil diameter 3 mm approximately. (c) (d) 2
2
The maximum distance at which these 18. One of the of Young’s double slits is
dots can be resolved by the eye is, [take covered with a glass plate as shown
wavelength of light, λ = 500 nm] in figure. The position of central
(a) 1 m (b) 5 m maximum will,
(c) 3 m (d) 6m
Glass slide Screen
13. In a Young’s double-slit experiment, the
slit separation is doubled. To maintain
the same fringe spacing on the screen,
the screen-to-slit distance D must be
changed to,
D
(a) 2D (b)
2
D (a) get shifted downwards 
(c) 2 D (d)
2 (b) get shifted upwards
14. Two coherent monochromatic light
beams of intensities I and 4I are (c) will remain the same
superposed. The maximum and (d) data insufficient to conclude
minimum possible intensities in the 19. Light transmitted by Nicol prism is,
resulting beam are [IIT-JEE 1988] (a) partiallypolarised
(a) 5I and I (b) 5I and 3I (b) unpolarised
(c) 9I and I (d) 9I and 3I (c) plane polarised
15. When light is incident on a soap film of (d) elliptically polarised
thickness 5×10–5 cm, the wavelength of 20. The transverse nature of light is shown
light reflected maximum in the visible in,
region is 5320 Å. Refractive index of
(a) interference
the film will be,
(b) diffraction
(a) 1.22 (b) 1.33
(c) scattering
(c) 1.51 (d) 1.83.
(d) polarisation
16. First diffraction minimum due to a
single slit of width 1.0×10–5 cm is at
30o. Then wavelength of light used is,

Unit 6  OPTICS 97

UNIT-6(XII-Physics_Vol-2).indd 97 7/31/2019 8:54:43 PM


Answers 19. Write a note on optical fibre.
1) b 2) b 3) d 4) a 5) b 20. Explain the working of an endoscope.
6) c 7) d 8) b 9) c 10) d 21. What are primary focus and secondary
11) d 12) b 13) a 14) c 15) b focus of concex lens?

16) b 17) a 18) b 19) c 20) d 22. What are the sign conventions followed
for lenses?
23. Arrive at lens equation from lens
Short Answer Questions
maker’s formula.
1. State the laws of reflection.
24. Obtain the equation for lateral
2. What is angle of deviation due to magnification for thin lens.
reflection?
25. What is power of a lens?
3. Give the characteristics of image
26. Derive the equation for effective focal
formed by a plane mirror.
length for lenses in contact.
4. Derive the relation between f and R for
27. What is angle of minimum deviation?
a spherical mirror.
28. What is dispersion?
5. What are the Cartesian sign conventions
29. How are rainbows formed?
for a spherical mirror?
30. What is Rayleigh’s scattering?
6. What is optical path? Obtain the
equation for optical path of a medium 31. Why does sky appear blue?
of thickness d and refractive index n. 32. What is the reason for reddish
7. State the laws of refraction. appearance of sky during sunset and
sunrise?
8. What is angle of deviation due to
refraction? 33. Why do clouds appear white?
9. What is principle of reversibility? 34. What are the salient features of
corpuscular theory of light?
10. What is relative refractive index?
35. What is wave theory of light?
11. Obtain the equation for apparent
depth. 36. What is electromagnetic wave theory
of light?
12. Why do stars twinkle?
37. Write a short note on quantum theory
13. What is critical angle and total internal
of light.
reflection?
38. What is a wavefront?
14. Obtain the equation for critical angle.
39. What is Huygens’ principle?
15. Explain the reason for glittering of
diamond. 40. What is interference of light?
16. What are mirage and looming? 41. What is phase of a wave?
17. Write a short notes on the prisms 42. Obtain the relation between phase
making use of total internal reflection. difference and path difference.
18. What is Snell’s window? 43. What are coherent sources?
44. What is intensity division?

98 Unit 6  OPTICS

UNIT-6(XII-Physics_Vol-2).indd 98 7/31/2019 8:54:43 PM


45. How does wavefront division provide 69. How is polarisation of light obtained
coherent sources? by scattering of light?
46. How do source and images behave as 70. Discuss about simple microscope and
coherent sources? obtain the equations for magnification
47. What is bandwidth of interference for near point focusing and normal
pattern? focusing.
48. What is diffraction? 71. What are near point and normal
49. Differentiate between Fresnel and focusing?
Fraunhofer diffraction. 72. Why is oil immersed objective
50. Discuss the special cases on first preferred in a microscope?
minimum in Fraunhofer diffraction. 73. What are the advantages and
51. What is Fresnel’s distance? Obtain the disadvantages of using a reflecting
equation for Fresnel’s distance. telescope?

52. Mention the differences between 74. What is the use of an erecting lens in a
interference and diffraction. terrestrial telescope?
53. What is a diffraction grating? 75. What is the use of collimator?
54. What are resolution and resolving 76. What are the uses of spectrometer?
power? 77. What is myopia? What is its remedy?
55. What is Rayleigh’s criterion? 78. What is hypermetropia? What is its
56. What is polarisation? remedy?

57. Differentiate between polarised and 79. What is presbyopia?


unpolarised light 80. What is astigmatism?
58. Discuss polarisation by selective
absorption. Long Answer Questions
59. What are polariser and analyser? 1. Derive the mirror equation and the
60. What are plane polarised, unpolarised equation for lateral magnification.
and partially polarised light? 2. Describe the Fizeau’s method to
61. State and obtain Malus’ law. determine speed of light.
62. List the uses of polaroids. 3. Obtain the equation for radius of
63. State Brewster’s law. illumination (or) Snell’s window.
64. What is angle of polarisation and obtain 4. Derive the equation for acceptance
the equation for angle of polarisation. angle and numerical aperture, of
optical fiber.
65. Discuss about pile of plates.
5. Obtain the equation for lateral
66. What is double refraction?
displacement of light passing through
67. Mention the types of optically active a glass slab.
crystals with example.
6. Derive the equation for refraction at
68. Discuss about Nicol prism. single spherical surface.

Unit 6  OPTICS 99

UNIT-6(XII-Physics_Vol-2).indd 99 7/31/2019 8:54:43 PM


7. Obtain lens maker’s formula and 22. Obtain the equation for resolving
mention its significance. power of optical instrument.
8. Derive the equation for thin lens and 23. Discuss about simple microscope and
obtain its magnification. obtain the equations for magnification
9. Derive the equation for effective focal for near point focusing and normal
length for lenses in out of contact. focusing.
10. Derive the equation for angle of 24. Explain about compound microscope
deviation produced by a prism and and obtain the equation for
thus obtain the equation for refractive magnification.
index of material of the prism. 25. Obtain the equation for resolving
11. What is dispersion? Obtain the power of microscope.
equation for dispersive power of a 26. Discuss about astronomical telescope.
medium. 27. Mention different parts of spectrometer
12. P
rove laws of reflection using and explain the preliminary
Huygens’ principle. adjustments.
13. Prove laws of refraction using Huygens’ 28. Explain the experimental
principle. determination of material of the prism
14. Obtain the equation for resultant using spectrometer.
intensity due to interference of light.
15. Explain the Young’s double slit Conceptual Questions:
experimental setup and obtain the 1. Why are dish antennas curved?
equation for path difference.
2. What type of lens is formed by a bubble
16. Obtain the equation for bandwidth in inside water?
Young’s double slit experiment.
3. It is possible for two lenses to produce
17. Obtain the equations for constructive zero power?
and destructive interference for
4. Why does sky look blue and clouds
transmitted and reflected waves in thin
look white?
films.
5. Why is yellow light preferred to during
18. Discuss diffraction at single slit and
fog?
obtain the condition for nth minimum.
6. Two independent monochromatic
19. Discuss the diffraction at a grating
sources cannot act as coherent sources,
and obtain the condition for the mth
why?
maximum.
7. Does diffraction take place at the
20. Discuss the experiment to determine
Young’s double slit?
the wavelength of monochromatic
8. Is there any difference between colored
light using diffraction grating.
light obtained from prism and colours
21. Discuss the experiment to determine
of soap bubble?
the wavelength of different colours
using diffraction grating.

100 Unit 6  OPTICS

UNIT-6(XII-Physics_Vol-2).indd 100 7/31/2019 8:54:43 PM


9. A small disc is placed in the path of the index n, it acts as a divergent lens of
light from distance source. Will the focal length 100 cm. What must be the
center of the shadow be bright or dark? value of n?
10. When a wave undergoes reflection at  [Ans: 5/3]
a denser medium, what happens to its 6. If the distance D between an object
phase? and screen is greater than 4 times the
focal length of a convex lens, then there
Numerical Problems are two positions of the lens for which
images are formed on the screen.
1. An object is placed at a certain distance
This method is called conjugate foci
from a convex lens of focal length 20 method. If d is the distance between
cm. Find the distance of the object if the two positions of the lens, obtain the
the image obtained is magnified 4 equation for focal length of the convex
times.  lens.
 [Ans: –15 cm.]  D 2 − d 2 
2. A compound microscope has a [Ans:  f = 
 4 D 
magnification of 30. The focal length 7. A beam of light of wavelength 600 nm
of eye piece is 5 cm. Assuming the from a distant source falls on a single
final image to be at least distance of slit 1 mm wide and the resulting
distinct vision, find the magnification diffraction pattern is observed on a
produced by the objective. screen 2 m away. What is the distance
 [Ans: 5] between the first dark fringe on either
3. An object is placed in front of a concave side of the central bright fringe?
mirror of focal length 20 cm. The  [Ans: 2.4 mm]
image formed is three times the size 8. In Young’s double slit experiment,
of the object. Calculate two possible the slits are 2 mm apart and are
distances of the object from the mirror. illuminated with a mixture of two
 [Ans: –40/3 cm and –80/3 cm] wavelength λ0 = 750 nm and λ = 900 nm.
4. A small bulb is placed at the bottom of What is the minimum distance from
a tank containing water to a depth of the common central bright fringe on
80 cm.What is the area of the surface a screen 2 m from the slits where a
of water through which light from the bright fringe from one interference
bulb can emerge out? Refractive index pattern coincides with a bright fringe
of water is 1.33. (Consider the bulb to from the other?
be a point source.)  [Ans: 4.5 mm]
 [Ans: 2.6 m ]
2
9. In Young’s double slit experiment, 62
5. A thin converging glass lens made fringes are seen in visible region for
of glass with refractive index 1.5 has sodium light of wavelength 5893 Å.
a power of + 5.0 D. When this lens If violet light of wavelength 4359 Å
is immersed in a liquid of refractive is used in place of sodium light, then

Unit 6  OPTICS 101

UNIT-6(XII-Physics_Vol-2).indd 101 7/31/2019 8:54:51 PM


what is the number of fringes seen? and the tube length is 6.5 cm. What is
[Ans: 84] the focal length of the eyepiece.
10. A compound microscope has a  [Ans: 2 cm]
magnifying power of 100 when
the image is formed at infinity. The
objective has a focal length of 0.5 cm

BOOKS FOR REFERENCE

1. Frances A. Jenkins and Harvey E. White, Fundamentals of Optics, 4th Edition, McGraw
Hill Book Company, (2011).
2. David Halliday, Robert Resnick and Jearl Walker,Fundamentals of Physics, 6th Edition,John
Wiley & Sons Inc., (2004).
3. H.C. Verma,Concepts of Physics [Part-1], 1st Edition, Bharathi Bhawan Publishers &
Distributers Pvt. Ltd., (2008).
4. Roger A. Freedman, Hugh D. Young, Sears and Zemansky’s University Physics, 12th
Edition, Pearson, (2011).

102 Unit 6  OPTICS

UNIT-6(XII-Physics_Vol-2).indd 102 7/31/2019 8:54:51 PM


CONCEPT MAP

Optics

Ray Optics

Reflection Refraction Optical


instruments

Refractive
Spherical mirror index – Real depth and Microscope Telescope
apparent depth

Mirror formula Simple Compound Refracting Reflecting


and magnification microscope microscope telescope telescope

Spherical Lens makers Lens formula Total internal


Dispersion and Power of lens
refracting formula and reflections and its
prism formula and its
surface magnification applications combination

Wave Optics

Huygens
principle Interference Diffraction Polarisation

Verification Principle of Single slit diffraction


superposition Brewsters law
laws of and expression for
reflection and width of central maxima
refraction
Young’s double
slit experiment Types of polarisation
Difference between
interference and
diffraction Polarisation Polarisation
by refraction by scattering

Application
of polarisation

Unit 6  OPTICS 103

UNIT-6(XII-Physics_Vol-2).indd 103 7/31/2019 8:54:51 PM


ICT CORNER
Optics

In this activity you will be able to explore


the behaviour of a Young’s double slit
experiment by adjusting the slit separation,
Topic: Young’s double slit
the distance to the screen, and the wave-
experiment.
length of the light.

STEPS:
• Open the browser and type
• ‘tutor-homework.com/Physics_Help/double_slit_experiment.html’ in the address bar.
• Change the slit separation (distance between two sources) andobservehow the pattern of
bright and dark fringes changes.
• What happens to the fringe width if distance between the source and screen decreases?
• Observe how does the fringe width in interference pattern vary with the wavelength of
incident light?
Observe the pattern of bright and dark fringes by clicking the Run button.

Step1 Step2

Step3 Step4

Note:
Use flash enabled browser or install flash player in your system.
URL:
http://tutor-homework.com/Physics_Help/double_slit_experiment.html
* Pictures are indicative only.
* If browser requires, allow Flash Player or Java Script to load the page.

104 Unit 6 OPTICS

UNIT-6(XII-Physics_Vol-2).indd 104 7/31/2019 8:54:53 PM


UNIT
DUAL NATURE OF RADIATION
7 AND MATTER

“If quantum mechanics has not profoundly shocked you, you have not understood it yet”
– Neils Bohr

In this unit, the students are exposed to


• the phenomenon of electron emission and its types
• the observations of Hertz, Hallwachs and Lenard
• photoelectric effect and its laws
• the concept of quantization of energy
• photo cell and its applications
• particle nature of radiation
• the wave nature of matter
• de Broglie equation and de Broglie wavelength of electron
• the construction and working of electron microscope
• Davisson and Germer experiment
• X-rays and its production
• X-rays spectra and its types

7.1 Classical physics which describes the


motion of the macroscopic objects treats
INTRODUCTION particles and waves as separate components
of physical reality. The mechanics of particles
and the optics of waves are traditionally
We are familiar with the concepts of independent subjects, each with its own
particle and wave in our everyday experience. experiments and principles.
Marble balls, grains of sand, atoms, electrons Electromagnetic radiations are regarded
and so on are some examples of particles while as waves because they exhibit wave
the examples of waves are sea waves, ripples in phenomena such as interference, diffraction
a pond, sound waves and light waves. and polarization under some suitable
Particle is a material object which is circumstances. Similarly, under other
considered as a tiny concentration of matter circumstances like black body radiation
(localized in space and time) whereas and photo electric effect, electromagnetic
wave is a broad distribution of energy radiations behave as though they consist of
(not localized in space and time). They, stream of particles.
both particles and waves, have the ability When electrons, protons and other
to carry energy and momentum from one particles are discovered, they are considered
place to another. as particles because they possess mass and
105

UNIT-7(XII-Physics_Vol-2)_Word flow.indd 105 7/31/2019 7:20:59 PM


charge. However, later experiments showed The minimum energy needed for an
that under certain circumstance, they electron to escape from the metal surface
exhibit wave-like properties. is called work function of that metal. The
In this unit, the particle nature of waves work function of the metal is denoted by φ0
(radiation) and then the wave nature of and is measured in electron volt (eV).
particles (matter) – that is, wave-particle
duality of radiation and matter are discussed
with the relevant experimental observations Note The SI unit of energy is
which support this dual nature. joule. But electron volt is
a commonly used unit of
energy in atomic and nuclear physics.
7.1.1 Electron emission
One electron volt is defined as the
In metals, the electrons in the outer most
kinetic energy gained by an electron
shells are loosely bound to the nucleus.
when accelerated by a potential
Even at room temperature, there are a large
difference of 1 V.
number of free electrons which are moving
inside the metal in a random manner. 1 eV = KE gained by the electron
Though they move freely inside the metal, = Work done by the electric field
they cannot leave the surface of the metal.
=qV
The reason is that when free electrons reach
the surface of the metal, they are attracted = 1.602 × 10–19 C × 1 V
by the positive nuclei of the metal. It is this = 1.602 × 10–19 J
attractive pull which will not allow free
electrons to leave the metallic surface at
room temperature. Suppose the maximum kinetic energy
In order to leave the metallic surface, the of the free electron inside the metal is
free electrons must cross a potential barrier 0.5 eV and the energy needed to overcome
created by the positive nuclei of the metal. the surface barrier of a metal is 3 eV, then
The potential barrier which prevents free the minimum energy needed for electron
electrons from leaving the metallic surface emission from the metallic surface is 3 – 0.5
is called surface barrier. = 2.5 eV. Here 2.5 eV is the work function
Free electrons possess some kinetic of the metal.
energy and this energy is different for The work function is different for
different electrons. The kinetic energy of the different metals and is a typical property of
free electrons is not sufficient to overcome metals and the nature of their surface. Table
the surface barrier. Whenever an additional 7.1 gives the approximate value of work
energy is given to the free electrons, they will function for various metals. The material
have sufficient energy to cross the surface with smaller work function is more effective
barrier. And they escape from the metallic in electron emission because extra energy
surface. The liberation of electrons from required to release the free electrons from
any surface of a substance is called electron the metal surface is smaller.
emission.

106 Unit 7 DUAL NATURE OF R ADIATION AND MAT TER

UNIT-7(XII-Physics_Vol-2)_Word flow.indd 106 7/31/2019 7:20:59 PM


Table 7.1  Work function of some materials

Work function Work function


Metal Symbol Metal Symbol
(eV) (eV)

Cesium Cs 2.14 Aluminium Al 4.28

Potassium K 2.30 Copper Cu 4.65

Sodium Na 2.75 Silver Ag 4.70

Calcium Ca 3.20 Platinum Pt 5.65

So the metal selected for electron


emission should have low work function.
The electron emission is categorized into
different types depending upon the form of
energy being utilized. There are mainly four
types of electron emission which are given
below.
Hot filament Electrons
i)  Thermionic emission
When a metal is heated to a high Figure 7.2  Thermionic emission from hot
temperature, the free electrons on the surface filament of cathode ray tube or X-ray tube
of the metal get sufficient energy in the form
of thermal energy so that they are emitted
from the metallic surface (Figure 7.1). This ii)  Field emission
type of emission is known as thermionic Electric field emission occurs when a
emission. very strong electric field is applied across
the metal. This strong field pulls the free
Metal Electrons liberated electrons and helps them to overcome the
surface barrier of the metal (Figure 7.3).
Examples: Field emission scanning electron
microscopes, Field-emission display etc.
Electrons Thermal energy
(a) (b)
Electrons emitted
Figure 7.1  Electrons in the (a) metal – +
(b) heated metal – +
– +
– +
The intensity of the thermionic emission – +
– +
(the number of electrons emitted) depends – +
on the metal used and its temperature. Metal Strong electric field
Examples: cathode ray tubes, electron
Figure 7.3  Field emission
microscopes, X-ray tubes etc (Figure 7.2).
Unit 7  DUAL NATURE OF R ADIATION AND MAT TER 107

UNIT-7(XII-Physics_Vol-2)_Word flow.indd 107 7/31/2019 7:21:00 PM


iii)  Photo electric emission 7.2
When an electromagnetic radiation of
suitable frequency is incident on the surface PHOTO ELECTRIC EFFECT
of the metal, the energy is transferred from
the radiation to the free electrons. Hence, the
7.2.1  Hertz, Hallwachs and
free electrons get sufficient energy to cross
Lenard’s observation
the surface barrier and the photo electric
emission takes place (Figure 7.4). The Hertz observation
number of electrons emitted depends on the Maxwell’s theory of electromagnetism
intensity of the incident radiation. Examples: predicted the existence of electromagnetic
Photo diodes, photo electric cells etc. waves and concluded that light itself is
just an electromagnetic wave. Then the
Radiation
Emitted electrons experimentalists tried to generate and detect
electromagnetic waves through various
experiments.
In 1887, Heinrich Hertz first became
successful in generating and detecting
electromagnetic wave with his high voltage
induction coil to cause a spark discharge
Free electrons Metal between two metallic spheres (we have
Figure 7.4  Photo electric emission learnt this in Unit 5 of XII standard physics).
When a spark is formed, the charges will
oscillate back and forth rapidly and the
iv)  Secondary emission
electromagnetic waves are produced.
When a beam of fast moving electrons
The electromagnetic waves thus
strikes the surface of the metal, the kinetic
produced were detected by a detector that
energy of the striking electrons is transferred
has a copper wire bent in the shape of a
to the free electrons on the metal surface.
circle. Although the detection of waves is
Thus the free electrons get sufficient kinetic
successful, there is a problem in observing
energy so that the secondary emission of
the tiny spark produced in the detector.
electron occurs (Figure 7.5). Examples:
In order to improve the visibility of
Image intensifiers, photo multiplier tubes etc.
the spark, Hertz made many attempts and
High Speed
p d electrons
e Secondary electrons finally noticed an important thing that small
detector spark became more vigorous when
it was exposed to ultraviolet light.
The reason for this behaviour of the
spark was not known at that time. Later it
was found that it is due to the photoelectric
emission. Whenever ultraviolet light is
Free electrons Metal incident on the metallic sphere, the electrons
Figure 7.5  Secondary emission of electrons on the outer surface are emitted which
caused the spark to be more vigorous.
108 Unit 7  DUAL NATURE OF R ADIATION AND MAT TER

UNIT-7(XII-Physics_Vol-2)_Word flow.indd 108 7/31/2019 7:21:00 PM


Lenard’s observation
It is interesting to note that
In 1902, Lenard studied this electron
the experiment of Hertz
emission phenomenon in detail. His simple
confirmed that light is an
electromagnetic wave. But the same experimental setup is as shown in Figure
experiment also produced the first 7.7. The apparatus consists of two metallic
evidence for particle nature of light. plates A and C placed in an evacuated quartz
bulb. The galvanometer G and battery B are
Hallwachs’ observation connected in the circuit.
In 1888, Wilhelm Hallwachs, a German
Ultraviolet Radiation
physicist, confirmed that the strange
behaviour of the spark is due to the action of Electrons
ultraviolet light with his simple experiment. G
A clean circular plate of zinc is mounted
C A
on an insulating stand and is attached to
a gold leaf electroscope by a wire. When
the uncharged zinc plate is irradiated by Quartz bulb
ultraviolet light from an arc lamp, it becomes
_ + I
positively charged and the leaves will open B
as shown in Figure 7.6(a).
Figure 7.7  Experimental setup of Lenard
Further, if the negatively charged zinc
plate is exposed to ultraviolet light, the leaves
When ultraviolet light is incident on
will close as the charges leaked away quickly
the negative plate C, an electric current
(Figure 7.6(b)). If the plate is positively
flows in the circuit that is indicated by the
charged, it becomes more positive upon
deflection in the galvanometer. On other
UV rays irradiation and the leaves will
hand, if the positive plate is irradiated by
open further (Figure 7.6(c)). From these
the ultraviolet light, no current is observed
observations, it was concluded that negatively
in the circuit.
charged electrons were emitted from the zinc
From these observations, it is concluded
plate under the action of ultraviolet light.
that when ultraviolet light falls on the negative
UV rays plate, electrons are ejected from it which are
attracted by the positive plate A. On reaching
Zn + + + + + Zn –––––––

Zn +++++++
+
the positive plate through the evacuated bulb,
+ – +
the circuit is completed and the current flows
+ – +
– +
– ––
in it. Thus, the ultraviolet light falling on the
++ + +
–– –– + +
+ +
+ – –– +
+ +
+ negative plate causes the electron emission
+ –– + +
from the surface of the plate.
(a) (b) (c)
Photoelectric effect
Figure 7.6  Irradiation of ultraviolet light The ejection of electrons from a
on (a) uncharged zinc plate (b) negatively
metal plate when illuminated by light
charged plate (c) positively charged plate
or any other electromagnetic radiation
Unit 7  DUAL NATURE OF R ADIATION AND MAT TER 109

UNIT-7(XII-Physics_Vol-2)_Word flow.indd 109 7/31/2019 7:21:00 PM


of suitable wavelength (or frequency) is the cathode (negative electrode) made up of
called photoelectric effect. Although these photosensitive material and is used to emit
electrons are not different from all other electrons. The anode (positive electrode) A
electrons, it is customary to call them as collects the electrons emitted from C. These
photoelectrons and the corresponding electrodes are taken in an evacuated glass
current as photoelectric current or photo envelope with a quartz window that permits
current. the passage of ultraviolet and visible light.
Metals like cadmium, zinc, magnesium The necessary potential difference
etc show photoelectric emission for between C and A is provided by high
ultraviolet light while some alkali metals tension battery B which is connected across
lithium, sodium, caesium respond well a potential divider arrangement PQ through
even to larger wavelength radiation a key K. C is connected to the centre
like visible light. The materials which terminal while A to the sliding contact J
eject photoelectrons upon irradiation of of the potential divider. The plate A can be
electromagnetic wave of suitable wavelength maintained at a desired positive or negative
are called photosensitive materials. potential with respect to C. To measure
both positive and negative potential of A
7.2.2  Effect of intensity of with respect to C, the voltmeter is designed
incident light on photoelectric to have its zero marking at the centre and is
current connected between A and C. The current is
measured by a micro ammeter mA in series.
Experimental setup If there is no light falling on the cathode
The apparatus shown in Figure 7.8 is C, no photoelectrons are emitted and the
employed to study the phenomenon of microammeter reads zero. When ultraviolet
photoelectric effect in detail. S is a source or visible light is allowed to fall on C,
of electromagnetic waves of known and the photoelectrons are liberated and are
variable frequency ν and intensity I. C is attracted towards anode. As a result, the

Quartz window S

Evacuated
Photo sensitive glass tube
plate
µA

C A P
K
J +
Photo electrons B
I V _
Q

Figure 7.8  Experimental setup for the study of photoelectric effect

110 Unit 7  DUAL NATURE OF R ADIATION AND MAT TER

UNIT-7(XII-Physics_Vol-2)_Word flow.indd 110 7/31/2019 7:21:01 PM


photoelectric current is set up in the circuit
which is measured using micro ammeter. Here, intensity of light means
Note
The variation of photocurrent with brightness. A bright light has
respect to (i) intensity of incident light more intensity than a dim
(ii) the potential difference between the light.
electrodes (iii) the nature of the material
and (iv) frequency of incident light can be
studied with the help of this apparatus. 7.2.3 Effect of potential
difference on photoelectric
Effect of intensity of incident light on current
photoelectric current To study the effect of potential difference
To study the effect of intensity of incident V between the electrodes on photoelectric
light on photoelectric current, the frequency current, the frequency and intensity of the
of the incident light and the accelerating incident light are kept constant. Initially the
potential V of the anode are kept constant. potential of A is kept positive with respect
Here the potential of A is kept positive with to C and the cathode is irradiated with the
respect to that of C so that the electrons given light.
emitted from C are attracted towards A. Now, the potential of A is increased and
Now, the intensity of the incident light is the corresponding photocurrent is noted. As
varied and the corresponding photoelectric the potential of A is increased, photocurrent
current is measured. is also increased. However a stage is reached
where photocurrent reaches a saturation
value (saturation current) at which all the
Frequency and
Accelerating photoelectrons from C are collected by
potential- constant A. This is represented by the flat portion
Photo electric current

of the graph between potential of A and


photocurrent (Figure 7.10).
When a negative (retarding) potential
is applied to A with respect to C, the
current does not immediately drop to zero
because the photoelectrons are emitted
with some definite and different kinetic
Light intensity, I energies. The kinetic energy of some of
the photoelectrons is such that they could
Figure 7.9 Variation of photocurrent overcome the retarding electric field and
with intensity
reach the electrode A.
A graph is drawn between light intensity When the negative (retarding)
along x-axis and the photocurrent along y-axis. potential of A is gradually increased, the
From the graph in Figure 7.9, it is evident that photocurrent starts to decrease because
photocurrent – the number of electrons emitted more and more photoelectrons are being
per second – is directly proportional to the repelled away from reaching the electrode
intensity of the incident light. A. The photocurrent becomes zero at a
Unit 7 DUAL NATURE OF R ADIATION AND MAT TER 111

UNIT-7(XII-Physics_Vol-2)_Word flow.indd 111 7/31/2019 7:21:01 PM


particular negative potential V0, called From equation (7.1),
stopping or cut-off potential.
K max = eV0 (in joule) (7.3)
(or)

Photo current
K max = V0 (in eV ) 
I3 > I2 > I1
I3
From the Figure 7.10, when the intensity
I2 of the incident light alone is increased, the
I1 saturation current also increases but the
Stopping potential value of V0 remains constant.
Thus, for a given frequency of the incident
light, the stopping potential is independent
_V 0 Collecting electrode
0
Retarding potential
of intensity of the incident light. This also
potential
implies that the maximum kinetic energy of
the photoelectrons is independent of intensity
Figure 7.10  Variation of photocurrent of the incident light.
with potential difference
Stopping potential is that the value of 7.2.4  Effect of frequency
the negative (retarding) potential given of incident light on stopping
to the collecting electrode A which is potential
just sufficient to stop the most energetic To study the effect of frequency of incident
photoelectrons emitted and make the light on stopping potential, the intensity of the
photocurrent zero. incident light is kept constant. The variation
At the stopping potential, even the of photocurrent with the collector electrode
most energetic electron is brought to rest. potential is studied for radiations of different
Therefore, the initial kinetic energy of the frequencies and a graph drawn between them
fastest electron (Kmax) is equal to the work is shown in Figure 7.11. From the graph,
done by the stopping potential to stop it it is clear that stopping potential vary over
(eV0). different frequencies of incident light.
1
K max = mυ2max = eV0 (7.1) Photo current
2 Saturation current
where υmax is the maximum speed of the
ν3 > ν2 > ν1
emitted photoelectron.
ν3
ν2
ν1
2eV0
υmax =
m –V03 –V02 –V01 0 Collecting electrode
Retarding potential potential
2×1.602×10−19
υmax = ×V0
9.1×10−31 Figure 7.11  Variation of photocurrent with
collector electrode potential for different
= 5.93×105 V0 (7.2) frequencies of the incident radiation


112 Unit 7  DUAL NATURE OF R ADIATION AND MAT TER

UNIT-7(XII-Physics_Vol-2)_Word flow.indd 112 7/31/2019 7:21:50 PM


Greater the frequency of the incident of the incident light. The saturation
radiation, larger is the corresponding current is also directly proportional to
stopping potential. This implies that as the the intensity of incident light.
frequency is increased, the photoelectrons are ii) Maximum kinetic energy of the photo
emitted with greater kinetic energies so that electrons is independent of intensity of
the retarding potential needed to stop the the incident light.
photoelectrons is also greater. iii) Maximum kinetic energy of the photo
electrons from a given metal is directly
proportional to the frequency of
Metal A Metal B
incident light.
iv) For a given surface, the emission of
Stopping ν ν0A > photoelectrons takes place only if the
potential
ν >ν0B frequency of incident light is greater
than a certain minimum frequency
ν0A ν0B called the threshold frequency.
Frequency of
incident radiation v) There is no time lag between incidence
of light and ejection of photoelectrons.
Figure 7.12  Variation of stopping Once photoelectric phenomenon has
potential with frequency of the incident been thoroughly examined through various
radiation for two metals
experiments, the attempts were made to
explain it on the basis of wave theory of
Now a graph is drawn between frequency light.
and the stopping potential for different
metals (Figure 7.12). From this graph, it is 7.2.6  Concept of
found that stopping potential varies linearly quantization of energy
with frequency. Below a certain frequency
called threshold frequency, no electrons Failures of classical wave theory
are emitted; hence stopping potential is From Maxwell’s theory (Refer unit 5
zero for that reason. But as the frequency of volume 1), we learnt that light is an
is increased above threshold value, the electromagnetic wave consisting of coupled
stopping potential varies linearly with the electric and magnetic oscillations that move
frequency of incident light. with the speed of light and exhibit typical
wave behaviour. Let us try to explain the
experimental observations of photoelectric
7.2.5  Laws of photoelectric
effect using wave picture of light.
effect
i) When light is incident on the target,
The above detailed experimental there is a continuous supply of energy to the
investigations of photoelectric effect electrons. According to wave theory, light
revealed the following results: of greater intensity should impart greater
i) For a given frequency of incident light, kinetic energy to the liberated electrons
the number of photoelectrons emitted (Here, Intensity of light is the energy
is directly proportional to the intensity delivered per unit area per unit time).
Unit 7  DUAL NATURE OF R ADIATION AND MAT TER 113

UNIT-7(XII-Physics_Vol-2)_Word flow.indd 113 7/31/2019 7:21:50 PM


But this does not happen. The (Given : The work function for cesium
experiments show that maximum kinetic is 1.90 eV and the power absorbed per unit
energy of the photoelectrons does not area is 1.60×10−6 Wm−2 which produces a
depend on the intensity of the incident light. measurable photocurrent in cesium.)
ii) According to wave theory, if a Solution
sufficiently intense beam of light is incident
on the surface, electrons will be liberated i) According to wave theory, the energy
from the surface of the target, however low in a light wave is spread out uniformly
the frequency of the radiation is. and continuously over the wavefront.
From the experiments, we know that For the sake of simplicity, the following
photoelectric emission is not possible below assumptions are made.
a certain minimum frequency. Therefore, a) Light is absorbed in the top atomic
the wave theory fails to explain the existence layer of the metal
of threshold frequency. b) For a given element, each atom
iii) Since the energy of light is spread absorbs an equal amount of energy
across the wavefront, the electrons which and this energy is proportional to its
receive energy from it are large in number. cross-sectional area A
Each electron needs considerable amount of c) Each atom gives this energy to one of
time (a few hours) to get energy sufficient the electrons.
to overcome the work function and to get The energy absorbed by each electron
liberated from the surface. in time t is given by
But experiments show that photoelectric E = IAt
emission is almost instantaneous process
(the time lag is less than 10–9 s after the With this energy absorbed, the most
surface is illuminated) which could not be energetic electron is released with Kmax by
explained by wave theory. overcoming the surface energy barrier or
Thus, the experimental observations work function ϕ0 and this is expressed as
of photoelectric emission could not be K max = IAt − ϕ0 
(1)
explained on the basis of the wave theory of
light. Thus, wave theory predicts that for a
unit time, at low light intensities when
EXA MP LE 7 .1
IA < ϕ0, no electrons are emitted. At higher
For the photoelectric emission from cesium, intensities, when IA ≥ ϕ0, electrons are
show that wave theory predicts that emitted. This implies that higher the light
i)  maximum kinetic energy of the intensity, greater will be Kmax.
photoelectrons (Kmax) depends on the Therefore, the predictions of
intensity I of the incident light wave theory contradict experimental
ii)  Kmax does not depend on the observations at both very low and very
frequency of the incident light and high light intensities.
iii)  the time interval between the Kmax is dependent only on the intensity
incidence of light and the ejection of under given conditions – that is, by suitably
photoelectrons is very long. increasing the intensity, one can produce
114 Unit 7  DUAL NATURE OF R ADIATION AND MAT TER

UNIT-7(XII-Physics_Vol-2)_Word flow.indd 114 7/31/2019 7:21:58 PM


photoelectric effect even if the frequency radiations emitted by a black body and the
is less than the threshold frequency. So the shape of its radiation curves.
concept of threshold frequency does not According to Planck, matter is composed
even exist in wave theory. of a large number of oscillating particles
ii)  According to wave theory, the (atoms) which vibrate with different
intensity of a light wave is proportional to frequencies. Each atomic oscillator - which
the square of the amplitude of the electric vibrates with its characteristic frequency -
2
field (E0 ). The amplitude of this electric emits or absorbs electromagnetic radiation
field increases with increasing intensity of the same frequency. It also says that
and imparts an increasing acceleration and i) If an oscillator vibrates with frequency v,
kinetic energy to an electron. its energy can have only certain discrete
Now I is replaced with a quantity values, given by the equation.
proportional to E02 in equation (1). This
means that Kmax should not depend at all En= nhν   n=1,2,3....(7.4)
on the frequency of the classical light wave
where h is a constant, called Planck’s
which again contradicts the experimental
constant.
results.
ii) The oscillators emit or absorb energy in
iii)  If an electron accumulates light
small packets or quanta and the energy
energy just enough to overcome the work
of each quantum is E = hν.
function, then it is ejected out of the atom
This implies that the energy of the
with zero kinetic energy. Therefore, from
oscillator is quantized – that is, energy is not
equation (1),
continuous as believed in the wave picture.
0 = IAt − ϕ 0 This is called quantization of energy.
ϕ0 ϕ0
t= =
IA I(π r 2 ) 7.2.7  Particle nature of light:
Einstein’s explanation
By taking
the atomic radius
r = 1.0×10 m and substituting the given
−10 Einstein extended Planck’s quantum
values of I and ϕ0, we can estimate the time concept to explain the photoelectric effect
interval as in 1905. According to Einstein, the energy
1.90×1.6×10−19 in light is not spread out over wavefronts but
t= is concentrated in small packets or energy
1.60×10−6 ×3.14 ×(1×10−10 )2
quanta. Therefore, light (or any other
= 0.61×107 s ≈ 71 days electromagnetic waves) of frequency v from
Thus, wave theory predicts that there is any source can be considered as a stream of
a large time gap between the incidence of quanta and the energy of each light quantum
light and the ejection of photoelectrons but is given by E = hν.
the experiments show that photo emission He also proposed that a quantum of light
is an instantaneous process. has linear momentum and the magnitude

Concept of quantization of energy of that linear momentum is p = .. The
c
Max Planck proposed quantum concept individual light quantum of definite energy
in 1900 in order to explain the thermal and momentum can be associated with a
Unit 7  DUAL NATURE OF R ADIATION AND MAT TER 115

UNIT-7(XII-Physics_Vol-2)_Word flow.indd 115 7/31/2019 7:22:31 PM


particle. The light quantum can behave as a Einstein’s explanation of photoelectric
particle and this is called photon. Therefore, equation
photon is nothing but particle manifestation When a photon of energy hν is incident
of light. on a metal surface, it is completely
Characteristics of photons: absorbed by a single electron and the
According to particle nature of light, electron is ejected. In this process, a part
photons are the basic constituents of of the photon energy is used for the
any radiation and possess the following ejection of the electrons from the metal
characteristic properties: surface (photoelectric work function ϕ0 )
and the remaining energy as the kinetic
i) The photons of light of frequency ν and energy of the ejected electron. From the
wavelength λ will have energy, given by law of conservation of energy,
E = hv = hc . 1
l hv = ϕ 0 + mυ2
ii) The energy of a photon is determined 2 (7.6)
by the frequency of the radiation and
not by its intensity and the intensity where m is the mass of the electron and υ
has no relation with the energy of the its velocity. This is shown in Figure 7.13(a).
individual photons in the beam.
iii) The photons travel with the velocity of
light and its momentum is given by
E = hv E = hv0
p = h = hv
l c
iv) Since photons are electrically neutral,
they are unaffected by electric and Kmax = hv – hv0
magnetic fields. K=0
v) When a photon interacts with matter
Metal Metal
(photon-electron collision), the total
energy, total linear momentum and
angular momentum are conserved. Since Figure 7.13 Emission of photoelectrons
photon may be absorbed or a new photon
may be produced in such interactions, the
If we reduce the frequency of the incident
number of photons may not be conserved.
light, the speed or kinetic energy of photo
electrons is also reduced. At some frequency
According to quantum ν0 of incident radiation, the photo electrons
Note
concept, intensity of light of are ejected with almost zero kinetic energy
given wavelength is defined (Figure 7.13(b)). Then the equation (7.6)
as the number of energy quanta or becomes
photons incident per unit area per unit
time, with each photon having same hv0 = ϕ0
energy. The unit is Wm–2. where ν0 is the threshold frequency. By

116 Unit 7 DUAL NATURE OF R ADIATION AND MAT TER

UNIT-7(XII-Physics_Vol-2)_Word flow.indd 116 7/31/2019 7:22:41 PM


rewriting the equation (7.6), we get
Kmax

Po sium

Ca diu ium
1
hv = hv0 + m2 (7.7)

So tass
lci m
Ce
2 Slope = h

um
The equation (7.7) is known as Einstein’s O
Frequency
photoelectric equation. −2.14eV
If the electron does not lose energy by −2.30eV
internal collisions, then it is emitted with −2.75eV
maximum kinetic energy Kmax. Then
−3.20eV
1
K max = mυ2max
2 Figure 7.15  Kmax vs ν graph for different
where υmax is the maximum velocity of metals
the electron ejected. The equation (7.6) is
rearranged as follows: Millikan also calculated the value of
Planck’s constant (h=6.626×10–34 Js) and
Kmax = hv – ϕ0(7.8) work function of many metals (Cs, K, Na,
Ca); these values are in agreement with the
theoretical prediction.
Explanation for the photoelectric effect:
Kmax
The experimentally observed facts of
photoelectric effect can be explained with
the help of Einstein’s photoelectric equation.
i)  As each incident photon liberates one
hv
electron, then the increase of intensity of the
v0 v light (the number of photons per unit area per
hv0
Frequency unit time) increases the number of electrons
emitted thereby increasing the photocurrent.
The same has been experimentally observed.
Figure 7.14  Kmax vs ν graph ii)  From Kmax = hv – ϕ0, it is evident
that Kmax is proportional to the frequency of
A graph between maximum kinetic the light and is independent of intensity of
energy Kmax of the photoelectron and the light.
frequency ν of the incident light is a straight iii)  As given in equation (7.7), there
line as shown in Figure 7.14. The slope of must be minimum energy (equal to the
the line is h and its y-intercept is –ϕ0. work function of the metal) for incident
Einstein’s equation was experimentally photons to liberate electrons from the
verified by R.A. Millikan. He drew Kmax metal surface. Below which, emission of
versus ν graph for many metals (cesium, electrons is not possible. Correspondingly,
potassium, sodium and lithium) as shown there exists minimum frequency called
in Figure 7.15 and found that the slope is threshold frequency below which there is no
independent of the metals. photoelectric emission.
Unit 7  DUAL NATURE OF R ADIATION AND MAT TER 117

UNIT-7(XII-Physics_Vol-2)_Word flow.indd 117 7/31/2019 7:22:50 PM


iv) According to quantum concept, the 7.2.8  Photo electric cells and
transfer of photon energy to the electrons their applications
is instantaneous so that there is no time lag
between incidence of photons and ejection Photo cell
of electrons. Photo electric cell or photo cell is a device
Thus, the photoelectric effect is explained which converts light energy into electrical
on the basis of quantum concept of light. energy. It works on the principle of photo
The nature of light: wave - particle duality electric effect. When light is incident on
We have learnt that wave nature of light the photosensitive materials, their electric
explains phenomena such as interference, properties will get affected, based on which
diffraction and polarization. Certain photo cells are classified into three types.
phenomena like black body radiation, They are
photoelectric effect can be explained by i) Photo emissive cell: Its working
assigning particle nature to light. Therefore, depends on the electron emission from
both theories have enough experimental a metal cathode due to irradiation of
evidences. light or other radiations.
In the past, many scientific theories have ii) Photo voltaic cell: Here sensitive
been either revised or discarded when they element made of semiconductor is used
contradicted with new experimental results. which generates voltage proportional to
Here, two different theories are needed to the intensity of light or other radiations.
answer the question: what is nature of light? iii) Photo conductive cell: In this, the
It is therefore concluded that light resistance of the semiconductor changes
possesses dual nature, that of both particle in accordance with the radiant energy
and wave. It behaves like a wave at some incident on it.
circumstances and it behaves like a particle In this section, we discuss about photo
at some other circumstances. emissive cell and its applications.
In other words, light behaves as a wave Photo emissive cell
during its propagation and behaves as a Construction:
particle during its interaction with matter.
It consists of an evacuated glass
Both theories are necessary for complete
or quartz bulb in which two metallic
description of physical phenomena. Hence,
electrodes – that is, a cathode and an
the wave nature and quantum nature
anode are fixed as shown in Figure 7.16.
complement each other.

A reader may find it difficult to understand how light can be both a wave and a
stream of particle. This is the case even for great scientist like Albert Einstein.
Einstein once wrote a letter to his friend Michel Besso in 1954 expressing
his frustration:
“All these fifty years of conscious brooding have brought me no closer to answer the
question, ‘What are light quanta?’ Of course today everyone thinks he knows the answer,
but he is deluding himself ”.

118 Unit 7  DUAL NATURE OF R ADIATION AND MAT TER

UNIT-7(XII-Physics_Vol-2)_Word flow.indd 118 7/31/2019 7:22:50 PM


The cathode C is semi-cylindrical in that switch on and off according to whether
shape and is coated with a photo sensitive it is night or day.
material. The anode A is a thin rod or wire Photo cells are used for reproduction
kept along the axis of the semi-cylindrical of sound in motion pictures and are used
cathode. A potential difference is applied as timers to measure the speeds of athletes
between the anode and the cathode during a race. Photo cells of exposure meters
through a galvanometer G. in photography are used to measure the
intensity of the given light and to calculate
the exact time of exposure.
Radiation

EX AM P L E 7 . 2
A radiation of wavelength 300 nm is incident
C A
on a silver surface. Will photoelectrons be
observed?

Solution:
Energy of the incident photon is
hc
E = hv = (in joules)
λ
_
G hc
+ E= (in eV)
eV
λe
B Substituting the known values, we get
Figure 7.16  Construction of photo cell
6.626×10−34 ×3×108
E=
300×10−9 ×1.6×10−19
Working:
When cathode is illuminated, electrons E = 4.14 eV
are emitted from it. These electrons are
From Table 7.1, the work function
attracted by anode and hence a current
of silver = 4.7 eV. Since the energy of
is produced which is measured by the
the incident photon is less than the work
galvanometer. For a given cathode, the
function of silver, photoelectrons are not
magnitude of the current depends on
observed in this case.
i) the intensity to incident radiation and
ii) the potential difference between anode
and cathode. EX AM P L E 7 . 3
Applications of photo cells: When light of wavelength 2200Å falls on
Photo cells have many applications, Cu, photo electrons are emitted from it.
especially as switches and sensors. Find (i) the threshold wavelength and
Automatic lights that turn on when it gets (ii) the stopping potential. Given: the work
dark use photocells, as well as street lights function for Cu is ϕ0 = 4.7eV .
Unit 7  DUAL NATURE OF R ADIATION AND MAT TER 119

UNIT-7(XII-Physics_Vol-2)_Word flow.indd 119 7/31/2019 7:23:14 PM


Solution ii) The number of photons reaching the
i)  The threshold wavelength is given by surface per second is
P
hc 6.626×10− 34 ×3×108 np = ×A
λ0 = = E
ϕ0 4.7×1.6 ×10− 19 2
= × 2×10−4
= 2643 Å 6.626×10−19
= 6.04×1014 photons / sec
ii)  Energy of the photon of wavelength  
2200 Å is The rate of emission of photoelectrons is

hc 6.626×10−34 ×3×108   = (0.40) n p = 0.4× 6.04×1014


E= =
λ 2200×10−10   = 2.415´1014 photoelectrons / sec

= 9.035×10

−19
J = 5.65 eV
EX AM P L E 7 . 5
We know that kinetic energy of fastest
Light of wavelength 390 nm is directed at
photo electron is
a metal electrode. To find the energy of
Kmax = hv – ϕ0 = 5.65 – 4.7 electrons ejected, an opposing potential
= 0.95 eV difference is established between it
and another electrode. The current of
From equation (7.2), K max = eV0
photoelectrons from one to the other is
K max 0.95×1.6×10−19 stopped completely when the potential
V0 = =
e 1.6×10−19 difference is 1.10 V. Determine i) the work
Therefore, stopping potential = 0.95 V function of the metal and ii) the maximum
wavelength of light that can eject electrons
EXA MP LE 7 .4 from this metal.
The work function of potassium is 2.2 eV. Solution
UV light of wavelength 3000 Å and intensity
i)  The work function is given by
2 Wm-2 is incident on the potassium surface. hc
i) Determine the maximum kinetic energy ϕ = hv
φ00 = – Kmax
hν − max = − eV0
λ
of the photo electrons ii) If 40% of incident
since Kmax = eV0
photons produce photo electrons, how
many electrons are emitted per second if  6.626×10−34 ×3×108 
=  − 1.6×10−19 ×1.10
the area of the potassium surface is 2 cm2 ?  × −9   
 390 10 

Solution = 5.10×10−19 −1.76×10−19 = 3.34×10−19 J

i)  The energy of the photon is = 2.09 eV


hc 6.626×10−34 ×3×108 ii)  The threshold wavelength is
E= =
λ 3000×10−10
hc 6.626×10− 34 ×3×108
E = 6.626×10 −19
J = 4.14 eV λ0 = =
ϕ0 3.33×10− 19
Maximum KE of the photoelectrons is = 5.969×10− 7 m = 5963 Å
Kmax = hv – ϕ0 = 4.14 – 2.2 = 1.94 eV

120 Unit 7  DUAL NATURE OF R ADIATION AND MAT TER

UNIT-7(XII-Physics_Vol-2)_Word flow.indd 120 7/31/2019 7:23:37 PM


7.3 7.3.2  De Broglie wave
length:
MATTER WAVES
The momentum of photon of frequency
ν is given by
7.3.1  Introduction - Wave hv h
p= = since c = vλ
nature of particles c λ
The wavelength of a photon in terms of
So far, we learnt that the characteristics of
its momentum is
particles and waves are different. A wave is
specified by its frequency, wavelength, wave h
λ= (7.9)
velocity, amplitude and intensity. It spreads p
out and occupies a relatively large region According to de Broglie, the above
of space. A particle specified by its mass, equation is completely a general one and
velocity, momentum and energy occupies a this is applicable to material particles as well.
definite position in space and is very small Therefore, for a particle of mass m travelling
in size. with speed υ , the wavelength is given by
Classical physics treated particles and
waves as distinct entities. But quantum h h
λ= = (7.10)
theory suggested dual character for mυ p
radiations – that is, radiation behaves as This wavelength of the matter waves
a wave at times and as a particle at other is known as de Broglie wavelength. This
times. equation relates the wave character (the wave
From this wave – particle duality of length λ) and the particle character (the
radiation, the concept of wave nature of momentum p) through Planck’s constant.
matter arises which we will see in this section.

De Broglie wave:
7.3.3  De Broglie wave length
The wave–particle duality of radiation of electrons:
was extended to matter by a French physicist
Louis de Broglie (pronounced as de Broy) in An electron of mass m is accelerated
1924. through a potential difference of V volt. The
Greatly influenced by the symmetry in kinetic energy acquired by the electron is
nature, de Broglie suggested that if radiation given by
like light can act as particles at times, then 1
mυ2 = eV
matter particles like electrons should also 2
act as waves at times. Therefore, the speed υ of the electron is
According to de Broglie hypothesis,
all matter particles like electrons, protons, 2eV
υ= (7.11)
neutrons in motion are associated with m
waves. These waves are called de Broglie Hence, the de Broglie wavelength of the
waves or matter waves. electron is

Unit 7  DUAL NATURE OF R ADIATION AND MAT TER 121

UNIT-7(XII-Physics_Vol-2)_Word flow.indd 121 7/31/2019 7:23:49 PM


L.T.
h h
λ= = (7.12)
mυ 2emV
F Electron gun
H.T.
Substituting the known values in the Thin aluminium
above equation, we get diaphragms

6.626×10−34
λ= Aluminium cylinder
2V ×1.6×10−19 ×9.11×10−31
12.27 ×10−10
= meter (or)
V Incident
12.27 beam
λ= Å
V Electron
detector
For example, if an electron is accelerated
through a potential difference of 100V, then Scattered
beam
its de Broglie wavelength is 1.227 Å. θ
Since the kinetic energy of the electron, Ni crystal
K = eV, then the de Broglie wavelength
associated with electron can be also written as Figure 7.17  Experimental set up of
h Davisson – Germer experiment
λ= (7.13)
2mK
The electrons scattered by Ni atoms
7.3.4  Davisson – Germer in different directions are received by
experiment the electron detector which measures
the intensity of scattered electron beam.
De Broglie hypothesis of matter waves
The detector is rotatable in the plane of
was experimentally confirmed by Clinton
Davisson and Lester Germer in 1927. They the paper so that the angle ϕ between the
demonstrated that electron beams are incident beam and the scattered beam can
diffracted when they fall on crystalline solids. be changed at our will. The intensity of the
Since crystal can act as a three-dimensional scattered electron beam is measured as a
diffraction grating for matter waves, the function of the angle θ.
electron waves incident on crystals are
diffracted off in certain specific directions.
V = 54 V
Figure 7.17 shows a schematic representation
Intensity of diffracted
electron beam

of the apparatus for the experiment.


The filament F is heated by a low tension
(L.T.) battery. Electrons are emitted from the
hot filament by thermionic emission. They
50°
are then accelerated due to the potential
difference between the filament and the 0° 30° 60° 90°
θ
anode aluminium cylinder by a high tension
(H.T.) battery. Electron beam is collimated by Figure 7.18  Variation of intensity of
using two thin aluminium diaphragms and is diffracted electron beam with the angle θ
allowed to strike a single crystal of Nickel.
122 Unit 7  DUAL NATURE OF R ADIATION AND MAT TER

UNIT-7(XII-Physics_Vol-2)_Word flow.indd 122 7/31/2019 7:23:59 PM


scattered wave shows a peak or maximum
It is to be noted that electrons at an angle of 50° to the incident electron
Note
are not the only particles beam. This peak in intensity is attributed
with which wave nature to the constructive interference of electrons
can be demonstrated. The particles diffracted from various atomic layers
like neutrons and alpha particle are of the target material. From the known
also associated with waves. They value of interplanar spacing of Nickel, the
undergo diffraction when they are wavelength of the electron wave has been
scattered by suitable crystals. Neutron experimentally calculated as 1.65Å.
diffraction studies are highly useful for The wavelength can also be calculated
investigating crystal structures. from de Broglie relation for V = 54 V from
equation (7.18) as
12.27 12.27
Diffraction is one of λ= Å=
Note V 54
the properties of waves.
λ = 1.67 Å
Whenever waves are incident
on an obstacle, they bend around the This value agrees well with the
edges of the obstacle. This bending of experimentally observed wavelength of
waves is called diffraction. The amount 1.65Å. Thus this experiment directly verifies
of bending depends on the wavelength de Broglie’s hypothesis of the wave nature of
of the waves. moving particles.
We have learnt in unit 6 that as the
wavelength of light is very small, 7.3.4 Electron Microscope
diffraction effects of light are very
small. In order to study diffraction of Principle
light, diffraction gratings are used. This is the direct application of wave
Since X-rays and de Broglie waves of nature of particles. The wave nature of
electrons have wavelengths (in the the electron is used in the construction of
order of 10–10m) much shorter than that microscope called electron microscope.
of the light wave, diffraction grating The resolving power of a microscope is
cannot be used for their diffraction. In inversely proportional to the wavelength
a crystal, the spacing between atomic of the radiation used for illuminating the
planes is comparable to the wavelength object under study. Higher magnification
of x-rays and de Broglie waves of as well as higher resolving power can be
electrons. Hence, for their diffraction, obtained by employing the waves of shorter
the crystals are used which serve as wavelengths.
three-dimensional grating. De Broglie wavelength of electron is
very much less than (a few thousands less)
that of the visible light being used in optical
Figure 7.18 shows the variation of
microscopes. As a result, the microscopes
intensity of the scattered electrons with
employing de Broglie waves of electrons
the angle θ for the accelerating voltage of
have very much higher resolving power than
54V. For a given accelerating voltage V, the
Unit 7 DUAL NATURE OF R ADIATION AND MAT TER 123

UNIT-7(XII-Physics_Vol-2)_Word flow.indd 123 7/31/2019 7:24:01 PM


Light source Electron source

Condenser lens
Condenser lens

Object Object
Objective lens
Objective lens

Intermediate
image Intermediate
image
Projector lens
Projector lens

Photographic
Screen plate
(a) (b) (c)

Figure 7.19  (a) Optical microscope (b) Electron microscope (c) Photograph of electron
microscope

optical microscope. Electron microscopes magnified image is obtained on the screen.


giving magnification more than 2,00,000 These electron microscopes are being used
times are common in research laboratories. in almost all branches of science.
Working
EX AM P L E 7 . 6
The construction and working of an
electron microscope is similar to that of an Calculate the momentum and the de
optical microscope except that in electron Broglie wavelength in the following cases:
microscope focussing of electron beam is i) an electron with kinetic energy 2 eV.
done by the electrostatic or magnetic lenses. ii) a bullet of 50 g fired from rifle with a
The electron beam passing across a suitably speed of 200 m/s
arranged either electric or magnetic fields iii) a 4000 kg car moving along the
undergoes divergence or convergence thereby highways at 50 m/s
focussing of the beam is done (Figure 7.19). Hence show that the wave nature of matter
The electrons emitted from the source is important at the atomic level but is not
are accelerated by high potentials. The beam really relevant at macroscopic level.
is made parallel by magnetic condenser lens.
When the beam passes through the sample Solution:
whose magnified image is needed, the beam i)  Momentum of the electron is
carries the image of the sample.
With the help of magnetic objective lens p = 2mK = 2×9.1×10−31 × 2×1.6×10−19
and magnetic projector lens system, the = 7.63×10−25 kg m s -1
124 Unit 7  DUAL NATURE OF R ADIATION AND MAT TER

UNIT-7(XII-Physics_Vol-2)_Word flow.indd 124 7/31/2019 7:24:04 PM


Magnified images of some objects:

Coloured scanning electron micrograph Needle and thread


of a common housefly (Muscadomestica).

Its de Broglie wavelength is is important at the atomic level but it is not


h 6.626×10−34
really relevant at the macroscopic level.
λ= = = 0.868×10−9 m
p 7.63×10−25
  = 8.68 Å EX AM P L E 7 . 7
ii)  Momentum of the bullet is Find the de Broglie wavelength associated
p = mυ = 0.050× 200 = 10kgms−1 with an alpha particle which is accelerated
Its de Broglie wavelength is through a potential difference of 400 V.
Given that the mass of the proton is
h 6.626×10−34
 λ= = = 6.626×10−33 m 1.67 × 10–27 kg.
p 10
iii)  Momentum of the car is Solution
An alpha particle contains 2 protons
p = mυ = 4000×50 = 2×105 kgms−1
and 2 neutrons. Therefore, the mass M
Its de Broglie wavelength is of the alpha particle is 4 times that of a
h 6.626×10−34 proton (mp) (or a neutron) and its charge q
 λ= = = 3.313×10−39 m
p 2×10 5 is twice that of a proton (+e).
The de Broglie wavelength associated
From these calculations, we notice
with it is
that electron has significant value of de
Broglie wavelength (≈10–9m which can h h
λ= =
be measured from difraction studies) but 2 MqV 2×(4m p )×(2e)×V
bullet and car have negligibly small de
6.626×10−34
Broglie wavelengths associated with them =
(≈10–33m and 10–39m respectively, which 2× 4×1.67 ×10−27 × 2×1.6×10−19 × 400
are not measurable by any experiment). 6.626×10−34
= = 0.00507 Å
This implies that the wave nature of matter 4× 20×10−23 1.67 ×1.6

Unit 7  DUAL NATURE OF R ADIATION AND MAT TER 125

UNIT-7(XII-Physics_Vol-2)_Word flow.indd 125 7/31/2019 7:24:20 PM


EXA MP LE 7 .8 answers this question has already been
discovered, even before Planck’s quantum
A proton and an electron have same de theory of radiation.
Broglie wavelength. Which of them moves
faster and which possesses more kinetic Discovery of x-rays
energy? Wilhelm Roentgen in 1895 discovered that
whenever fast moving electrons fall on certain
Solution materials, a highly penetrating radiation is
h
We know that λ = emitted. Since their origin was not known at
2mK that time, they were called x-rays.
Since proton and electron have same de
X-rays are electromagnetic waves of short
Broglie wavelength, we get
wavelength ranging from 0.1 to 100Å. They
h h K p me
= (or) = travel along straight lines with the velocity
2m p K p 2me K e Ke mp of light and are not affected by electric and
Since me < m p , K p < K e , the electron has magnetic fields. X-ray photons are highly
more kinetic energy than the proton. energetic because of its high frequency or
short wavelength. Therefore, they can pass
1
Kp mp υ2p υp K pme through materials which are opaque to
=2  (or)  υ = visible light.
Ke 1 K e mp
me υ2e e
2 The quality of x-rays is measured in terms
υp Kp
of their penetrating power which depends
me2 m me
= 2
= e  since = on the velocity with which the electrons
υe mp mp Ke mp
strike the target material and the atomic
Since me < mp , υ p < υe , the electron moves number of target material. The intensity
faster than the proton. of x-rays is dependent on the number of
electrons striking the target.
Production of x-rays
7.4
X-rays are produced in x-ray tube which
X – RAYS is essentially a discharge tube as shown in
Figure 7.20. A tungsten filament F is heated
to incandescence by a battery. As a result,
Introduction electrons are emitted from it by thermionic
Quantum theory of radiation explains emission.
photoelectric effect in which the electrons The electrons are accelerated to high
are emitted due to the incidence of speeds by the voltage applied between
photons and the energy is transferred from the filament F and the anode. The target
photons to the electrons. Immediately, a materials like tungsten, molybdenum are
question arises: Is the reverse process also embedded in the face of the solid copper
possible? anode. The face of the target is inclined at
This means that whether electron kinetic an angle with respect to the electron beam
energy can be transformed into photon so that x-rays can leave the tube through its
energy or not. The phenomenon which side.

126 Unit 7  DUAL NATURE OF R ADIATION AND MAT TER

UNIT-7(XII-Physics_Vol-2)_Word flow.indd 126 7/31/2019 7:24:59 PM


X-ray Evacuated tube
50 kV Tungsten target
Target
F

Relative intensity
Electrons 40 kV
Cathode Anode
L.T.
30 kV
H.T.

20 kV
Wavelength
Figure 7.21  (a) X-ray spectra of tungsten
at various accelerating potentials
Figure 7.20  Production of x-rays Kα

When high-speed electrons strike the
target, they are decelerated suddenly and
Relative intensity
lose their kinetic energy. As a result, x-ray Tungsten, 35 kV
photons are produced. Since most of the
kinetic energy of the bombarding electrons
gets converted into heat, targets made of
high-melting-point metals and a cooling Molybdenum,
system are usually employed. 35 kV
Wavelength
X-ray spectra
X-rays are produced when fast moving Figure 7.21  (b) X-ray spectra of tungsten
and molybdenum at 35 kV accelerating
electrons strike the metal target. The intensity of
potential
the x-rays when plotted against its wavelength
gives a curve called x-ray spectrum (Figure Though classical electromagnetic theory
7.21(a) and (b)). X-ray spectra consist of two suggests the emission of radiations from
parts: a continuous spectrum and a series of accelerating electrons, it could not explain
peaks superimposed on it. two features exhibited by x-ray spectra.
The continuous spectrum consists of These features are given below.
radiations of all possible wavelengths with (i) For a given accelerating voltage, the
a certain minimum wavelength λ 0 which lower limit for the wavelength of
depends on the voltage across the electrodes. continuous x-ray spectra is same for all
The peaks are characteristics of the material targets. This minimum wavelength is
of the target and hence they are called called cut-off wavelength.
characteristic spectrum. Figure 7.21(a) (ii) The intensity of x-rays is significantly
depicts the x-ray spectra of tungsten at various increased at certain well-defined
accelerating voltages and Figure 7.21(b) shows wavelengths as shown in the case
the x-ray spectra of tungsten and molybdenum of characteristic x-ray spectra for
at a particular accelerating voltage. molybdenum (Figure 7.21(b)).
Unit 7  DUAL NATURE OF R ADIATION AND MAT TER 127

UNIT-7(XII-Physics_Vol-2)_Word flow.indd 127 7/31/2019 7:25:01 PM


But these two features could be explained where λ 0 is the cut-off wavelength.
on the basis of photon theory of radiation. Substituting the known values in the above
Continuous x-ray spectra equation, we get
When a fast moving electron penetrates and
12400
approaches a target nucleus, the interaction λ0 = Å (7.14)
V
between the electron and the nucleus either
accelerates or decelerates it which results in a The relation given by equation (7.14) is
change of path of the electron. The radiation known as the Duane – Hunt formula.
produced from such decelerating electron is The value of λ 0 depends only on the
called Bremsstrahlung or braking radiation accelerating potential and is same for all
(Figure 7.22). targets. This is in good agreement with the
experimental results. Thus, the production
Electron of KE E1
Photon of energy
of continuous x-ray spectrum and the origin
hν=E1–E2 of cut – off wavelength can be explained on
the basis of photon theory of radiation.
Characteristic x – ray spectra:
+ X – ray spectra show some narrow peaks
at some well – defined wavelengths when
the target is hit by fast electrons. The line
E1>E2 spectrum showing these peaks is called
Electron of KE E2
characteristic x – ray spectrum. This
x – ray spectrum is due to the electronic
Figure 7.22  Bremsstrahlung photon transitions within the atoms.
from a decelerating electron When an energetic electron penetrates
into the target atom and removes some of
The energy of the photon emitted is
the K-shell electrons. Then the electrons
equal to the loss of kinetic energy of the
from outer orbits jump to fill up the vacancy
electron. Since an electron may lose part or
so created in the K-shell. During the
all of its energy to the photon, the photons
downward transition, the energy difference
are emitted with all possible energies
between the levels is given out in the form of
(or frequencies). The continuous x-ray
x– ray photon of definite wavelength. Such
spectrum is due to such radiations.
wavelengths, characteristic of the target,
When an electron gives up all its energy,
constitute the line spectrum.
then the photon is emitted with highest
From the Figure 7.23, it is evident that
frequency ν0 (or lowest wavelength λ 0 ).
K-series of lines in the x-ray spectrum of
The initial kinetic energy of an electron
an element arises due to the electronic
is given by eV where V is the accelerating
transitions from L, M, N, . . levels to the
voltage. Therefore, we have
K-level. Similarly, the longer wavelength
hv0 = eV (or) hc = eV L-series originates when an L-electron
λ0
hc is knocked out of the atom and the
λ0 =
eV corresponding vacancy is filled by the

128 Unit 7  DUAL NATURE OF R ADIATION AND MAT TER

UNIT-7(XII-Physics_Vol-2)_Word flow.indd 128 7/31/2019 7:25:25 PM


radiograph containing a deep shadow of the
bones and a light shadow of the flesh may
O be obtained. X-ray radiographs are used to
n=4 detect fractures, foreign bodies, diseased
N
Mα Mγ organs etc.

n=3 2)  Medical therapy
M Since x-rays can kill diseased tissues,
Lα Lγ
they are employed to cure skin diseases,
Lβ Lδ malignant tumours etc.
n=2 3)  Industry
L
X-rays are used to check for flaws in
welded joints, motor tyres, tennis balls and
wood. At the custom post, they are used for
detection of contraband goods.
4)  Scientific research


Kγ Kε X-ray diffraction is important tool

to study the structure of the crystalline
materials – that is, the arrangement of atoms
and molecules in crystals.

n=1 EX AM P L E 7 . 9
K
Calculate the cut-off wavelength and cut-
off frequency of x-rays from an x –ray tube
Figure 7.23  Origin of characteristic of accelerating potential 20,000 V.
x-ray spectra
Solution
electronic transitions from M, N, O,... and The cut-off wavelength of the characteristic
so on. x-rays is
The Kα and Kβ of the K-series of 12400 12400
molybdenum are shown by the two peaks in λ = Å= Å
V 20000
its x-ray spectrum in Figure 7.21(b). = 0.62 Å
Applications of x-rays: The corresponding frequency is
X-rays are being used in many fields. c 3×108
ν0 = = = 4.84×1018 Hz
Let us list a few of them. λ 0 0.62×10 −10

1)  Medical diagnosis
X-rays can pass through flesh more
easily than through bones. Thus an x-ray

Unit 7  DUAL NATURE OF R ADIATION AND MAT TER 129

UNIT-7(XII-Physics_Vol-2)_Word flow.indd 129 7/31/2019 7:25:30 PM


SUMMARY
„„ Particle is a material object which is considered as a tiny concentration of matter
(localized in space and time) whereas wave is a broad distribution of energy
(not localized in space and time).
„„ The liberation of electrons from any surface of a substance is called electron emission.
„„ The minimum energy needed for an electron to escape from the metal surface is called
work function of that metal.
„„ 1 eV is equal to 1.602 × 10–19 J.
„„ The emission of electrons by supplying thermal energy is known as thermionic emission.
„„ Electric field emission occurs when a very strong electric field is applied across the
metal.
„„ The emission of electrons due to irradiation of light is called photoelectric emission.
„„ Secondary emission is the process in which electrons are emitted due to the bombardment
of fast moving electrons.
„„ The photoelectric current (i.e. the number of electrons emitted per second) is directly
proportional to the intensity of the incident light.
„„ Stopping potential is that the value of the negative (retarding) potential given to the
collecting electrode A which is just sufficient to stop the most energetic photoelectrons
emitted and make the photocurrent zero.
„„ The stopping potential is independent of intensity of the incident light.
„„ Maximum kinetic energy of the photoelectrons is independent of intensity of the
incident light.
„„ For a given surface, the emission of photoelectrons takes place only if the frequency
of incident light is greater than a certain minimum frequency called the threshold
frequency.
„„ According to Planck, a matter is composed of a large number of oscillating particles
(atoms) which vibrate with different frequencies.
„„ According to Einstein, the energy in light is not spread out over wavefronts but is
concentrated in small packets or energy quanta.
„„ The individual light quantum of definite energy and momentum can be associated with
a particle. This particle is named photon.
„„ Light behaves as a wave during its propagation and behaves as a particle during its
interaction with matter.
„„ Photo electric cell or photo cell is a device which converts light energy into electrical
energy.
„„ According to de Broglie hypothesis, all matter particles like electrons, protons, neutrons
in motion are associated with waves. These waves are called de Broglie waves or matter
waves.
„„ Wave nature of the electron is used in the construction of microscope called electron
microscope.

130 Unit 7  DUAL NATURE OF R ADIATION AND MAT TER

UNIT-7(XII-Physics_Vol-2)_Word flow.indd 130 7/31/2019 7:25:31 PM


„„ De Broglie hypothesis of matter waves was experimentally confirmed by Clinton
Davisson and Lester Germer in 1927.
„„ Whenever fast moving electrons fall on the materials, a highly penetrating radiation,
x-rays, is emitted.
„„ Continuous x-ray spectrum consists of radiations of all possible wavelengths with a
certain minimum wavelength λ 0 .
„„ Characteristic x-ray spectra show some narrow peaks at some well – defined
wavelengths when the target is hit by fast electrons.

CONCEPT MAP

Dual nature of radiation and matter

Photoelectric
Electron emission effect Matter waves X-rays

Observations of De Broglie
Thermionic Hertz, Hallwachs Introduction
emission and Lenard hypothesis

Field emission Effect of intensity De Broglie equation Production of


X-rays

Photoelectric Effect of potential De Broglie


difference wavelength of X-ray spectra
emission
electron

Davisson-Germer Continuous
Secondary emission Effect of frequency Experiment X-ray spectra

Quantisation of Characteristic
energy Electron microscope X-ray spectra

Einstein’s
photoelectric Applications
equation

Photocells and uses

Unit 7  DUAL NATURE OF R ADIATION AND MAT TER 131

UNIT-7(XII-Physics_Vol-2)_Word flow.indd 131 7/31/2019 7:25:31 PM


E V AL U A T I O N

I Multiple Choice Questions 5. If a light of wavelength 330 nm is


1. The wavelength λe of an electron and incident on a metal with work function
λp of a photon of same energy E are 3.55 eV, the electrons are emitted. Then
related by (NEET 2013) the wavelength of the emitted electron
is (Take h = 6.6 × 10–34 Js)
a. λ p µ λ e b. λ p µ λ e
a. < 2.75×10−9 m b. ≥ 2.75×10−9 m
1
c. λ p µ d. λ p µ λ 2e c. ≤ 2.75×10−12 m d. < 2.5×10−10 m
λe
6. A photoelectric surface is illuminated
2. In an electron microscope, the electrons
successively by monochromatic
are accelerated by a voltage of 14 kV. If
light of wavelength λ and λ 2 . If the
the voltage is changed to 224 kV, then
the de Broglie wavelength associated maximum kinetic energy of the emitted
with the electrons would photoelectrons in the second case is
3 times that in the first case, the work
a. increase by 2 times
function at the surface of material is
b. decrease by 2 times (NEET 2015)
c. decrease by 4 times hc 2hc
a) b)
d. increase by 4 times λ λ
hc
3. A particle of mass 3 × 10–6 g has the same c) hc d)
wavelength as an electron moving with 3λ 2λ
a velocity 6×106 m s−1 . The velocity of 7. In photoelectric emission, a radiation
the particle is whose frequency is 4 times threshold
a. 1.82×10−18 m s−1 frequency of a certain metal is incident
on the metal. Then the maximum
b. 9×10−2 m s−1
possible velocity of the emitted electron
c. 3×10−31 m s−1 will be
d. 1.82×10−15 m s−1 hv0 6hv0
a) b)
4. When a metallic surface is illuminated m m
with radiation of wavelength λ , the hv0 hv0
c) 2 d)
stopping potential is V. If the same m 2m
surface is illuminated with radiation of 8. Two radiations with photon energies
wavelength 2λ , the stopping potential 0.9 eV and 3.3 eV respectively are falling
V on a metallic surfacesuccessively. If the
is . The threshold wavelength for the
4 work function of the metal is 0.6 eV,
metallic surface is (NEET 2016)
then the ratio of maximum speeds of
a. 4λ b. 5λ emitted electrons will be
5 a) 1:4 b) 1:3
c. λ d. 3λ
2 c) 1:1 d)1:9

132 Unit 7 DUAL NATURE OF R ADIATION AND MAT TER

UNIT-7(XII-Physics_Vol-2)_Word flow.indd 132 7/31/2019 7:27:19 PM


9. A light source of wavelength 520  nm 2 2 2

emits 1.04 × 1015 photons per second c. hc - me c 2 - e B R


λ 2me
while the second source of 460 nm
2
produces 1.38 × 1015 photons per second.  
d. hc − 2m  eBR 
Then the ratio of power of second source λ
e  
 2me 
to that of first source is 14. The work functions for metals A, B
a) 1.00 b) 1.02 and C are 1.92 eV, 2.0 eV and 5.0 eV
c) 1.5 d) 0.98 respectively. The metals which will
10. The mean wavelength of light from sun emit photoelectrons for a radiation of
is taken to be 550 nm and its mean wavelength 4100Å is/are
power is 3.8 × 1026W. The number of a. A only
photons received by the human eye per b. both A and B
second on the average from sunlight is
c. all these metals
of the order of
a) 1045 b) 1042 d. none
c) 1054 d) 1051 15. Emission of electrons by the absorption
of heat energy is called………emission.
11. The threshold wavelength for a metal
a. photoelectric
surface whose photoelectric work
function is 3.313 eV is b. field
a) 4125Å b) 3750Å c. thermionic
c) 6000Å d) 2062.5 Å d. secondary
12. A light of wavelength 500 nm is incident Answers
on a sensitive plate of photoelectric 1. d 2. c 3. d 4. d 5. b
work function 1.235 eV. The kinetic
6. d 7. b 8. b 9. c 10. a
energy of the photo electrons emitted
is be (Take h = 6.6 × 10–34 Js) 11. b 12. c 13. d 14. b 15. c
a) 0.58 eV b) 2.48 eV
II Short Answer Questions
c) 1.24 eV d) 1.16 eV
13. Photons of wavelength λ are incident 1. Why do metals have a large number of
on a metal. The most energetic free electrons?
electrons ejected from the metal are 2. Define work function of a metal. Give
bent into a circular arc of radius R by its unit.
a perpendicular magnetic field having 3. What is photoelectric effect?
magnitude B. The work function of the 4. How does photocurrent vary with the
metal is (KVPY-SX 2016) intensity of the incident light?
hc e 2 B2 R2
a. − me + 5. Give the definition of intensity of light
λ 2me
and its unit.
2
  6. How will you define threshold
b. hc + 2me  eBR 
λ  2m  frequency?
 e

Unit 7  DUAL NATURE OF R ADIATION AND MAT TER 133

UNIT-7(XII-Physics_Vol-2)_Word flow.indd 133 7/31/2019 7:27:54 PM


7. What is a photo cell? Mention the 7. Explain the quantum concept of light.
different types of photocells. 8. Obtain Einstein’s photoelectric
8. Write the expression for the de Broglie equation with necessary explanation.
wavelength associated with a charged 9. Explain experimentally observed facts
particle of charge q and mass m, when of photoelectric effect with the help of
it is accelerated through a potential V. Einstein’s explanation.
9. State de Broglie hypothesis. 10. Give the construction and working of
10. Why we do not see the wave properties photo emissive cell.
of a baseball? 11. Derive an expression for de Broglie
11. A proton and an electron have same wavelength of electrons.
kinetic energy. Which one has greater 12. Briefly explain the principle and
de Broglie wavelength. Justify. working of electron microscope.
12. Write the relationship of de Broglie 13. Describe briefly Davisson – Germer
wavelength λ associated with a particle experiment which demonstrated the
of mass m in terms of its kinetic wave nature of electrons.
energy K.
IV. Numerical problems
13. Name an experiment which shows
1. How many photons per second emanate
wave nature of the electron. Which
from a 50 mW laser of 640 nm?
phenomenon was observed in this
experiment using an electron beam? [Ans: 1.61×1017 s−1 ]
14. An electron and an alpha particle have 2. Calculate the maximum kinetic
same kinetic energy. How are the de energy and maximum velocity of
Broglie wavelengths associated with the photoelectrons emitted when
them related? the stopping potential is 81V for the
photoelectric emission experiment.

III Long Answer Questions  [[Ans: 1.3×10–17J; 5.3×106 ms–1]]


3. Calculate the energies of the photons
1. What do you mean by electron associated with the following radiation:
emission? Explain briefly various (i) violet light of 413 nm (ii) X-rays of
methods of electron emission. 0.1 nm (iii) radio waves of 10 m.
2. Briefly discuss the observations of [Ans: 3eV ; 12424eV ;1.24×10−7 eV ]
Hertz, Hallwachs and Lenard. 4. A 150 W lamp emits light of mean
3. Explain the effect of potential difference wavelength of 5500 Å . If the efficiency
on photoelectric current. is 12%, find out the number of photons
4. Explain how frequency of incident emitted by the lamp in one second.
light varies with stopping potential. [Ans: 4.98´1019 ]
5. List out the laws of photoelectric effect. 5. How many photons of frequency 1014 Hz
6. Explain why photoelectric effect will make up 19.86 J of energy?
cannot be explained on the basis of [Ans: 3´1020 ]
wave nature of light.

134 Unit 7  DUAL NATURE OF R ADIATION AND MAT TER

UNIT-7(XII-Physics_Vol-2)_Word flow.indd 134 7/31/2019 7:28:02 PM


6. What should be the velocity of the 11. UV light of wavelength 1800 Å is
electron so that its momentum equals incident on a lithium surface whose
that of 4000 Å wavelength photon. threshold wavelength 4965 Å .
[Ans: 1818ms-1 ] Determine the maximum energy of the
7. When a light of frequency 9´1014 Hz is electron emitted.
incident on a metal surface, photoelectrons  [Ans: 4.40 eV]
are emitted with a maximum speed of 12. Calculate the de Broglie wavelength of
8×105 ms−1 . Determine the threshold a proton whose kinetic energy is equal
frequency of the surface. to 81.9 × 10–15 J. (Given: mass of proton
[Ans: 4.61´1014 Hz ] is 1836 times that of electron).
8. When a 6000Å light falls on the [Ans: 4×10−14 m ]
cathode of a photo cell and produced 13. A deuteron and an alpha particle are
photoemission. If a stopping potential accelerated with the same potential.
of 0.8 V is required to stop emission Which one of the two has i) greater
of electron, then determine the value of de Broglie wavelength
(i) frequency of the light (ii) energy of associated with it and ii) less kinetic
the incident photon (iii) work function energy? Explain.
of the cathode material (iv) threshold [Ans: λ d = 2λ α and K d = K α ]
2
frequency and (v) net energy of the 14. An electron is accelerated through a
electron after it leaves the surface. potential difference of 81V. What is the
[Ans: 5×1014 Hz; 2.07 eV; 1.27 eV; de Broglie wavelength associated with
3.07×1014 Hz; 0.8 eV ] it? To which part of electromagnetic
spectrum does this wavelength
9. A 3310 Å photon liberates an electron correspond?
from a material with energy 3×10−19 J [Ans: λ =1.36 Å and x-rays]
while another 5000 Å photon ejects 15. The ratio between the de Broglie
an electron with energy 0.972×10−19 J wavelengths associated with protons,
from the same material. Determine accelerated through a potential of 512 V
the value of Planck’s constant and the and that of alpha particles accelerated
threshold wavelength of the material. through a potential of X volts is found
[Ans: 6.62×10−34 Js; 6620×10−10 m ] to be one. Find the value of X.
10. At the given point of time, the  [Ans: 64 V]
earth receives energy from sun at
4 cal cm-2 min-1. Determine the number
of photons received on the surface of the
Earth per cm2 per minute. (Given : Mean
wavelength of sun light = 5500 Å )
[Ans: 4.65´1019 ]

Unit 7  DUAL NATURE OF R ADIATION AND MAT TER 135

UNIT-7(XII-Physics_Vol-2)_Word flow.indd 135 7/31/2019 7:28:41 PM


BOOK FOR REFERENCES

1. Arthur Beiser, Shobhit Mahajan, Rai Choudhury, Concepts of Modern Physics, Sixth
Edition, McGraw Hill Education (India) Private Limited.
2. H.S. Mani and G.K. Mehta, Introduction to Modern Physics, Affiliated East-West Press
Pvt. Ltd.
3. H.C.Verma, Concepts of Physics, Volume 1 and 2, BharathiBhawan publishers.
4. Halliday, Resnick and Walker, Principles of Physics, Wiley publishers.

136 Unit 7  DUAL NATURE OF R ADIATION AND MAT TER

UNIT-7(XII-Physics_Vol-2)_Word flow.indd 136 7/31/2019 7:28:41 PM


ICT CORNER
Dual nature of radiation and matter

In this activity you will be able to visualize how


Topic: Photoelectric effect
light knocks electrons off a metal target and
describe the photoelectric effect experiment.
STEPS:
• Open the browser and type “https://phet.colorado.edu/en/simulation/legacy/photoelectric” in the
address bar.
• Change intensity of light and observe how the intensity of light will affect the photo electric current
and the energy of electrons
• By adjusting the value of wavelength and observe how the wavelength of light will affect the photo
electric current and the energy of electrons
• Adjust the value of voltage from the battery and analyse the effect of potential difference on the
photoelectric current.
• Change the material of the target and analyse how it will affect the current and the energy of electrons.
• Study the photo electric current – voltage graph and Photo electric current - intensity graph obtained
from this experiment.
Step1 Step2

Step3 Step4

Note:
Install Java application if it is not in your browser.
You can download all the phet simulation and works in off line fromhttps://phet.
colorado.edu/en/offline-access .
URL:
https://phet.colorado.edu/en/simulation/legacy/photoelectric
* Pictures are indicative only.
* If browser requires, allow Flash Player or Java Script to load the page.

Unit 7 DUAL NATURE OF R ADIATION AND MAT TER 137

UNIT-7(XII-Physics_Vol-2)_Word flow.indd 137 7/31/2019 7:28:42 PM


UNIT

8 ATOMIC AND NUCLEAR PHYSICS

All of physics is either impossible or trivial. It is impossible until you understand it, and then it becomes trivial
– Ernest Rutherford

LEARNING OBJECTIVES

In this unit, the students are exposed to


• electric discharge through the gases
• determination of specific charge by J.J. Thomson experiment
• determination of electronic charge by Millikan’s oil drop experiment
• atom models – J.J. Thomson and Rutherford
• Bohr atom model and hydrogen atom
• atomic spectrum and hydrogen spectrum
• structure and properties of nucleus
• various classification of nuclei based on atomic and mass number
• mass defect and binding energy
• relation between stability and binding energy curve
• alpha, beta and gamma decay
• law of radioactive decay
• nuclear fission and fusion
• elementary ideas of nuclear reactors
• qualitative idea of elementary particles

8.1 In earlier classes, we have studied that


INTRODUCTION anything which occupies space is called
matter. Matter can be classified into solids,
Earth
liquids and gases. In our daily life, we use
water for drinking, petrol for vehicles,
Atom
Apple we inhale oxygen, stainless steel vessels
Same size
for cooking, etc. Experiences tell us that
Same size behaviour of one material is not same
Atom
as another, this means that the physical
Atom Apple
Apple Earth and chemical properties are different for
different materials. In order to understand
Figure 8.1 Comparision of size of an atom
with that of an apple and comparision of this, we need to know the fundamental
size of an apple with that of the Earth constituents of materials.

138

UNIT-8(XII-Physics_Vol-2)_Word flow.indd 138 7/31/2019 7:32:37 PM


When an object is divided repeatedly, But by special arrangement, one can make a
the process of division could not be done gas to conduct electricity.
beyond a certain stage in a similar way and A simple and convenient device used
we end up with a small speck. This small to study the conduction of electricity
speck was defined as an atom. The word through gases is known as gas discharge
atom in Greek means ‘without division tube. The arrangement of discharge tube
or indivisible’. The size of an atom is is shown in Figure 8.2. It consists of a long
very very small. For an example, the size closed glass tube (of length nearly 50 cm
of hydrogen atom (simplest among other and diameter of 4 cm) inside of which
atoms) is arround 10 –10 m. An American the gas in pure form is filled usually. The
Physicist Richard P. Feynman said that small opening in the tube is connected to
if the atom becomes the size of an apple, a high vacuum pump and a low-pressure
then the apple becomes the size of the gauge. This tube is fitted with two metallic
earth as shown in Figure 8.1. Such a small plates known as electrodes which are
entity is an atom. connected to secondary of an induction
In this unit, we first discuss the theoretical coil. The electrode connected to positive
models of atom to understand its structure. of secondary is known as anode and the
The Bohr atom model is more successful than electrode to the negative of the secondary
J. J. Thomson and Rutherford atom models. is cathode. The potential of secondary is
It explained many unsolved issues in those maintained about 50 kV.
days and also gave better understanding of
chemistry.
Later, scientists observed that even the atom
is not the fundamental entity. It consists of
electrons and nucleus. Around 1930, scientists
discovered that nucleus is also made of proton
and neutron. Further research discovered that
even the proton and neutron are made up of (a)
fundamental entities known as quarks.
In this context, the remaining part of this High
potential difference
unit is written to understand the structure —
and basic properties of nucleus. Further how
+
the nuclear energy is produced and utilized
are discussed.
C A

8.2 Gas
ELECTRIC DISCHARGE
THROUGH GASES (b)

Gases at normal atmospheric pressure are Figure 8.2 Discharge tube (a) real picture
poor conductors of electricity because they (b) schematic diagram
do not have free electrons for conduction.
Unit 8 Atomic and Nuclear physics 139

UNIT-8(XII-Physics_Vol-2)_Word flow.indd 139 7/31/2019 7:32:37 PM


Suppose the pressure of the gas in discharge 8.2.1  Determination of specific
tube is reduced to around 110 mm of Hg using e
charge   of an electron –
vacuum pump, it is observed that no discharge  m
takes place. When the pressure is kept near 100 Thomson’s experiment
mm of Hg, the discharge of electricity through Thomson’s experiment is considered as
the tube takes place. Consequently, irregular one among the landmark experiments for the
streaks of light appear and also crackling sound birth of modern physics. In 1887, J. J. Thomson
is produced. When the pressure is reduced to made remarkable improvement in the scope
the order of 10 mm of Hg, a luminous column of study of gases in discharge tubes. In the
known as positive column is formed from presence of electric and magnetic fields, the
anode to cathode. cathode rays are deflected. By the variation of
When the pressure reaches to around electric and magnetic fields, mass normalized
0.01 mm of Hg, positive column disappears. charge or the specific charge (charge per unit
At this time, a dark space is formed between mass) of the cathode rays is measured.
anode and cathode which is often called
Deflecting
Crooke’s dark space and the walls of the plates
Fluorescent screen

tube appear with green colour. At this stage, A


S
+
P
C
some invisible rays emanate from cathode _
N
O

called cathode rays, which are later found Battery


P'
High voltage
be a beam of electrons. battery

Properties of cathode rays Magnet

(1) Cathode rays possess energy and


momentum and travel in a straight line with Figure 8.3  Arrangement of J.J. Thomson
experiment to determine the specific
high speed of the order of 107m s-1. It can
charge of an electron
be deflected by application of electric and
magnetic fields. The direction of deflection
The arrangement of J. J. Thomson’s
indicates that they are negatively charged
experiment is shown in Figure 8.3. A
particles.
highly evacuated discharge tube is used and
(2) When the cathode rays are allowed
cathode rays (electron beam) produced at
to fall on matter, they produce heat. They
cathode are attracted towards anode disc A.
affect the photographic plates and also
Anode disc is made with pin hole in order
produce fluorescence when they fall on
to allow only a narrow beam of cathode
certain crystals and minerals.
rays. These cathode rays are now allowed
(3) When the cathode rays fall on a
to pass through the parallel metal plates,
material of high atomic weight, x-rays are
maintained at high voltage as shown in
produced.
Figure 8.3. Further, this gas discharge tube
(4) Cathode rays ionize the gas through
is kept in between pole pieces of magnet
which they pass.
such that both electric and magnetic fields
(5) The speed of cathode rays is up to
th are perpendicular to each other. When
 1 
  of the speed of light. the cathode rays strike the screen, they
10 
produce scintillation and hence bright spot
140 Unit 8  Atomic and Nuclear physics

UNIT-8(XII-Physics_Vol-2)_Word flow.indd 140 7/31/2019 7:32:38 PM


is observed. This is achieved by coating the 2

screen with zinc sulphide. eV = 1 mυ2 ⇒ e = υ


2 m 2V
(i) Determination of velocity of cathode Substituting the value of velocity from
rays equation (8.1), we get

e = 1 E 2 (8.2)
m 2V B 2
Substituting the values of E, B and V, the
S
specific charge can be determined as
+ e = 1.7 ×1011 C kg −1
e– E FE B v m
FB
_
(iii) 
Deflection of charge only due to
uniform electric field
N When the magnetic field is turned off,
the deflection is only due to electric field.
The deflection in vertical direction is due to
the electric force.
Figure 8.4  Electric force balancing the
magnetic force – the path of electron Fe = eE (8.3)
beam is a straight line
Let m be the mass of the electron and by
For a fixed electric field between the applying Newton’s second law of motion,
plates, the magnetic field is adjusted such acceleration of the electron is
that the cathode rays (electron beam) strike
at the original position O (Figure 8.3). This ae = 1 Fe (8.4)
m
means that the magnitude of electric force is
balanced by the magnitude of force due to Substituting equation (8.4) in equation
magnetic field as shown in Figure 8.4. Let e (8.3),
be the charge of the cathode rays, then
eE = eB υ
ae = 1 eE = e E
m m
⇒ υ = E (8.1)
B
+
(ii) Determination of specific charge y

Since the cathode rays (electron beam) e–

are accelerated from cathode to anode, the E


_
potential energy of the electron beam at the
l
cathode is converted into kinetic energy of
Screen
the electron beam at the anode. Let V be
the potential difference between anode and Figure 8.5  Deviation of path by applying
cathode, then the potential energy is eV. uniform electric field
Then from law of conservation of energy,
Unit 8  Atomic and Nuclear physics 141

UNIT-8(XII-Physics_Vol-2)_Word flow.indd 141 7/31/2019 7:32:39 PM


Let y be the deviation produced from (iv) Deflection of charge only due to
original position on the screen as shown uniform magnetic field
in Figure 8.5. Let the initial upward Suppose that the electric field is switched
velocity of cathode ray be u = 0 before off and only the magnetic field is switched
entering the parallel electric plates. Let on. Now the deflection occurs only due
t be the time taken by the cathode rays to magnetic field. The force experienced
to travel in electric field. Let l be the by the electron in uniform magnetic field
length of one of the plates, then the time applied perpendicular to its path is
taken is
Fm = e υB (in magnitude)
t= l (8.5) Since this force provides the centripetal
υ
force, the electron beam undergoes a semi-
Hence, the deflection yʹ of cathode rays
circular path. Therefore, we can equate Fm to
is (note: u = 0 and ae = e E ) 2
m centripetal force mυ .
R
y ′ = ut + 1 at 2 ⇒ y ′ = ut + 1 ae t 2
2 2 2

   
2 Fm = e υB = m υ
= 1  e E  l  R
2  m  υ  where v is the velocity of electron beam
2 2 at the point where it enters the magnetic
y′ = 1 e l B (8.6) field and R is the radius of the circular path
2m E
traversed by the electron beam.
Therefore, the deflection y on the screen
is eB = m υ ⇒ e = υ (8.9)
R m BR
y ∝ y′ ⇒ y = C y′
Further, substituting equation (8.1) in
where C is proportionality constant
equation (8.10), we get
which depends on the geometry of the
discharge tube and substituting yʹ value in e = E (8.10)
equation 8.6, we get m B2R
2 2 By knowing the values of electric field,
y =C1 e l B (8.7)
2m E magnetic field and the radius of circular
path, the value of specific charge  e  can
Rearranging equation (8.7) as  m
be calculated, which is also consistant with
e = 2 yE (8.8)
other two methods.
m Cl 2 B 2

Substituting the values on RHS, the The specific charge is


value of specific charge is calculated as Note independent of
e = 1.7 × 1011 C kg −1 (a) gas used
m (b) nature of the electrodes

142 Unit 8 Atomic and Nuclear physics

UNIT-8(XII-Physics_Vol-2)_Word flow.indd 142 7/31/2019 7:32:42 PM


8.2.2  Determination of These two parallel plates are enclosed in a
charge of an electron – chamber with glass walls. Further, plates A
Millikan’s oil drop experiment and B are given a high potential difference
around 10 kV such that electric field acts
Millikan’s oil drop experiment is another vertically downward. A small hole is made
important experiment in modern physics at the centre of the upper plate A and
which is used to determine one of the atomizer is kept exactly above the hole
fundamental constants of nature known as to spray the liquid. When a fine droplet
charge of an electron (Figure 8.6 (a)). of highly viscous liquid (like glycerine)
By adjusting electric field is sprayed using atomizer, it falls freely
suitably, the motion of oil downward through the hole of the top plate
drop inside the chamber can only under the influence of gravity.
be controlled – that is, it can Few oil drops in the chamber can acquire
be made to move up or down electric charge (negative charge) because
or even kept balanced in the of friction with air or passage of x-rays in
field of view for sufficiently long time. between the parallel plates. Further the
Atomizer
chamber is illuminated by light which
is passed horizontally and oil drops can
(+)
be seen clearly using microscope placed
Light
source
perpendicular to the light beam. These drops
(–) Telescope
can move either upwards or downward.
Let m be the mass of the oil drop and q
(a) be its charge. Then the forces acting on the
droplet are
To Atomizer
(a)  gravitational force Fg = mg
(b)  electric force Fe = qE
+ A
(c)  buoyant force Fb
Microscope (d)  viscous force Fv

Light source Fb Fv Fb Fe
– B

(b)
Figure 8.6  Millikan’s experiment (a) real Oil drop
picture and schematic picture (b) Side
view picture
Fg Fg
The experimental arrangement is shown (a) (b)
in Figure 8.6 (b). The apparatus consists of
two horizontal circular metal plates A and Figure 8.7  Free body diagram of the oil
B each with diameter around 20 cm and drop – (a) without electric field (b) with
electric field
are separated by a small distance 1.5 cm.

Unit 8  Atomic and Nuclear physics 143

UNIT-8(XII-Physics_Vol-2)_Word flow.indd 143 7/31/2019 7:32:43 PM


(a) Determination of radius of the droplet From the free body diagram as shown in
When the electric field is switched off, Figure 8.7 (a), the force balancing equation is
the oil drop accelerates downwards. Due to Fg = Fb + Fυ
the presence of air drag forces, the oil drops
   
easily attain its terminal velocity and moves ρ 4 πr 3  g = σ  4 πr 3  g + 6πr υη
3  3 
with constant velocity. This velocity can
be carefully measured by noting down the 4 πr 3 (ρ − σ) g = 6πr υη
time taken by the oil drop to fall through 3
a predetermined distance. The free body 2 πr 3 (ρ − σ) g = 3πr υη
diagram of the oil drop is shown in Figure 8.7 3
(a), we note that viscous force and buoyant
1
force balance the gravitational force.  9 ηυ  2
r= 
Let the gravitational force acting on the  2(ρ − σ) g  (8.11)
oil drop (downward) be Fg = mg
Let us assume that oil drop to be spherical Thus, equation (8.11) gives the radius of
in shape. Let ρ be the density of the oil drop, the oil drop.
and r be the radius of the oil drop, then the (b) Determination of electric charge
mass of the oil drop can be expressed in When the electric field is switched on,
terms of its density as charged oil drops experience an upward
m electric force (qE). Among many drops,
ρ= one particular drop can be chosen in the
V
volume of the  field of view of microscope and strength
 

⇒ m = ρ 4 πr 3  
 4 3

of the electric field is adjusted to make that
3  sphere,V = πr 
3 particular drop to be stationary. Under these
The gravitational force can be written in circumstances, there will be no viscous force
terms of density as acting on the oil drop. Then, from the free
  body diagram shown Figure 8.7 (b), the net
Fg = mg ⇒ Fg = ρ 4 πr 3  g force acting on the oil droplet is
3 
Let σ be the density of the air, the Fe + Fb = Fg
upthrust force experienced by the oil drop
due to displaced air is ⇒ qE + 4 πr 3σg = 4 πr 3ρg
3 3
 
Fb = σ  4 πr 3  g
3  ⇒ qE = 4 πr 3 (ρ − σ) g
3
Once the oil drop attains a terminal
velocity υ, the net downward force acting
⇒ q = 4 πr 3 (ρ − σ) g (8.12)
on the oil drop is equal to the viscous force 3E
acting opposite to the direction of motion
Substituting equation (8.11) in equation
of the oil drop. From Stokes law, the viscous
(8.12), we get
force on the oil drop is 1

Fv = 6πr υη 18 π  η3 υ3 2
q=  
E  2(ρ − σ) g 
144 Unit 8  Atomic and Nuclear physics

UNIT-8(XII-Physics_Vol-2)_Word flow.indd 144 7/31/2019 7:32:47 PM


Millikan repeated this experiment 8.3.1  J
 . J. Thomson’s Model
several times and computed the charges on (Water melon model)
oil drops. He found that the charge of any oil
In this model, the atoms are visualized
drop can be written as integral multiple of a
as homogeneous spheres which contain
basic value, −1.6 ×10−19 C, which is nothing
uniform distribution of positively charged
but the charge of an electron.
particles (Figure 8.8 (a)). The negatively
8.3 charged particles known as electrons are
ATOM MODELS embedded in it like seeds in water melon
as shown in Figure 8.8 (b).
Introduction 10 m
–10

Around 400 B.C, Greek philosophers


Leucippus and Democretus proposed the _ _
_
concept of atom, ‘Every object on continued _
_
subdivision ultimately yields atoms’. Later, _
many physicists and chemists tried to
understand the nature with the idea of atoms. Positively charged
Many theories were proposed to explain the material

properties (physical and chemical) of bulk Figure 8.8  (a) Atom (b) Water melon
materials on the basis of atomic model.
For instance, J. J. Thomson proposed a The atoms are electrically neutral, this
theoretical atom model which is based on implies that the total positive charge in an
static distribution of electric charges. Since atom is equal to the total negative charge.
this model fails to explain the stability of atom, According to this model, all the charges
one of his students E. Rutherford proposed the are assumed to be at rest. But from classical
electrodynamics, no stable equilibrium
first dynamic model of an atom. Rutherford
points exist in electrostatic configuration
gave atom model which is based on results of
(this is known as Earnshaw’s theorem)
an experiment done by his students (Geiger
and hence such an atom cannot be stable.
and Marsden). But this model also failed to
Further, it fails to explain the origin of
explain the stability of the atom.
spectral lines observed in the spectrum of
Later, Niels Bohr who is also a student of
hydrogen atom and other atoms.
Rutherford proposed an atomic model for
hydrogen atom which is more successful than 8.3.2  Rutherford’s model
other two models. Niels Bohr atom model
In 1911, Geiger and Marsden did a
could explain the stability of the atom and also
remarkable experiment based on the advice
the origin of line spectrum. There are other
of their teacher Rutherford, which is known
atom models, such as Sommerfeld’s atom
as scattering of alpha particles by gold foil.
model and atom model from wave mechanics The experimental arrangement is shown
(quantum mechanics). But we will restrict in Figure 8.9. A source of alpha particles
ourselves only to very simple (mathematically (radioactive material, example polonium) is
simple) atom model in this section.
Unit 8  Atomic and Nuclear physics 145

UNIT-8(XII-Physics_Vol-2)_Word flow.indd 145 7/31/2019 7:32:47 PM


Flash of
light
Lead Lead Screem

Block Microscope
Fluorescent
α Screen
θ
+

Polonium Scattering
sample angle

Gold
foil

Figure 8.9  Schematic diagram for scattering of alpha particles experiment by Rutherford

kept inside a thick lead box with a fine hole experiment. In this experiment, alpha
as seen in Figure 8.9. The alpha particles particles (positively charged particles) are
coming through the fine hole of lead box allowed to fall on the atoms of a metallic
pass through another fine hole made on the gold foil. The results of this experiment are
lead screen. These particles are now allowed given below and are shown in Figure 8.10,
to fall on a thin gold foil and it is observed Rutherford expected the nuclear model to be
that the alpha particles passing through gold as seen in Figure 8.10 (a) but the experiment
foil are scattered through different angles. A showed the model as in Figure 8.10 (b).
movable screen (from 0° to 180°) which is (a) Most of the alpha particles are un-
made up of zinc sulphide (ZnS) is kept on deflected through the gold foil and went
the other side of the gold foil to collect the straight.
alpha particles. Whenever alpha particles (b) Some of the alpha particles are deflected
strike the screen, a flash of light is observed through a small angle.
which can be seen through a microscope. (c) A few alpha particles (one in thousand)
Rutherford proposed an atom model are deflected through the angle more
based on the results of alpha scattering than 90°

Expected Observed
Number of scattered particles detected

107
_
_ _ 106

_ _ _ 105
_
_ + 104

_ _ 103
102
10
0 20 40 60 80 100 120 140 160 180
(a) (b) (c) Scattering angle θ (in degree)

Figure 8.10  In alpha scattering experiment – (a) Rutherford expected (b) experiment
result (c) The variation of alpha particles scattered N(θ) with scattering angle θ

146 Unit 8  Atomic and Nuclear physics

UNIT-8(XII-Physics_Vol-2)_Word flow.indd 146 7/31/2019 7:32:47 PM


(d) Very few alpha particles returned back of the nucleus and the alpha particle just
(back scattered) –that is, deflected back before it gets reflected back through 180° is
by 180° defined as the distance of closest approach
In Figure 8.10 (c), the dotted points r0 (also known as contact distance). At this
are the alpha scattering experiment data distance, all the kinetic energy of the alpha
points obtained by Geiger and Marsden particle will be converted into electrostatic
and the solid curve is the prediction from potential energy (Refer unit 1, volume 1 of
Rutherford’s nuclear model. It is observed +2 physics text book).
that the Rutherford’s nuclear model is in
1 mυ 2 = 1 (2e)(Ze)
good agreement with the experimental data. 2 0
4 πε 0 r0
Conclusion made by Rutherford based on
the above observation ⇒ r0 = 1 2 Ze 2 = 1 2 Ze 2
From the experimental observations, 2 (
4 πε 0 1 mυ2
0 )
4 πε 0 Ek

Rutherford proposed that an atom has a lot


of empty space and contains a tiny matter where Ek is the kinetic energy of the
known as nucleus whose size is of the alpha particle.This is used to estimate the
order of 10-14m. The nucleus is positively size of the nucleus but size of the nucleus
charged and most of the mass of the atom is always lesser than the distance of closest
is concentrated in nucleus. The nucleus is approach. Further, Rutherford calculated
surrounded by negatively charged electrons. the radius of the nucleus for different
Since static charge distribution cannot be in nuclei and found that it ranges from
a stable equilibrium, he suggested that the 10–14 m to 10–15 m.
electrons are not at rest and they revolve (b) Impact parameter
around the nucleus in circular orbits like
planets revolving around the sun.
(a) Distance of closest approach

Alpha particle
b + θ
Impact parameter
Alpha particles r0 gold nucleus
distance of closed approach 90°

Figure 8.11  Distance of closest approach Nucleus


and impact parameter Figure 8.12  Impact parameter

When an alpha particle moves straight The impact parameter (b) (see Figure 8.12)
towards the nucleus, it reaches a point is defined as the perpendicular distance
where it comes to rest momentarily and between the centre of the gold nucleus and
returns back as shown in Figure 8.11. The the direction of velocity vector of alpha
minimum distance between the centre particle when it is at a large distance. The
Unit 8  Atomic and Nuclear physics 147

UNIT-8(XII-Physics_Vol-2)_Word flow.indd 147 7/31/2019 7:32:48 PM


relation between impact parameter and Hence, Rutherford model could not account
scattering angle can be shown as for the stability of atoms.
    (b) According to this model, emission
b ∝ cot  θ  ⇒ b = K cot  θ  (8.13) of radiation must be continuous and
2 2
must give continuous emission spectrum
1 2 Ze 2 but experimentally we observe only line
where K = and θ is called
4 πε 0 mυ20 (discrete) emission spectrum for atoms.
scattering angle. Equation (8.13) implies
that when impact parameter increases,
the scattering angle decreases. Smaller 8.3.3  Bohr atom model
the impact parameter, larger will be the In order to overcome the limitations of
deflection of alpha particles. the Rutherford atom model in explaining the
Drawbacks of Rutherford model stability and also the line spectrum observed
for a hydrogen atom (Figure 8.14), Niels Bohr
Rutherford atom model helps in the
made modifications of Rutherford atom
calculation of the diameter of the nucleus
model. He is the first person to give better
and also the size of the atom but has the
theoretical model of the structure of an atom
following limitations:
to explain the line spectrum of hydrogen
(a) This model fails to explain the
atom. The following are the assumptions
distribution of electrons around the nucleus
(postulates) made by Bohr.
and also the stability of the atom.

++ Figure 8.14  The line spectrum of


__
e hydrogen

Postulates of Bohr atom model:


Instability of atom
(a) The electron in an atom moves
Figure 8.13  Spiral in motion of an around nucleus in circular orbits under the
electron around the nucleus influence of Coulomb electrostatic force
of attraction. This Coulomb force gives
According to classical electrodynamics, necessary centripetal force for the electron
any accelerated charge emits electromagnetic to undergo circular motion.
radiations. Due to emission of radiations, (b) Electrons in an atom revolve around
it loses its energy. Hence, it can no longer the nucleus only in certain discrete orbits
sustain the circular motion. The radius of called stationary orbits where it does not
the orbit, therefore, becomes smaller and radiate electromagnetic energy. Only those
smaller (undergoes spiral motion) as shown discrete orbits allowed are stable orbits.
in Figure 8.13 and finally the electron should
The angular momentum of the electron
fall into the nucleus and the atoms should
in these stationary orbits are quantized –
disintegrate. But this does not happen.
that is, it can be written as integer or integral

148 Unit 8  Atomic and Nuclear physics

UNIT-8(XII-Physics_Vol-2)_Word flow.indd 148 7/31/2019 7:32:49 PM


The circumference of an electron’s orbit
multiple of h called as reduced Planck’s
2π of radius r must be an integral multiple of de
constant – that is,  (read it as h-bar) and Broglie wavelength – that is,
the integer n is called as principal quantum
2πr = nλ (8.14)
number of the orbit.
h  where n = 1,2,3,......
l = n  where  =

But the de Broglie wavelength (λ) for an
This condition is known as angular electron of mass m moving with velocity υ is
momentum quantization condition.
λ = h where h is called Planck’s constant.
According to quantum mechanics, mυ
particles like electrons have dual nature (Refer Thus from equation (8.14),
unit 7, volume 2 of +2 physics text book). The
 
standing wave pattern of the de Broglie wave 2πr = n  h 
 mυ 
associated with orbiting electron in a stable
orbit is shown in Figure 8.15. mυr = n h

For any particle of mass m undergoing
circular motion with radius r and velocity υ, the
magnitude of angular momentum l is given by
r l = r (mυ)
mυr = l = n
(c) Energy of orbits are not continuous
but discrete. This is called the quantization
of energy. An electron can jump from one
orbit to another orbit by absorbing or
emitting a photon whose energy is equal
λ
to the difference in energy (ΔE) between
the two orbital levels (Figure 8.16)
∆E = E final − Einitial = hv = h c
λ
n=3 where c is the speed of light and λ is the
wavelength of the radiation used and v is the
frequency of the radiation. Thus, the frequency
Forbidden of the radiation emitted is related only to change
n = 3.3
in atom’s energy and it does not depend on
frequency of electron’s orbital motion.
n=5 _
_e
E E
f f
hv hv
_ _
_ E E _
e i i e

Figure 8.15  Standing wave pattern for Figure 8.16  Absorption and emission of
electron in a stable orbit radiation

Unit 8  Atomic and Nuclear physics 149

UNIT-8(XII-Physics_Vol-2)_Word flow.indd 149 7/31/2019 7:32:51 PM


E X A M P L E 8. 1 Proton is assumed to be stationary
The radius of the 5 orbit of hydrogen
th
Electron revolves in nth orbit
atom is 13.25 Å. Calculate the wavelength of radius rn with
speed υn
of the electron in the 5th orbit.
υn
Solution: rn
Proton _
+
2πr = nλ M, +e F Electron
m, e_
2 × 3.14 × 13.25Å = 5 × λ Electrostatic attraction
provides centripetal
∴ λ = 16.64Å acceleration

E X A M P L E 8. 2
Figure 8.17  Electron revolving around
Find the (i) angular momentum the nucleus
(ii) velocity of the electron in the 5th orbit
of hydrogen atom. Let Z be the atomic number of the atom,
then +Ze is the charge of the nucleus. Let
(h = 6.6 × 10–34 Js, m = 9.1 × 10–31 kg) –e be the charge of the electron. From
Solution Coulomb’s law, the force of attraction
(i) Angular momentum is given by between the nucleus and the electron is
 (+Ze)(− −ee)) ˆr 
l = n = nh FFcoloumb = 11 (+Ze)(
coloumb =
2π πε00
44πε rrnn
22

−34
= 5× 6.6 ×10 = 5.25×10−34 kgm2s−1 =−
= − 11 Ze Ze22
ˆr 
2 × 3.14 πε00 rrnn22
44πε
This force provides necessary centripetal
(ii) Velocity is given by
force
Velocity υ = l  mυn2 ˆr 
mr F centripetal =
(5.25×10−34 kgm 2s−1 ) rn
= where m be the mass of the electron that
(9.1×10−31 kg)(13.25×10−10 m)
moves with a velocity υn in a circular orbit.
υ= 4.4 ×105 ms−1
Therefore,
 
Radius of the orbit of the electron and F coloumb = F centripetal
velocity of the electron 2
Consider an atom which contains the 1 Ze 2 = mυn
4 πε 0 rn2 rn
nucleus at rest and an electron revolving
around the nucleus in a circular orbit of 4 πε 0 (mυnrn )2
radius rn as shown in Figure 8.17. Nucleus rn = (8.15)
Zme 2
is made up of protons and neutrons. Since
From Bohr’s assumption, the angular
proton is positively charged and neutron is
momentum quantization condition,
electrically neutral, the charge of a nucleus
mυnrn = ln = n,
is purely the total charge of protons.

150 Unit 8  Atomic and Nuclear physics

UNIT-8(XII-Physics_Vol-2)_Word flow.indd 150 7/31/2019 7:32:53 PM


4 πε 0 (mυnrn )2
∴ rn =
Zme 2
4 πε 0 (n)2 4 πε 0n2 2
rn = =
Zme 2 Zme 2 r2= 4r1
 ε h2  n2 ∴ h r3= 9r1
   rn =  0
    (  = )(8.16)
 πme  Z
2 2π r1

where nÎ . Since, ε0, h, e and π are


constants. Therefore, the radius of the orbit
r4= 16r1
becomes
n2
rn = a0
Z Figure 8.18  Variation of radius of the
ε 0h 2 orbit with principal quantum number
where a0 = = 0.529 Å. This is
πme 2
known as Bohr radius which is the smallest υn = h Z
radius of the orbit in an atom. Bohr radius 2πma0 n
is also used as unit of length called Bohr.
1 Bohr = 0.53 Å. For hydrogen atom (Z = 1), υn µ 1
n
the radius of nth orbit is
Note that the velocity of electron
rn = a0n2 decreases as the principal quantum number
For the first orbit (ground state), increases as shown in Figure 8.19. This curve
is the rectangular hyperbola. This implies
r1 = a0 = 0.529 Å that the velocity of electron in ground state
For the second orbit (first excited state), is maximum when compared to excited
states.
r2 = 4a0 = 2.116 Å
For the third orbit (second excited state), Vn

r3 = 9a0 = 4.761 Å
and so on. υn µ 1
n
Thus the radius of the orbit from centre
increases with n, that is, rn ∝ n2 as shown in
Figure 8.18. O n
Further, Bohr’s angular momentum
quantization condition leads to Figure 8.19  Variation of velocity of
the electron in the orbit with principal
mυnrn = mυna0n2 = n h quantum number

Unit 8  Atomic and Nuclear physics 151

UNIT-8(XII-Physics_Vol-2)_Word flow.indd 151 7/31/2019 7:32:55 PM


The energy of an electron in the nth orbit Notice that the energy of the first excited
Since the electrostatic force is a state is greater than the ground state, second
conservative force, the potential energy for excited state is greater than the first excited
the nth orbit is state and so on. Thus, the orbit which is
(+Ze)(−e) 2 closest to the nucleus (r1) has lowest energy
Un = 1 = − 1 Ze
4 πε 0 rn 4 πε 0 rn (minimum energy compared with other
1 Z 2
me 4  ε 0h2 n2  orbits). So, it is often called ground state
= − 2 2 2  rn =  
4ε0 h n  πme 2 Z  energy (lowest energy state). The ground
state energy of hydrogen (–13.6 eV ) is
The kinetic energy for the nth orbit is used as a unit of energy called Rydberg
4 2 (1 Rydberg = –13.6 eV ).
1 2 me
KEn = mυn = 2 2 2 Z
2 The negative value of this energy is
8 ε 0h n
because of the way the zero of the potential
This implies that Un = –2 KEn. Total energy is defined. When the electron is
energy in the nth orbit is taken away to an infinite distance (very far
distance) from nucleus, both the potential
En = KEn + U n = KEn − 2KEn = − KEn
energy and kinetic energy terms vanish
4 2
En = − me2 2 Z2 and hence the total energy also vanishes.
8ε 0h n The energy level diagram along with the
For hydrogen atom (Z = 1), shape of the orbits for increasing values of n
are shown in Figure 8.20. It shows that the
4
energies of the excited states come closer and
En = − me2 2 12 joule (8.17)
8 ε 0h n closer together when the principal quantum
number n takes higher values.
where n stands for principal quantum
number. The negative sign in equation E X A M P L E 8. 3
(8.17) indicates that the electron is bound
(a) Show that the ratio of velocity of an
to the nucleus.
electron in the first Bohr orbit to the speed
Substituting the values of mass and
of light c is a dimensionless number.
charge of an electron (m and e), permittivity
of free space ε0 and Planck’s constant h and (b) Compute the velocity of electrons in
expressing in terms of eV, we get ground state, first excited state and second
excited state in Bohr atom model.
En = −13.6 12 eV
n Solution

 or the first orbit (ground state), the total


F (a) The velocity of an electron in nth orbit is
energy of electron is E1= – 13.6 eV. υn = h Z
For the second orbit (first excited state), 2πma0 n
the total energy of electron is E2= –3.4 eV. ε 0h 2
For the third orbit (second excited state), where a0 = = Bohr radius. Substituting
πme 2
the total energy of electron is E3= –1.51 eV for a0 in υn,
and so on.

152 Unit 8  Atomic and Nuclear physics

UNIT-8(XII-Physics_Vol-2)_Word flow.indd 152 7/31/2019 7:32:57 PM


Free electron

Energy, J Energy, eV
n=∞ 0 0
−19
n=5 —0.87×10 —0.54
n=4 −19
—1.36×10 —0.85
n=3 −19
—2.42×10 —1.51 Excited states

n=2 −19
—5.43×10 —3.40

n=1
—21.76×10−19 —13.6 Ground state

Figure 8.20  Energy levels of a hydrogen atom

2  2  (1.6 ×10−19 C)2


υn = e Z = c  e  Z = αcZ α=
2 ε 0h n  2ε 0hc  n n 2 ×(8.854 ×10−12 C 2 N−1m−2 )
where c is the speed of light in free space or (1.6 ×10−19 C)2
vacuum and its value is c = 3 × 108 m s–1and (6.6 ×10−34 Nms)×(3×108 ms−1 )
α is called fine structure constant. υ1 2
=α= e
For a hydrogen atom, Z = 1 and for the first c 2ε 0hc
orbit, n = 1, the ratio of velocity of electron 1 1
≈ = which is a dimensionless
in first orbit to the speed of light in vacuum 136.9 137
number
or free space is

Unit 8  Atomic and Nuclear physics 153

UNIT-8(XII-Physics_Vol-2)_Word flow.indd 153 7/31/2019 7:32:58 PM


⇒α= 1
137
(b) Using fine structure constant, the
velocity of electron can be written as

υn = αcZ
n m M
For hydrogen atom (Z = 1) the velocity of COM
electron in nth orbit is r2 r1
υn = c 1 = (2.19 ×106 ) 1 ms−1
137 n n
For the first orbit (ground state), the
Let V be the velocity of the nuclear motion
velocity of electron is
and υ be the velocity of electron motion.
υ1 = 2.19 ×106 ms−1 Since the total linear momentum of the
system is zero,
For the second orbit (first excited state),
the velocity of electron is −mυ + M υ = 0 or

υ2 = 1.095×106 ms−1 MV = mυ = p
  
pe + pn = 0 or
For the third orbit (second excited state),  
the velocity of electron is p e = pn = p
Hence, the kinetic energy of the system is
υ3 = 0.73×106 ms−1
pn2 p2 p2  1 1 
Here, υ1 > υ2 > υ3 KE = + e =  + 
2 M 2m 2  M m 
Let 1 + 1 = 1 . Here the reduced mass
E X A M P L E 8. 4 M m µm

The Bohr atom model is derived with the is, µ m = mM


M +m
assumption that the nucleus of the atom
is stationary and only electrons revolve Therefore, the kinetic energy of the system
around the nucleus. Suppose the nucleus is p2
now is KE =
also in motion, then calculate the energy of 2µ m
this new system.
Since the potential energy of the system is
Solution same, the total energy of the hydrogen can
be expressed by replacing mass by reduced
Let the mass of the electron be m and
mass, which is
mass of the nucleus be M. Since there is
µ e4
no external force acting on the system, the En = − m2 2 12
centre of mass of hydrogen atom remains 8 ε 0h n
at rest. Hence, both nucleus and electron Since the nucleus is very heavy compared
move about the centre of mass as shown in to the electron, the reduced mass is closer
figure. to the mass of the electron.

154 Unit 8  Atomic and Nuclear physics

UNIT-8(XII-Physics_Vol-2)_Word flow.indd 154 7/31/2019 7:33:01 PM


EI = E2 – E1 = –3.4 eV – (–13.6 eV) = 10.2 eV
In 1931, H.C. Urey and co-
Note workers noticed that in the Similarly, the excitation energy for
shorter wavelength region an electron from ground state (n = 1) to
of the hydrogen spectrum lines, faint second excited state (n = 3) is called second
companion lines are observed. From excitation energy, which is
the isotope displacement effect (isotope
EII = E3 – E1 = –1.51 eV – (–13.6 eV) = 12.1 eV
shift), the isotope of the same element will
have slightly different spectral lines. The and so on.
presence of these faint lines confirmed Excitation potential is defined as
the existence of isotopes of hydrogen
excitation energy per unit charge.
atom (which is named as Deuterium).
First excitation potential is,
On calculating wavelength or wave
number difference between the faint EI = eVI ⇒ VI = 1 EI = 10.2 volt
e
and bright spectral lines, atomic mass
Second excitation potential is,
of deuterium is measured to be twice
that of atomic mass of hydrogen atom. EII = eVII ⇒ VII = 1 EII = 12.1 volt
Bohr atom model could not explain e
this isotopic shift. Thus by considering and so on.
nuclear motion (although the movement
of the nucleus is much smaller, it is Ionization energy and ionization potential
observed) into account in the Bohr atom An atom is said to be ionized when an
model, the wave number or wavelength electron is completely removed from the
difference between hydrogen atom and atom – that is, it reaches the state with energy
deuterium is theoretically calculated En→∞ . The minimum energy required to
which perfectly agreed with the remove an electron from an atom in the
spectroscopic measured values. ground state is known as binding energy
The difference between hydrogen or ionization energy.
atom and deuterium is in the number Eionizaation = E∞ − E1 = 0 − (−13.6 eV)
of neutron. Hydrogen atom contains = 13.6 eV
an electron and a proton, whereas
When an electron is in nth state of an atom,
deuterium has an electron, a proton
the energy spent to remove an electron from
and a neutron.
that state – that is, its ionization energy is
 
Excitation energy and excitation potential Eionization = E∞ − En = 0 − − 132.6 Z 2 eV
 n 
The energy required to excite an
electron from lower energy state to any = 132.6 Z 2 eV
n
higher energy state is known as excitation
energy. At normal room temperature, the
The excitation energy for an electron electron in a hydrogen atom (Z=1) spends
from ground state (n = 1) to first excited most of its time in the ground state. The
state (n = 2) is called first excitation energy, amount of energy spent to remove an
which is electron from the ground state of an atom

Unit 8 Atomic and Nuclear physics 155

UNIT-8(XII-Physics_Vol-2)_Word flow.indd 155 7/31/2019 7:33:02 PM


Table 8.1
Physical quantity Ground state First excited state Second excited state
Radius (rn ∝ n2) 0.529 Å 2.116 Å 4.761 Å

Velocity (vn ∝ n-1) 2.19 × 106 m s-1 1.095 × 106 m s-1 0.73 × 106 m s-1

Total Energy (En ∝ n-2) –13.6 eV –3.4 eV –1.51 eV

(E = 0 for n→∞) is known as first ionization (c) When a photon with energy 42 eV
energy (13.6 eV). Then, the hydrogen atom and another photon with energy 56 eV are
is said to be in ionized state or simply called made to collide with this atom, does this
as hydrogen ion, denoted by H+. If we supply atom absorb these photons?
more energy than the ionization energy, the (d) Determine the radius of its first Bohr
excess energy will be the kinetic energy of orbit.
the free electron.
Ionization potential is defined as (e) Calculate the kinetic and potential
ionization energy per unit charge. energies in the ground state.
Solutions
Vionization = 1 Eionization = 132.6 Z 2 V
e n (a) Given that
Thus, for a hydrogen atom (Z =1), the
ionization potential is En = − 542.4 eV
n
V = 132.6 volt For n = 1, the ground state energy
n E1 = –54.4 eV and for n = 2, E2 = –13.6 eV.
The radius, velocity and total energy in Similarly, E3 = –6.04 eV, E4 = –3.4 eV and
ground state, first excited state and second so on.
excited state is listed in Table 8.1. For large value of principal quantum
number – that is, n = ∞, we get E∞ = 0 eV.
E X A M P L E 8. 5
0 n=∞
—3.4 n=4
Suppose the energy of a hydrogen–like —6.04 n=3
atom is given as En = − 542.4 eV where —13.6 n=2
n
nÎ . Calculate the following: —54.4(eV) n=1

(a)  Sketch the energy levels for this atom (b) For a hydrogen-like atom, ground state
and compute its atomic number. energy is
(b) If the atom is in ground state, compute
its first excitation potential and also its E1 = − 132.6 Z 2 eV
n
ionization potential.

156 Unit 8  Atomic and Nuclear physics

UNIT-8(XII-Physics_Vol-2)_Word flow.indd 156 7/31/2019 7:33:03 PM


where Z is the atomic number. Hence, But note that E2 – E1 ≠ 42 eV, E3 – E1 ≠ 42 eV,
comparing this energy with given energy, E4 – E1 ≠ 42 eV and E3 – E2 ≠ 42 eV.
we get, – 13.6 Z2 = – 54.4 ⇒ Z = ±2. Since,
atomic number cannot be negative number, For all possibilities, no difference in energy
Z = 2. is an integer multiple of photon energy.
Hence, photon A is not absorbed by this
(c) The first excitation energy is atom. But for Photon B, E4 – E1 = 51 eV,
EI = E2 − E1 = −13.6 eV − (−54.4 eV) which means, Photon B can be absorbed
= 40.8 eV by this atom.

Hence, the first excitation potential is (e) Since total energy is equal to negative of
kinetic energy in Bohr atom model, we get
(40.8 eV)
VI = 1 EI =
e e  
KEn = −En = −− 542.4 eV
= 40.8 volt  n 
The first ionization energy is = 542.4 eV
n
Eionization = E∞ − E1 = 0 − (−54.4 eV)
Potential energy is negative of twice the
= 54.4 eV
kinetic energy, which means,
Hence, the first ionization potential is  
(54.4 eV) U n = −2KEn = −2  542.4 eV
 n 
Vionization = 1 Eionization =
e e
= 54.4 volt = − 1082.8 eV
n
(d) Consider two photons to be A and B. For a ground state, put n =1
Given that photon A with energy 42 eV Kinetic energy is KE1 = 54.4 eV and
and photon B with energy 51 eV Potential energy is U1 = –108.8 eV
From Bohr assumption, difference in energy
levels is equal to photon energy, then atom 8.3.4  Atomic spectra
will absorb energy, otherwise, not.
Materials in the solid, liquid and gaseous
E2 − E1 = −13.6 eV − (−54.4 eV)
states emit electromagnetic radiations
= 40.8 eV ≈ 41eV when they are heated up and these emitted
Similarly, radiations usually belong to continuous
E3 − E1 = −6.04 eV − (−54.4 eV) spectrum. For example, when white light
= 48.36 eV is examined through a spectrometer,
electromagnetic radiations of all wavelengths
E4 − E1 = −3.4 eV − (−54.4 eV) are observed which is a continuous spectrum.
= 51eV In early twentieth century, many scientists
E3 − E2 = −6.04 eV − (−13.6 eV) spent considerable time in understanding the
= 7.56 eV characteristic radiations emitted by the atoms
of individual elements exposed to a flame or
and so on. electrical discharge. When they are viewed

Unit 8  Atomic and Nuclear physics 157

UNIT-8(XII-Physics_Vol-2)_Word flow.indd 157 7/31/2019 7:33:06 PM


High voltage
difference
Diffraction
grating
Line spectrum

Low-pressure gas

Figure 8.21  Spectrum of an atom

or photographed, instead of a continuous photons. Once the electrons get sufficient


spectrum, the radiation contains of a set energy as given by Bohr’s postulate (c), it
of discrete lines, each with characteristic absorbs energy with particular wavelength
wavelength. In other words, the wavelengths (or frequency) and jumps from its stationary
of the light obtained are well defined and the state (original state) to higher energy state.
positions and intensities are characteristic of Those wavelengths (or frequencies) for
the element as shown in Figure 8.21. which the colours are not observed are seen
This implies that these spectra are unique as dark lines in the absorption spectrum as
to each element and can be used to identify shown in Figure 8.22 (b).
the element of the gas (like finger print used Hydrogen spectrum
to identify a person) – that is, it varies from n=5 n=4 n=3 n=5 n=4
one gas to another gas. This uniqueness of
line spectra of elements made the scientists
to determine the composition of stars, sun λ(nm)
and also used to identify the unknown
400 500 600 700 400 500
compounds.
(a) Emission spectrum of hydrogen (b) Absorption
Hydrogen spectrum (in wavelength) (in w
When the hydrogen gas enclosed Hydrogen
in a spectrum
tube is heated
n = 5up,nit
= 4emits electromagnetic
n=3 n=5 n=4 n=3
radiations of certain sharply-defined
characteristic wavelength (line spectrum),
called hydrogen emission spectrum (Refer
λ(nm) λ(nm)
unit 5, volume 1 of +2 physics text book).
The emission
400 spectra500 of hydrogen
600are shown700 400 500 600 700
(a) Emission
in Figure 8.22(a). spectrum of hydrogen (b) Absorption spectrum of hydrogen
(in wavelength)
When any gas is heated up, the thermal (in wavelength)
energy is supplied to excite the electrons.
Similarly by passing light on the atoms, Figure 8.22  Hydrogen spectrum
electrons can be excited by absorbing (a) emission (b) absorption

158 Unit 8  Atomic and Nuclear physics

UNIT-8(XII-Physics_Vol-2)_Word flow.indd 158 7/31/2019 7:33:06 PM


Since electrons in excited states have very limit. These series are named as Lyman series,
small life time, these electrons jump back to Balmer series, Paschen series, Brackett series,
ground state through spontaneous emission Pfund series, etc. The wavelengths of these
in a short duration of time (approximately spectral lines perfectly agree with the equation
10–8 s) by emitting the radiation with same derived from Bohr atom model.
wavelength (or frequency) corresponding
1 = R  1 − 1  = v
to the colours it absorbed (Figure 8.22 (a)). λ  n2 m2   (8.18)

This is called emission spectroscopy.
The wavelengths of these lines can be where v is known as wave number
calculated with great precision. Further, the which is inverse of wavelength, R is known
emitted radiation contains wavelengths both as Rydberg constant whose value is 1.09737
lesser and greater than the visible spectrum. x 107 m-1and m and n are positive integers
such that m > n. The various spectral series
n=5 Ionised atom
are discussed below:
n=4 (Continuous energy levels)
E=0
—0.85
—1.5 n=3 Excited (a) Lyman series
Paschen series states
—3.4
n=2 Put n = 1 and m = 2,3,4....... in equation
Balmer
—5 series (8.18). The wave number or wavelength of
Energy (eV)

The lowest energy level is


called the ground state and
spectral lines of Lyman series which lies in
all other states are called as
excited states
ultra-violet region is
—10

 
n=1 Ground state v = 1 = R  12 − 12 
—13.6
—15 Lyman
λ 1 m 
c hc (to calculate wavelengths
λ= =
series v E2— E1 and frequencies)
(b) Balmer series
Wavelength, λ Put n = 2 and m = 3,4,5....... in equation
(8.18). The wave number or wavelength of
1875 nm
820 nm
122 nm

365 nm

656 nm
91 nm

spectral lines of Balmer series which lies in


visible region is
 
v = 1 = R  12 − 12 
Lyman
λ 2 m 
series Balmer series Paschen series

(c) Paschen series


UV Visible light IR
Put n = 3 and m = 4,5,6....... in equation
Figure 8.23  Spectral series – Lyman, (8.18). The wave number or wavelength of
Balmer, Paschen series
spectral lines of Paschen series which lies in
infra-red region (near IR) is
Notice that the spectral lines of hydrogen as
shown in Figure 8.23 are grouped in separate  
v = 1 = R  12 − 12 
series. In each series, the distance of separation λ 3 m 
between the consecutive wavelengths
(d) Brackett series
decreases from higher wavelength to the lower
wavelength, and also wavelength in each series Put n = 4 and m = 5,6,7........ in equation
approach a limiting value known as the series (8.18). The wave number or wavelength of

Unit 8  Atomic and Nuclear physics 159

UNIT-8(XII-Physics_Vol-2)_Word flow.indd 159 7/31/2019 7:33:07 PM


spectral lines of Brackett series which lies in 8.4
infra-red region (middle IR) is NUCLEI
 
v = 1 = R  12 − 12 
λ 4 m  Introduction
(e) Pfund series In the previous section, we have
discussed various preliminary atom models,
Put n = 5 and m = 6,7,8........ in equation
Rutherford’s alpha particle scattering
(8.18). The wave number or wavelength of
experiment and Bohr atom model. These
spectral lines of Pfund series which lies in
played a vital role to understand the structure
infra-red region (far IR) is
of the atom and the nucleus. In this section,
 
v = 1 = R  12 − 12  the structure and the properties of the
λ 5 m  nucleus, and its classifications are discussed.
Different spectral series are listed in
Table 8.2. 8.4.1  Composition of nucleus
Atoms have a nucleus surrounded by
Table 8.2
electrons. The nucleus contains protons
n m Series Name Region and neutrons. The neutrons are electrically
1 2,3,4..... Lyman Ultraviolet neutral (q = 0) and the protons have positive
2 3,4,5..... Balmer Visible charge (q =+ e) equal in magnitude of the

charge of the electron (q = –e). The number


3 4,5,6..... Paschen Infrared
of protons in the nucleus is called the
4 5,6,7...... Brackett Infrared atomic number and it is denoted by Z. The
5 6,7,8..... Pfund Infrared number of neutrons in the nucleus is called
neutron number (N). The total number
Limitations of Bohr atom model of neutrons and protons in the nucleus is
The following are the drawbacks of Bohr called the mass number and it is denoted
atom model by A. Hence, A = Z+N.
(a) Bohr atom model is valid only for The two constituents of nucleus namely
hydrogen atom or hydrogen like-atoms neutrons and protons, are collectively
but not for complex atoms. called nucleons. The mass of a proton is
(b) When the spectral lines are closely 1.6726 ×10−27 kg which is roughly 1836
examined, individual lines of hydrogen times the mass of the electron. The mass of
spectrum is accompanied by a number a neutron is slightly greater than the mass of
of faint lines. These are often called fine the proton and it is equal to 1.6749 ×10−27 kg.
structure. This is not explained by Bohr To specify the nucleus of any element, we
atom model. use the following general notation
(c) Bohr atom model fails to explain the A
X
Z
intensity variations in the spectral lines.
where X is the chemical symbol of
(d) The distribution of electrons in atoms is
the element, A is the mass number and
not completely explained by Bohr atom
Z is the atomic number. For example,
model.
160 Unit 8  Atomic and Nuclear physics

UNIT-8(XII-Physics_Vol-2)_Word flow.indd 160 7/31/2019 7:33:09 PM


the nitrogen nucleus is represented by only in nuclear reactions in the laboratory
7N . It implies that nitrogen contains or by cosmic rays.
15

15 nucleons of which 7 are protons The chemical properties of any atom are
(Z = 7) and 8 are neutrons (N = A – Z = 8). determined only by electrons, the isotopes
Note that once the element is specified, of any element have same electronic
the value of Z is known and subscript Z is structure and same chemical properties. So
sometimes omitted. For example, nitrogen the isotopes of the same element are placed
nucleus is simply denoted as 15 N and we call in the same location in the periodic table.
it as ‘nitrogen fifteen’. Isobars:
Since the nucleus is made up of positively
Isobars are the atoms of different
charged protons and electrically neutral
elements having the same mass number
neutrons, the overall charge of the nucleus is
A, but different atomic number Z. In other
positive and it has the value of +Ze. But the
words, isobars are the atoms of different
atom is electrically neutral which implies that
chemical element which has same number
the number of electrons in the atom is equal
of nucleon. For example 1640 S , 1740Cl , 1840 Ar ,
to the number of protons in the nucleus.
19 K and 20 Ca are isobars having same mass
40 40

8.4.2  I  sotopes, isobars, and number 40 and different atomic number.


isotones Unlike isotopes, isobars are chemically
Isotopes: different elements. They have different
physical and chemical properties.
In nature, there are atoms of a particular
element whose nuclei have same number of Isotones:
protons but different number of neutrons. Isotones are the atoms of different
These kinds of atoms are called isotopes. elements having same number of neutrons.
In other words, isotopes are atoms of 5 B and 6 C are examples of isotones which
12 13

the same element having same atomic 7 neutrons.


number Z, but different mass number A.
For example, hydrogen has three isotopes 8.4.3  A
 tomic and nuclear
and they are represented as 11H (hydrogen), masses
1 H (deuterium),and 1 H (tritium). Note that The mass of nuclei is very small when it
2 3

all the three nuclei have one proton and, expressed in SI units (about 10-25 kg or less).
hydrogen has no neutron, deuterium has 1 Therefore, it is more convenient to express it
neutron and tritium has 2 neutrons. in terms of another unit namely, the atomic
The number of isotopes for the particular mass unit (u). One atomic mass unit (u)
element and their relative abundances is defined as the 1/12th of the mass of the
(percentage) vary with each element. For isotope of carbon 126 C , the most abundant
example, carbon has four main isotopes: naturally occurring isotope of carbon.
6 C , 6 C , 6 C and 6 C . But in nature, the In other words
11 12 13 14

percentage of 6 C is approximately 98.9%,


12

that of 136 C is 1.1% and that of 146 C is mass of 126 C atom 1.9926 ×10−26
1u= =
0.0001%. The other carbon isotope 116 C , do 12 12
−27
not occur naturally and it can be produced = 1.660 ×10 kg

Unit 8  Atomic and Nuclear physics 161

UNIT-8(XII-Physics_Vol-2)_Word flow.indd 161 7/31/2019 7:33:20 PM


In terms of this atomic mass unit, the different methods have been carried out
mass of the neutron = 1.008665 u, the on the nuclei of various atoms. The nuclei
mass of the proton =1.007276 u, the mass are found to be approximately spherical in
of the hydrogen atom = 1.007825 u and the shape. It is experimentally found that radius
mass of 126 C = 12u. Note that usually mass of nuclei for Z > 10, satisfies the following
specified is the mass of the atoms, not mass empirical formula
of the nucleus. To get the nuclear mass of
1
particular nucleus, the mass of electrons has R = R0 A 3  (8.19)
to be subtracted from the corresponding
atomic mass. Experimentally the atomic Here A is the mass number of the nucleus
mass is determined by the instrument and the constant R0 = 1.2 F, where 1 F =
called Bainbridge mass spectrometer. If we 1 × 10–15 m.The unit fermi (F) is named after
determine the atomic mass of the element Enrico Fermi.
without considering the effect of its isotopes,
we get the mass averaged over different
isotopes weighted by their abundances. E X A M P L E 8. 7
197
Calculate the radius of 79 Au nucleus.
E X A M P L E 8. 6
Solution
Calculate the average atomic mass of
chlorine if no distinction is made between According to the equation (8.19),
its different isotopes? 1
R = 1.2 × 10 −15 × (197) 3 = 6.97 × 10 −15 m
Solution Or R = 6.97 F
The element chlorine is a mixture of
75.77% of 1735Cl and 24.23% of 17
37
Cl . So the
average atomic mass will be E X A M P L E 8. 8
75.77 24.23 Calculate the density of the nucleus with
× 34.96885u + × 36.96593u
100 100 mass number A.
= 35.453u
Solution
In fact, the chemist uses the average atomic
mass or simply called chemical atomic From equation (8.19), the radius of the
1
weight (35.453 u for chlorine) of an element. nuclei satisfy the equation = R0 A 3 . Then
So it must be remembered that the atomic the volume of the nucleus
mass which is mentioned in the periodic
table is basically averaged atomic mass. V = 4 πR 3 = 4 πR03 A
3 3
By ignoring the mass difference between
8.4.4  S
 ize and density of the the proton and neutron, the total mass of
nucleus the nucleus having mass number A is equal
The alpha particle scattering experiment to A.m where m is mass of the proton and
and many other measurements using is equal to 1.6726 x 10-27 kg.

162 Unit 8  Atomic and Nuclear physics

UNIT-8(XII-Physics_Vol-2)_Word flow.indd 162 7/31/2019 7:33:22 PM


Mass of 6 electrons = 6 × 0.00055 u = 0.0033 u
+ n + Proton
n n + The expected mass of carbon-12 nucleus
+ n
+ + = 6.05196 u + 6.04362 u = 12.09558 u
n
n + + But using mass spectroscopy, the
+ n n Neutron
atomic mass of carbon-12 atom is found
to be 12 u. So if we subtract the mass of 6
Nuclear density electrons (0.0033  u) from 12 u, we get the
carbon-12 nuclear mass which is equal
ρ = mass of the nuclei = A.m = m to 11.9967 u. Note that the experimental
Volume ofthe nuclei 4 πR 3 A 4 πR 3
3 0 3 0 mass of carbon-12 nucleus is less than the
total mass of its individual constituents by
The above expression shows that the
∆m = 0.09888 u . This difference in mass ∆m
nuclear density is independent of the mass
is called mass defect. In general, if M, mp,
number A. In other words, all the nuclei
and mn are mass of the nucleus ( ZA X ), the
(Z > 10) have the same density and it is an
mass of a proton and the mass of a neutron
important characteristics of the nuclei.
respectively, then the mass defect is given by
We can calculate the numerical
value of this density by substituting the
corresponding values.
( )
∆m = Zm p + Nmn − M (8.20)

Where has this mass disappeared? The


ρ= 1.67 ×10−27 = 2.3×1017 kg m-3.
4 π ×(1.2 ×10−15 )3 answer was provided by Albert Einstein
3 with the help of famous mass-energy
relation ( E = mc 2 ) . According to this
It implies that nucleons are extremely
relation, the mass can be converted into
tightly packed in the nucleus and compare
energy and energy can be converted into
this density with the density of water which
mass. In the case of the carbon-12 nucleus,
is 103 kg m-3.
when 6 protons and 6 neutrons combine to
form carbon-12 nucleus, mass equal to mass
A single teaspoon of nuclear
defect disappears and the corresponding
matter would weigh about
energy is released. This is called the binding
trillion tons.
energy of the nucleus (BE) and is equal to
( ∆m) c 2 . In fact, to separate the carbon-12
8.4.5  Mass defect and nucleus into individual constituents, we
binding energy must supply the energy equal to binding
energy of the nucleus.
It is experimentally found out that the
We can write the equation (8.20) in
mass of any nucleus is always less than the
terms of binding energy
sum of the mass of its individual constituents.
For example, consider the carbon-12 nucleus
which is made up of 6 protons and 6 neutrons. ( )
BE = Zm p + Nmn − M c 2 (8.21)
Mass of 6 neutrons = 6 ×1.00866 u = 6.05196 u
It is always convenient to work with
Mass of 6 protons = 6 × 1.00727 u = 6.04362 u
the mass of the atom than the mass of the
Unit 8  Atomic and Nuclear physics 163

UNIT-8(XII-Physics_Vol-2)_Word flow.indd 163 7/31/2019 7:33:27 PM


nucleus. Hence by adding and subtracting The binding energy of the 24 He nucleus is
the mass of the Z electrons, we get 28 MeV.

(
BE = Zm p + Zme + Nmn − M − Zme c 2 ) (8.22)
8.4.6 Binding energy curve
( )
BE =  Z m p + me + Nmn − M − Zme  c 2 In the previous section, the origin of the
where m p + me = mH (mass of hydrogen binding energy is discussed. Now we can
atom) find the average binding energy per nucleon
BE . It is given by
BE = [ ZmH + Nmn − ( M + Zme ) ] c 2 (8.23)
BE =
[ Zm H + Nmn − M A ] c 2
(8.25)
Here M + Zme = M A where M A is the A
mass of the atom of an element ZA X .
The average binding energy per nucleon
Finally, the binding energy in terms of
is the energy required to separate single
the atomic masses is given by
nucleon from the particular nucleus. BE
BE = [ ZmH + Nmn − M A ] c 2 (8.24) is plotted against A of all known nuclei. It
gives a curve as seen in Figure 8.24.
10
Using Einstein’s mass-energy 16
8O 56
26 Fe
120
avg. binding energy per nucleon (MeV)

50 Sn
Note equivalence, the energy 4
2He
8 238
92 U
equivalent of one atomic
mass unit 1u = 1.66 × 10−27 × (3 × 108 ) 2 6
= 14.94 × 10 −11 J ≈ 931MeV
4

E X A M P L E 8. 9 2
3
2He

2
1H
Compute the binding energy of He 4
2
0
nucleus using the following data: Atomic 0 50 100 150 200 250
Number of nucleons, A (mass number)
mass of Helium atom, M A ( He) = 4.00260 u
and that of hydrogen atom, mH = 1.00785 u. Figure 8.24 Avg. binding energy of the
nucleons
Solution:
Important inferences from of the average
Binding energy BE = [ ZmH + Nmn − M A ] c 2
binding energy curve:
For helium nucleus, Z = 2, N = A–Z = 4–2 = 2
(1) The value of BE rises as the mass number
Mass defect increases until it reaches a maximum
∆m = ( 2 × 1.00785 u ) + ( 2 × 1.008665 u ) − 4.00260 u  value of 8.8 MeV for A = 56 (iron) and
then it slowly decreases.
0785 u ) + ( 2 × 1.008665 u ) − 4.00260 u  ∆m = 0.03038 u
(2) The average binding energy per nucleon
B.E = 0.03038 u × c 2 is about 8.5 MeV for nuclei having mass
B.E = 0.03038 × 931MeV = 28 MeV number between A = 40 and 120. These
elements are comparatively more stable
1uc = 931MeV 
2
and not radioactive.

164 Unit 8 Atomic and Nuclear physics

UNIT-8(XII-Physics_Vol-2)_Word flow.indd 164 7/31/2019 7:33:41 PM


(3) For higher mass numbers, the curve the electrostatic repulsive force between two
reduces slowly and BE for uranium is protons separated by a distance 10−15 m
about 7.6 MeV. They are unstable and
( )
2
radioactive. q2 1.6 × 10 −19
F = k × 2 = 9 × 10 ×
9
≈ 230 N
From Figure 8.24, if two light nuclei with r ( )
2
10 −15
A<28 combine with a nucleus with A<56,
the binding energy per nucleon is more for The acceleration experienced by a proton
final nucleus than initial nuclei. Thus, if due to the force of 230 N is
the lighter elements combine to produce a F 230 N
nucleus of medium value A, a large amount a= = −27
≈ 1.4 × 1029 m s −2 .
m 1.67 × 10 kg
of energy will be released. This is the basis
of nuclear fusion and is the principle of the This is nearly 1028 times greater than the
hydrogen bomb. acceleration due to gravity. So if the protons in
(4) If a nucleus of heavy element is split the nucleus experience only the electrostatic
(fission) into two or more nuclei of force, then the nucleus would fly apart in an
medium value A, the energy released instant. Then how protons are held together in
would again be large. The atom bomb is nucleus?
based on this principle and huge energy of From this observation, it was concluded
atom bombs comes from this fission when that there must be a strong attractive force
it is uncontrolled. Fission is explained in between protons to overcome the repulsive
the section 8.7 Coulomb’s force. This attractive force which
holds the nucleus together is called strong
E X A M P L E 8. 10
nuclear force. The properties of strong nuclear
Compute the binding energy per nucleon force were understood through various
of 24 He nucleus. experiments carried out between 1930s and
1950s. A few properties of strong nuclear force
Solution
are
From example 8.9, we found that the BE of (i) The strong nuclear force is of very short
2 He =28 Mev range, acting only up to a distance of a
4

few Fermi. But inside the nucleus, the


Binding energy per nucleon = B.E = 28
repulsive Coulomb force or attractive
MeV/4 = 7 MeV.
gravitational forces between two protons
are much weaker than the strong nuclear
8.5 force between two protons. Similarly, the
NUCLEAR FORCE gravitational force between two neutrons
is also much weaker than strong nuclear
force between the neutrons. So nuclear
Nucleus contains protons and neutrons.
force is the strongest force in nature.
From electrostatics, we learnt that like charges
(ii) The strong nuclear force is attractive
repel each other. In the nucleus, the protons
and acts with an equal strength between
are separated by a distance of about a few
proton-proton, proton-neutron, and
Fermi (10−15 m ), they must exert on each other
neutron – neutron.
a very strong repulsive force. For example,
Unit 8  Atomic and Nuclear physics 165

UNIT-8(XII-Physics_Vol-2)_Word flow.indd 165 7/31/2019 7:33:46 PM


(iii) Strong nuclear force does not act on the The phenomenon of radioactivity was
electrons. So it does not alter the chemical first discovered by Henri Becquerel in
properties of the atom. 1896. Later, Marie Curie and her husband
Pierre Curie did a series of experiments
8.6 in detail to understand the phenomenon
RADIOACTIVITY of radioactivity. In India, Saha Institute
of Nuclear Physics (SINP), Kolkata is the
In the binding energy curve, the stability premier institute pursuing active research
of the nucleus that has Z > 82 starts to in nuclear physics.
decrease and these nuclei are called unstable
nuclei. Some of the unstable nuclei decay
During early days of nuclear
naturally by emitting some kind of particles Note physics research, the term
to form a stable nucleus. The elements ‘radiation’ was used to denote
of atomic number Z > 82 and isotopes of the emanations from radioactive
lighter nuclei belong to naturally-occurring nuclei. Now we know that α rays
radioactive nuclei. Each of these radioactive are in fact 24 He nuclei and β rays are
nuclei decays to another nucleus by the electrons or positrons. Certainly, they
emission of 24 He nucleus ( α - decay) or are not electromagnetic radiation.
electron or positron ( β - decay) or gamma The γ ray alone is electromagnetic
rays ( γ - decay). radiation.
The phenomenon of spontaneous
emission of highly penetrating radiations
such as α, β and γ rays by an element is 8.6.1 Alpha decay
called radioactivity and the substances
When unstable nuclei decay by emitting
which emit these radiations are called
an α-particle ( 24 He nucleus), it loses two
radioactive elements. These radioactive
protons and two neutrons. As a result, its
elements can be heavy elements (Z > 82),
atomic number Z decreases by 2, the mass
isotopes of lighter and heavy elements and
number decreases by 4. We write the alpha
these isotopes are called radioisotopes. For
decay process symbolically in the following
example, carbon isotope 146 C is radioactive
way
but 126 C is not.
Radioisotopes have a variety of A
X→ A− 4
Y + 24 He (8.26)
Z Z −2
applications such as carbon dating, cancer
treatment, etc. When radioactive nucleus Here X is called the parent nucleus and
undergoes decay, the mass of the system Y is called the daughter nucleus.
decreases – that is, the mass of the initial Example: Decay of Uranium 238 92 U to

nucleus before decay is always greater than thorium 90Th with the emission of 24 He
234

the sum of the mass of the final nucleus nucleus (α-particle)


and that of the emitted particle. When this 238
U→ 234
Th + 24 He
92 90
difference in mass ∆m < 0 , it appears as
the energy according to Einstein’s relation As already mentioned, the total mass of the
E = ∆m c 2 . daughter nucleus and 24 He nucleus is always

166 Unit 8 Atomic and Nuclear physics

UNIT-8(XII-Physics_Vol-2)_Word flow.indd 166 7/31/2019 7:33:54 PM


less than that of the parent nucleus. The E X A M P L E 8. 1 1
difference in mass ( ∆m = mX − mY − mα )
is released as energy called disintegration (a) Calculate the disintegration energy
energy Q and is given by when stationary 232 92 U nucleus decays
to thorium 90Th with the emission of
228

α particle. The atomic masses are of


Q = (mX − mY − mα ) c 2 (8.27)
92 U = 232.037156 u , 90Th = 228.028741u
232 228

and 2 He = 4.002603 u
4

Note that for spontaneous decay (natural


(b) Calculate kinetic energies of Th
228
radioactivity) Q >0. In alpha decay process, 90
and α-particle and their ratio.
the disintegration energy is certainly positive
(Q > 0). In fact, the disintegration energy Q Solution
is also the net kinetic energy gained in the
The difference in masses
decay process or if the parent nucleus is at
∆m = (mU − mTh − mα )
rest, Q is the total kinetic energy of daughter
nucleus and the 24 He nucleus. Suppose Q = (232.037156–228.028741 – 4.002603)u
< 0, then the decay process cannot occur The mass lost in this decay = 0.005812 u
spontaneously and energy must be supplied
Since 1u = 931MeV, the energy Q released
to induce the decay.
is
Q = (0.005812u)×(931MeV / u)
In alpha decay, why does = 5.41 MeV
Note the unstable nucleus emit
This disintegration energy Q appears as
2 He nucleus? Why it does
4

not emit four separate nucleons? the kinetic energy of α particle and the
After all 24 He consists of two daughter nucleus.
protons and two neutrons. For In any decay, the total linear momentum
example, if 23892 U nucleus decays must be conserved.
into 90Th by emitting four separate
234

nucleons (two protons and two Total linear momentum of the parent
neutrons), then the disintegration nucleus = total linear momentum of the
energy Q for this process turns out daughter nucleus +α particle
to be negative. It implies that the Since before decay, the uranium nucleus is
total mass of products is greater at rest, its momentum is zero.
92 U ) nucleus.
than that of parent( 238
This kind of process cannot occur By applying conservation of momentum,
in nature because it would violate we get
conservation of energy. In any decay  
0 = mTh υTh + mα υα
process, the conservation of energy,  
mα υα = −mTh υTh
conservation of linear momentum
and conservation of angular It implies that the alpha particle and
momentum must be obeyed. daughter nucleus move in opposite
directions.

Unit 8 Atomic and Nuclear physics 167

UNIT-8(XII-Physics_Vol-2)_Word flow.indd 167 7/31/2019 7:34:02 PM


In magnitude mα υα = mTh υTh β− decay:
mTh In β- decay, the atomic number of the
The velocity of α particle υα = υ
mα Th nucleus increases by one but mass number
mTh remains the same. This decay is represented
Note that >1 , so υα > υTh . The ratio by

of the kinetic energy of α particle to the A
X → Z +A1Y + e− + ν
Z  (8.28)
daughter nucleus
K . Eα 1 mα υ2 α
2
It implies that the element X becomes Y
= by giving out an electron and antineutrino
K .ETh 1 m υ2
2 Th Th
( ν ). In otherwords, in each β- decay, one
By substituting, the value of υα into the
neutron in the nucleus of X is converted
above equation, we get
into a proton by emitting an electron (e–)
K .Eα mTh 228.02871 and antineutrino. It is given by
= = = 57
K .ETh mα 4.002603
n → p + e− + ν
The kinetic energy of α particle is 57 times
greater than the kinetic energy of the Where p -proton, ν -antineutrino.
daughter nucleus ( 228 Example: Carbon ( 146 C ) is converted
90Th ).
into nitrogen ( 147 N ) through β- decay.
The disintegration energy Q = total kinetic
energy of products 14
6 C → 147 N + e− + ν
K .Eα + K .ETh = 5.41 MeV
β+ decay:
57 K .ETh + K .ETh = 5.41MeV In β+ decay, the atomic number is
5.41 decreased by one and the mass number
K .ETh = MeV = 0.093MeV remains the same. This decay is represented by
58
K .Eα = 57 K .ETh = 57 × 0.093 = 5.301 MeV A
Z X → Z−A1Y + e + + ν
 (8.29)
In fact, 98% of total kinetic energy is taken It implies that the element X becomes Y
by the α particle. by giving out an positron and neutrino ( ν ).
In otherwords, for each β+ decay, a proton
in the nucleus of X is converted into a
8.6.2  Beta decay neutron by emitting a positron (e+) and a
In beta decay, a radioactive nucleus emits neutrino. It is given by
either electron or positron. If electron (e–) is p → n + e+ + ν
emitted, it is called β- decay and if positron
(e+) is emitted, it is called β+ decay. The However a single proton (not inside
positron is an anti-particle of an electron any nucleus) cannot have β+ decay due to
whose mass is same as that of electron and energy conservation, because neutron mass
charge is opposite to that of electron – that is, is larger than proton mass. But a single
+e. Both positron and electron are referred neutron (not inside any nucleus) can have
to as beta particles. β- decay.

168 Unit 8  Atomic and Nuclear physics

UNIT-8(XII-Physics_Vol-2)_Word flow.indd 168 7/31/2019 7:34:09 PM


A very interesting application of alpha decay is in smoke detectors which
prevent us from any hazardous fire.
Battery holder Radioactive source Current
Alarm
housing detector

– Radioactive
+ source

+ –
Ions
Fire smoke

Buzzer

The smoke detector uses around 0.2 mg of man-made weak radioactive isotope called
americium ( 241
95 Am ). This radioactive source is placed between two oppositely charged
metal plates and α radiations from 241 95 Am continuously ionize the nitrogen, oxygen
molecules in the air space between the plates. As a result, there will be a continuous flow
of small steady current in the circuit. If smoke enters, the radiation is being absorbed by
the smoke particles rather than air molecules. As a result, the ionization and along with
it the current is reduced. This drop in current is detected by the circuit and alarm starts.
The radiation dosage emitted by americium is very much less than safe level, so it can
be considered harmless.

Example: Sodium ( 1122 Na ) is converted allowed discrete energies whereas in beta


into neon ( 1022 Ne ) through β+ decay. decay, it was found that the beta particle
22 22 (i.e, electron) have a continuous range of
11 Na → 10 Ne + e + + ν
energies. But the conservation of energy and
It is important to note that the electron momentum gives specific single values for
or positron which comes out from nuclei electron energy and the recoiling nucleus
during beta decay never present inside the Y. It seems that the conservation of energy,
nuclei rather they are produced during the momentum are violated and could not be
conversion of neutron into proton or proton explained why energy of beta particle have
into neutron inside the nucleus. continuous range of values. So beta decay
Neutrino: remained as a puzzle for several years.
After a detailed theoretical and
Initially, it was thought that during beta
experimental study, in 1931 W.Pauli
decay, a neutron in the parent nucleus is
proposed a third particle which must be
converted to the daughter nuclei by emitting
present in beta decay to carry away missing
only electron as given by
energy and momentum. Fermi later named
A
Z X→ A
Z +1Y + e−  (8.30) this particle the neutrino (little neutral one)
since it has no charge, have very little mass.
But the kinetic energy of electron
For many years, the neutrino (symbol ν
coming out of the nucleus did not match
, Greek nu) was hypothetical and could
with the experimental results. In alpha
not be verified experimentally. Finally, the
decay, the alpha particle takes only certain
Unit 8  Atomic and Nuclear physics 169

UNIT-8(XII-Physics_Vol-2)_Word flow.indd 169 7/31/2019 7:34:12 PM


neutrino was detected experimentally in an electron of maximum of energy
1956 by Fredrick Reines and Clyde Cowan. 13.4 MeV.
Later Reines received Nobel prize in physics (2) it undergoes beta decay to an excited
in the year 1995 for his discovery. state of carbon ( 126 C * ) by emitting an
The neutrino has the following properties electron of maximum energy 9.0 MeV
· It has zero charge followed by gamma decay to ground
· It has an antiparticle called anti-neutrino. state by emitting a photon of energy
· Recent experiments showed that the 4.4 MeV.
neutrino has very tiny mass. It is represented by
12
· It interacts very weakly with the matter. 5 B → 126 C + e− + ν
Therefore, it is very difficult to detect. In 12
C * → 126 C + γ
fact, in every second, trillions of neutrinos 6

coming from the sun are passing through


In this decay process, the
our body without any interaction. daughter nucleus is in an
excited state, denoted by
12C∗, and the beta decay is
8.6.3  Gamma decay 6
followed by a gamma decay.
In α and β decay, the daughter
nucleus is in the excited state most 12B
5
of the time. The typical life time of
excited state is approximately 10-11s. e–
So this excited state nucleus immediately 13.4 MeV
ENERGY

returns to the ground state or lower energy e– 12C∗


6
state by emitting highly energetic photons
4.4 MeV γ
called γ rays. In fact, when the atom is in
the excited state, it returns to the ground 12C
6
state by emitting photons of energy in the
order of few eV. But when the excited state In this decay process, the
nucleus returns to its ground state, it emits daughter nucleus 126 C is left
in the ground state.
a highly energetic photon (γ rays) of energy
in the order of MeV. The gamma decay is Figure 8.25  Gamma decay
given by
A
Z X * → ZA X + gamma ( γ ) rays (8.31)

Here the asterisk(*) means excited state 8.6.4  Law of radioactive


nucleus. In gamma decay, there is no change decay
in the mass number or atomic number of In the previous section, the decay
the nucleus. process of a single radioactive nucleus was
Boron ( 125 B ) has two beta decay modes as discussed. In practice, we have bulk material
shown in Figure 8.25: of radioactive sample which contains a
(1) it undergoes beta decay directly into vast number of the radioactive nuclei and
ground state carbon ( 126 C ) by emitting not all the radioactive nucleus in a sample

170 Unit 8  Atomic and Nuclear physics

UNIT-8(XII-Physics_Vol-2)_Word flow.indd 170 7/31/2019 7:34:14 PM


decay at the same time. It decays over a
N
 ln N  = −λt
  N0
period of time and this decay is basically a
 
random process. It implies that we cannot ln  N  = −λt
predict which nucleus is going to decay or  N 0 
rather we can determine like probabilistic Taking exponentials on both sides, we get
basis (like tossing a coin). We can calculate
approximately how many nuclei in a sample N = N 0 e−λt (8.35)
are decayed over a period of time.
[Note: e ln x = e y ⇒ x = e y ]
At any instant t, the number of decays
 dN  Equation (8.35) is called the law of
per unit time, called rate of decay   is radioactive decay. Here N denotes the
dt
proportional to the number of nuclei ( N ) at number of undecayed nuclei present at any
the same instant. time t and N 0 denotes the number of nuclei
dN at initial time t=0. Note that the number of
∝N
dt atoms is decreasing exponentially over the
By introducing a proportionality time. This implies that the time taken for all
constant, the relation can be written as the radioactive nuclei to decay will be infinite.
Equation (8.35) is plotted in Figure 8.26.
dN = −λN (8.32)
dt N0
Number of undecayed nuclei

N = N0e–λt
Here proportionality constant λ is called
decay constant which is different for different
radioactive sample and the negative sign in N0/2
the equation implies that the N is decreasing
with time. N0/4
By rewriting the equation (8.32), we get N0/8
N0/16

dN = −λNdt (8.33) T1/2 2T1/2 3T1/2 4T1/2 Time t


Here dN represents the number of nuclei Figure 8.26  Law of radioactive decay
decaying in the time interval dt.
Let us assume that at time t = 0 s, the We can also define another useful
number of nuclei present in the radioactive quantity called activity (R) or decay rate
sample is N 0 . By integrating the equation which is the number of nuclei decayed per
(8.33), we can calculate the number of dN
second and it is denoted as R = . Note
undecayed nuclei N at any time t. dt
From equation (8.33), we get that activity R is a positive quantity.
From equation (8.35), we get
dN = −λ dt
N (8.34)
R = dN = λN 0 e−λt (8.36)
N t
dt
dN = − λ dt
∫ N ∫
N0 0 R = R0 e−λt (8.37)

Unit 8  Atomic and Nuclear physics 171

UNIT-8(XII-Physics_Vol-2)_Word flow.indd 171 7/31/2019 7:34:18 PM


where R0 = λN 0 initially present to reduce to one half of
the initial amount.
The equation (8.37) is also equivalent
The half-life is the important
to radioactive law of decay. Here R0 is
characteristic of every radioactive sample.
the activity of the sample at t=0 and R
Some radioactive nuclei are known to have
is the activity of the sample at any time t.
half-life as long as 1014 years and some
From equation (8.37), activity also shows
nucleus have very shorter life time (10-14s).
exponential decay behavior. The activity R
We can express half-life in terms of the
also can be expressed in terms of number of
decay constant. At t = T1/ 2 , the number of
undecayed atoms present at any time t.
N0
− λt
From equation (8.37), since N = N 0 e , undecayed nuclei N = .
2
we write By substituting this value in to the
R = λN (8.38) equation (8.35), we get
N0
Equation (8.35) implies that the activity = N 0 e−λT1/2
2
at any time t is equal to the product of decay
constant and number of undecayed nuclei 1 = e−λT1/2 or e λT1/2 = 2
2
at the same time t. Since N decreases over
Taking logarithm on both sides and
time, R also decreases.
rearranging the terms,
The SI unit of activity R is Becquerel and
one Becquerel (Bq) is equal to one decay per
T1/2 = ln 2 = 0.6931 (8.39)
second. There is also another standard unit λ λ
for the activity called Curie(Ci).
1 Curie =1 Ci = 3.7 × 1010 decays per second One should not think that
Note shorter half-life material
1 Ci = 3.7 × 1010 Bq
is safer than longer half-
life material because it will not last
long. The shorter half-life sample will
Initially one curie was
Note defined as number of decays
have higher activity and it is more
‘radioactive’ which is more harmful.
per second in 1 g of radium
and it is equal to 3.7 × 1010 decays/s. If the number of atoms present at t=0
N0
is N 0 , then atoms remain undecayed
2
N0
in first half-life and atoms remain
8.6.5 Half-life 4
undecayed after second half life and so on.
It is difficult to calculate the time taken
In general, after n half-lives, the number of
by a given a sample of N atoms to decay.
nuclei remaining undecayed is given by
However, we can calculate the time taken by
n
the given sample of atoms to reduce some  1
N =   N0 (8.40)
fraction of the initial amount.  2
We can define the half-life T1/ 2 as the where n can be integer or non-integer.
time required for the number of atoms Since the activity of radioactive sample also
172 Unit 8 Atomic and Nuclear physics

UNIT-8(XII-Physics_Vol-2)_Word flow.indd 172 7/31/2019 7:34:23 PM


obeys the exponential decay law, we can also Using mean life, the half-life can be
write an equation for an activity similar to rewritten as
equation (8.36).
T1/2 = t ln 2 = 0.6931 t (8.44)
After n half-lives, the activity or decay
rate of any radioactive sample is
Mean life : Not for examination
n
 1 The integration in the equation (8.42)
R =   R0 (8.41)
 2 can be performed using integration by
parts.
Mean life (τ):
∞ ∞
When the radioactive nucleus undergo
∫ λN t e λN 0 ∫ t e−λt dt
− λt
0 dt
the decay, the nucleus which disintegrates
t= 0
= 0
first has zero life time and the nucleus which N0 N0
decay last has an infinite lifetime. The actual ∞
life time for each nucleus varies from zero t = λ ∫ t e−λt dt
to infinity. Therefore, it is meaningful to 0

define average life or mean life time t, that u=t dv = e−λt dt


the nucleus survives before it decays.
The mean life time of the nucleus is the ∞  t e−λt 
∞ ∞
 e−λt 
t = λ∫ t e  
∫  −λ  dt
− λt
ratio of sum or integration of life times dt =λ   − λ
 −λ  0
of all nuclei to the total number nuclei 0 0

present initially. By substituting the limits, the first


The total number of nuclei decaying in term in the above equation becomes zero.
the time interval from t to t + ∆t is equal ∞

t = ∫ e−λt dt = − 1 e−λt  = 1



to R∆t = λN 0e−λt ∆t . It implies that until the
time t, this R∆t number of nuclei lived. So 0
λ 0 λ
the life time of these R∆t nuclei is equal to be
tR∆t . In the limit ∆t ® 0 , the total life time
of all the nuclei would be the integration of E X A M P L E 8. 1 2
tRdt from the limit t = 0 to t = ∞ .
Mean life Calculate the number of nuclei of carbon-14
undecayed after 22,920 years if the initial
∞ ∞ number of carbon-14 atoms is 10,000. The
∫ t [ Rdt ] ∫ t l N 0 e − l t dt  half-life of carbon-14 is 5730 years.
t=
0
= 0
(8.42)
   N0 N0 Solution

After a few integration (refer box item), To get the time interval in terms of half-
the expression for mean life time, life,
t 22, 920 yr
n= = =4
1 T1/ 2 5730 yr
t= (8.43)
l
The number of nuclei remaining undecayed
Note that mean life and decay constant is after 22,920 years,
inversely proportional to each other.
Unit 8  Atomic and Nuclear physics 173

UNIT-8(XII-Physics_Vol-2)_Word flow.indd 173 7/31/2019 7:34:27 PM


n 4 (c) Activity after 2 hours can be
 1  1
N =   N 0 =   × 10, 000 calculated in two different ways:
 2  2
Method 1: R = R0 e–λt
N = 625
At t = 2 hr = 7200 s
–3
R = 3.75 × 103 × e–7200 ×1.155 ×10
E X A M P L E 8. 13
R = 3.75 × 103 × 2.4 ×10–4 = 0.9 Ci
A radioactive sample has 2.6 µg of pure n
 
7 N which has a half-life of 10 minutes. Method 2: R =  1  R 0
13

(a) How many nuclei are present initially? 2


(b) What is the activity initially? (c) What 120 min
Here n = = 12
is the activity after 2 hours? (d) Calculate 10 min
mean life of this sample.  
12
R =  1  × 3.75×103 ≈ 0.9 Ci
2
Solution
(a) To find N0, we have to find the T1/2 10 × 60
number of 137 N atoms in 2.6µg . The (d) mean life t = =
0.6931 0.6931
atomic mass of nitrogen is 13. Therefore, = 865.67 s
13 g of 137 N contains Avogadro number
( 6.02 × 1023 ) of atoms.
8.6.6  Carbon dating
In 1 g, the number of 13
7 N is equal to be
6.02 ´1023 atoms The interesting application of beta decay
. So the number of 13
N
13 7
is radioactive dating or carbon dating. Using
atoms in 2.6µg is this technique, the age of an ancient object
can be calculated. All living organisms
23
N 0 = 6.02 ×10 × 2.6 ×10−6 = 12.04 ×1016 atoms absorb carbon dioxide (CO2) from air
13
to synthesize organic molecules. In this
(b) To find the initial activity R0 , we absorbed CO2, the major part is 126 C and very
have to evaluate decay constant λ small fraction ( 1.3 × 10−12 ) is radioactive 146 C
0.6931 = 0.6931 = 1.155×10−3 s−1 whose half-life is 5730 years.
λ=
T1/2 10 × 60 Carbon-14 in the atmosphere is always
decaying but at the same time, cosmic
Therefore
rays from outer space are continuously
R0 = λN 0 = 1.155×10-3 ×12.04 ×1016
bombarding the atoms in the atmosphere
=13.90 ×1013 decays/s which produces 146 C . So the continuous
= 13.90 ×1013 Bq production and decay of 146 C in the
atmosphere keep the ratio of 146 C to 126 C
In terms of a curie, always constant. Since our human body,
13 tree or any living organism continuously
R 0 = 13.90 ×1010 = 3.75×103 Ci absorb CO2 from the atmosphere, the ratio
3.7 ×10
of 146 C to 126 C in the living organism is also
since 1Ci = 3.7 ×1010Bq
nearly constant. But when the organism
174 Unit 8  Atomic and Nuclear physics

UNIT-8(XII-Physics_Vol-2)_Word flow.indd 174 7/31/2019 7:34:36 PM


dies, it stops absorbing CO2. Since 146 C starts Solution
to decay, the ratio of 146 C to 126 C in a dead To calculate the age, we need to know the
organism or specimen decreases over the initial activity (R0) of the characol (when
years. Suppose the ratio of 146 C to 126 C in the the sample was alive).
ancient tree pieces excavated is known, then
the age of the tree pieces can be calculated. The activity R of the sample
R = R0 e−λt (1)
E X A M P L E 8. 14
To find the time t, rewriting the above
Keezhadi (கீழடி), a small hamlet, has R
equation (1), e λt = 0
become one of the very important R
archeological places of Tamilandu. It is By taking the logarithm on both sides, we
located in Sivagangai district. A lot of R 
artefacts (gold coins, pottery, beads, iron get t = 1 ln  0  (2)
λ  R 
tools, jewellery and charcoal, etc.) have
Here R = 38 decays/s=38 Bq.
been unearthed in Keezhadi which have
given substantial evidence that an ancient To find decay constant, we use the equation
urban civilization had thrived on the banks
λ = 0.6931 = 0.6931
of river Vaigai. To determine the age of T1/2 5730 yr × 3.156 ×107 s / yr
those materials, the charcoal of 200 g sent ∴
for carbon dating is given in the following [ 1yr = 365.25 × 24 × 60 × 60 s = 3.156 × 107 s]
figure (b). The activity of 146 C is found to be λ = 3.83×10−12 s−1
38 decays/s. Calculate the age of charcoal.
To find the initial activity R0 , we use
the equation R0 = λN 0 . Here N 0 is the
number of carbon-14 atoms present in
the sample when it was alive. The mass of
the characol is 200 g. In 12 g of carbon,
there are 6.02 × 1023 carbon atoms. So 200
g contains,

6.02 ×1023 atoms / mol
× 200 ≈ 1×1025 atoms
Figure (a)  Keezhadi – excavation site 12 g / mol
When the tree(sample) was alive, the ratio
of 146 C to 126 C is 1.3 × 10−12 . So the total
number of carbon-14 atoms is given by
N 0 = 1×1025 ×1.3×10−12 = 1.3×1013 atoms
The initial activity
R0 = 3.83×10−12 ×1.3×1013 ≈ 50 decays / s
= 50 Bq
Figure (b) – Characol which was sent for By substituting the value of R0 and λ in the
carbon dating equation (2), we get
Unit 8  Atomic and Nuclear physics 175

UNIT-8(XII-Physics_Vol-2)_Word flow.indd 175 7/31/2019 7:34:43 PM


1  50  electron, and antineutrino with the half life
t= × ln  
3.83 × 10 −12
 38  of 13 minutes.
Neutrons are classified according to their
t=
0.27
× 1012  7 × 1010 sec kinetic energy as (i) slow neutrons (0 to 1000 eV)
3.83 (ii) fast neutrons (0.5 MeV to 10 MeV).
In years The neutrons with average energy of about
7 ×1010 s 0.025 eV in thermal equilibrium are called
t= ≈ 2200 years
3.156 ×107 s / yr thermal neutron, because at 298K, the
thermal energy kT  0.025eV . Slow and fast
In fact, the excavated materials were sent
neutrons play a vital role in nuclear reactors.
for carbon dating to USA by Archeological
Department of Tamilnadu and the report 8.7
confirmed that the age of Keezhadi NUCLEAR FISSION
artefacts lies between 2200 years to 2500
years (Sangam era- 400 BC to 200 BC).
In 1939, German scientists Otto Hahn
The Keezhadi excavations experimentally
and F.  Strassman discovered that when
proved that urban civilization existed in
uranium nucleus is bombarded with a
Tamil Nadu even 2000 years ago!
neutron, it breaks up into two smaller nuclei
of comparable masses with the release of
8.6.7  Discovery of Neutrons energy. The process of breaking up of the
nucleus of a heavier atom into two smaller
In 1930, two German physicists Bothe
nuclei with the release of a large amount of
and Becker found that when beryllium
energy is called nuclear fission. The fission
was bombarded with α particles, highly
is accompanied by the release of neutrons.
penetrating radiation was emitted. This
The energy that is released in the nuclear
radiation was capable of penetrating the
fission is of many orders of magnitude
thick layer of lead and was unaffected by
greater than the energy released in chemical
the electric and magnetic fields. Initially, it
reactions.
was thought as γ radiation. But in the year
Uranium undergoes fission reaction in
1932, James Chadwick discovered that those
90 different ways. The most common fission
radiations are not EM waves but they are
reactions of 235
92 U nuclei are shown here.
particles of mass little greater than the mass
of the proton and had no charge. He called U + 01n → U * → 141
56 Ba + 36 Kr + 3 0 n + Q
235 236 92 1

them as neutrons. The above reaction can


92 92
 (8.45)
be written as
9
Be + 24 He → 126 C + 01n U + 01n →
235
92 U * → 140
236
92 54 Xe + 38 Sr + 2 0 n + Q
94 1


4
(8.46)
where 01n denotes neutron.
Here Q is energy released during the
Neutrons are stable inside the nucleus.
decay of each uranium nuclei. When the
But outside the nucleus they are unstable. If
slow neutron is absorbed by the uranium
the neutron comes out of the nucleus (free
nuclei, the mass number increases by one
neutron), it decays with emission of proton,
and goes to an excited state 236
92 U . But this
*

176 Unit 8  Atomic and Nuclear physics

UNIT-8(XII-Physics_Vol-2)_Word flow.indd 176 7/31/2019 7:34:48 PM


excited state does not last longer than 10-12s Mass defect ∆m = 236.054398 u – 235.829095 u
and decay into two daughter nuclei along = 0.225303 u
with 2 or 3 neutrons. From each reaction,
So the energy released in each fission =
on an average, 2.5 neutrons are emitted. It is
0.225303 × 931MeV ≈ 200 MeV
shown in Figure 8.27
n
This energy first appears as kinetic
Neutron
capture
energy of daughter nuclei and neutrons. But
later, this kinetic energy is transferred to the
surrounding matter as heat.
Chain reaction:
When one 235
92 U nucleus undergoes fission,
Excited
nucleus
the energy released might be small. But
from each fission reaction, three neutrons
are released. These three neutrons cause
further fission in another three 235 92 U nuclei
Fission which in turn produce nine neutrons. These
n Daughter nine neutrons initiate fission in another 27
Neutrons nuclei
92 U nuclei and so on. This is called a chain
235

n reaction and the number of neutrons goes on


n increasing almost in geometric progression.
It is shown in Figure 8.28.
Figure 8.27  Nuclear fission There are two kinds of chain reactions:
(i) uncontrolled chain reaction (ii) controlled
Energy released in fission: chain reaction. In an uncontrolled chain
We can calculate the energy (Q) released reaction, the number of neutrons multiply
in each uranium fission reaction. We choose indefinitely and the entire amount of energy
the most favorable fission which is given in released in a fraction of second.
the equation (8.45). The atom bomb is an example of nuclear
fission in which uncontrolled chain reaction
235
92 U + 01n → 236 * 141 92 1
92 U → 56 Ba + 36 Kr + 3 0 n + Q occurs. Atom bombs produce massive
destruction for mankind. During World
Mass of 235
U = 235.045733 u
92
War II, in the year 1946 August 6 and 9, USA
Mass of 01n = 1.008665 u dropped two atom bombs in two places of
Japan, Hiroshima and Nagasaki. As a result,
Total mass of reactant = 236.054398 u
lakhs of people were killed and the two cities
Mass of 141
56 Ba = 140.9177 u were completely destroyed. Even now the
people who are living in those places have
Mass of 92
Kr = 91.8854 u
36
side effects caused by the explosion of atom
Mass of 3 neutrons = 3.025995 u bombs.
The total mass of products = 235.829095 u It is possible to calculate the typical
energy released in a chain reaction. In the

Unit 8  Atomic and Nuclear physics 177

UNIT-8(XII-Physics_Vol-2)_Word flow.indd 177 7/31/2019 7:34:52 PM


One incoming neutron
causes a fission event in
a 235U nucleus.
235 U
92
92Kr
36
141
56 Ba 135
Several neutrons
138I 51 I
53
from the initial 138Xe 98 Nb
54 41
fission event cause 95Y
fission in additional 95 39
235U nuclei. 38Sr

The number of neutrons


and the number of fission 235
events grow rapidly. 92 U

Figure 8.28  Nuclear chain reaction

first step, one neutron initiates the fission E X A M P L E 8. 1 5


of one nucleus by producing three neutrons Calculate the amount of energy released
and energy of about 200 MeV. In the second when 1 kg of 235 92 U undergoes fission
step, three nuclei undergo fission, in third reaction.
step nine nuclei undergo fission, in fourth
step 27 nucleus undergo fission and so Solution
on. In the 100th step, the number of nuclei 235 g of 235
92 U has 6.02 × 10 atoms. In one
23

which undergoes fission is around 2.5 × 1040 gram of 235


92 U , the number of atoms is equal
. The total energy released after 100th step
6.02 × 1023
is 2.5 × 1040 × 200MeV = 8 × 1029 J . It is really to = 2.56 × 1021 .
235
an enormous amount of energy which is
equivalent to electrical energy required in So the number of atoms in 1 kg of U=
235
92
2.56 × 10 × 1000 = 2.56 × 10
21 24
Tamilnadu for several years.
If the chain reaction is controllable, Each 235
92 U nucleus releases 200 MeV of

then we can harvest an enormous amount energy during the fission. The total energy
of energy for our needs. It is achieved in a released by 1kg of 235
92 U is

controlled chain reaction. In the controlled Q = 2.56 × 1024 × 200 MeV = 5.12 × 1026 MeV
chain reaction, the average number of
neutron released in each stage is kept as one By converting in terms of joules,
such that it is possible to store the released Q = 5.12 × 1026 × 1.6 × 10 −13 J = 8.192 × 1013 J .
energy. In nuclear reactors, the controlled
In terms of Kilowatt hour,
chain reaction is achieved and the produced
8.192 × 1013
energy is used for power generation or for Q= = 2.27 × 107 kWh
3.6 × 106
research purpose.

178 Unit 8  Atomic and Nuclear physics

UNIT-8(XII-Physics_Vol-2)_Word flow.indd 178 7/31/2019 7:34:59 PM


This is enormously large energy which is with plutonium or polonium is used as the
enough to keep 100 W light bulb operating neutron source. During fission of 235 92 U , only
for 30,000 years. To produce this much energy fast neutrons are emitted but the probability of
through chemical reaction, around 20,000 tons initiating fission by it in another nucleus is very
of TNT(tri nitro toluene) has to be exploded. low. Therefore, slow neutrons are preferred for
Nuclear reactor: sustained nuclear reactions.

Nuclear reactor is a system in which the Moderators: The moderator is a material


nuclear fission takes place in a self-sustained used to convert fast neutrons into slow
controlled manner and the energy produced is neutrons. Usually the moderators are chosen
used either for research purpose or for power in such a way that it must be very light nucleus
generation. The first nuclear reactor was built having mass comparable to that of neutrons.
in the year 1942 at Chicago, USA by physicist Hence, these light nuclei undergo collision
Enrico Fermi. The main parts of a nuclear with fast neutrons and the speed of the
reactor are fuel, moderator and control rods. neutron is reduced (Note that a billiard ball
In addition to this, there is a cooling system striking a stationary billiard ball of equal mass
which is connected with power generation set would itself be stopped but the same billiard
up. ball bounces off almost with same speed when
it strikes a heavier mass. This is the reason for
Fuel: The fuel is fissionable material,
using lighter nuclei as moderators). Most of
usually uranium or plutonium. Naturally
the reactors use water, heavy water (D2O) and
occurring uranium contains only 0.7% of
graphite as moderators. The blocks of uranium
92 U and 99.3% are only 92 U . So the 92 U
235 238 238
stacked together with blocks of graphite (the
must be enriched such that it contains at least
moderator) to form a large pile is shown in the
2 to 4% of 235
92 U . In addition to this, a neutron
Figure 8.29 (a) & (b).
source is required to initiate the chain reaction
for the first time. A mixture of beryllium

Control rods Hot liquid


Heat exchanger
Shielding Steam

Electric
Turbine
generator

Water
Pump
Uranium containers Pump

Cold liquid
Figure 8.29  (a) Block diagram of Nuclear reactor

Unit 8  Atomic and Nuclear physics 179

UNIT-8(XII-Physics_Vol-2)_Word flow.indd 179 7/31/2019 7:35:01 PM


Hot water Heat Electric
(or liquid exchanger generator
Core Steam turbine
(fuel and moderator) sodium) Steam

Water

Control
rods
Condenser
Containment
vessel (shielding) Pump

Pump
Shielding
Cooling water
Figure 8.29 (b) Schematic diagram of nuclear reactor

Control rods: The control rods are used to surrounded by a concrete wall of thickness of
adjust the reaction rate. During each fission, about 2 to 2.5 m.
on an average 2.5 neutrons are emitted and in
Cooling system:The cooling system
order to have the controlled chain reactions,
removes the heat generated in the reactor
only one neutron is allowed to cause another
core. Ordinary water, heavy water and liquid
fission and the remaining neutrons are
sodium are used as coolant since they have
absorbed by the control rods.
very high specific heat capacity and have large
Usually cadmium or boron acts as control
boiling point under high pressure. This coolant
rod material and these rods are inserted into
passes through the fuel block and carries away
the uranium blocks as shown in the Figure
the heat to the steam generator through heat
8.29 (a) and (b). Depending on the insertion
exchanger as shown in Figure 8.29(a) and (b).
depth of control rod into the uranium, the
The steam runs the turbines which produces
average number of neutrons produced per
electricity in power reactors.
fission is set to be equal to one or greater
than one. If the average number of neutrons
India has 22 nuclear reactors
produced per fission is equal to one, then Note in operation. Nuclear
reactor is said to be in critical state. In fact, all reactors are constructed in
the nuclear reactors are maintained in critical two places in Tamilnadu, Kalpakkam
state by suitable adjustment of control rods. If and Kudankulam. Even though
it is greater than one, then reactor is said to be nuclear reactors are aimed to cater to
in super-critical and it may explode sooner or our energy need, in practice nuclear
may cause massive destruction. reactors now are able to provide only
Shielding: For a protection against 2% of energy requirement of India.
harmful radiations, the nuclear reactor is

180 Unit 8 Atomic and Nuclear physics

UNIT-8(XII-Physics_Vol-2)_Word flow.indd 180 7/31/2019 7:35:01 PM


8.8 into heat. When the temperature is high
enough to initiate the thermonuclear fusion,
NUCLEAR FUSION
they start to release enormous energy which
tends to stabilize the star and prevents it
When two or more light nuclei (A<20) from further collapse.
combine to form a heavier nucleus, then
The sun’s interior temperature is around
it is called nuclear fusion. In the nuclear
1.5 × 107 K . The sun is converting 6 × 1011 kg
fusion, the mass of the resultant nucleus is
hydrogen into helium every second and it
less than the sum of the masses of original
has enough hydrogen such that these fusion
light nuclei. The mass difference appears as
lasts for another 5 billion years. When the
energy. The nuclear fusion never occurs at
hydrogen is burnt out, the sun will enter into
room temperature unlike nuclear fission.
new phase called red giant where helium
It is because when two light nuclei come
will fuse to become carbon. During this
closer to combine, it is strongly repelled by
stage, sun will expand greatly in size and all
the coulomb repulsive force.
its planets will be engulfed in it.
To overcome this repulsion, the two light
According to Hans Bethe, the sun is
nuclei must have enough kinetic energy
powered by proton-proton cycle of fusion
to move closer to each other such that the
reaction. This cycle consists of three steps
nuclear force becomes effective. This can
and the first two steps are as follows:
be achieved if the temperature is very much
1
greater than the value 107 K. When the 1 H + 11H → 12 H + e + + ν (8.44)
surrounding temperature reaches around
1
107K, lighter nuclei start fusing to form 1 H + 12 H → 23 H + γ (8.45)
heavier nuclei and this resulting reaction is
A number of reactions are possible in the
called thermonuclear fusion reaction.
third step. But the dominant one is
Energy generation in stars:
3
2 H + 23 H → 24 H + 11H + 11H (8.46)
The natural place where nuclear fusion
occurs is the core of the stars, since its The overall energy production in the
temperature is of the order of 107K. In fact, above reactions is about 27 MeV. The
the energy generation in every star is only radiation energy we received from the sun is
through thermonuclear fusion. Most of the due to these fusion reactions.
stars including our Sun fuse hydrogen into
helium and some stars even fuse helium into Elementary particles:
heavier elements. An atom has a nucleus surrounded by
The early stage of a star is in the form electrons and nuclei is made up of protons
of cloud and dust. Due to their own and neutrons. Till 1960s, it was thought
gravitational pull, these clouds fall inward. that protons, neutrons and electrons are
As a result, its gravitational potential energy fundamental building blocks of matter. In
is converted to kinetic energy and finally 1964, physicist Murray Gellman and George

Unit 8  Atomic and Nuclear physics 181

UNIT-8(XII-Physics_Vol-2)_Word flow.indd 181 7/31/2019 7:35:03 PM


Zweig theoretically proposed that protons in nature. Our planets are bound to the sun
and neutrons are not fundamental particles; through gravitational force of the sun. In +2
in fact they are made up of quarks. These volume 1, we have learnt that between two
quarks are now considered elementary charges there exists electromagnetic force
particles of nature. Electrons are fundamental and it plays major role in most of our day-to-
or elementary particles because they are day events. In this unit, we have learnt that
not made up of anything. In the year 1968, between two nucleons, there exists a strong
the quarks were discovered experimentally nuclear force and this force is responsible
by Stanford Linear Accelerator Center for stability of the nucleus. In addition
(SLAC), USA. There are six quarks namely, to these three forces, there exists another
up, down, charm, strange, top and bottom fundamental force of nature called the weak
and their antiparticles. All these quarks have force. This weak force is even shorter in
fractional charges. For example, charge of range than nuclear force. This force plays
2
up quark is + e and that of down quark is an important role in beta decay and energy
3
1
− e. production of stars. During the fusion of
3 hydrogen into helium in sun, neutrinos and
According to quark model, proton is enormous radiations are produced through
made up of two up quarks and one down
weak force. The detailed mechanism of
quark and neutron is made up of one up
weak force is beyond the scope of this book
quark and two down quarks as shown in the
and for further reading, appropriate books
Figure 8.30.
can be referred.
u d Gravitational, electromagnetic, strong
+2e –1e and weak forces are called fundamental
3 3
forces of nature. It is very interesting to
d d u u
realize that, even for our day-to-day life, we
–1e –1e +2e +2e
3 3 3 3 require these four fundamental forces. To
put it in simple words: We are in the Earth
Neutron Proton
because of Earth’s gravitational attraction
Figure 8.30  Constituents of nucleons on our body. We are standing on the surface
of the earth because of the electromagnetic
The study of elementary particles is force between atoms of the surface of the
called particle physics and it is an active area earth with atoms in our foot. The atoms in
of research even now. Till date, more than our body are stable because of strong nuclear
20 Nobel prizes have been awarded in the force. Finally, the lives of species in the earth
field of particle physics. depend on the solar energy from the sun
Fundamental forces of nature: and it is due to weak force which plays vital
It is known that there exists gravitational role during nuclear fusion reactions going
force between two masses and it is universal on in the core of the sun.

182 Unit 8  Atomic and Nuclear physics

UNIT-8(XII-Physics_Vol-2)_Word flow.indd 182 7/31/2019 7:35:04 PM


SUMMARY

„„ A device used to study the conduction of electricity through gases is known as gas
discharge tube
„„ Charge per unit mass is known as specific charge or normalized charge, and it is
independent of gas used and also nature of electrodes used
„„ The minimum distance between alpha particle and centre of the nucleus just before
it gets reflected back by 180° is defined as distance of closest approach r0
„„ The impact parameter (b) (see Figure 8.12) is defined as the perpendicular distance
between the centre of the gold nucleus and the direction of velocity vector of alpha
particle when it is at a large distance.
„„ According to Bohr atom model, angular momentum is quantized.
2
„„ The radius of the orbit in Bohr atom model is rn = a0 n
Z
ε 0h 2
„„ The radius of first orbit is a0 = = 0.529 Å also known as Bohr radius
πme 2
„„ The velocity of electron in nth orbit is υn = h Z
2πma0 n
„„ The fine structure constant is α = 1 which is a dimensionless constant
137 4 2
„„ The total energy of electron in the nth orbit is En = − me2 2 Z2 = −13.6 12 eV
8 ε 0h n n
„„ The energy required to excite an electron from the lower energy state to any higher
energy state is known as excitation energy and corresponding potential supplied is
known as excitation potential.
„„ The minimum energy required to remove an electron from an atom which is in
ground state is known as ionization energy.
„„ The potential difference through which an electron should be accelerated to get
ionization energy is known as ionization potential.
„„ The wavelength of spectral lines of Lyman series lies in ultra-violet region
„„ The wavelength of spectral lines of Balmer series lies in visible region while those of
Paschen and Brackett series lie in infra-red region
„„ The nucleus of element X having atomic number Z and mass number A is represented
by ZA X
„„ The radius of nucleus (Z > 10) of mass number A is given by R = R0 A1/ 3 where
R0 = 1.2 F
„„ The density of nucleus ρ = 2.3×1017 kg m−3
„„ If M, mp and mn are masses of a nucleus ( ZA X ), proton and neutron respectively , then
the mass defect is ∆m = ( Zm p + Nmn ) − M
„„ The binding energy of nucleus B.E = ( Zm p + Nmn − M ) c 2
„„ The binding energy per nucleon is maximum for iron which is 8.8 MeV.

Unit 8  Atomic and Nuclear physics 183

UNIT-8(XII-Physics_Vol-2)_Word flow.indd 183 7/31/2019 7:35:08 PM


„„ Alpha decay: ZA X → A−4
Y + 24 He
Z −2
A
„„ β- decay: Z X → Z +A1Y + e− + ν
A
„„ β+ decay: Z X → Z−A1Y + e + + ν
„„ Gamma decay: ZA X * → ZA X + γ
„„ Law of radioactive decay: N = N 0 e−λt
n
 1
„„ In general, after n half lives, the number of nuclei undecayed is N =   N 0
 2
„„ The relation between half-life and decay constant T1/2 = ln 2
λ
„„ If a heavier nucleus decays into lighter nuclei, it is called nuclear fission
„„ If two lighter nuclei fuse to heavier nuclei, it is called nuclear fusion
„„ In nuclear reactors, the nuclear chain reaction is controlled. In stars, the energy
generation is through nuclear fusion.

184 Unit 8  Atomic and Nuclear physics

UNIT-8(XII-Physics_Vol-2)_Word flow.indd 184 7/31/2019 7:35:10 PM


CONCEPT MAP

Atomic and Nuclear Physics

Atom models Nucleus

J.J. Thomson Properties

α - scattering Binding
experiment energy

Rutherford Redioactive
decays

Bohr atom Nuclear


model fusion & fission

Hydrogen Elementary
spectrum particles

Unit 8  Atomic and Nuclear physics 185

UNIT-8(XII-Physics_Vol-2)_Word flow.indd 185 7/31/2019 7:35:10 PM


EVALUATION

I Multiple
Multiple Choice
choice Questions
questions (c) B
Vwis increased by 14.4 times
V
(a) (b) a
1. (d) B
Vais decreased by 14.4 times
Vw
1. Suppose
The speed an of
alpha particle
light in anaccelerated
isotropic
by a potential
medium depends on,of V volt is allowed to 7. Theλratio of the wavelengthsVa λfor the
(c) w (d) a
collide with a nucleus whose atomic transition
λa from n =2 to n = V1w λinw Li++,
(a) its intensity
number is Z, then the distance of 6. He
Starsand H is due to,
+

(b)its wavelength twinkle


closest approach of alpha particle to (a)1: 2:ection
3 (b) 1: 4: 9
(a) refl
the nucleus is of propagation
(c) the nature
(c)
(d) the Zmotion of the source w.r.to (b) 3:2:1 (d) 4: 9: 36
total internal reflection
V 8. Th
(a) 14
mediumV. 4 Å (b) 14 . 4 Å (c)erefraction
electric potential between a
Z proton and an electron is given by
2. A rod ofZ length 10 cm lies alongV the (d) polarisation
(c) 1.44 axis
Å of a concave (d) 1mirror
.44 Åof r
principal V Z V = V0 alnbiconvex
7. When lens rof
 r  , where is a constant.
0 glass having
focal length 10 cm in such
2. In a hydrogen atom, the electron a way that refractive index 1.47 is dipped in a
0
Assume that Bohr atom model is
its end closer
revolving in tothe
the fourth
pole is 20 cm away
orbit, has liquid, it acts as a plane sheet of glass.
applicable to potential, then variation
from the
angular mirror. The
momentum equallength
to of the This implies that the liquid must have
of radius of nth orbit rn with the principal
image is, (AIPMT Main 2012) refractive index,
(a) h (b) h quantum number n is
(a) 2.5 cm π 5cm
(b) (a) less than one
1
4hcm
(c) 10 2h15cm
(d) (a) rn ∝ than that of glass (b) rn ∝ n
(b) less
(c) (d) n
π π
3. An object is placed in front of a convex (c) greater than that of glass
1
3. mirror
Atomicofnumber of H-like
focal length of atom
f andwith
the (c) n ∝ 2to that of glass (d) rn ∝ n
(d) requal
2

ionization for n = 1 of n
maximum potential
and minimum 122.4 Vdistance is
8. If
Thetheradius of radius
curvature
9. nuclear of 27ofAl curved
is 3.6
an object
(a) 1 from
(b) 2 the (c)mirror
3 such
(d) 4 that the surfacethe at aapproximate
thin planoconvex lensradius
is 10
fermi, nuclear
image
4. Th formed
e ratio between is real
the fiand magnified.
rst three orbits cm and
of 64
Cu theis refractive index is 1.5. If the
(IEE Main 2009)]
of hydrogen atom is plane
(a) 2.4surface is silvered, then (b) 1.2the focal
(a) 1:2:3
(a) 2f and c c and ∞
(b) 2:4:6
(b) length will be,
(c) 4.8 (d)3.6
(c) 1:4:9
(c) f and O (d) 1:3:5
(d) None of (a) 5 cm (b) 10 cm
10. The nucleus is approximately spherical
these (c) 15 cm Then the surface (d) 20area
cm of
5. The charge of cathode rays is in shape.
4. For light incident from air onto a slab of 9. nucleus
An air bubble
(a) positive (b) negative havinginmass glassnumber
slab of refractive
A varies
refractive index 2. Maximum possible index 1.5 (near normal incidence) is 5
(c) neutral (d) not defined as
angle of refraction is, cm deep
6. In J.J. Th omson e/m experiment,oa beam (a) A2 / 3 when viewed from (b)one
A4 /surface
3

(a) 30o (b) 45 and 3 cm deep when viewed from the


of electron is replaced by that of muons (c) A1/ 3 (d) A5/3
(c) 60o (d) 90o opposite face. The thickness of the slab
(particle with same charge as that of 11. The mass of a 37 Li nucleus is 0.042 u
5. If the velocity and wavelength is,
electrons but mass 208 times of light
that of less than the sum of the masses of all
in air is V and
electrons).a No defl λ and that in water
a ection condition is
is (a) 8 cm (b) 10 cm
its nucleons. The binding energy per
Vw and λwonly
achieved , thenif the refractive index of (c) 12 cmof 7 Li nucleus is nearly (d) 16 cm
nucleon
water is,
3
(a) B is increased by 208 times 10. (a)
A ray46of MeVlight travelling in(b) a transparent
5.6 MeV
(b) B is decreased by 208 times medium of refractive index n falls, on a
(c) 3.9MeV (d)23 MeV

186 Unit 8 Atomic and Nuclear physics

UNIT-8(XII-Physics_Vol-2)_Word flow.indd 186 7/31/2019 7:35:17 PM


12. M p denotes the mass of the proton Answers
and M n denotes mass of a neutron. 1) c 2) d 3) c 4) c 5) b
A given nucleus of binding energy B,
6) c 7) d 8) b 9) c 10) A
contains Z protons and N neutrons.
11) b 12) c 13) b 14) c 15) b
The mass M(N,Z) of the nucleus is
given by(where c is the speed of light)
(a) M ( N , Z ) = NM n + ZM p − Bc 2 II Short answer questions

(b) M ( N , Z ) = NM n + ZM p + Bc 2 1. What are cathode rays?.


2. Write the properties of cathode rays.
(c) M ( N , Z ) = NM n + ZM p − B / c 2
3. Give the results of Rutherford alpha
(d) M ( N , Z ) = NM n + ZM p + B / c 2 scattering experiment.
13. A radioactive nucleus (initial mass 4. Write down the postulates of Bohr
number A and atomic number Z) atom model.
emits 2α and 2 positrons. The ratio of
5. What is meant by excitation energy.
number of neutrons to that of proton
in the final nucleus will be 6. Define the ionization energy and
ionization potential.
A−Z −4 A− Z −2
(a) (b) 7. Write down the draw backs of Bohr
Z −2 Z −6
A− Z −4 A − Z − 12
atom model.
(c) (d) 8. What is distance of closest approach?
Z −6 Z −4
14. The half-life period of a radioactive 9. Define impact parameter.
element A is same as the mean life 10. Write a general notation of nucleus of
time of another radioactive element B. element X. What each term denotes?
Initially both have the same number of 11. What is isotope? Give an example.
atoms. Then 12. What is isotone? Give an example.
(a) A and B have the same decay rate 13. What is isobar? Give an example.
initially
14. Define atomic mass unit u.
(b) A and B decay at the same rate always
15. Show that nuclear density is almost
(c) B will decay at faster rate than A constant for nuclei with Z > 10.
(d) A will decay at faster rate than B. 16. What is mass defect?
15. A system consists of N 0 nucleus at 17. What is binding energy of a nucleus?
t=0. The number of nuclei remaining Give its expression.
after half of a half-life (that is, at time
1 18. Calculate the energy equivalent of 1
t = T1 ) atomic mass unit.
2 2
N0 N 19. Give the physical meaning of binding
(a) (b) 0 energy per nucleon.
2 2
20. What is meant by radioactivity?
N N
(c) 0 (d) 0
4 8

Unit 8  Atomic and Nuclear physics 187

UNIT-8(XII-Physics_Vol-2)_Word flow.indd 187 7/31/2019 7:35:25 PM


21. Give the symbolic representation of 9. Discuss the gamma decay process with
alpha decay, beta decay and gamma example.
decay. 10. Obtain the law of radioactivity.
22. In alpha decay, why the unstable 11. Discuss the properties of neutrino and
nucleus emits 24 He nucleus? Why it its role in beta decay.
does not emit four separate nucleons? 12. Explain the idea of carbon dating.
23. What is mean life of nucleus? Give the 13. Discuss the process of nuclear fission
expression. and its properties.
24. What is half-life of nucleus? Give the 14. Discuss the process of nuclear fusion
expression. and how energy is generated in stars?
25. What is meant by activity or decay 15. Describe the working of nuclear reactor
rate? Give its unit. with a block diagram.
26. Define curie. 16. Explain in detail the four fundamental
27. What are the constituent particles of forces.
neutron and proton? 17. Briefly explain the elementary particles
of nature.
III Long answer questions
1. Explain the J.J. Thomson experiment Exercises
to determine the specific charge of 1. Consider two hydrogen atoms HA
electron. and HB in ground state. Assume that
2. Discuss the Millikan’s oil drop hydrogen atom HA is at rest and
experiment to determine the charge of hydrogen atom HB is moving with a
an electron. speed and make head-on collide on the
3. Derive the energy expression for stationary hydrogen atom HA. After
hydrogen atom using Bohr atom the strike, both of them move together.
model. What is minimum value of the kinetic
4. Discuss the spectral series of hydrogen energy of the moving hydrogen atom
atom. HB, such that any one of the hydrogen
5. Explain the variation of average atoms reaches one of the excitation
binding energy with the mass number state.
by graph and discuss its features.  [Ans: 20.4 eV]
6. Explain in detail the nuclear force. 2. In the Bohr atom model, the frequency
7. Discuss the alpha decay process with of transitions is given by the following
example. expression

8. Discuss the beta decay process with 1 1 


v = Rc  2 − 2  , where n < m,
examples. n m 

188 Unit 8  Atomic and Nuclear physics

UNIT-8(XII-Physics_Vol-2)_Word flow.indd 188 7/31/2019 7:35:26 PM


Consider the following transitions: ratio of decayed numbers of A and B
Transitions m→n nuclei after 80 minutes.
1  [Ans: 5:4]
3→2
2 7. On your birthday, you measure the
2 →1
activity of the sample 210 Bi which has a
3 3 →1 half-life of 5.01 days. The initial activity
that you measure is 1µCi . (a) What is
Show that the frequency of these the approximate activity of the sample
transitions obey sum rule (which is on your next birthday? Calculate (b)
known as Ritz combination principle) the decay constant (c) the mean life (d)
 [Ans: v3→2 + v2→1 = v3→1 ] initial number of atoms.

3. (a) A hydrogen atom is excited by [Ans: (a) 10-22 µCi (b) 1.6 × 10−6 s −1
radiation of wavelength 97.5 nm. Find
the principal quantum number of the (c) 7.24days (d) 2.31 × 1010 ]
excited state. 8. Calculate the time required for 60%
(b) Show that the total number of lines of a sample of radon undergo decay.
in emission spectrum is n(n -1) and Given T1/ 2 of radon =3.8 days
2
compute the total number of possible  [Ans: 5.022 days]
lines in emission spectrum.
9. Assuming that energy released by
 [Ans: (a) n =4 (b) 6 possible transitions]
the fission of a single 235 92 U nucleus
4. Calculate the radius of the earth if is 200MeV, calculate the number of
the density of the earth is equal to the fissions per second required to produce
density of the nucleus.[mass of earth 1 watt power.
5.97 × 1024 kg ]. [Ans: 3.125 × 1010 ]
[Ans: 180 m ]
10. Show that the mass of radium ( 22688 Ra )
5. Calculate the mass defect and the
with an activity of 1 curie is almost a
binding energy per nucleon of the
gram. Given T1/ 2 =1600 years.
47 Ag nucleus. [atomic mass of Ag =
108

107.905949] 11. Characol pieces of tree is found from


 ∆m = 0.990391u and  an archeological site. The carbon-14
Ans:  
  B.E = 8.5MeV / A  content of this characol is only 17.5%
that of equivalent sample of carbon
6. Half lives of two radioactive elements
from a living tree. What is the age of
A and B are 20 minutes and 40 minutes
tree?
respectively. Initially, the samples have
equal number of nuclei. Calculate the  [Ans: 1.44 × 104 yr]

Unit 8  Atomic and Nuclear physics 189

UNIT-8(XII-Physics_Vol-2)_Word flow.indd 189 7/31/2019 7:35:33 PM


BOOKS FOR REFERENCE

1. Introduction to Modern Physics, 5. Physics for scientist and engineers with


H.S. Mani and G.K. Mehta, East-West modern physics, Serway and Jewett,
Press, New Delhi Brook/Coole publishers, 8th edition
2. Concepts of Modern Physics, Arthur 6. Physics for scientist and engineers with
Beiser, McGraw Hill, 6th edition modern physics, Paul Tipler and Gene
Mosca, Sixth edition, W.H.Freeman
3. Concepts of Physics – H. C. Verma,
and Company
Volume 2, Bharati Bhawan Publisher
4. Fundamentals of Physics, Halliday,
Resnick and Walker, Wiley Publishers,
10th edition

190 Unit 8  Atomic and Nuclear physics

UNIT-8(XII-Physics_Vol-2)_Word flow.indd 190 7/31/2019 7:35:33 PM


ICT CORNER
Atomic and Nuclear physics

In this activity you will be able to(i) experimentally demon- Topic: Millikan's oil
strate the concept of Millikan’s oil drop experiment (ii) find the
terminal velocity of the drop and (iii) find the charge on a drop. drop experiment

STEPS:
• Open the browser and type“vlab.amrita.edu” in the address bar.
• Click ‘Physical Sciences’tab. Thenclick ‘Modern Physics Virtual Lab’ and then click ‘Millikan's oil drop
experiment’. Go to “simulator” tab to do the experiment.
• Click on ‘START’ button. Click on Combo box to choose the oil.
• Click ‘START’ button of stop watch and notice the time taken t1 by a drop, to travel distance l1 between
l1
any two points. Calculate the terminal velocity υ1 =
t1
• Click ‘Voltage On’ to suspend the same oil drop in air, which is the balancing voltage V.
• Click the ‘X Ray ON’ button and notice the time taken t2 by same drop to travel distance l2 between any
l
two points. Calculate the terminal velocity υ2 = 2
t2
6πηr (υ1 + υ2 )d
• Charge of drop is calculated using the equation q = . r-radius of oil drop (can be
V
measure using telescope), η-viscosity of air (1.81 × 10-5 kgm-1s-1), d is the distance between the plates.

Step1 Step2

Step3 Step4

Note:
1. One time sign up is needed to do simulation. Then login using that username and
password.
2. Install flash player in your browserif it is not there.
URL:
http://vlab.amrita.edu/index.php?sub=1&brch=195&sim=357&cnt=4
* Pictures are indicative only.
* If browser requires, allow Flash Player or Java Script to load the page.

Unit 8 Atomic and Nuclear physics 191

UNIT-8(XII-Physics_Vol-2)_Word flow.indd 191 7/31/2019 7:35:34 PM


UNIT

9 SEMICONDUCTOR ELECTRONICS

Electronics is clearly the winner of the day


– John Ford.

LEARNING OBJECTIVES

In this unit, the students are exposed to,


• Energy band diagram in semiconductors
• Types of semiconductors
• Formation of p-n junction diode and its V-I characteristics
• Rectification process
• Special purpose diodes
• Transistors and their immediate applications
• Digital and analog signals
• Logic gates
• Boolean algebra
• De Morgan’s theorem

9.1
INTRODUCTION in 1897. This was followed by a vacuum
triode implemented by Lee De Forest to
Electronics has become a part of our amplify electrical signals. This led to the
daily life. All gadgets like mobile phones, introduction of tetrode and pentode tubes.
computers, televisions, music systems etc Subsequently, the transistor era began
work on the electronic principles. Electronic with the invention of bipolar junction
circuits are used to perform various transistor by Bardeen, Brattain and Shockley
operations in devices like air conditioners, in 1948 for which Nobel prize was awarded
microwave oven, dish washers and washing in 1956. The emergence of Germanium
machines. Besides this, its applications are and Silicon semiconductor materials made
widespread in all fields like communication this transistor gain popularity, in turn its
systems, medical diagnosis and treatments application in different electronic circuits.
and even handling money through ATMs. The following years witnessed the
invention of the integrated circuits (ICs)
Evolution of Electronics: that helped to integrate the entire electronic
The history of electronics began with the circuit on a single chip which is small
invention of vacuum diode by J.A. Fleming in size and cost-effective. Since 1958
192

UNIT-9_XII-Physics_Vol-2.indd 192 7/31/2019 7:30:39 PM


ICs capable of holding several thousand
electronic components on a single chip such
as small-scale, medium-large scale, and
very-large scale integration started coming
into existence. Digital integrated circuits
became another robust IC development that (a)
enhanced the architecture of computers.All
these radical changes led to the introduction
of microprocessor in 1969 by Intel.
The electronics revolution, in due course
of time, accelerated the computer revolution.
Now the world is on its way towards small
particles of nano-size, far too small to see. (b) (c)
This helps in the miniaturization to an
unimaginable size. A room-size computer
during its invention has now emerged as
a laptop, palmtop, iPad, etc. In the recent
past, IBM revealed the smallest computer
whose size is comparable with the tip of the (d) (e)
rice grain, measuring just 0.33 mm on each Figure: 9.1 Evolution of computers
side. (a) one of the world’s first computers
(b) desktop computer (c) laptop computer
Electronics is the branch of physics
(d) palmtop computer (e) Thinnest
incorporated with technology towards the computer revealed by IBM
design of circuits using transistors and
microchips. It depicts the behaviour and
movement of electrons in a semiconductor,
The world’s first
vacuum, or gas. Electronics deals with
computer ‘ENIAC’ was
electrical circuits that involve active
invented by J. Presper Eckert
components such as transistors, diodes,
and John Mauchly at the University of
integrated circuits, and sensors, associated
Pennsylvania. The construction work
with the passive components like resistors,
started in 1943 and got over in 1946. It
inductors, capacitors, and transformers.
occupied an area of around 1800 square
This chapter deals with semiconductor
feet. It had 18,000 vacuum tubes and it
devices like p-n junction diodes, bipolar
weighed around 50 tons.
junction transistors and logic circuits.

Passive Components:
Note components that cannot 9.1.1 Energy band diagram
generate power in a circuit. of solids

Active components: components that In an isolated atom, the electronic energy


can generate power in a circuit. levels are widely separated and are far apart
and the energy of the electron is decided by the
Unit 9 Semic onductor electronics 193

UNIT-9_XII-Physics_Vol-2.indd 193 7/31/2019 7:30:40 PM


orbit in which it revolves around the nucleus. of very large number of closely spaced
However, in the case of a solid, the atoms energy levels in a very small energy range
are closely spaced and hence the electrons in is known as energy band.
the outermost energy levels of nearby atoms The energy band formed due to the
influence each other. This changes the nature valence orbitals is called valence band
of the electron motion in a solid from that of and that formed due to the unoccupied
an isolated atom to a large extent. orbitals is called the conduction band. The
The valence electrons in an atom are energy gap between the valence band and
responsible for the bonding nature. Let us the conduction band is called forbidden
consider an atom with one electron in the energy gap. Electrons cannot exist in the
outermost orbit. It means that the number forbidden energy gap.
of valence electrons is one. When two such The representation of the valence band
atoms are brought close to each other, the and conduction band is shown in Figure
valence orbitals are split into two. Similarly 9.2(a). EV represents the maximum energy
the unoccupied orbitals of each atom will of the valence band and EC represents
also split into two. The electrons have the minimum energy of the conduction band.
choice of choosing any one of the orbitals as The forbidden energy gap, Eg = EC– Ev. The
the energy of both the orbitals is the same. kinetic energy of the electron increases
When the third atom of the same element is from bottom to top (near the nucleus to the
brought to this system, the valence orbitals farthest) and the potential energy decreases
of all the three atoms are split into three. indicating that the electrons in the orbitals
The unoccupied orbitals also will split into closer to the nucleus are bound with large
three. potential energy. Hence, the electrons closer
In reality, a solid is made up of millions of to nucleus require a lot of energy to be
atoms. When millions of atoms are brought excited. The electrons in the valence band
close to each other, the valence orbitals and are less bound to the nucleus and can be
the unoccupied orbitals are split according easily excited.
to the number of atoms. In this case, the
energy levels will be closely spaced and will The energy levels of the orbiting
be difficult to differentiate the orbitals of Note electrons are measured in
one atom from the other and they look like electron volts, (eV).
a band as shown in Figure 9.2. This band
Free
Conduction band electrons
EC CB CB
CB
Conduction
Forbidden band
energy gap Eg = 6eV Eg = 1eV
Valence
EV band
VB VB
Valence band VB Holes
Insulator Semiconductor Metal
Electron

Figure: 9.2 (a) Schematic representation of valence band, conduction band, and forbidden
energy gap. Energy band structure of (b) Insulators (c) Semiconductors (d) Metals

194 Unit 9 Semic onductor electronics

UNIT-9_XII-Physics_Vol-2.indd 194 7/31/2019 7:30:40 PM


9.1.2 Classification of in the solid can break the covalent bond
materials between the atoms (covalent bond is
formed due to the sharing of electrons to
The classification of solids into attain stable electronic configuration). This
insulators, metals, and semiconductors can releases some electrons from valence band
be explained with the help of the energy to conduction band. Since free electrons
band diagram. are small in number, the conductivity of
9.1.2.1 Insulators the semiconductors is not as high as that
The energy band structure of insulators of the conductors. The resistivity value of
is shown in Figure 9.2(b). The valence band semiconductors is from 10–5 to 106 Ωm.
and the conduction band are separated by
a large energy gap. The forbidden energy In semiconductors, electrons
gap is approximately 6 eV in insulators. The Note in the valence band are bound
gap is very large that electrons from valence electrons which cannot move.
band cannot move into conduction band Hence, they cannot contribute for
even on the application of strong external conduction.
electric field or the increase in temperature.
When the temperature is increased
Therefore, the electrical conduction is not
further, more number of electrons is
possible as the free electrons are almost
promoted to the conduction band and
nil and hence these materials are called
increases the conduction. Thus, we can
insulators. Its resistivity is in the range of
say that the electrical conduction increases
1011–1019 Ωm.
with the increase in temperature. In other
9.1.2.2 Metals words, resistance decreases with increase
In metals, the valence band and in temperature. Hence, semiconductors are
conduction band overlap as shown in Figure said to have negative temperature coefficient
9.2(c). Hence, electrons can move freely of resistance. The most important elemental
into the conduction band which results semiconductor materials are Silicon (Si) and
in a large number of free electrons in the Germanium (Ge). The forbidden energy
conduction band. Therefore, conduction gaps for Si and Ge at at room temperature
becomes possible even at low temperatures. are 1.1 eV and 0.7 eV respectively.
The application of electric field provides
sufficient energy to the electrons to drift in a 9.2
particular direction to constitute a current. TYPES OF
For metals, the resistivity value lies between SEMICONDUCTRORS
10–2 and 10–8 Ωm.

9.1.2.3 Semiconductors 9.2.1 Intrinsic


In semiconductors, there exists a narrow semiconductors
forbidden energy gap ( Eg < 3 eV ) between A semiconductor in its pure form
the valence band and the conduction band. without impurity is called an intrinsic
At a finite temperature, thermal agitations semiconductor. Here, impurity means
Unit 9 Semic onductor electronics 195

UNIT-9_XII-Physics_Vol-2.indd 195 7/31/2019 7:30:41 PM


any other atom in the crystal lattice. The
Silicon lattice is shown in Figure 9.3(a).
Si Si Si
Each Silicon atom has four electrons in the
Covalent bond
outermost orbit and is covalently bonded
Si Si Si
with the neighbouring atoms to form the
lattice. The band diagram for this case show
Si Si Si Valence electron in Figure 9.4(a).
A small increase in temperature is
sufficient enough to break some of the
(a) covalent bonds and release the electrons
free from the lattice as shown in Figure
Free electron
9.3(b). As a result, some states in the
Si Si Si valence band become empty and the same
Hole
number of states in the conduction band
Si Si Si
will be occupied as shown in Figure 9.4(b).
The vacancies produced in the valence
band are called holes. As the holes are
Si Si Broken
Si
covelant bond deficiency of electrons, they are treated to
possess positive charges. Hence, electrons
and holes are the two charge carriers in
(b) semiconductors.
Figure: 9.3  (a) Two dimensional crystal In intrinsic semiconductors, the number
lattice of Silicon (b) The presence of free of electrons in the conduction band is equal
electron, hole, and broken covalent bond
to the number of holes in the valence band.
in the intrinsic Silicon crystal.
The conduction is due to the electrons in the

Thermally
Generated
electrons in the
EC conduction band
EC
Vacancy (hole) created
Eg (Band gap)
Eg (Band gap) due to the release
EV of an electron
EV

(a) (b)
Figure: 9.4  (a) Valence band and conduction band of intrinsic semiconductor.
(b) Presence of thermally generated electrons in the conduction band and vacancy
created due to the shift of electron from valence band to conduction band at room
temperature.

196 Unit 9  Semic onductor electronics

UNIT-9_XII-Physics_Vol-2.indd 196 7/31/2019 7:30:41 PM


conduction band and holes in the valence 9.2.2.1 n-type semiconductor
band. These currents are represented as I e A n-type semiconductor is obtained by
and I h respectively. doping a pure Germanium (or Silicon) crystal
with a dopant from group V pentavalent
elements like Phosphorus, Arsenic, and
Definition of a hole: When an
Antimony as shown in Figure 9.5(a). The
Note electron is excited, covalent
dopant has five valence electrons while the
bond is broken. Now octet rule
Germanium atom has four valence electrons.
will not be satisfied. Thus each excited
electron leaves a vacancy to complete During the process of doping, a few of the
bonding. This ‘deficiency’ of electron is Germanium atoms are replaced by the group
termed as a ‘hole’ V dopants. Four of the five valence electrons
of the impurity atom are bound with the
4 valence electrons of the neighbouring
The total current ( I ) is always the sum replaced Germanium atom. The fifth valence
of the electron current ( I e ) and the hole electron of the impurity atom will be loosely
current ( I h ) . I = I e + I h attached with the nucleus as it has not formed
An intrinsic semiconductor behaves the covalent bond.
like an insulator at 0 K. The increase in The energy level of the loosely attached
temperature increases the number of charge fifth electron from the dopant is found
carriers (electrons and holes). The schematic just below the conduction band edge and
diagram of the intrinsic semiconductor in
band diagram is shown in Figure 9.4(b). +4
Si
+4
Si
+4
Si
The intrinsic carrier concentration is the
number electron in the conduction band or
the number of holes in the valence band in an +4
Si
+5
Si
+4
Si

intrinsic semiconductor. Free electron

9.2.2 Extrinsic semiconductors +4 +4 +4


Si Si Si
The carrier concentration in an
(a)
intrinsic semiconductor is not sufficient
enough to develop efficient electronic
devices. Another way of increasing the Conduction band
carrier concentration in an intrinsic Fermi
semiconductor is by adding impurity level
atoms. The process of adding impurities Donar
to the intrinsic semiconductor is called energy
levels Valence band
doping. It increases the concentration of
charge carriers (electrons and holes) in the
semiconductor and in turn, its electrical Figure 9.5 n-type extrinsic
conductivity. The impurity atoms are called semiconductor: (a) Free electron which
dopants and its order is approximately 100 is loosely attached to the lattice
ppm (parts per million). (b) Representation of donar energy level.

Unit 9 Semic onductor electronics 197

UNIT-9_XII-Physics_Vol-2.indd 197 7/31/2019 7:30:42 PM


is called the donor energy level as shown 9.2.2.2  p-type semiconductor
in Figure9.5(b). At room temperature, Here, a trivalent atom from group III
these electrons can easily move to the elements such as Boron, Aluminium, Gallium
conduction band with the absorption of and Indium is added to the Germanium
thermal energy. It is shown in the Figure or Silicon substrate. The dopant with
9.6. Besides, an external electric field also three valence electrons are bound with the
can set free the loosely bound electrons neighbouring Germanium atom as shown in
and lead to conduction. Figure 9.7(a). As Germanium atom has four
valence electrons, one electron position of the
Electrons from
EC dopants dopant in the Germanium crystal lattice will
EO Donar energy
remain vacant. The missing electron position
Eg(Band gap) level in the covalent bond is denoted as a hole.
EV
Thermally Generated
hole
+4 +4 +4
Si Si Si
(b) Hole

Figure 9.6  Thermally generated +4 +3 +4


Si B Si
hole in the valence band and the free
electrons generated by the dopants
in the conduction band (n-type +4 +4 +4
semiconductor). Si Si Si

It is important to note that the energy (a)


required for an electron to jump from the
valence band to the conduction band E g ( ) Conduction band
in an intrinsic semiconductor is 0.7 eV Acceptor
energy
for Ge and 1.1 eV for Si, while the energy levels
required to set free a donor electron is only Fermi
0.01 eV for Ge and 0.05 eV for Si. level
Valence band
The group V pentavalent impurity atoms
donate electrons to the conduction band (b)
and are called donor impurities. Therefore, Figure 9.7  P-type extrinsic semiconductor
each impurity atom provides one extra (a) Hole generated by the dopant
electron to the conduction band in addition (b) Representation of acceptor energy level.
to the thermally generated electrons. These
thermally generated electrons leave holes in To make complete covalent bonding
valence band. Hence, the majority carriers with all four neighbouring atoms, the
of current in an n-type semiconductor are dopant is in need of one more electron.
electrons and the minority carriers are These dopants can accept electrons from
holes. Such a semiconductor doped with the neighbouring atoms. Therefore, this
a pentavalent impurity is called an n-type impurity is called an acceptor impurity.
semiconductor. The energy level of the hole created by each
198 Unit 9  Semic onductor electronics

UNIT-9_XII-Physics_Vol-2.indd 198 7/31/2019 7:30:43 PM


Thermally as shown in Figure 9.9(a). Since the n-region
generated has a high electron concentration and the
Ec electron
Eo p-region a high hole concentration, electrons
diffuse from the n-side to the p-side. This
Eg(Band gap)
causes diffusion current which exists due to
Acceptor energy
the concentration difference of electrons.
EV level The electrons diffusing into the p-region
may occupy holes in that region and make
Holes from
dopants it negative. The holes left behind by these
electrons in the n-side are equivalent to
(a) the diffusion of holes from the p-side to
Figure 9.8  Thermally generated electron the n-side. If the electrons and holes were
in the conduction band and the holes not charged, this diffusion process would
generated by the dopants in the valence continue until the concentration of electrons
band (p-type semiconductor).
PN Junction
impurity atom is just above the valence
band and is called the acceptor energy level,
as shown in Figure 9.7(b). p n
For each acceptor atom, there will be
a hole in the valence band in addition to
the thermally generated holes. In such
an extrinsic semiconductor, holes are the (a)
(a)

majority carriers and thermally generated Diffusion

electrons are minority carriers as shown in


Figure 9.8. The semiconductor thus formed
is called a p-type semiconductor. p n

The n-type and p-type


semiconductor are neutral as (b)
we are topping neutral atoms Depletion region
to the intrinsic semiconductors.
– – + +

– – + +

9.3 p – – + +
n
DIODES – – + +

– – + +

9.3.1  P-N Junction formation (c) Immobile ions

9.3.1.1  Formation of depletion layer Figure: 9.9  (a) P-N junction (b) Diffusion
of electrons across the junction (c) Presence
A p-n junction is formed by joining
of immobile ions in the depletion region
n-type and p-type semiconductor materials
Unit 9  Semic onductor electronics 199

UNIT-9_XII-Physics_Vol-2.indd 199 7/31/2019 7:30:43 PM


and holes on the two sides were the same, This is due to the fact that the immobile ions
as happens if two gasses come into contact on both sides establish an electric potential
with each other. difference across the junction. Therefore, an
But, in a p-n junction, when the electrons electron trying to diffuse into the interior of
and holes move to the other side of the the depletion region encounters a negative
junction, they leave behind exposed charges wall of ions repelling it backwards. If the
on dopant atom sites, which are fixed in the free electron has enough energy, it can break
crystal lattice and are unable to move. On through the wall and enter into the p-region,
the n-side, positive ion cores are exposed where it can recombine with a hole and create
and on the p- side, negative ion cores are another negative ion.
exposed as shown in Figure 9.9(b). An
Depletion region
electric field E forms between the positive
ion cores in the n-type material and negative – – + +

ion cores in the p-type material. The electric – – + +

field sweeps free carriers out of this region p – – + +


n
and hence it is called depletion region as it is – – + +

depleted of free carriers. A barrier potential – – + +

Vb due to the electric field E is formed at the


junction as shown in Figure 9.9(c).
+ –
As this diffusion of charge carriers from
both sides continues, the negative ions Figure 9.10  Barrier potential formed
form a layer of negative space charge region across the junction
along the p-side. Similarly, a positive space
charge region is formed by positive ions on The strength of the electric potential
the n-side. The positive space charge region difference across the depletion region keeps
attracts electrons from p-side to n-side and increasing with the crossing of each electron
the negative space charge region attracts until equilibrium is reached; at this point, the
holes from n-side to p-side. This moment internal repulsion of the depletion layer stops
of carriers happen in this region due to the further diffusion of free electrons across the
formed electric field and it constitutes a junction. This difference in potential across
current called drift current. The diffusion the depletion layer is called the barrier
current and drift current flow in the potential as shown in Figure 9.10. At 25 0C,
opposite direction and at one instant they this barrier potential approximately equals
both become equal. Thus, a p-n junction is 0.7 V for Silicon and 0.3 V for Germanium.
formed.

9.3.1.2 Junction potential or barrier 9.3.2  P-N Junction diode


potential A p-n junction diode is formed when a
The recombination of charge carriers takes p-type semiconductor is fused with an-type
place only to a certain point beyond which the semiconductor. It is a device with single p-n
depletion layer acts like a barrier to further junction as shown in Figure 9.11(a). The
diffusion of free charges across the junction. circuit symbol is shown in Figure 9.11(b).
200 Unit 9  Semic onductor electronics

UNIT-9_XII-Physics_Vol-2.indd 200 7/31/2019 7:30:43 PM


the junction. In addition, the accelerated
P N electrons experience a strong attraction by
(a) the positive potential applied to the p-side.
This results in the movement of electrons
Anode Cathode
towards the p-side and in turn, holes towards
the n-side. When the applied voltage is
+
_
increased, the width of the depletion region
Conventional current flow and hence the barrier potential are further
(b) reduced. This results in a large number of
Figure 9.11  p-n junction diode electrons passing through the junction
(a) Schematic representation resulting in an exponential rise in current
(b) Circuit symbol through the junction.
Depletion region
Holes Electrons
9.3.2.1  Biasing a diode – +

Biasing means providing external – +

energy to charge carriers to overcome the


+ p – +
n –
barrier potential and make them move in – +

a particular direction. The charge carriers


– +

can either move towards the junction or away


+ –
from the junction. The external voltage
applied to the p-n junction is called bias Figure 9.12  Schematic representation of
a p-n junction diode under forward bias
voltage. Depending on the polarity of the
external source to the p-n junction we have
Reverse Bias
two types of biasing
If the positive terminal of the battery is
1. Forward bias
connected to the n-side and the negative
2. Reverse bias
potential to the p-side, the junction is said
Forward Bias to be reverse biased as shown in Figure 9.13.
If the positive terminal of the external Depletion region
Holes Electrons
voltage source is connected to the p-side – – – + + +

and the negative terminal to the n-side,


p n
– – – + + +

it is called forward biased as shown in – – – – + + + +

Figure  9.12. The application of a forward – – – + + +

bias potential makes the electrons move into – – – + + +

the n-side and the holes into the p-side. This


– +
initiates the recombination with the ions
near the junction which in turn reduces the Figure 9.13  Schematic representation of
width of the depletion region and hence the a p-n junction diode under reverse bias
barrier potential.
The electron from the n-side is now As the positive potential is connected
accelerated towards the p-side as it to the n-type material, the electrons in the
experiences a reduced barrier potential at n-type material are attracted towards the
Unit 9  Semic onductor electronics 201

UNIT-9_XII-Physics_Vol-2.indd 201 7/31/2019 7:30:44 PM


positive terminal in turn, the holes in the the diode is varied by varying the biasing
p-type material move towards the negative voltage across the dc power supply. The
terminal (both away from the junction). It forward bias voltage and the corresponding
increases the immobile ions at the junction. forward bias current are noted. A graph is
The net effect is the widening of the plotted by taking the forward bias voltage (V)
depletion region. This leads to an increase along the x-axis and the current (I) through
in the barrier potential. Consequently, the the diode along the y-axis. This graph is
majority charge carriers from both sides called the forward V-I characteristics of the
experience a great barrier to cross the p-n junction diode and is shown in Figure
junction. This reduces the diffusion current 9.14(b). Three inferences can be brought out
across the junction effectively. from the graph:
Yet, a small current flows across the
R mA
junction due to the minority charge carriers + –
in both regions. The reverse bias for majority + +
charge carriers serves as the forward bias for (0 –12V) V
– –
minority charge carriers. The current that
flows under a reverse bias is called the reverse (a)
saturation current. It is represented as I s .
The reverse saturation current is I (mA) Forward
independent of the applied voltage and is current

depended only on the thermally generated


minority charge carriers. Even a small Forward
bias
voltage is sufficient enough to drive the
minority charge carriers across the junction.
V
The reverse saturation current Forward voltage
Note of a silicon diode doubles for Knee
every 10 °C rise in temperature. voltage

(b)
9.3.3 Characteristics of a Figure 9.14 p-n junction diode
junction diode (a) diode under forward bias (b) forward
9.3.3.1 Forward characteristics characteristics
It is the study of the variation in current
through the diode with respect to the applied (i) At room temperature, a potential
voltage across the diode when it is forward difference equal to the barrier potential
biased. is required before a reasonable
The p-n junction diode is forward biased forward current starts flowing across
as shown in Figure 9.14(a). An external the diode. This voltage is known as
resistance (R) is used to limit the flow of threshold voltage or cut-in voltage or
current through the diode. The voltage across knee voltage (Vth). It is approximately
202 Unit 9 Semic onductor electronics

UNIT-9_XII-Physics_Vol-2.indd 202 7/31/2019 7:30:44 PM


0.3 V for Germanium and 0.7 V for R µA
+ –
Silicon. The current flow is negligible
when the applied voltage is less than – +
the threshold voltage. Beyond the (0 –15V)
+ V
threshold voltage, increase in current –
is significant even for a small increase
(a)
in voltage.
(ii) The graph clearly infers that the current
Reverse +I (mA)
flow is not linear and is exponential.
breakdown
Hence it does not obey Ohm’s law. voltage
(iii) The forward resistance ( rf ) of the –V
diode is the ratio of the small change
in voltage ( ∆V ) to the small change in Reverse
voltage
∆V 50 µA
current ( ∆I ) , rf = .
∆I Germanium
(iv) Thus the diode behaves as a conductor 20 µA Silicon
when it is forward biased.
(b) –I (µA)
However, if the applied voltage is increased
beyond a rated value, it will produce an Figure 9.15  p-n junction diode
extremely large current which may destroy (a) diode under reverse bias (b) reverse
the junction due to overheating. This is called characteristics
as the breakdown of the diode and the voltage
at which the diode breaks down is called the
breakdown voltage. Thus, it is safe to operate
a diode well within the threshold voltage and I (mA)
the breakdown voltage. Forward
current
9.3.3.2  Reverse characteristics Forward
The circuit to study the reverse bias
characteristics is shown in Figure 9.15(a). In
the reverse bias, the p-region of the diode
Reverse
is connected to the negative terminal and
voltage (V)
n-region to the positive terminal of the dc
power supply. (V) Forward
A graph is drawn between the reverse bias voltage
Reverse
voltage and the current across the junction, bias
which is called the reverse characteristics of
Reverse
a p-n junction diode. It is shown in Figure
current
9.15(b). Under this bias, a very small current I (µA)
in µA, flows across the junction. This is
Figure 9.16  Forward and reverse
due to the flow of the minority charge charactristics of a diode
carriers called the leakage current or reverse

Unit 9  Semic onductor electronics 203

UNIT-9_XII-Physics_Vol-2.indd 203 7/31/2019 7:30:45 PM


saturation current. Besides, the current E X A M P L E 9. 2
is almost independent of the voltage. The
reverse bias voltage can be increased only up Consider an ideal junction diode. Find the
to the rated value otherwise the diode will value of current flowing through AB is
enter into the breakdown region. A
1kΩ B
+3 V –7 V

Ideal diode: It acts like a


Note conductor when it is forward Solution
biased. When it is reverse The barrier potential of the diode is
biased, it acts like an insulator. The neglected as it is an ideal diode.
barrier potential is assumed to be
The value of current flowing through AB
zero and hence it behaves like a
resistor. can be obtained by using Ohm’s law
V 3 − (−7) 10
I= = 3
= 3 = 10 −2 A = 10 mA
R 1 × 10 10
The forward and reverse charactristics
are given in one graph as
shown in Figure 9.16.
9.3.4 Rectification
Rectification is the process of converting
E X A M P L E 9. 1 alternating current into direct current is
An ideal diode and a 5 Ω called rectification.In this section, we will
resistor are connected in series with a 15 discuss two types of rectifiers namely, half
V power supply as shown in figure below. wave rectifier and full wave rectifier.
Calculate the current that flows through 9.3.4.1 Half wave rectifier circuit
the diode. The half wave rectifier circuit is shown
in Figure 9.17(a). The circuit consists of a
+ transformer, a p-n junction diode and a
15V – 5Ω resistor. In a half wave rectifier circuit, either
a positive half or the negative half of the AC
input is passed through while the other half
is blocked. Only one half of the input wave
Solution
reaches the output. Therefore, it is called
The diode is forward biased and it is half wave rectifier. Here, a p-n junction
an ideal one. Hence, it acts like a closed diode acts as a rectifying diode.
switch with no barrier voltage. Therefore,
current that flows through the diode can During the positive half cycle
be calculated using Ohm’s law. When the positive half cycle of the ac
V = IR input signal passes through the circuit,
terminal A becomes positive with respect
to terminal B. The diode is forward biased
I = V = 15 = 3 A
R 5 and hence it conducts. The current flows

204 Unit 9 Semic onductor electronics

UNIT-9_XII-Physics_Vol-2.indd 204 7/31/2019 7:30:46 PM


through the load resistor RL and the AC for electronic equipments. A constant or
voltage developed across RLconstitutes the a steady voltage is required which can be
output voltage V0 and the waveform of the obtained with the help of filter circuits and
diode current is shown in Figure 9.17(b). voltage regulator circuits.
During the negative half cycle Efficiency (h) is the ratio of the output
dc power to the ac input power supplied to
When the negative half cycle of the ac
the circuit. Its value for half wave rectifier
input signal passes through the circuit,
is 40.6 %
terminal A is negative with respect to
terminal B. Now the diode is reverse biased
If the direction of the diode is
and does not conduct and hence no current
Note reversed, the negative half of
passes through RL. The reverse saturation
the ac signal is passed through
current in a diode is negligible. Since there
and the positive half is blocked.
is no voltage drop across RL, the negative
half cycle of ac supply is suppressed at the
output. The output waveform is shown in 9.3.4.1 Full wave rectifier
Figure 9.17b. The positive and negative half cycles
of the AC input signal pass through the
A I + full wave rectifier circuit and hence it is
Diode called the full wave rectifier. The circuit
AC
Input RL is shown in Figure 9.18(a). It consists of
– two p-n junction diodes, a center tapped
B (a)

D1
M A
Vpeak Input
voltage
Current flow
during positive half cycle
AC – R +
Time C
G
0 Input
Current flow
during negative half cycle

–Vpeak N B
D2
(b)
Output
voltage
Vpeak
Input voltage

Time
0
0
(b) Time
–Vpeak

Figure 9.17 (a) Input ac signal (b) half


wave rectifier circuit (c) input and output Output voltage

waveforms
0
(c) Time

The output of the half wave rectifier


Figure 9.18 (a) Full wave rectifier circuit
is not a steady dc voltage but a pulsating
(b) Input and output waveforms
wave. This pulsating voltage is not sufficient
Unit 9 Semic onductor electronics 205

UNIT-9_XII-Physics_Vol-2.indd 205 7/31/2019 7:30:46 PM


transformer, and a load resistor ( RL ) .
The centre is usually taken as the ground Centre tap transformer: There
Note is a facility to tap at halfway
or zero voltage reference point. Due to
point in the secondary
the centre tap transformer, the output
windings. This helps to measure the
voltage rectified by each diode is only
induced voltage from one end of the
one half of the total secondary voltage.
secondary to the centre point. If the
During positive half cycle centre tap point is grounded then the
When the positive half cycle of the ac voltage applied across the secondary
input signal passes through the circuit, will be divided by half. For example,
terminal M is positive, G is at zero potential if the voltage applied across the
secondary is 240 V, then the voltage
and N is at negative potential. This forward
across one end and the centre tap point
biases diode D1 and reverse biases diode
is 120 V.
D2. Hence, being forward biased, diode D1
conducts and current flows along the path
MD1 AGC. As a result, positive half cycle
of the voltage appears across RL in the
9.3.5 Breakdown mechanism
direction G to C
The reverse current or the reverse
During negative half cycle
saturation current due to the minority
When the negative half cycle of the ac
charge carriers is small. If the reverse bias
input signal passes through the circuit,
applied to a p-n junction is increased beyond
terminal N is positive, G is at zero
a point, the junction breaks down and the
potential and M is at negative potential.
reverse current rises sharply. The voltage at
This forward biases diode D2 and reverse
which this happens is called the breakdown
biases diode D1. Hence, being forward
voltage and it depends on the width of the
biased, diode D2 conducts and current
depletion region, which in turn depends on
flows along the path ND2 BGC. As a result,
the doping level.
negative half cycle of the voltage appears
A normal p-n junction diode gets
across RL in the same direction from G
damaged at this point. Specially designed
to C
diodes like Zener diode can be operated at
Hence in a full wave rectifier both
this regime and can be used for the purpose
postive and negative half cycles of the
of voltage regulation in circuits. There are
input signal pass through the circuit in the
two mechanisms that are responsible for
same direction as shown in Figure 9.18(b).
breakdown under increasing reverse voltage.
Though both positive and negative half
cycles of ac input are rectified, the output is 9.3.5.1 Zener breakdown
still pulsating in nature. Heavily doped p-n junctions have
The efficiency (η) of full wave rectifier narrow depletion layers of the order of
is twice that of a half wave rectifier and is <10-6 m. When a reverse voltage across this
found to be 81.2 %. It is because both the junction is increased to the breakdown limit,
positive and negative half cycles of the ac a very strong electric field of strength 3 × 107
input source are rectified. V m–1 is set up across the narrow layer. This
206 Unit 9 Semic onductor electronics

UNIT-9_XII-Physics_Vol-2.indd 206 7/31/2019 7:30:48 PM


electric field is strong enough to break or 9.3.6 Zener diode
rupture the covalent bonds in the lattice and
thereby generating electron-hole pairs. This Zener diode is a reverse biased heavily
effect is called Zener effect. doped Silicon diode named after its inventor
Even a small further increase in reverse C. Zener. It is specially designedto be operated
voltage produces a large number of charge in the breakdown region. The doping level
carriers. Hence the junction has very low of the Silicon diode can be varied to have a
resistance in the breakdown region. This wide range of breakdown voltages from 2 V
process of emission of electrons due to the to over 1000 V.
rupture of bands in from the lattice due to As explained in the previous section,
strong electric field is known as internal field Zener breakdown occurs due to the breaking
emission or field ionization. The electric field of covalent bonds by the strong electric field
required for this is of the order of 106 V m–1. set up in the depletion region by the reverse
voltage. It produces an extremely large number
9.3.5.2 Avalanche breakdown of electrons and holes which constitute the
Avalanche breakdown occurs in lightly reverse saturation current. The current is
doped junctions which have wide depletion limited by both external resistance and power
layers. Here, in this case, the electric field is dissipation of the diode. A Zener diodes is
not strong enough to produce breakdown. shown in Figure 9.19(a) and its circuit symbol
Alternatively, the thermally generated of Zener diode is shown in Figure 9.19(b).
minority charge carriers accelerated by the It looks like an ordinary p-n junction
electric field gains sufficient kinetic energy, diode except the cathode lead approximating
collide with the semiconductor atoms while the shape of a ‘z’ letter. The arrow head points
passing through the depletion region. This the conventional flow of forward current.
leads to the breaking of covalent bonds and In Figure 9.19(a), black ring indicates the
in turn generates electron-hole pairs. cathode lead.
The newly generated charge carriers are
also accelerated by the electric field resulting
in more collisions and further production
Figure 9.19 Zener diode (a) commercial
of charge carriers. This cumulative process
picture(b) circuit symbol
leads to an avalanche of charge carriers
across the junction and consequently
9.3.6.1 V-I Characteristics of Zener diode
reduces the reverse resistance. The diode
The circuit to study the forward and reverse
current increases sharply.
characteristics of a Zener diode is shown in
For a reverse voltage of, Figure 9.20(a) and Figure 9.20 (b). The V-I
Note (i) less than 4V → Zener characteristics of a Zener diode is shown in
Figure 9.20(c). The forward characteristic of
effect predominates
a Zener diode is similar to that of an ordinary
(ii) greater than 6V → Avalanche effect
p-n junction diode. It starts conducting
predominates
approximately around 0.7 V. However, the
(iii) between 4 and 6V → both effects
reverse characteristics is highly significant in
are present.
Zener diode. The increase in reverse voltage
Unit 9 Semic onductor electronics 207

UNIT-9_XII-Physics_Vol-2.indd 207 7/31/2019 7:30:48 PM


normally generates very small reverse current. I Z (max ) → maximum current limited by
While in Zener diode, when the reverse maximum power dissipation.
voltage is increased to the breakdown voltage
The Zener diode is operated in the
(VZ), the increase in current is very sharp. The
reverse bais having the voltage greater than
voltage remains almost constant throughout
VZ and current less than IZ(max). The reverse
the breakdown region. In Figure 9.20(c), IZ(max)
characteristic is not exactly vertical which
represents the maximum reverse current.
means that the diode possesses some small
If the reverse current is increased further,
resistance called Zener dynamic impedance.
the diode will be damaged. The important
Zener resistance is the inverse of the slope
parameters on the reverse characteristics are
in the breakdown region. It means an
VZ → Zener breakdown voltage increase in the Zener current produces only
I Z (min) → minimum current to sustain a very small increase in the reverse voltage.
breakdown However this can be neglected. The voltage
of an ideal Zener diode does not change
(a) mA
R + – once it goes into breakdown. It means that
VZ remains almost constant even when I Z
+ + increases considerably.
+ (0 –12V)
– – V

The maximum reverse bias
Note that can be applied before
(b) R mA entering into the Zener region
+ – is called the Peak inverse voltage.
Commercially referred as PIV rating.
– +
+ (0 –15V)
– +
V
Applications

The zener diode can be used as
1. Voltage regulators
(c) Forward
current (+IF) 2. Peak clippers
3. Calibrating voltages
Forward
bias 4. Provide fixed reference voltage in a
region
network for biasing
–VZ
5. Meter protection against damage from
accidental application of excessive voltage.
Reverse bias Forward bias (+VF)
(–VR)
Knee
Iz(min) voltage
0.3V Germanium
9.3.6.2 Zener diode as a voltage regulator
“Zener” breakdown 0.7V Silicon

Constant
region A Zener diode working in the breakdown
zener voltage
IZ(max)
region can serve as a voltage regulator. It
maintains a constant output voltage even
Reverse
current (–IR) when input voltage Vi or load current I L
varies. The circuit used for the same is shown
Figure 9.20 Zener diode (a) forward bias
(b) reverse bias (c) V-I characteristics in Figure 9.21. Here, in this circuit the input
voltage Vi is regulated at a constant voltage,
208 Unit 9 Semic onductor electronics

UNIT-9_XII-Physics_Vol-2.indd 208 7/31/2019 7:30:49 PM


Vz (Zener voltage) at the output represented Solution
as V0 using a Zener diode. The output voltage Voltage across AB is VZ = 9V
is maintained constant as long as the input
Voltage drop across R = 15 - 9 = 6V
voltage does not fall belowVz .
Therefore current through the resistor R,
Ri
I = 6 3 = 6 mA
IL
+
I 1×10
Iz Voltage across the load resistor = VAB = 9V
Vi RL Vo
Vz Current through load resistor,
V
I L = AB = 9 3 = 4.5 mA
RL 2 ×10

The current through the Zener diode,
Figure 9.21  Circuit to study voltage
I Z = I − I L = 6 mA − 4.5 mA =1.5 mA
regulation by Zener diode

When the potential developed across 9.3.7  Optoelectronic devices


the diode is greater than VZ ,the diode
Optoelectronics deals with devices which
moves into the Zener breakdown region. It
convert electrical energy into light and light
conducts and draws relatively large current
into electrical energy through semiconductors.
through the series resistance Ri . The total
Optoelectronic device is an electronic device
current I passing through Ri equals the sum
which utilizes light for useful applications. We
of diode current I Z and load current I L
will discuss some important optoelectronic
( )
I = I Z + I L . It is to be noted that the total
devices namely, light emitting diodes, photo
current is always less than the maximum
diodes and solar cells.
Zener diode current.
Under all conditions Vo = VZ 9.3.7.1  Light Emitting Diode (LED)
Thus, output voltage is regulated. LED is a p-n junction diode which emits
visible or invisible light when it is forward
biased. Since, electrical energy is converted
E X A M P L E : 9. 3 into light energy, this process is also called
Find the current through the Zener diode electroluminescence. The circuit symbol of
when the load resistance is 1 KΩ. Use diode LED is shown in Figure 9.22(a).
approximation. The cross-sectional view of a
commercial LED is shown in Figure
I RZ = 1 KΩ IL
A 9.22(b). It consists of a p-layer, n-layer
and a substrate. A transparent window is
used to allow light to travel in the desired
15 V
VZ = 9 V RL= 2KΩ direction. An external resistance in series
with the biasing source is required to limit
the forward current through the LED.
In addition, it has two leads; anode and
B
cathode.
Unit 9  Semic onductor electronics 209

UNIT-9_XII-Physics_Vol-2.indd 209 7/31/2019 7:30:52 PM


(a) emitted. For non-radiative recombination,
energy is liberated in the form of heat.
The colour of the light is determined
Anode Cathode by the energy band gap of the material.
Therefore, LEDs are available in a wide range
of colours such as blue (SiC), green (AlGaP)
and red (GaAsP). Now a days, LED which
(b)
Emitted Light emits white light (GaInN) is also available.
Applications
Cathode P- layer
Active region · Indicator lamps on the front panel of the
Diode N-layer
Substrate scientific and laboratory
Anode
equipments.
Transparent plastic case
· Seven-segment displays.
Terminal pins
· Traffic signals, exit
+ – (c) signs, emergency vehicle
lighting etc.
· Industrial process control, position
P-type N-type
encoders, bar graph readers.

Hole Electron
Conduction band
E X A M P L E: 9.4
Light
Fermi level
Determine the wavelength of light emitted
Recombi-

Band gap
nation

(Forbidden band)
from LED which is made up of GaAsP
Valence band
semiconductor whose forbidden energy
Figure 9.22  (a) Circuit symbol of LED gap is 1.875 eV. Mention the colour of the
(b) Inside view of LED (c) Schematic light emitted (Take h = 6.6 × 10-34 Js).
diagram to explain recombination process
Solution
When the p-n junction is forward biased,
the conduction band electrons on n-side and
E g = hc
valence band holes on p-side diffuse across the λ
junction. When they cross the junction, they Therefore,
become excess minority carriers (electrons −34 8
λ= hc = 6.6 ×10 × 3×−10
in p-side and holes in n-side). These excess Eg 1.875×1.6 ×10 19
minority carriers recombine with oppositely = 660 nm
charged majority carriers in the respective
The wavelength 660 nm corresponds to
regions, i.e. the electrons in the conduction
red colour light.
band recombine with holes in the valence
band as shown in the Figure 9.22(c).
9.3.7.2 Photodiodes
During recombination process, energy
A p-n junction diode which converts an
is released in the form of light (radiative)
optical signal into electric current is known
or heat (non-radiative). For radiative
as photodiode. Thus, the operation of
recombination, a photon of energy hv is
210 Unit 9  Semic onductor electronics

UNIT-9_XII-Physics_Vol-2.indd 210 7/31/2019 7:30:53 PM


photodiode is exactly opposite to that of an Applications
LED. Photo diode words in reverse bias. Its · Alarm system
circuit symbol is shown in Figure 9.23(a). · Count items on a conveyer belt
The direction of arrows indicates that the · Photoconductors
light is incident on the photo diode. · Compact disc players, smoke detectors
The device consists of a p-n junction · Medical applications such as detectors
semiconductor made of photosensitive material for computed tomography etc.
kept safely inside a plastic case as shown in
Figure 9.23(b). It has a small transparent 9.3.7.2 Solar cell
window that allows light to be incident on the A solar cell, also known as photovoltaic
p-n junction. Photodiodes can generate current cell, converts light energy directly into
when the p-n junction is exposed to light and electricity or electric potential difference
hence are called as light sensors. by photovoltaic effect. It is basically a p-n
(a) (b)
junction which generates emf when solar
radiation falls on the p-n junction. A solar
Anode Cathode
cell is of two types: p-type and n-type.
Both types use a combination of p-type
and n-type Silicon which together forms the
Figure 9.23  (a) Circuit symbol p-n junction of the solar cell. The difference
(b) Schematic view of photodiode
is that p-type solar cells use p-type Silicon as
the base with an ultra-thin layer of n-type
When a photon of sufficient energy (hu )
Silicon as shown in Figure 9.24, while n-type
strikes the depletion region of the diode, some
solar cell uses the opposite combination.
of the valence band electrons are elevated
The other side of the p-Silicon is coated with
into conduction band, in turn holes are
metal which forms the back electrical contact.
developed in the valence band. This creates
On top of the n-type Silicon, metal grid is
electron-hole pairs. The amount of electron-
deposited which acts as the front electrical
hole pairs generated depends on the intensity
contact. The top of the solar cell is coated
of light incident on the p-n junction.
with anti-reflection coating and toughened
These electrons and holes are swept across
glass.
the p-n junction by the electric field created
In a solar cell, electron–hole pairs are
by reverse voltage before recombination
generated due to the absorption of light
takes place. Thus, holes move towards the
n-side and electrons towards the p-side.
When the external circuit is made, the
electrons flow through the external circuit Sunlight (Photons)

and constitute the photocurrent. Front electrical contacts


Toughened glass
When the incident light is zero, there Anti-reflective coating
– – – – – – – –
exists a reverse current which is negligible. Load p-type silicon (base)
n-type silicon
p-n Junction
+ + + + + + +
This reverse currentin the absence of any Black electrical contact
Polymer backsheet
incident light is called dark current and is
Figure 9.24  Cross-sectional view of a
due to the thermally generated minority
solar cell
carriers.
Unit 9  Semic onductor electronics 211

UNIT-9_XII-Physics_Vol-2.indd 211 7/31/2019 7:30:53 PM


near the junction. Then the charge carriers Bipolar Junction Transistor
are separated due to the electric field of the The BJT consists of a semiconductor
depletion region. Electrons move towards (Silicon or Germanium) crystal in which an
n–type Silicon and holes move towards n-type material is sandwiched between two
p-type Silicon layer. The electrons reaching p-type materials (PNP transistor) or a p-type
the n-side are collected by the front contact material sandwiched between two n-type
and holes reaching p-side are collected by materials (NPN transistor). To protect it
the back electrical contact. Thus a potential against moisture, it is sealed inside a metal or
difference is developed across solar cell.When a plastic case. The two types of transistors with
an external load is connected to the solar cell, their circuit symbols are shown in Figure 9.25.
photocurrent ­­flows
­
through the load.
Emitter Collector
Many solar cells are connected together JCB
E C
either in series or in parallel combination E C
N P N
to form solar panel or module. Many solar
JEB
panels are connected with each other to form Base
B
B
solar arrays. For high power applications, NPN Transistor
solar panels and solar arrays are used.
(a)
Applications: Emitter Collector
JCB
E C
· Solar cells are widely used in calculators,
E C
watches, toys, portable power supplies, P N P

etc. JEB B
Base
· Solar cells are used in satellites and space B

applications PNP Transistor

· Solar panels are used to generate electricity. (b)


Figure 9.25  Schematic Diagram of
(a) NPN transistor and circuit symbol
9.4
(b) PNP transistor and circuit symbol
THE BIPOLAR JUNCTION
TRANSISTOR [BJT]
The three regions formed are called
Introduction as emitter, base and collector which are
provided with terminals or ohmic contacts
In 1951, William Schockley invented
labeled as E, B, and C. As a BJT has two p-n
the modern version of transistor. It is
junctions, two depletion layers are formed
a semiconductor device that led to a
across the emitter-base junction (JEB) and
technological revolution in the twentieth
collector-base junction (JCB) respectively. The
century. The heat loss in transistor is
circuit symbol carries an arrowhead at the
very less. This has laid the foundation of
emitter lead pointing from p to n indicating
integrated chips which contain thousands
the direction of conventional current.
of miniaturized transistors. The emergence
of the integrated chips led to increasing Emitter:
applications in the fast developing The main function of the emitter is
electronics industry. to supply majority charge carriers to the
212 Unit 9  Semic onductor electronics

UNIT-9_XII-Physics_Vol-2.indd 212 7/31/2019 7:30:54 PM


collector region through the base region.
Hence, emitter is more heavily doped than In a PNP transistor, base and
Note collector will be negative with
the other two regions.
respect to emitter indicated
Base: by the middle letter N whereas base
Base is very thin (10-6 m) and very lightly and collector will be positive in an
doped compared to the other two regions. NPN transistor [indicated by the
middle letter P]
Collector:
The main function of collector is to
collect the majority charge carriers supplied 9.4.1 Transistor circuit
by the emitter through the base. Hence, configurations
collector is made physically larger than the There are three types of circuit
other two as it has to dissipate more power. connections for operating a transistor based
Its is modarately dopped. on the terminal that is used in common to
both input and output circuits.
Because of the differing size
and the amount of dopping, 9.4.1.1 Common-Base configuration
the emitter cannot be replaced The base is common to both the input
by the collector and vice versa. and output circuits. The schematic and
circuit symbol are shown in Figure 9.26(a)
Transistor Biasing and 9.26(b). The input current is the
emitter current IE and the output current
The application of suitable dc voltages
is the collector current IC. The input signal
across the transistor terminals is called biasing.
is applied between emitter and base, the
Different modes of transistor biasing output is measured between collector and
Forward Active: base.
In this bias the emitter-base junction
is forward biased and the collector-base E
N P N
C
IE IC
junction is reverse biased. The transistor is in B

the active mode of operation. In this mode, VEB – +


VCB
+ IB –
the transistor functions as an amplifier.
Saturation: (a)
Here, the emitter-base junction and
IE E C
collector-base junction are forward biased. IC
B
The transistor has a very large flow of – +
VEB VCB
currents across the junctions. In this mode, + IB –

transistor is used as a closed switch.


Cut-off: (b)

In this bias, the emitter-base junction and Figure 9.26 NPN transistor in common
collector-base junction are reverse biased. base configuration (a) schematic circuit
Transistor in this mode is an open switch. diagram (b) circuit symbol

Unit 9 Semic onductor electronics 213

UNIT-9_XII-Physics_Vol-2.indd 213 7/31/2019 7:30:54 PM


9.4.1.2 Common-Emitter configuration IE

In this configuration, the emitter is


common to both the input and output loops E
N +
as shown in Figure 9.27. Base current, IB is B
P – VEC
IB
the input current and the collector current, + N
– V C
IC is the output current. The input signal is BC IC

applied between the emitter and base and (a)

the output is measured between the collector IE

and the emitter. RE

E
+ V
IC B – EC
IB
C VBC
+
– C
N
IC

B
P + V
IB – CE
(b)
VBE + N

IE
E Figure 9.28 NPN transistor in common
collector configuration (a) schematic
(a) circuit diagram (b) circuit symbol
IC
9.4.2 Transistor action in the
C common base mode
IB B +V
VBE + E –
CE
The operation of an NPN transistor in

IE the common base mode is explained below.
The current flow in a common base NPN
(b)
transistor in the forward active mode is
Figure 9.27 NPN transistor in common shown in Figure 9.29.
emitter configuration (a) schematic
circuit diagram (b) circuit symbol n++ P n+
Electrons
E C
9.4.1.3 Common- Collector configuration iE
Recombination
Here, the collector is common to both iB1 iB2

the input and output circuits as shown in VBE


B
VCB
iB
Figure 9.28. The base current IB is the input
current, the emitter current IE is the output
current. The input signal is applied between Figure 9.29 Flow of current in a NPN
the base and the collector, the output is transistor
measured between the emitter and collector. Basically, a BJT can be considered as
two p-n junction diodes connected back-
As the output is taken from
to-back. In the forward active bias of the
Note the emitter in common
transistor, the emitter-base junction is
collector configuration, it is
called an emitter follower. forward biased by a dc power supply VEB
and the collector-base junction is reverse

214 Unit 9 Semic onductor electronics

UNIT-9_XII-Physics_Vol-2.indd 214 7/31/2019 7:30:54 PM


biased by the bias power supply VCB. The I E = I B + I C . [Since the base current is very
forward bias decreases the depletion region small, we can write, I E ≈ I C ]
across the emitter-base junction and the
reverse bias increases the depletion region There is another component of collector
across the collector-base junction. Hence, current due to the thermally generated
the barrier potential across the emitter-base electrons called reverse saturation current,
junction is decreased and the collector-base denoted as ( I CO ) . This factor is temperature
junction is increased. The voltage across sensitive. Therefore, care must be taken
the emitter-base junction is represented as towards the stability of the system at high
VEB and the collector-base junction as VCB. temperatures.
In an NPN transistor, the majority charge The ratio of the collector current to the
carriers in the emitter are electrons. As it emitter current is called the forward current
is heavily doped, it has a large number of gain (α dc ) of a transistor.
electrons. The forward bias across the emitter- IC
base junction causes the electrons in the α dc =
IE
emitter region to flow towards the base region
and constitutes the emitter current ( I E ) . The α of a transistor is a measure of the
The electrons after reaching the base region quality of a transistor. Higher the value of
recombine with the holes in the base region. α better is the transistor. It means that the
Since the base region is very narrow and collector current is closer to the emitter
lightly doped, all the electrons will not have current. The value of α is less than unity and
sufficient holes to recombine and hence most ranges from 0.95 to 0.99. This indicates that
of the electrons reach the collector region. the collector current is 95% to 99% of the
Eventually, the electrons that reach the emitter current.
collector region will be attracted by the
1. The conventional flow of
collector terminal as it has positive potential
Note current is based on the
and flows through the external circuit. This
direction of the motion
constitutes the collector current ( I C ) . The
of holes
holes that are lost due to recombination
2. In NPN transistor, current enters
in the base region are replaced by the
from the base into the emitter.
positive potential of the bias voltage VEE
3. In a PNP transistor, current enters
and constitute the base current ( I B ) . The
from the emitter into the base.
magnitude of the base current will be in
4. The emitter-base junction has low
micoamperes as against milliamperes for
resistance and the collector-base
emitter and collector currents.
junction has high resistance.
It is to be noted that if the emitter current
is zero, then the collector current is almost
Working of a PNP transistor
zero. It is therefore imperative that a BJT is
called a current controlled device. Applying The working of a PNP transistor is similar
Kirchoff ’s law, we can write the emitter to the NPN transistor except for the fact that
current as the sum of the collector current the emitter current I E is due to holes and the
and the base current. base current I B is due to electrons. However,

Unit 9 Semic onductor electronics 215

UNIT-9_XII-Physics_Vol-2.indd 215 7/31/2019 7:30:56 PM


the current through the external circuit is The static characteristics of the BJT are
due to the flow of electrons. 1. Input characteristics
2. Output characteristics
E X A M P L E 9. 5
3. Transfer characteristics
In a transistor connected in the common
9.4.3.1  Input Characteristics
base configuration, α =0.95 , I E =1 mA .
Calculate the values of I C and I B . Input Characteristics curves give the
relationship between the base current ( I B )
Solution and base to emitter voltage (VBE ) at constant
α=
IC collector to emitter voltage (VCE ) and are
IE shown in Figure 9.31.
I C = α I E = 0.95×1= 0.95 mA Initially, the collector to emitter voltage
I E = I B + IC (VCE) is set to a particular voltage (above
0.7 V to reverse bias the junction). Then
∴ I B = I C − I E =1− 0.95 = 0.05 mA
the base-emitter voltage (VBE) is increased
in suitable steps and the corresponding
9.4.3  Static Characteristics base-current (IB) is recorded. A graph is
of Transistor in Common plotted with VBE along the x-axis and IB
Emitter Mode along the y-axis. The procedure is repeated
The know-how of certain parameters for different values of VCE.
like the input resistance, output resistance, I
B(µA)
and current gain of a transistor are very VCE=0V

important for the effective use of transistors 80


VCE=10V
in circuits. The circuit to study the static 70

characteristics of an NPN transistor in the 60 VCE=20V

ΔI
common emitter mode is given in Figure 50 B
Base current

9.30. The bias supply voltages VBB and VCC 40


bias the base-emitter junction and collector- 30
emitter junction respectively. The junction 20
potential at the base-emitter is represented 10
ΔVBE

as VBE and the collector-emitter as VCE. The


0 0.2 0.4 0.6 0.8 1.0
rheostats R1and R2 are used to vary the base VBE(V)

Base-emitter voltage
and collector currents respectively. VK

IC

IB _ +
Figure 9.31  Input characteristics of
B C
mA
+
a NPN transistor in common emitter
+ _
VBB R1
µΑ
E
VCE
_
R2
VCC
configuration
VBE IE

The following observations are made from


the graph.
Figure 9.30  Static characteristics of · The curve looks like the forward
a NPN transistor in common emitter
characteristics of an ordinary p-n
configuration
junction diode.
216 Unit 9  Semic onductor electronics

UNIT-9_XII-Physics_Vol-2.indd 216 7/31/2019 7:30:58 PM


· There exists a threshold voltage or knee IB. The four important regions in the output
voltage (Vk ) below which the base characteristics are:
current is very small. The value is 0.7 V
IC
for Silicon and 0.3 V for Germanium Active region
transistors. Beyond the knee voltage, the IB = 4 mA
base current increases with the increase IB =3 mA
ΔI

in base-emitter voltage. C

IB =2 mA
· It is also noted that the increase in the

Saturation region
collector-emitter voltage decreases IB =1 mA
the base current. This shifts the curve
outward. This is because the increase in IB =0 mA
0 VCE
collector-emitter voltage increases the ΔVCE

Cut-off region
width of the depletion region in turn,
reduces the effective base width and Figure 9.32  Output characteristics of
thereby the base current. a NPN transistor in common emitter
configuration
Input resistance
The ratio of the change in base-emitter
(i).  Saturation region
voltage ( ∆VBE ) to the change in base current
When VCE is increased above 0 V, the
( ∆I B ) at a constant collector-emitter voltage I C increases rapidly to a saturation value
(VCE ) is called the input resistance (Ri). The almost independent of I B (Ohmic region,
input resistance is not linear in the lower
OA) called knee voltage. Transistors are
region of the curve.
always operated above this knee voltage.
 ∆V 
Ri =  BE  (ii)  Cut-off region
 ∆I B 
A small collector current ( I C ) exists
CB V

The input resistance is high for a


even after the base current ( I B ) is reduced
transistor in common emitter configuration.
to zero. This current is due to the presence
of minority carriers across the collector-base
9.4.3.2 Output Characteristics junction and the surface leakage current
The output characteristics give the ( ICEO ) . This region is called as the cut-off
relationship between the variation in the region, because the main collector current
collector current (∆ I C ) with respect to the is cut-off.
variation in collector-emitter voltage (∆ VCE )
(iii)  Active region
at constant input current ( I B ) as shown in
In this region, the emitter-base junction
Figure 9.32.
is forward biased and the collector-base
Initially, the base current (IB) is set to
junction is reverse biased. The transistor in
a particular value. Then collector-emitter
this region can be used for voltage, current
voltage (VCE) is increased in suitable steps
and power amplification.
and the corresponding collector current (IC)
is recorded. A graph is plotted with the VCE (iv)  Breakdown region
along the x-axis and IC along the y-axis. This If the collector-emitter voltage (VCE )
procedure is repeated for different values of is increased beyond the rated value given
Unit 9  Semic onductor electronics 217

UNIT-9_XII-Physics_Vol-2.indd 217 7/31/2019 7:31:01 PM


by the manufacturer, the collector current
( IC ) increases enormously leading to the
junction breakdown of the transistor. This IC
avalanche breakdown can damage the (mA) ∆IC

transistor. ∆IB

Output Resistance
IB(µA)
The ratio of the change in the collector-
emitter voltage ( ∆VCE ) to the corresponding Figure 9.33  Current transfer
characteristics of a NPN transistor
change in the collector current (∆ I C ) at
common emitter configuration
constant base current ( I B ) is called output
resistance (RO).
9.4.3.4  Relation between α and β
 ∆V  There is a relation between current
Ro =  CE 
 ∆I C  I gain in the common base configuration α
B
and current gain in the common
β emitter
The output resistance for transistor α=
configuration β which is given below.
1+ β
in common emitter configuration is very
low. α=
β β= α
1+ β 1− α
9.4.3.3  Current transfer characteristics
E X A M Pβ L =E α9. 6
This gives the variation of collector 1− α
current ( I C ) with changes in base current The output characteristics of a transistor
( I B ) at constant collector-emitter voltage connected in common emitter mode is
(VCE ) as shown in Figure 9.33. It is seen shown in the figure. Determine the value
that a small I C flows even when I B is zero. of IC when VCE = 15 V. Also determine the
This current is called the common emitter value of IC when VCE is changed to 10 V
leakage current ( I CEO ), which is due to the I C (mA)
flow of minority charge carriers. 8

90 μA
Forward current gain 7
80 μA
70 μA
The ratio of the change in collector 6
60 μA
current ( ∆I C ) to the change in base current 5 50 μA
( ∆I B ) at constant collector-emitter voltage 40 μA

(VCE ) is called forward current gain (β).


4
30 μA
3
(Active region)
 ∆I  20 μA
β=  C  2
 ∆I B 
VCE 10 μA
1

Its value is very high and it generally IB = 0 μA


0 5 10 15 20 V (V)
ranges from 50 to 200. It depends on VCESat (Cut-off region)
CE

the construction of the transistor and will


be provided by the manufacturer. There When VCE = 15 V, IC = 1.5 μA
are transistors with b as high as 1000 as
well. When VCE is changed to 10 V, IC = 1.4 μA

218 Unit 9  Semic onductor electronics

UNIT-9_XII-Physics_Vol-2.indd 218 7/31/2019 7:31:03 PM


The collector current is · Presence of dc source at the input
independent of the collector- (saturation region):
Note
emitter voltage in the active When a high input voltage (Vin = +5V )
region. is applied, the base current (IB) increases
and in turn increases the collector current.
E X A M P L E : 9. 7 The transistor will move into the saturation
region (turned ON). The increase in
In the circuit shown in the figure, the input collector current (IC) increases the voltage
voltage Vi is 20 V, VBE = 0 V and VCE = 0 V. drop across RC , thereby lowering the output
What are the values of IB , IC , β? voltage, close to zero. The transistor acts
20 V like a closed switch and is equivalent to ON
condition.
R C 4 kΩ · Absence of dc source at the input (cut-
off region):
C A low input voltage (Vin = 0 V ), decreases
RB the base current (IB) and in turn decreases
Vi B
500 kΩ the collector current (IC). The transistor
E will move into the cut-off region (turned
OFF). The decrease in collector current
(IC) decreases the drop across RC , thereby
increasing the output voltage, close to +5 V.
Vi 20 V The transistor acts as an open switch which
IB = = = 40 μA V = 0V 
RB 500 kΩ  BE  is considered as the OFF condition.
V 20 V It is manifested that, a high input gives
I C = CC = = 5 mA VCE = 0V 
RC 4 kΩ a low output and a low input gives a high
I 5 mA output. In addition, we can say that the
β= C= = 125
I B 40 μA output voltage is opposite to the applied
input voltage. Therefore, a transistor can
9.4.4 Transistor as a switch be used as an inverter in computer logic
circuitry.
The transistor in saturation and cut-off
regions functions like an electronic switch
that helps to turn ON or OFF a given circuit 9.4.5 Operating Point
by a small control signal. The circuit is The operating point is a point where
shown in Figure 9.34. the transistor can be operated efficiently.
IC
A line that is drawn with the values
RB IB
C
RC VCC (when I C = 0) and I C (when VCE
= 0) is called the dc load line. The dc
B NPN
VO
E

Vin VBE IE VCC load line superimposed on the output


characteristics of a transistor is used to
learn the operating point of the transistor
Figure 9.34 Transistor as a switch as shown in Figure 9.35.

Unit 9 Semic onductor electronics 219

UNIT-9_XII-Physics_Vol-2.indd 219 7/31/2019 7:31:05 PM


IC Solution
IC = VCC B
RC DC load line β = 120
IB = 4 mA 25V 25
Base current I B = = = 25 μA
P IB =3 mA 1 M Ω 1×106
IC
O IB =2 mA β=
IB
R IB =1 mA
IC = βIB

0 A IB =0 mA I C =120×25 μA
VCE
VCE = VCC
I C =3 mA
Figure 9.35  Output characteristics of a VCE = VCC − I C RC
transistor in common emitter mode with
the dc load line VCE = 25 − 3 mA × 5 k
VCE =10 V
Points P,  Q, R in Figure 9.35 are called Q
I CmA
points or quiescent points which determine B
6
the operating point or the working point of
4
a transistor. If the operating point is chosen Q
at the middle of the dc load line (point Q), 2
the transistor can effectively work as an
A
amplifier. The operating point determines
0 5 10 15 20 25 30
the maximum signal that can be obtained VCE
without being distorted.
For a transistor to work as a open switch,
the Q point can be chosen at the cut-off 9.4.6  Transistor as an
region and to work as a closed switch, the Q amplifier
point can be chosen in the saturation region.
A transistor operating in the active
E X A M P L E : 9. 8 region has the capability to amplify weak
signals. Amplification is the process of
The current gain of a common emitter
increasing the signal strength (increase
transistor circuit shown in figure is 120.
in the amplitude).If a large amplification
Draw the dc load line and mark the Q point
is required, the transistors are cascaded
on it. (VBE to be ignored).
with coupling elements like resistors,
VCC = 25 V capacitors, and transformers which is called
IB IC
as multistage amplifiers.
RL 5 kΩ
RB 1MΩ Here, the amplification of an electrical
C signal is explained with a single stage
B
VCE
transistor amplifier as shown in Figure
E 9.36(a). Single stage indicates that the
IE
circuit consists of one transistor with the
allied components. An NPN transistor
220 Unit 9  Semic onductor electronics

UNIT-9_XII-Physics_Vol-2.indd 220 7/31/2019 7:31:07 PM


is connected in the common emitter result, the base current ( I B ) increases.
configuration. Consequently, the collector current ( I C )
+VCC
increases β times. This increases the voltage
Input AC signal

R1
drop across RC which in turn decreases the
collector-emitter voltage (VCE ) . Therefore,
RC +ve
0
IC CC –ve
C1 IB
C
VO the input signal in the positive direction
B
NPN produces an amplified signal in the negative
IE
E
direction at the output. Hence, the output
VS +ve
R2 RE CE 0 signal is reversed by 180° as shown in
0V
–ve
Figure 9.36(b).
  · During the negative half cycle
Figure 9.36  (a) Transistor as an amplifier Input signal (Vs ) decreases the forward
(b) Input and output waveform showing voltage across the emitter-base. As a result,
1800 phase reversal. base current ( I B ) decreases and in turn
To start with, the Q point or the operating increases the collector current ( I C ) . The
point of the transistor is fixed so as to get the increase in collector current ( I C ) decreases
maximum signal swing at the output (neither the potential drop across RC and increases
towards saturation point nor towards cut- the collector-emitter voltage (VCE ) . Thus,
off). A load resistance, RC is connected in the input signal in the negative direction
series with the collector circuit to measure produces an amplified signal in the positive
the output voltage. The capacitor C1 allows direction at the output.Therefore, 1800 phase
only the ac signal to pass through. The reversal is observed during the negative half
emitter bypass capacitor CE provides a low cycle of the input signal as well as shown in
reactance path to the amplified ac signal. Figure 9.36(b).
The coupling capacitor CC is used to couple
one stage of the amplifier with the next stage 9.4.7  Transistor as an
while constructing multistage amplifiers. VS oscillator
is the sinusoidal input signal source applied An electronic oscillator basically
across the base-emitter. The output is taken converts dc energy into ac energy of
across the collector-emitter. high frequency ranging from a few Hz
 IC  to several MHz. Hence, it is a source of
Collector current, I C =β I B b =  alternating current or voltage. Unlike an
 I B 
amplifier, oscillator does not require any
Applying Kirchhoff ’s voltage law in the external signal source.
output loop, the collector-emitter voltage is Basically, there are two types of
given by oscillators: Sinusoidal and non-sinusoidal.
VCE = VCC − I C RC Sinusoidal oscillators generate oscillations in
Working of the amplifier the form of sine waves at constant amplitude
· During the positive half cycle and frequency as shown in Figure 9.37(a).
Input signal (Vs ) increases the forward Whereas non-sinusoidal oscillators generate
voltage across the emitter-base. As a complex non-sinusoidal waveforms like

Unit 9  Semic onductor electronics 221

UNIT-9_XII-Physics_Vol-2.indd 221 7/31/2019 7:31:10 PM


Square-wave, Triangular-wave or Sawtooth- feedback circuit provides a portion of the
wave as shown in Figure 9.36(b). output to the tank circuit to sustain the
V
oscillations without energy loss. Hence, an
V
oscillator does not require an external input
t
signal. The output is said to be self-sustained.
t
Amplifier
(a)
(b)
V
The transistor amplifier circuit is already
V
explained in section {9.4.5}.

t t Amplifier Vo
R

(d)
Tank circuit + +
C1 – –
(C) L
Feedback
Figure: 9.37  (a) sinusoidal waveform Vf Network

(b) square waveform (c) ramp waveform (a) (b)


(d) triangular waveform Figure 9.39  (a) Block diagram of an
oscillator (b) tank circuit
Sinusoidal oscillations can be of two types:
Damped and undamped. If the amplitude Feedback network
of the electrical oscillations decreases with
The circuit used to feedback a portion of
time due to energy loss, it is called damped
the output to the input is called the feedback
oscillations as shown in Figure 9.38(a).
network. If the portion of the output fed to
On the other hand, the amplitude of the
the input is in phase with the input, then the
electrical oscillations remains constant with
magnitude of the input signal increases. It is
time in undamped oscillations as shown in
necessary for sustained oscillations.
Figure 9.38(b).
Tank circuit
V V
The LC tank circuit consists of an
inductance and a capacitor connected in
t t
parallel as shown in Figure 9.39. Whenever
energy is supplied to the tank circuit from a
DC source, the energy is stored in inductor
(a) (b)
and capacitor alternatively. This produces
Figure 9.38  (a) Damped oscillations electrical oscillations of definite frequency.
(b) Undamped oscillations (Refer section 4.9.1, Volume 1 of XII std.
Physics text book)
Transistor Oscillator But in practical oscillator circuits there
An oscillator circuit consists of a tank will be loss of energy across resistors,
circuit, an amplifier and a feedback circuit inductor coils and capacitors. A small
as shown in Figure 9.39. The tank circuit amount of energy is used up in overcoming
generates electrical oscillations and acts as these losses during every cycle of charging
the ac input source to the transistor amplifier. and discharging of the capacitor. Due to this,
Amplifier amplifies the input ac signal. The the amplitude of the oscillations decreases
222 Unit 9  Semic onductor electronics

UNIT-9_XII-Physics_Vol-2.indd 222 7/31/2019 7:31:10 PM


gradually. Hence, the tank circuit produces Resonant frequency, f o = 1
damped electrical oscillations. Therefore, 2π LC
in order to produce undamped oscillations, 1
On simplifying we get C =
a positive feedback is provided from the 4 π f o2 L
2

output circuit to the input circuit. When frequency is equal to 500 kHz
The frequency of oscillations is
C= 1 =
determined by the values of L and C using
4 × 3.14 × (500 × 103 )2 × 150 × 10 −6
2
the equation.
=676 pF
f= 1
2π LC When frequency is equal to 1500 kHz
C= 1 = 50 pF
Barkhausen conditions for sustained 4 × 3.14 × (1500 × 103 )2 × 150 × 10 −6
2

oscillations = 75 pF
The following condition called Therefore, the capacitor range is 75 – 676 pF
Barkhausen conditions should be satisfied
for sustained oscillations in the oscillator. 9.5
· The loop phase shift must be 00 or integral
DIGITAL ELECTRONICS
multiples of 2π.
· The loop gain must be unity. Aβ =1
Digital Electronics is the sub-branch
Here, A→Voltage gain of the amplifier,
of electronics which deals with digital
b →feedback ratio; (fraction of the
signals. It is increasingly used in numerous
output that is fed back to the input)
applications ranging from high end
There are different types of oscillator processor circuits to miniature circuits
circuits based on the different types of for signal processing, communication etc.
tank circuits. Examples: Hartley oscillator, Digital signals are preferred than analog
Colpitt’s oscillator, Phase shift oscillator, signals due to their better performance,
and Crystal oscillator. accuracy, speed, flexibility, and immunity to
Applications of oscillators noise.
· to generate a periodic sinusoidal or non
sinusoidal wave forms. 9.5.1  Analog and Digital
· to generate RF carriers. Signals
· to generate audio tones There are 2 different types of signals
· to generate clock signal in digital circuits. used in Electronics. They are (i) Analog
· as sweep circuits in TV sets and CRO. signals and (ii) Digital signals. An analog
signal is a continuously varying voltage or
E X A M P L E : 9. 9
current with respect to time. Such signals
Calculate the range of the variable capacitor have been employed in rectifying circuits
that is to be used in a tuned-collector and transistor amplifier circuits.
oscillator which has a fixed inductance of Digital signals are signals which contain
150 μH. The frequency band is from 500 only discreate values of voltages. Digital
kHz to 1500 kHz. signals need two states: switch ON and OFF.
Unit 9  Semic onductor electronics 223

UNIT-9_XII-Physics_Vol-2.indd 223 7/31/2019 7:31:12 PM


ON is considered as one state and OFF is Digital electronics deals with logical
considered as the other state. It can also be operations. The variables are called logical
defined as high (ON) or low (OFF) state, variables. The operators like logical addition
closed (ON) or open (OFF). These high (+) and logical multiplication (.) are called
and low states are defined using binary logical operators. When the logical operators
numbers 1or 0 in Boolean Algebra. The (+, .) operate on logical variables (A, B), it
state 1 represents the terms: circuit on, high gives logical constant (Y). The equation that
voltage, a closed switch. Similarly a 0 state represents this operation is called logical
represents circuit off, low voltage or an open statement.
switch. For example,
Positive and Negative Logic Logical operator: +
Logical variable: A, B
In digital systems, there exists two
Logical constant: Y
voltage levels: 5V (high) and 0V (low). In a
Logical Statement: Y = A + B
positive logic system; a binary 1 stands for
5V and 0 stands for 0V while in negative The possible combinations of inputs
logic system, 1 state for 0V and 0 state for and the corresponding output is given in
5V as shown in Figure 9.40. table called truth table. The circuits which
perform the basic logical operations such
(a) as logical addition, multiplication and
1
+5v inversion are discussed below.
AND gate
0 0
0v
Postive logic Circuit symbol
(b)
0
The circuit symbol of a two input AND
+5v gate is shown in Figure 9.41(a). A and B are
inputs and Y is the output. It is a logic gate
1 1
0v and hence A, B, and Y can have the value of
Negative logic
either 1 or 0.
Figure 9.40  (a) Positive (b) Negative A
AND Y
logics B

(a)
9.5.2  Logic gates
Inputs Output
A logic gate is an electronic circuit which A B Y = A+ B
functions based on digital signals. The logic 0 0 0
gates are considered as the basic building 0 1 0
blocks of most of the digital systems. It has
1 0 0
one output with one or more inputs. There
1 1 1
are three types of basic logic gates: AND, OR,
(b)
and NOT. The other logic gates are Ex-OR,
Figure 9.41  (a) Two input AND gate
NAND, and NOR. They can be constructed
(b) Truth Table
from the basic logic gates.
224 Unit 9  Semic onductor electronics

UNIT-9_XII-Physics_Vol-2.indd 224 7/31/2019 7:31:12 PM


Boolean equation:
Y = A. B A NOT Y
It performs logical multiplication and is
different from arithmetic multiplication. (a)
Logic operation Inputs Output
The output of AND gate is high (1) only A Y=A
when all the inputs are high (1). The rest of 0 1
the cases the output is low. Hence the output 1 0
of AND gate is high (1) only when all the
inputs are high. It is represented in the truth (b)
table (Figure 9.41(b)). Figure 9.43  (a) NOT gate (b) Truth Table
OR gate
Boolean equation
Circuit Symbol
The circuit symbol of a two input OR Y=A
gate is shown in Figure 9.42(a). A and B are Logic operation
inputs and Y is the output. The output is the complement of the
A Y
Inputs Output input. It is represented with an overbar. It is
OR
B A B Y = A+ B also called as inverter. The truth table infers
0 0 0 that the output Y is 1 when input A is 0 and
(a)
0 1 1 vice versa. The truth table of NOT is shown
1 0 1 in Figure 9.43(b).
1 1 1 NAND gate
(b) The circuit symbol of NAND gate is
Figure 9.42  (a) Two input OR gate shown in Figure 9.44(a). A and B are inputs
(b) Truth Table and Y is the output.

Boolean equation: A
Z A
NAND
AND NOT
A + B =Y B Y B Y

It performs logical addition and is (a)


different from arithmetic addition.
Input Output Output
Logic operation
(AND) (NAND)
The output of OR gate is high (logic 1
state) when either of the inputs or both are A B Z= A.B Y = A.B
high. The truth table of OR gate is shown in 0 0 0 1
Figure 9.42(b). 0 1 0 1
1 0 0 1
NOT gate
1 1 1 0
Circuit symbol
The circuit symbol of NOT gate is shown (b)
in Figure 9.43(a). A is the input and Y is the Figure 9.44  (a)Two input NAND gate
(b) Truth Table
output.

Unit 9  Semic onductor electronics 225

UNIT-9_XII-Physics_Vol-2.indd 225 7/31/2019 7:31:13 PM


Boolean equation The output is high when all the inputs
Y = A.B are low. The output is low for all other
Logic operation combinations of inputs. The truth table of
The output Y equals the complement NOR gate is shown in Figure 9.45(b).
of AND operation. The circuit is an AND Ex-OR gate
gate followed by a NOT gate. Therefore, it Circuit symbol
is summarized as NAND. The output is at The circuit symbol of Ex-OR gate is
logic zero only when all the inputs are high. shown in Figure 9.46(a). A and B are inputs
The rest of the cases, the output is high and Y is the output. The Ex-OR operation is
(Logic 1 state). The truth table of NAND denoted as ⊕.
gate is shown in Figure 9.44(b).
Boolean equation
NOR gate Y = A. B + A. B
Circuit symbol Y = A⊕ B
The circuit symbol of NOR gate is shown
Logic operation
in Figure 9.45(a). A and B are inputs and
The output is high only when either
Y is the output.
of the two inputs is high. In the case of an
A
OR Z
NOT Y
A
NOR Y
Ex-OR gate with more than two inputs, the
B B
output will be high when odd number of
(a) inputs are high. The truth table of Ex-OR
Inputs Output Output gate is shown in Figure 9.46(b).
(OR) (NOR)
A B Z=A+B Y = A+B A
0 0 0 1 Y
B
0 1 1 0
1 0 1 0 (a)
1 1 1 0
Inputs Output
(b)
(Ex-OR)
Figure: 9.45  (a) NOR gate (b) Truth
Table A B Y = A⊕ B
0 0 0
Boolean equation 0 1 1
1 0 1
Y = A+B
1 1 0
Logic operation
(b)
Y equals the complement of OR operation
Figure 9.46  (a) Ex-OR gate (b) Truth
(A OR B). The circuit is an OR gate followed
Table
by a NOT gate and is summarized as NOR.

226 Unit 9  Semic onductor electronics

UNIT-9_XII-Physics_Vol-2.indd 226 7/31/2019 7:31:14 PM


E X A M P L E 9. 10 Boole in 1854. Later the importance of
Boolean algebra was realized in the design
What is the output Y in the following of computer circuits. Today we are in a
circuit, when all the three inputs A, B, and digital world and most of the comforts that
C are first 0 and then 1?  we experience is due to digitization with the
A foundation based on Boolean algebra.
X
P
B Q Y
C The concept of high (1) and
low (0) is not a new one. In fact,
Solution it was applied in telephone
switching circuits by Shannon in 1938.
A B C X = A.B Y = X .C
0 0 0 0 1
Laws of Boolean Algebra
1 1 1 1 0
The NOT, OR and AND operations
discussed in 9.5.2 are the Boolean
operations. The results of these operations
E X A M P L E 9. 11
can be summarised as:
In the combination of the following gates,
Complement law
write the Boolean equation for output Y in
terms of inputs A and B.
A Y=A
A
1 0
B Y = 0 =1
Y 1 Y =1 = 0
2

The complement law can be realised as


Solution A= A
The output at the 1 AND gate: AB
st

OR laws
The output at the 2nd AND gate: A B
A B Y = A+B
The output at the OR gate: Y = A . B + A . B 0 0 Y = 0+0 = 0
0 1 Y = 0+1 = 1
9.6 1 0 Y = 1+0 = 1
BOOLEAN ALGEBRA 1 1 Y = 1+1 = 1
The OR laws can be realised as
Boolean Algebra is basically a choice 1st law A+0=A
between two options (i) yes or no (ii) high or 2nd law A+1=1
low. These two options in Boolean algebra 3rd law A+A=A
are represented by binary numbers 0 or 1. It is
a concept that relates logic and mathematics
4th law A+ A =1
which is a century old, made up by George

Unit 9  Semic onductor electronics 227

UNIT-9_XII-Physics_Vol-2.indd 227 7/31/2019 7:31:16 PM


AND laws Both cases generate same outputs for
same inputs. It can be verified using the
A B Y = A.B
following truth table.
0 0 Y = 0.0 = 0
0 1 Y = 0.1 = 0 A B A+B A + B A B A. B
1 0 Y = 1.0 = 0
1 1 Y = 1.1 = 1 0 0 0 1 1 1 1
The AND laws can be realised as 0 1 1 0 1 0 0
1st law A.0=0 1 0 1 0 0 1 0
2nd law A.1=A
3rd law A.A=A 1 1 1 0 0 0 0
4th law
A . A= 0
From the above truth table, we can
The Boolean operations obey the conclude A + B = A . B .
following laws.
Thus De Morgan’s First Theorem is
Commutative laws proved. It also says that a NOR gate is equal
A+B=B+A to a bubbled AND gate.
A.B=B.A The corresponding logic circuit diagram
Associative laws is shown in Figure 9.47.
A + (B + C) = (A + B) + C
A
A . (B . C) = (A .B) . C Y
A
Y
B B
Distributive laws
A( B + C) = AB + AC Figure 9.47  NOR gate equals bubbled
AND gate
A + BC = (A + B) (A + C)
The above laws are used to simplify
complicated expressions and to simplify the 9.5.6.2  De Morgan’s Second Theorem
logic circuitry. The second theorem states that the
complement of the product of two inputs is
9.5.6  DE MORGAN’S THEOREM equal to the sum of its complements.
Proof
9.5.6.1  De Morgan’s First Theorem
The Boolean equation for NAND gate is
The first theorem states that the Y = AB
complement of the sum of two logical inputs The Boolean equation for bubbled OR
is equal to the product of its complements. gate is Y = A + B
Proof A and B are the inputs and Y is the
The Boolean equation for NOR gate is output. The above two equations produces
Y =A+B the same output for the same inputs. It can
The Boolean equation for a bubbled be verified by using the truth table
AND gate is Y = A . B
228 Unit 9  Semic onductor electronics

UNIT-9_XII-Physics_Vol-2.indd 228 7/31/2019 7:31:17 PM


of transistors, resistors, capacitors,
A B A.B A.B A B A+ B
etc. integrated on a small flat piece of
0 0 0 1 1 1 1 semiconductor material that is normally
0 1 0 1 1 0 1 Silicon.
1 0 0 1 0 1 1
1 1 1 0 0 0 0

From the above truth table we can


conclude A . B = A + B
Thus De Morgan’s First Theorem is
proved. It also says, a NAND gate is equal
to a bubbled OR gate.
The corresponding logic circuit diagram
is shown in Figure 9.48
A A
Y
B Y
B

Figure 9.48  NAND gate equals bubbled


OR gate

E X A M P L E : 9. 12

Simplify the Boolean identity


AC + ABC = AC Figure 9.49  Circuits with integrated chips

Solution Integrated circuits (ICs) are the


Step 1: AC (1 + B) = AC.1 [OR law-2] keystone of modern electronics. With
the advancement in technology and the
Step 2: AC . 1 = AC [AND law – 2]
emergence of Very Large Scale Integration
Therefore, AC + ABC = AC (VLSI) era it is possible to fit more and more
Circuit Description transistors on chips of same piece.
A AC
ICs have two main advantages over
B ordinary circuits: cost and performance.
A
Y=AC + ABC = B
Y=AC The size, speed, and capacity of chips
A
B
C ABC have progressed enormously with the
advancement in technology. Computers,
Thus the given statement is proved. mobile phones, and other digital home
appliances are now made possible by
9.5.7  Integrated Chips the small size and low cost of ICs. ICs
An integrated circuit is also referred can function as an amplifier, oscillator,
as an IC or a chip or a microchip (Figure timer, microprocessor and computer
9.49). It consists of thousands to millions memory.
Unit 9  Semic onductor electronics 229

UNIT-9_XII-Physics_Vol-2.indd 229 7/31/2019 7:31:18 PM


These extremely small ICs can perform Digital ICs usually find their applications
calculations and store data using either in computers, networking equipment, and
digital or analog technology. Digital ICs use most consumer electronics. Analog ICs
logic gates, which work only with values or linear ICs work with continuous values.
of ones and zeros. A low signal sent to a This means a component on a linear IC can
component on a digital IC will result in a take any value and output another value.
value of 0, while a high signal creates a value Linear ICs are typically used in audio and
of 1. radio frequency amplification.

SUMMARY

„„ Energy bands in solids are used to classify them into metals, insulators, and
semiconductors
„„ Doping increases the majority charge carriers and hence its conductivity.
„„ In a N-type semiconductor, electrons are the majority charge carriers and holes are
the minority carriers
„„ In P-type semiconductor, holes are the majority charge carriers and electrons are the
minority charge carriers
„„ A depletion region is formed in an unbiased PN junction. It is devoid of mobile
charge carriers. Instead, it has immobile ions
„„ When a PN junction diode is forward biased, the depletion region decreases and the
diode conducts once after the barrier potential is crossed. It acts like a closed switch.
„„ A PN junction diode in reverse biased condition functions as a open switch as it does
not conduct. The depletion region increases.
„„ A forward biased PN junction diode functions as a rectifier. Rectification is the
process of converting an AC current into DC current
„„ The half wave rectifier rectifies one half of the input signal and produces a pulsating
output.
„„ Full wave rectifier rectifies both the halves of the input signal.
„„ The efficiency of the full wave rectifier is two times the efficiency of the half wave
rectifier
„„ The two mechanisms that is responsible for breakdown under increasing reverse
voltage: Zener and Avalanche breakdown
„„ Zener breakdown happens in a heavily doped PN junction diode when a strong
electric field is applied.
„„ Avalanche breakdown occurs in lightly doped junctions which have wide depletion
layers. It is due to the breaking of covalent bonds by the thermally generated minority
charge carriers.
„„ Zener diode is a heavily doped PN junction diode works in the reverse biased
direction

230 Unit 9  Semic onductor electronics

UNIT-9_XII-Physics_Vol-2.indd 230 7/31/2019 7:31:18 PM


„„ Light emitting diode is a forward biased semiconductor device that emits emits
visible or invisible light when energized. The recombination of minority charge
carriers with the majority charge carriers in the respective regions release energy in
the form of Photons.
„„ A PN junction diode made of photosensitive material converts an optical signal into
electric current is called a photodiode.
„„ When a photon of sufficient energy strikes the diode, it creates an electron-hole
pair. These electrons and holes are swept across the p-n junction by the electric field
created by reverse voltage before recombination takes place and in turn generates
photo current.
„„ A solar cell is an electrical device that converts the energy of light directly into
electricity by the photovoltaic effect.
„„ A bipolar junction transistor is a semiconductor device is of two types: NPN and
PNP.
„„ BJT has three regions: emitter, base, and collector
„„ To operate the transistor in the active region, emitter base must be forward biased
and collector base must be reverse biased.
„„ A BJT can be operated in three different configurations: Common base, common
emitter, common collector.
„„ The forward current gain in common base configuration α gives the ratio of the
collector current to emitter current.
„„ The forward current gain in common emitter configuration β gives the ratio of the
collector current to the base current
„„ The BJT connected in common emitter configuration functions as a switch
„„ The BJT connected in common emitter configuration can be used as an amplifier.
There exists a phase reversal of 1800 between the input signal and the amplified
output signal.
„„ A transistor amplifier combined with a tank circuit and positive feedback acts as an
oscillator
„„ The logic gates are logical circuits provides output only for a combination of inputs.
„„ The basic logic gates are AND, OR, and NOT
„„ Boolean algebra is used to simplify complicated expressions and hence to simplify
the logic circuit.
„„ De Morgan’s First theorem states that the complement of the sum of two inputs is
equal to the product of its complements.
„„ The second theorem states that the complement of the product of two inputs is equal
to the sum of its complements.

Unit 9  Semic onductor electronics 231

UNIT-9_XII-Physics_Vol-2.indd 231 7/31/2019 7:31:18 PM


CONCEPT MAP

Energy BandsEnergy Bands

Metals Insulators Semiconductors

Intrinsic Extrinsic

p-type n-type

Diodes

p-n junction diode Special purpose diodes

Rectifier Zener Diode LED Photo diode Solar cell

PNP NPN

Transistors Characteristics Applications

Switch Amplifier Oscillator

Logicgates

AND OR NOT Ex-OR NAND NOR

Boolean algebra De morgan’s theorems

232 Unit 9  Semic onductor electronics

UNIT-9_XII-Physics_Vol-2.indd 232 7/31/2019 7:31:18 PM


EVALUATION

I. Multiple choice questions


7. The light emitted in an LED is due to
1. The barrier potential of a silicon diode a. Recombination of charge carriers
is approximately,
b. Reflection of light due to lens action
a. 0.7 V b. 0.3V
c. Amplification of light falling at the
c. 2.0 V d. 2.2V junction
2. Doping a semiconductor results in d. Large current capacity.
a. The decrease in mobile charge 8. When a transistor is fully switched on,
carriers it is said to be
b. The change in chemical properties a. Shorted
c. The change in the crystal structure b. Saturated
d. The breaking of the covalent bond c. Cut-off
3. A forward biased diode is treated as d. Open
a. An open switch with infinite 9. The specific characteristic of a common
resistance emitter amplifier is
b. A closed switch with a voltage drop a. High input resistance
of 0V
b. Low power gain
c. A closed switch in series with a
c. Signal phase reversal
battery voltage of 0.7V
d. Low current gain
d. A closed switch in series with a small
10. To obtain sustained oscillation in an
resistance and a battery.
oscillator,
4. If a half –wave rectified voltage is fed to
a. Feedback should be positive
a load resistor, which part of a cycle the
load current will flow? b. Feedback factor must be unity
a. 00–900 b. 900–1800 c. Phase shift must be 0 or 2π
c. 00–1800 d. 00–3600 d. All the above
5. The primary use of a zener diode is 11. If the input to the NOT gate is A =
1011, its output is
a. Rectifier b. Amplifier
a. 0100 b. 1000
c. Oscillator d. Voltage regulator
c. 1100 d. 0011
6. The principle in which a solar cell
operates 12. The electrical series circuit in digital
form is
a. Diffusion
a. AND b. OR
b. Recombination
c. NOR d. NAND
c. Photovoltaic action
d. Carrier flow

Unit 9 Semic onductor electronics 233

UNIT-9_XII-Physics_Vol-2.indd 233 7/31/2019 7:31:18 PM


13. Which one of the following represents II. Short Answer Questions
forward bias diode? (NEET) 1. Define electron motion in a
semiconductor.
a.

0V R –2 V
2. Distinguish between intrinsic and
extrinsic semiconductors.
b.

–4 V R –3 V
3. What do you mean by doping?
c.
4. How electron-hole pairs are created in
–2 V R +2 V
a semiconductor material?
d.
5. A diode is called as a unidirectional
–3 V R +5 V
device. Explain
6. What do you mean by leakage current
14. The given electrical network is in a diode?
equivalent to (NEET)
7. Draw the output waveform of a full
wave rectifier.
A 8. Distinguish between avalanche and
B Y zener breakdown.
a. AND gate 9. Discuss the biasing polarities in an
NPN and PNP transistors.
b. OR gate
10. Explain the current flow in a NPN
c. NOR gate
transistor
d. NOT gate
11. What is the phase relationship between
15. The output of the following circuit is 1
the AC input and output voltages in a
when the input ABC is (NEET 2016)
common emitter amplifier? What is
the reason for the phase reversal?
A 12. Explain the need for a feedback circuit
B
in a transistor oscillator.
Y
C 13. Give circuit symbol, logical operation,
truth table, and Boolean expression of
a. 101 AND, OR, NOT, NAND, NOR, and
b. 100 EX-OR gates
c. 110 14. State De Morgan’s first and second
theorems.
d. 010
III. Long Answer Questions
Answers
1. Elucidate the formation of a N-type
1. a 2. c 3. d 4. c 5. d and P-type semiconductors.
6. c 7. a 8. b 9. c 10. d 2. Explain the formation of PN junction
11. a 12. a 13. a 14. c 15. a diode. Discuss its V–I characteristics.

234 Unit 9  Semic onductor electronics

UNIT-9_XII-Physics_Vol-2.indd 234 7/31/2019 7:31:19 PM


3. Draw the circuit diagram of a half wave 2. Four silicon diodes and a 10 Ω resistor
rectifier and explain its working are connected as shown in figure below.
4. Explain the construction and working Each diode has a resistance of 1Ω. Find
of a full wave rectifier. the current flows through the 18Ω
5. What is an LED? Give the principle of resistor. [Ans: 0.13 A]
operation with a diagram.
C
6. Write notes on Photodiode.
D1 D3
7. Explain the working principle of a solar
cell. Mention its applications. A B
8. Sketch the static characteristics of a
D2 D4
common emitter transistor and bring
out the essence of input and output D
characteristics. + –
9. Describe the function of a transistor E F
as an amplifier with the neat circuit
diagram. Sketch the input and output 3. Assuming VCEsat = 0.2 V and β = 50,
wave form. find the minimum base current (IB)
10. Transistor functions as a switch. required to drive the transistor given in
Explain. the figure to saturation.  [Ans: 56 µA]
11. State Boolean laws. Elucidate how
they are used to simplify Boolean 3V
IC
expressions with suitable example.
12. State and prove De Morgan’s First and
1kΩ
Second theorems.

IV. Numerical Problems IB

1. The given circuit has two ideal diodes


connected as shown in figure below.
Calculate the current flowing through
the resistance R1  [Ans: 2.5 A]
2Ω

R1
D1 D2

10 V
R2 3Ω R3 2Ω

Unit 9  Semic onductor electronics 235

UNIT-9_XII-Physics_Vol-2.indd 235 7/31/2019 7:31:19 PM


4. A transistor having α =0.99 and 5. In the circuit shown in the figure, the
VBE = 0.7V, is given in the circuit. BJT has a current gain (β) of 50. For an
Find the value of the collector current. emitter – base voltage VEB = 600 mV,
 [Ans: 5.33 mA] calculate the emitter – collector voltage
VEC (in volts). [Ans: 2 V]
+12V

1k 3V

1k
10k

1k
60kΩ 500kΩ

BOOK FOR REFERENCES

1. Charles Kittel , Introduction to Solid State Physics, John Wiley & Sons, 2012
2. Rita John, Solid State Physics, McGraw Hill Education, 2016
3. Robert L. Boylestad, Louis Nashelsky, Electronic Devices and Circuit Theory , Pearson
Prentice Hall, 2011
4. Jacob Millman,   Christos Halkias,   Chetan Parikh,   Millman’s Integrated Electronics,
McGraw Hill Education, 2017
5. B.L.Theraja, R.S. Sedha, Principles of Electronics Devices and Circuits (Analog and Digital),
S. Chand & Company, 2011
6. Albert Paul Malvino, Donald P. Leach, Goutam Saha, Digital principles and applications,
McGraw Hill Education, 2014
7. V.K.Metha, Rohit Metha, Principles of Electronics, S. Chand & Company, 2010.

236 Unit 9  Semic onductor electronics

UNIT-9_XII-Physics_Vol-2.indd 236 7/31/2019 7:31:19 PM


ICT CORNER
Semiconductor electronics

In this activity you will be able to


(i) Construct , manipulate and simulate
the logic circuits. (ii) verify the truth tables Topic: Logic gates
of AND, OR, NOT, EX-OR, NAND and
NOR gates

STEPS:
• Open the browser and type “circuitverse.org/simulator” in the address bar.
• Click ‘Gates’ tab from the circuit elements. Select the gate you want to verify and drag it in
to the stage.
• Nodes in the logic gates are connected through wires. Wires can be drawn by dragging
from the nodes with the help of mouse.
• Select ‘input tool’ from input tab. Drag and keep it as two inputs.
• Select ‘output tool’ or ‘digital LED’ from output tab. Drag and keep it as output.
• Verify the truth tables of AND, OR, NOT, EX-OR, NAND and NOR gates. You can verify
De Morgan’s first and second theorems.

Step1 Step2

Step3 Step4

Note:
Login with the help of your mail id if you want to save your project in online.
URL:
https://circuitverse.org/simulator
* Pictures are indicative only.
* If browser requires, allow Flash Player or Java Script to load the page.

Unit 9 Semic onductor electronics 237

UNIT-9_XII-Physics_Vol-2.indd 237 7/31/2019 7:31:19 PM


UNIT

10
COMMUNICATION SYSTEMS

Good communication is the bridge between confusion and clarity


– Nat Turner

In this unit, the students are exposed to


• Basic elements of communication system
• Need for modulation and its types
• Propagation of electromagnetic waves through space
• Satellite communication
• Fiber optic communication
• RADAR
• Internet
• Global positioning system
• Applications of communication technology in fishing, mining, and agriculture sectors.

10.1 the shortest possible time. However, the 20th


century witnessed a leap over the development
INTRODUCTION of communication, meeting the demands of
speed and secured transfer of data. Every sector
in the world is experiencing a significant
Communication exists since the dawn of life
profit with the advent of Global Positioning
in this world. Growth in science and technology
System (GPS), satellite, mobile, and optical
removed the locational disadvantage
communications. This unit provides a
effectively. Information can be exchanged
glimpse of the basic concepts of electronic
from one person to another anywhere on this
communication and its applications.
Earth. Right from the developments made in
communication by great scientists like J.C. 10.2
Bose, G. Marconi, and Alexander Graham
Bell, communication has witnessed leaps and MODULATION
bounds. The communication industry is one
of the largest in size and is the oldest since The transmission of information through
communication through telegraph (1844), short distances does not require complicated
telephone (1876), and Radio (1887) started techniques. The energy of the information
centuries back. The intensive research in the signal is sufficient enough to be sent directly.
mid- and late nineteenth century leads to the However if the information, for example,
development of long-distance transmission in audio frequency (20 to 20,000 Hz) needs to
238

UNIT-10(XII-Physics_Vol-2).indd 238 7/31/2019 7:20:15 PM


be transmitted to long distances across the is called amplitude modulation. Here the
world, certain techniques are required to frequency and the phase of the carrier signal
transmit the information without any loss. remain constant. Amplitude modulation is
For long distance transmission, the low used in radio and TV broadcasting.
frequency baseband signal (input signal) is The signal shown in Figure 10.1(a) is the
superimposed onto a high frequency radio message signal or baseband signal that carries
signal by a process called modulation. In the information. Figure 10.1(b) shows the high-
modulation process, a very high frequency frequency carrier signal and Figure  10.1(c)
signal called carrier signal (radio signal) is gives the amplitude modulated signal.
used to carry the baseband signal. We can see clearly that the carrier wave is
As the frequency of the carrier signal modified in proportion to the amplitude of
is very high, it can be transmitted to long the baseband signal.
distances with less attenuation. The carrier Amplitude
Baseband Signal
signal is usually a sine wave signal. Also, the
Am
(a)
carrier signal will be more compatible with
Time
the communication medium like free space
and can propagate with greater efficiency.
Amplitude
Carrier Signal (b)
Carrier signal does not have
Note
information. Ac

Time

A sinusoidal carrier wave can be


represented as ec = Ec sin (2πνc t + ϕ), where Amplitude Amplitude Modulated

Ec is the amplitude, νc is the frequency and


Signal (c)
Ac
ϕ is the initial phase of the carrier wave at
any instant of time t. Time
Three characteristics in the carrier Minimum
signal can be modified by the baseband Amplitude
Envelope of
Modulated Signal
signal during the process of modulation: Maximum
Amplitude
amplitude, frequency and phase of the
carrier signal. Figure 10.1 Amplitude Modulation
(a) baseband signal (b) carrier signal
There are 3 types of modulation based
(c) modulated signal
on which parameter is modified. They are
(i) amplitude modulation, (ii) frequency Advantages of AM
modulation, and (iii) phase modulation.
i) Easy transmission and reception
ii) Lesser bandwidth requirements
10.2.1 AMPLITUDE iii) Low cost
MODULATION (AM) Limitations of AM
If the amplitude of the carrier signal is i) Noise level is high
modified according to the instantaneous ii) Low efficiency
amplitude of the baseband signal, then it iii) Small operating range
Unit 10 C OMMUNICATION SYSTEMS 239

UNIT-10(XII-Physics_Vol-2).indd 239 7/31/2019 7:20:15 PM


10.2.2  FREQUENCY in the positive direction (A, C). The increase
MODULATION (FM) in amplitude in the negative half cycle (B,
D) reduces the frequency of the modulated
The frequency of the carrier signal is
wave (Figure 10.2(c)).
modified according to the instantaneous
When the frequency of the baseband
amplitude of the baseband signal in
signal is zero (no input signal), there is no
frequency modulation. Here the amplitude
change in the frequency of the carrier wave.
and the phase of the carrier signal remain
It is at its normal frequency and is called
constant. Increase in the amplitude of the
as centre frequency or resting frequency.
baseband signal increases the frequency of
Practically this is the allotted frequency of
the carrier signal and vice versa. This leads
the FM transmitter.
to compressions and rarefactions in the
frequency spectrum of the modulated wave Advantages of FM
as shown in Figure 10.2. Louder signal leads i) Large decrease in noise. This leads to
to compressions and relatively weaker signals an increase in signal-noise ratio.
to rarefactions. ii) The operating range is quite large.
Amplitude
A Baseband Signal C iii) The transmission efficiency is very
(a) high as all the transmitted power is
0 Time useful.
iv) FM bandwidth covers the entire
B D frequency range which humans can
Amplitude (b) hear. Due to this, FM radio has better
Carrier Signal
quality compared to AM radio.
0 Time Limitations of FM
i) FM requires a much wider channel.
ii) FM transmitters and receivers are
Amplitude
Frequency Modulated
Signal
(c) more complex and costly.
iii) 
In FM reception, less area is covered
0 compared to AM.
Time

A B C D 10.2.3  PHASE MODULATION


(PM)
Figure 10.2  Frequency Modulation
(a) baseband signal (b) carrier signal The instantaneous amplitude of the
(c) frequency modulated signal baseband signal modifies the phase of
the carrier signal keeping the amplitude
When the amplitude of the baseband and frequency constant is called phase
signal is zero in Figure 10.2(a), the modulation (Figure 10.3). This modulation
frequency of the modulated signal is the is used to generate frequency modulated
same as the carrier signal. The frequency signals. It is similar to frequency modulation
of the modulated wave increases when the except that the phase of the carrier is varied
amplitude of the baseband signal increases instead of varying frequency.
240 Unit 10  C OMMUNICATION SYSTEMS

UNIT-10(XII-Physics_Vol-2).indd 240 7/31/2019 7:20:15 PM


Advantages of PM
i) FM signal produced from PM signal is
B (b) F
A
Carrier (fc)
very stable.
C E
Baseband
Input (fm)
(a) ii) The centre frequency called resting
D
frequency is extremely stable.
(c)

PM waveform Comparison between FM


φc

φc Max

φc

φc Min

φc

φc Max
Phase
lead
Phase
lead
Phase
leg
Phase
leg
Phase
lead
Phase
lead Note and PM
PM wave is similar to FM
Figure 10.3 Phase Modulation
(a) carrier signal (b) baseband signal wave. PM generally uses a smaller
(c) phase modulated signal bandwidth than FM. In other words,
in PM, more information can be sent
The carrier phase changes according in a given bandwidth. Hence, phase
to increase or decrease in the amplitude of modulation provides high transmission
the baseband signal. When the modulating speed on a given bandwidth.
signal goes positive, the amount of phase lead
increases with the amplitude of the modulating
10.3
signal. Due to this, the carrier signal is
compressed or its frequency is increased. THE ELEMENTS OF
On the other hand, the negative half cycle AN ELECTRONIC
of the baseband signal produces a phase lag COMMUNICATION SYSTEM
in the carrier signal. This appears to have
stretched the frequency of the carrier wave. Electronics plays a major role in
Hence similar to frequency modulated communication. Electronic communication
wave, phase modulated wave also comprises is nothing but the transmission of sound,
of compressions and rarefactions. When the text, pictures, or data through a medium.
signal voltage is zero (A, C and E) the carrier Long distance transmission uses free
frequency is unchanged. space as a medium. This section provides
The frequency shift in carrier wave sufficient information on how voice signal is
frequency exists in phase modulation transmitted by a transmitter through space
as well. The frequency shift depends on and received by the receiver at the receiving
(i) amplitude of the modulating signal and end.
(ii) the frequency of the signal.
Elements of an electronic communication
„ If a square wave is used system
Note
as the baseband signal, then The elements of the basic
phase reversal takes place in communication system are explained with
the modulated signal. the block diagram shown in Figure 10.4.
„ FM and PM waves are completely
1. Information (Baseband or input signal)
different for square wave
modulating signal. Information can be in the form of a
sound signal like speech, music, pictures, or
Unit 10 C OMMUNICATION SYSTEMS 241

UNIT-10(XII-Physics_Vol-2).indd 241 7/31/2019 7:20:15 PM


computer data which is given as input to the oscillator, modulator, and power amplifier.
input transducer. The transmitter is located at the broadcasting
station.
2.  Input transducer
Amplifier: The transducer output is very
A transducer is a device that converts weak and is amplified by the amplifier.
variations in a physical quantity (pressure, Oscillator: It generates high-frequency carrier
temperature, sound) into an equivalent wave (a sinusoidal wave) for long distance
electrical signal or vice versa. In transmission into space. As the energy of
communication system, the transducer a wave is proportional to its frequency, the
converts the information which is in the carrier wave has very high energy.
form of sound, music, pictures or computer Modulator: It superimposes the baseband
data into corresponding electrical signals. signal onto the carrier signal and generates
The electrical equivalent of the original the modulated signal.
information is called the baseband signal. Power amplifier: It increases the power
The best example for the transducer is the level of the electrical signal in order to cover
microphone that converts sound energy a large distance.
into electrical energy.
4.  Transmitting antenna
3. Transmitter It radiates the radio signal into space
It feeds the electrical signal from the in all directions. It travels in the form of
transducer to the communication channel. electromagnetic waves with the velocity of
It consists of circuits such as amplifier, light (3 × 108 m s–1).

Transmission Communication
Channel

Input
Transducer Amplifier
Power
Modulator Amplifer

Oscillator

Reception
Communication
Channel Base Band
Signal

Radio Output
Demodulator Amplifier Transducer
Receiver

Carrier
waves

Figure 10.4  Block diagram of transmission and reception of voice signals

242 Unit 10  C OMMUNICATION SYSTEMS

UNIT-10(XII-Physics_Vol-2).indd 242 7/31/2019 7:20:16 PM


5.  Communication channel signal. Then the baseband signal is detected
Communication channel is used to carry and amplified using amplifiers. Finally, it is
the electrical signal from transmitter to fed to the output transducer.
receiver with less noise or distortion. The 8. Repeaters
communication medium is basically of two Repeaters are used to increase the range
types: wireline communication and wireless or distance through which the signals are
communication. sent. It is a combination of transmitter and
Wireline communication (point- receiver. The signals are received, amplified,
point communication) uses mediums like and retransmitted with a carrier signal of
wires, cables and optical fibers. These different frequency to the destination. The
systems cannot be used for long distance best example is the communication satellite
transmission as they are connected in space.
physically. Examples are telephone,
intercom and cable TV. 9.  Output transducer
Wireless communication uses free space It converts the electrical signal back to its
as a communication medium. The signals are original form such as sound, music, pictures
transmitted in the form of electromagnetic or data. Examples of output transducers
waves with the help of a transmitting antenna. are loudspeakers, picture tubes, computer
Hence wireless communication is used monitor, etc.
for long distance transmission. Examples 10. Attenuation
are mobile, radio or TV broadcasting, and
The loss of strength of a signal while
satellite communication.
propagating through a medium is known as
6. Noise attenuation.
It is the undesirable electrical signal 11. Range
that interfaces with the transmitted signal.
It is the maximum distance between the
Noise attenuates or reduces the quality of
source and the destination up to which the
the transmitted signal. It may be man-made
signal is received with sufficient strength.
(automobiles, welding machines, electric
motors etc.) or natural (lightning, radiation
from sun and stars and environmental 10.3.1  BANDWIDTH
effects). Noise cannot be completely The frequency range over which the
eliminated. However, it can be reduced baseband signals or the information
using various techniques. signals such as voice, music, picture, etc. is
7. Receiver transmitted is known as bandwidth. Each
The signals that are transmitted through of these signals has different frequencies.
the communication medium are received The type of communication system
with the help of a receiving antenna and are depends on the nature of the frequency
fed into the receiver. The receiver consists band for a given signal. Bandwidth gives
of electronic circuits like demodulator, the difference between the upper and lower
amplifier, detector etc. The demodulator frequency limits of the signal. It can also be
extracts the baseband signal from the carrier defined as the portion of the electromagnetic
Unit 10  C OMMUNICATION SYSTEMS 243

UNIT-10(XII-Physics_Vol-2).indd 243 7/31/2019 7:20:16 PM


spectrum occupied by the signal. If ν1 and ν2 An example
are the lower and upper-frequency limits of Let us consider two baseband signals.
a signal then the bandwidth, BW = ν2 – ν1. One signal is modulated and the other is not
modulated.
The frequency of the original baseband
10.3.2  BANDWIDTH OF signal is taken as ν = 10 kHz while the
TRANSMISSION SYSTEM modulated signal is ν = 1 MHz.
The range of frequencies required to The height of the antenna required to
transmit a piece of specified information transmit the original baseband signal of
­
in a particular channel is called channel frequency ν = 10 kHz is
bandwidth or the bandwidth of the
transmission system. This corresponds to λ c 3×108
 h1 = = = = 7.5 km (10.2)
the spectrum that is assigned to be used by the 4 4v 4×10×103
system. For example, amplitude modulation The height of the antenna required to
system requires a channel bandwidth of transmit the modulated signal of frequency
10 kHz to transmit a 5 kHz signal, whereas ν = 1 MHz is
a single side-band system requires only
a 5 kHz channel bandwidth for the same   3×108
λ c
h2 = = = = 75 m (10.3)
5 kHz signal. This is because in amplitude 4 4v 4×1×106
modulation, the channel bandwidth is
twice the signal frequency. Therefore, it is Comparing equations (10.2) and (10.3),
required to reduce the channel bandwidth we can infer that it is practically feasible to
to accommodate more number of channels construct an antenna of height 75 m while
in the available electromagnetic spectrum. the one with 7.5 km is not possible. It clearly
In some applications, modulation is selected manifests that modulated signals reduce the
based on this. antenna height and are required for long
distance transmission.
10.4
10.5
ANTENNA SIZE
PROPAGATION OF
ELECTROMAGNETIC WAVES
Antenna is used at both transmitter and
receiver end. Antenna height is an important The information signal modulated with
parameter to be discussed. The height of the the carrier wave (radio wave) is transmitted
=λ.
antenna must be a multiplehof by an antenna. This travels through space
4
and is received by the receiving antenna
at the other end. The frequencies from
h= λ  (10.1)
4 2 kHz to 400 GHz are transmitted through
wireless communication. The strength of
where λ is wavelength ( λ= c ), c is the the electromagnetic wave keeps decreasing
v
velocity of light and ν is the frequency of the while traveling from transmitter to the
signal to be transmitted. receiver. The electromagnetic wave

244 Unit 10  C OMMUNICATION SYSTEMS

UNIT-10(XII-Physics_Vol-2).indd 244 7/31/2019 7:20:17 PM


transmitted by the transmitter travels in which leads to the attenuation of the
three different modes to reach the receiver wave.
according to its frequency range: • Tilting of the wave: As the wave
progresses, the wavefront starts gradually
• Ground wave propagation (or) surface
tilting according to the curvature of the
wave propagation (nearly 2 kHz to
Earth. This increase in the tilt decreases
2 MHz)
the electric field strength of the wave.
• Sky wave propagation (or) ionospheric
Finally, at some distance, the surface
propagation (nearly 3 MHz to 30 MHz)
wave dies out due to energy loss.
• Space wave propagation (nearly 30
MHz to 400 GHz) Atmosphere

10.5.1  GROUND WAVE Radio Home


tower Earth
PROPAGATION (a)

If the electromagnetic waves Ionosphere


transmitted by the transmitter glide over
the surface of the earth to reach the receiver,
then the propagation is called ground
(2)
wave propagation. The corresponding (1) (3)
waves are called ground waves or surface
waves. The pictorial representation is shown A B

in Figure 10.5(a). wav
e
Skip zone
Sky
wa
und age tance ve
Grocover
Both transmitting and receiving Sk y w ave sk
ip dis cov
era
ge
(b)
antennas must be close to the earth. The
size of the antenna plays a major role in Transmitter
Direct wave
Receiver

deciding the efficiency of the radiation of Refl


ecte
signals. d wave

During transmission, the electrical Earth


(c)
signals are attenuated over a distance. Some
reasons for attenuation are as follows: Communication satellite

• Increasing distance: The attenuation


of the signal depends on (i) power of Ionosphere Space wave

the transmitter (ii) frequency of the


transmitter, and (iii) condition of the
LOS
earth surface. LOS
Sky wave
Ground water
• Absorption of energy by the Earth:
When the transmitted signal in the Earth
form of EM wave is in contact with the (d)

Earth, it induces charges in the Earth


Figure 10.5 Propagation of EM waves
and constitutes a current. Due to this, (a) Ground wave (b) Sky wave (c) Space wave
the earth behaves like a leaky capacitor (d) Summary of all modes of propagation

Unit 10  C OMMUNICATION SYSTEMS 245

UNIT-10(XII-Physics_Vol-2).indd 245 7/31/2019 7:20:17 PM


The frequency of the ground waves is ultraviolet rays, cosmic ray, and other
mostly less than 2 MHz as high frequency high energy radiations like α, β rays from
waves undergo more absorption of energy sun, the air molecules in the ionosphere
at the earth’s atmosphere. The medium wave get ionized. This produces charged
signals received during the day time use ions and these ions provide a reflecting
surface wave propagation. medium for the reflection of radio waves
It is mainly used in local broadcasting, or communication waves back to earth
radio navigation, for ship-to-ship, ship- within the permitted frequency range.
to-shore communication and mobile The phenomenon of bending the radio
communication. waves back to earth is nothing but the total
internal reflection.
This is the reason why the EM waves
10.5.2 SKY WAVE are transmitted at a critical angle to ensure
PROPAGATION that the waves undergo total reflection and
The mode of propagation in which reaches the ground without escaping into
the electromagnetic waves radiated from space.
an antenna, directed upwards at large The shortest distance between the
angles gets reflected by the ionosphere transmitter and the point of reception of
back to earth is called sky wave the sky wave along the surface is called as
propagation or ionospheric propagation. the skip distance shown in Figure 10.5(b).
The corresponding waves are called sky The electromagnetic waves are
waves (Figure 10.5(b)). transmitted from the ground at particular
The frequency range of EM waves angles. When the angle of emission
in this mode of propagation is 3 to 30 increases, the reception of ground waves
MHz. EM waves of frequency more decreases. At one point there will be no
than 30 MHz can easily penetrate reception due to ground waves and marked
through the ionosphere and does not as A in the Figure 10.5(b).
undergo reflection. It is used for short If the angle of emission is increased
wave broadcast services. Medium and further, the reception of sky waves starts
high frequencies are for long-distance marked as B in the Figure 10.5(b). There
radio communication. Extremely long- is a zone in between where there is no
distance communication is also possible reception of electromagnetic waves
as the radio waves can undergo multiple neither ground nor sky, called as skip
reflections between the earth and the zone or skip area.
ionosphere. A single reflection helps
the radio waves to travel a distance of The higher the frequency,
Note
approximately 4000 km. higher is the skip distance
Ionosphere acts as a reflecting surface. and for a frequency less than
It is at a distance of approximately 50 the critical frequency, skip distance is
km and spreads up to 400 km above the zero.
Earth surface. Due to the absorption of

246 Unit 10 C OMMUNICATION SYSTEMS

UNIT-10(XII-Physics_Vol-2).indd 246 7/31/2019 7:20:17 PM


10.5.3  SPACE WAVE The range or distance (d) of coverage of
PROPAGATION the propagation depends on the height (h)
of the antenna given by the equation,
The process of sending and receiving
information signal through space is
d = 2 Rh (10.4)
called space wave communication
(Figure   10.5(c)). The electromagnetic
where, R is the radius of the earth which is,
waves of very high frequencies above 30
R = 6400 km
MHz are called as space waves. These
The distance of coverage is shown
waves travel in a straight line from the
pictorially in Figure 10.6.
transmitter to the receiver. Hence, it is
used for a line of sight communication EX AM P L E 1 0 . 1
(LOS).
A transmitting antenna has a height of
For high frequencies, the transmission
40 m and the height of the receiving
towers must be high enough so that the
antenna is 30 m. What is the maximum
transmitted and received signals (direct
distance between them for line-of-sight
waves) will not encounter the curvature
communication? The radius of the earth is
of the earth and hence travel with less
6.4×106 m.
attenuation and loss of signal strength.
Certain waves reach the receiver after
getting reflected from the ground. d1 d2

The communication systems


like television broadcast, satellite h1 h2
communication, and RADAR are based on T R
space wave propagation. Microwaves having
high frequencies (super high frequency
band) are used against radio waves due to Solution:
certain advantages: larger bandwidth, high The total distance d between the
data rates, better directivity, small antenna transmitting and receiving antennas will
size, low power consumption, etc. be the sum of the individual distances of
coverage.

d d = d1 + d2
= 2Rh1 + 2Rh2

d1 + d 2 = 2 R ( h1 + h2)
h d 1 + d 2 = 2 × 6.4 × 106 ×( 40 + 30 )
d1 + d 2 = 16 × 102 5 × (6.32 + 5.48)

d1 + d2 = 42217 m = 42.217 km
Figure 10.6  Distance of coverage

Unit 10  C OMMUNICATION SYSTEMS 247

UNIT-10(XII-Physics_Vol-2).indd 247 7/31/2019 7:20:20 PM


10.6 Applications

SATELLITE Satellites are classified into different


COMMUNICATION types based on their applications. Some
satellites are discussed below.
i) Weather Satellites: They are used to
The satellite communication is a mode monitor the weather and climate of
of communication of signal between Earth. By measuring cloud mass, these
transmitter and receiver via satellite. The satellites enable us to predict rain and
message signal from the Earth station is dangerous storms like hurricanes,
transmitted to the satellite on board via an cyclones etc.
uplink (frequency band 6 GHz), amplified ii) Communication satellites: They are
by a transponder and then retransmitted used to transmit television, radio,
to another earth station via a downlink internet signals etc. Multiple satellites
(frequency band 4 GHz) (Figure 10.7). are used for long distances.
The high-frequency radio wave signals iii) Navigation satellites: These are
travel in a straight line (line of sight) may employed to determine the geographic
come across tall buildings or mountains or location of ships, aircrafts or any other
even encounter the curvature of the earth. A object.
communication satellite relays and amplifies
such radio signals via transponder to reach 10.7
distant and far off places using uplinks FIBRE OPTIC
and downlinks. It is also called as a radio COMMUNICATION
repeater in sky. The applications are found
to be in all fields and are discussed below.
The method of transmitting
information from one place to another in
Earth
transmitting
station
terms of light pulses through an  optical
Earth Uplink fiber is called fiber optic communication.
It is in the process of replacing wire
Satellite transmission in communication systems.
Downlink (in geostationary orbit)
Earth
receiving
station

Figure 10.8  Optical fibers

Light has very high frequency (400 THz –


790 THz) than microwave radio systems. The
Figure 10.7  Satellite communication system fibers are made up of silica glass or silicon
dioxide which is highly abundant on Earth.
248 Unit 10  C OMMUNICATION SYSTEMS

UNIT-10(XII-Physics_Vol-2).indd 248 7/31/2019 7:20:21 PM


Now it has been replaced with materials such
as chalcogenide glasses, fluoroaluminate Most transatlantic
crystalline materials because they provide telecommunication cables
larger infrared wavelength and better between the United States of
transmission capability. America and Europe are fiber optic.
As fibers are not electrically conductive,
it is preferred in places where multiple 10.8
channels are to be laid and isolation is
RADAR AND
required from electrical and electromagnetic
APPLICATIONS
interference.
Applications Radar basically stands for Radio
Optical fiber system has a number Detection and Ranging System. It is
of applications namely, international one of the important applications of
communication, inter-city communication, communication systems and is mainly
data links, plant and traffic control and used to sense, detect, and locate distant
defense applications. objects like aircraft, ships, spacecraft, etc.
The angle, range, or velocity of the objects
Merits
that are invisible to the human eye can be
i) Fiber cables are very thin and weigh determined.
lesser than copper cables. Radar uses electromagnetic waves for
ii) 
This system has much larger band communication. The electromagnetic
width. This means that its information signal is initially radiated into space by an
carrying capacity is larger. antenna in all directions. When this signal
iii) 
Fiber optic system is immune to strikes the targeted object, it gets reflected
electrical interferences. or reradiated in many directions. This
iv) 
Fiber optic cables are cheaper than reflected (echo) signal is received by the
copper cables. radar antenna which in turn is delivered
Demerits to the receiver. Then, it is processed and
i)  Fiber optic cables are more fragile amplified to determine the geographical
when compared to copper wires. statistics of the object. The range is
ii) It is an expensive technology. determined by calculating the time taken
by the signal to travel from RADAR to the
Fiber optic cables provide target and back.
the fastest transmission rate Applications
compared to any other form
Radars find extensive applications in
of transmission. It can provide data
almost all fields. A few are mentioned below.
speed of 1 Gbps for homes and business.
i) In military, it is used for locating and
Multimode fibers operate at the speed
detecting the targets.
of 10 Mbps. Recent developments in
ii) It is used in navigation systems such
optical communication provide the
as ship borne surface search, air search
data speed at the rate of 25 Gb km/s
and weapons guidance systems.
Unit 10  C OMMUNICATION SYSTEMS 249

UNIT-10(XII-Physics_Vol-2).indd 249 7/31/2019 7:20:21 PM


iii) To measure precipitation rate and wind ii) Transmission of news across the globe
speed in meteorological observations, is done within a few seconds.
Radars are used. iii) Using Internet of Things (IoT), it
iv) It is employed to locate and rescue is made possible to control various
people in emergency situations. devices from a single device. Example:
home automation using a mobile
10.9 phone.
iv)  It enables smart classrooms, online
MOBILE COMMUNICATION
availability of notes, monitoring
student activities etc. in the field of
Mobile communication is used to
education.
communicate with others in different
locations without the use of any physical
Recently, the mobile
connection like wires or cables. It allows
communication technology
the transmission over a wide range of area
has evolved through
without the use of the physical link. It enables
various stages like 2G, 3G, 4G, 5G,
the people to communicate with each other
WiMAX, Wibro, EDGE, GPRS and
regardless of a particular location like office,
many others. This helps to increase
house, etc. It also provides communication
the speed of communication and the
access to remote areas.
range of coverage. The connectivity
issues have decreased with reliable and
secure connections. The GPS (Global
Positioning System) and GSM (Global
System for Mobile communication)
technology play an important role in
mobile communication. This increases
the utilization of bandwidth of the
network, sharing of the networks, error
detections, etc. Many methods like
digital switching, TDMA, CDMA have
Figure 10.9  Mobile communication
been used to ease the communication
process.
It provides the facility of roaming – that is,
the user may move from one place to another
without the need of compromising on the 10.10
communication. The maintenance and
cost of installation of this communication INTERNET
network are also cheap.
Applications Internet is a fast growing technology
i) It is used for personal communication in the field of communication system with
and cellular phones offer voice and multifaceted tools. It provides new ways and
data connectivity with high speed. means to interact and connect with people.
250 Unit 10  C OMMUNICATION SYSTEMS

UNIT-10(XII-Physics_Vol-2).indd 250 7/31/2019 7:20:22 PM


Internet is the largest computer network
recognized globally that connects millions
of people through computers. It finds
extensive applications in all walks of life.

To store all the information


available on the internet, you
would need over 1 billion
DVDs or 200 million Blu-ray discs.

Applications:
i) Search engine: The search engine is
basically a web-based service tool used
to search for information on World
Wide Web.
Figure 10.10  Constellation of GPS
ii) Communication: It helps millions of
satellites around Earth
people to connect with the use of social
networking: emails, instant messaging
The software then processes the data it
services and social networking tools.
accepts from each satellite to estimate the
iii) E-Commerce: Buying and selling of
location of the receiver.
goods and services, transfer of funds
are done over an electronic network. Applications
Global positioning system is highly
10.11 useful many fields such as fleet vehicle
GLOBAL POSITIONING management (for tracking cars, trucks and
SYSTEM buses), wildlife management (for counting
of wild animals) and engineering (for
GPS stands for Global Positioning making tunnels, bridges etc).
System. It is a  global  navigation satellite
system that offers geolocation and time 10.12
information to a GPS receiver anywhere on
APPLICATION OF
or near the Earth.
INFORMATION AND
GPS system works with the assistance
COMMUNICATION
of a satellite network. Each of these
TECHNOLOGY IN
satellites broadcasts a precise signal like an
AGRICULTURE, FISHERIES
ordinary radio signal. These signals that
AND MINING
convey the location data are received by
a low-cost aerial which is then translated
by the GPS software. The software is able (i) Agriculture
to recognize the satellite, its location, and The implementation of information
the time taken by the signals to travel from and communication technology (ICT)
each satellite. in agriculture sector enhances the
Unit 10  C OMMUNICATION SYSTEMS 251

UNIT-10(XII-Physics_Vol-2).indd 251 7/31/2019 7:20:24 PM


productivity, improves the living standards (ii) Fisheries
of farmers and overcomes the challenges a) Satellite vessel monitoring system
and risk factors. helps to identify fishing zones.
a) ICT is widely used in increasing food b) Use of barcodes helps to identify time
productivity and farm management. and date of catch, species name, quality
b) It helps to optimize the use of water, of fish.
seeds and fertilizers etc. (iii) Mining
c) Sophisticated technologies that a) ICT in mining improves operational
include robots, temperature and efficiency, remote monitoring and
moisture sensors,  aerial  images, and disaster locating system.
GPS technology can be used. b) Information and communication
d) Geographic information systems are technology provides audio-visual
extensively used in farming to decide warning to the trapped underground
the suitable place for the species to be miners.
planted. c) It helps to connect remote sites.

SUMMARY

„„ The basic elements required for the transmission and reception of a signal through
long distance using electromagnetic waves are transducer, amplifier, carrier signal,
modulator, power amplifier, medium of transmission, transmitting and receiving
antenna, demodulator, detector.
„„ For long-distance transmission, the baseband signal is modulated with the carrier wave.
„„ If the amplitude of the carrier signal is modified with the instantaneous amplitude of
the baseband signal then it is called amplitude modulation.
„„ The frequency of the carrier signal is modified with the instantaneous amplitude of
the baseband signal in frequency modulation.
„„ The instantaneous amplitude of the baseband signal modifies the phase of the carrier
signal keeping the amplitude and frequency constant is called phase modulation

„„ The height of the transmitting and receiving antenna must be a multiple of λ .


4
„„ If the EM waves transmitted by the transmitter glide over the surface of the earth to
reach the receiver, then the propagation of EM waves is called ground wave propagation.
„„ Electromagnetic waves radiated from an antenna, directed upwards at large angles
gets reflected by the ionosphere back to earth is called sky wave propagation.
„„ The process of sending and receiving information signal through space is called
space wave communication.
„„ The satellite communication is a mode of communication of signal between
transmitter and receiver via satellite.
„„ Fiber-optic communication  is a method of transmitting information by sending
pulses of light through an optical fiber.

252 Unit 10  C OMMUNICATION SYSTEMS

UNIT-10(XII-Physics_Vol-2).indd 252 7/31/2019 7:20:24 PM


„„ Radar basically stands for Radio Detection and Ranging System. It is one of the
important applications of communication systems for remote sensing.
„„ Mobile Communication is used to communicate with others in different locations
without the use of any physical connection like wires or cables.
„„ GPS stands for Global Positioning System that offers geolocation and time.
„„ Communication technology is used extensively in sectors like fisheries, mining, and
agriculture.

CONCEPT MAP

Communication system

Input signal (information signal) Transducer Amplifier Modulator

Antenna Transmitter

Communication channel

Wired Wireless

Ground wave propagation Sky wave propagation Space wave propagation


[nearly 2 kHz to 2 MHz] [nearly 3 MHz to 30 MHz] [nearly 30 MHz to 400 GHz]

Satellite communication RADAR GPS

Fibre optic communication

Demodulator Receiving Antenna

Detector Amplifier Transducer

Back to information signal

Unit 10  C OMMUNICATION SYSTEMS 253

UNIT-10(XII-Physics_Vol-2).indd 253 7/31/2019 7:20:24 PM


E V AL U A T I O N

I Multiple Choice Questions Answers


1. The output transducer of the 1. a 2. c 3. b 4. a 5. c
communication system converts the
radio signal into ---------------- II Short answers
(a) Sound 1. Give the factors that are responsible for
(b) Mechanical energy transmission impairments.
(c) Kinetic energy 2. Distinguish between wireline and
(d) None of the above wireless communication? Specify the
range of electromagnetic waves in
2. The signal is affected by noise in a which it is used.
communication system
3. Explain centre frequency or resting
(a) At the transmitter frequency in frequency modulation.
(b) At the modulator 4. What does RADAR stand for?
(c) In the channel 5. What do you mean by Internet of
(d) At the receiver Things?
3. The variation of frequency of carrier III Long Answers
wave with respect to the amplitude of
1. What is modulation? Explain the
the modulating signal is called ---------
types of modulation with necessary
---------
diagrams.
(a) Amplitude modulation
2. Elaborate on the basic elements of
(b) Frequency modulation communication system with the
(c) Phase modulation necessary block diagram.
(d) Pulse width modulation 3. Explain the three modes of propagation
4. The internationally accepted frequency of electromagnetic waves through
deviation for the purpose of FM space.
broadcasts. 4. What do you know about GPS? Write a
(a) 75 kHz (b) 68 kHz few applications of GPS.
(c) 80 kHz (d) 70 kHz 5. Give the applications of ICT in mining
and agriculture sectors.
5. The frequency range of 3 MHz to 30 6. Modulation helps to reduce the antenna
MHz is used for size in wireless communication –
(a) Ground wave propagation Explain.
(b) Space wave propagation 7. Fiber optic communication is gaining
(c) Sky wave propagation popularity among the various
(d) Satellite communication transmission media -justify.

254 Unit 10 C OMMUNICATION SYSTEMS

UNIT-10(XII-Physics_Vol-2).indd 254 7/31/2019 7:20:24 PM


BOOK FOR REFERENCES

1. B.L.Theraja, R.S. Sedha, Principles of Electronics Devices and Circuits (Analog and
Digital), S. Chand & Company, 2011.
2. K.D.Prasad, Antenna and Wave Propagation, Satya Prakashan, 2007.
3. U A Bakshi; A V Bakshi; K A Bakshi, Antenna and Wave Propagation, Technical
Publications, 2014.

ICT CORNER
Communication systems

In this activity you will be able to visualize


how the amplitude of the carrier wave is
Topic: Amplitude
changed in accordance with the intensity of
Modulation
the signal.

STEPS:
• Go to ‘academo.org’ page. Click Physics → Waves → Amplitude Modulation or open the browser and
type “academo.org/demos/amplitude-modulation/” in the address bar.
• Adjust the carrier wave (say f1 = 100 Hz, 200 Hz, etc.) and observe how the modulated wave changes.
• Adjust the signal wave (Say f2 = 10 Hz, 20 Hz, etc.) and observe how the modulated wave changes.
• Adjust the amplitude of the signal wave and observe how the modulated wave changes in accordance
with intensity of the signal.

Step1 Step2

Note:
Login with the help of your mail id if you want to save your project in online.
URL:
https://academo.org/demos/amplitude-modulation/
* Pictures are indicative only.
* If browser requires, allow Flash Player or Java Script to load the page.

Unit 10 C OMMUNICATION SYSTEMS 255

UNIT-10(XII-Physics_Vol-2).indd 255 7/31/2019 7:20:24 PM


UNIT
RECENT DEVELOPMENTS
11 IN PHYSICS

‘There’s Plenty of Room at the Bottom: An Invitation to Enter a New Field of Physics’
-Richard Feynman

256

Unit-11.indd 256 7/31/2019 7:18:33 PM


Unit 11 R ecent devel opments in physics 257

Unit-11.indd 257 7/31/2019 7:18:33 PM


258 Unit 11  R ecent devel opments in physics

Unit-11.indd 258 7/31/2019 7:18:34 PM


Unit 11  R ecent devel opments in physics 259

Unit-11.indd 259 7/31/2019 7:18:35 PM


260 Unit 11  R ecent devel opments in physics

Unit-11.indd 260 7/31/2019 7:18:37 PM


Unit 11  R ecent devel opments in physics 261

Unit-11.indd 261 7/31/2019 7:18:37 PM


262 Unit 11  R ecent devel opments in physics

Unit-11.indd 262 7/31/2019 7:18:37 PM


11.2.6 Applications of Nano technology

Optical Biomedical and


engineering and
communication drug delivery

Electronics Agriculture
and food

Metallurgy Cosmetics
and materials and paints

Defense and
security Biotechnology

Energy Textile
storage Applications

Nanotechnology

Applications of nanomaterial based products in differernt areas

Automotive industry Chemical industry Engineering


Lightweight construction Fillers for paint systems Wear protection for
Painting (fillers, base coat, Coating systems based tools and machines
clear coat) on nanocomposites (anti blocking coatings,
Catalysts Impregnation of papers scratch resistant
Tires (fillers) Switchable adhesives coatings on plastic
Sensors Magnetic fluids parts, etc.)
Coatings for wind- Lubricant-free bearings
screen and car bodies

Electronic industry Construction Medicine


Data memory Construction materials Drug delivery systems
Displays Thermal insulation Active agents
Laser diodes Flame retardants Contrast medium
Glass fibres Surface-functionalised Medical rapid tests
Optical switches building materials for Prostheses and
Filters (IR-blocking) wood, floors, stone, implants
Conductive, antistatic facades, tiles, roof tiles, etc. Antimicrobial agents
coatings Facade coatings and coatings
Groove mortar Agents in cancer therapy

Unit 11  R ecent devel opments in physics 263

Unit-11.indd 263 7/31/2019 7:18:38 PM


Textile/fabrics/ Energy Cosmetics
non-wovens

Surface-processed Fuel cells Sun protection


textiles Solar cells Lipsticks
Smart clothes Batteries Skin creams
Capacitors Tooth paste

Food and drinks Household Sports/ outdoor


Package materials Ceramic coatings for Ski wax
Storage life sensors irons Antifogging of
Additives Odors catalyst glasses/goggles
Clarification of fruit Cleaner for glass, Antifouling coatings
juices ceramic, floor, for ships/boats
windows Reinforced tennis
rackets and balls

11.2.7 Possible harmful effects of nanoparticles


The research on the harmful impact of application of nanotechnology is also equally
important and fast developing. The major concern here is that the nanoparticles have
the dimensions same as that of the biological molecules such as proteins. They may
easily get absorbed onto the surface of living organisms and they might enter the
tissues and fluids of the body.
The adsorbing nature depends on the surface of the nanoparticle. Indeed, it is possi-
ble to deliver a drug directly to a specific cell in the body by designing the surface of a
nanoparticle so that it adsorbs specifically onto the surface of the target cell.
The interaction with living systems is also affected by the dimensions of the nanopar-
ticles. For instance, nanoparticles of a few nanometers size may reach well inside
biomolecules, which is not possible for larger nanoparticles. Nanoparticles can also
cross cell membranes. It is also possible for the inhaled nanoparticles to reach the
blood, to reach other sites such as the liver, heart or blood cells.
Researchers are trying to understand the response of living organisms to the pres-
ence of nanoparticles of varying size, shape, chemical composition and surface
characteristics.

264 Unit 11  R ecent devel opments in physics

Unit-11.indd 264 7/31/2019 7:18:38 PM


DISEASES ASSOCIATED TO NANOPARTICLE EXPOSURE
(Not for examination)

Neurological diseases:
NANOPARTICLES Brain
Parkinson’s disease
INTERNALIZED Alzheimer’s disease
IN CELLS
Nanoparticle inhalation

Mitochondrion Asthma
Bronchitis
Nucleus
Lungs Emphysema
Cytoplasm Cancer
Heart
Membrane Arrythmia
Heart disease
Lipid vesicle
Death
Circulatory
system

Nanoparticles
ingestion

Gastro-Intestinal
System Diseases of
Crohn’s disease Other organs unknown
Colon cancer etiology in
kidneys, liver

Orthopedic implant
wear debris
Auto-immune diseases
Dermatitis Lymphatic Podoconiosis
Kaposi’s sarcoma
Urticaria system
Vasculitis

Auto-immune
diseases
Skin dermatitis

Unit 11 R ecent devel opments in physics 265

Unit-11.indd 265 7/31/2019 7:18:38 PM


11.3 Robotics
11.3.1 What is robotics?
Robotics is an integrated study of mechanical engineering, electronic engineering,
computer engineering, and science. Robot is a mechanical device designed with
electronic circuitry and programmed to perform a specific task. These automated
machines are highly significant in this robotic era where they can take up the role
of humans in certain dangerous environments that are hazardous to people like
defusing bombs, finding survivors in unstable ruins, and exploring mines and
shipwrecks.

Co-defender Services Co-inhabitant


Security

Logistics

Monitoring Intelligent
Inspection Co-Robot
Manufacure
Human-
& Automation
Robot
Macro
Interfaces
Intelligent
transportation Manufacture
&
Automation
Micro/Nano
Unmanned
vehicles Medical
Surgery
Rehabilitate
Co-explorer orthotics
prosthetics Co-worker

In 1954, George Devol invented the first digitally operated programmable robot
called Unimate. George Devol and Joseph Engelberger, the father of the modern
robotics industry formed the world’s first robot company in 1956. In 1961,
Unimate, was operated in a General Motors automobile factory for moving car
parts around in New Jersey.

266 Unit 11  R ecent devel opments in physics

Unit-11.indd 266 7/31/2019 7:18:39 PM


11.3.2 COMPONENTS OF ROBOTICS
The robotic system mainly consists of sensors, power supplies, control systems, manipulators and
necessary software.

Most robots are composed of


3 main parts:
Key components
1. The Controller ‐ also known as the
"brain" which is run by a computer Power conversion
program. It gives commands for the Sensors Actuators
moving parts to perform the job. unit
2. Mechanical parts - motors, pistons,
grippers, wheels, and gears that make Controller
the robot move, grab, turn, and lift.
3. Sensors - to tell the robot about its User
surroundings. It helps to determine Manipulator
the sizes and shapes of the objects interface linkage base
around, distance between the objects,
and directions as well.

11.3.3 TYPES OF ROBOTS


HUMAN ROBOT
Certain robots are made to resemble humans
in appearance and replicate the human
activities like walking, lifting, and sensing, etc.

1. Power conversion unit: Robots are powered by batteries, solar power, and hydraulics.
2. Actuators: Converts energy into movement. The majority of the actuators produce rotational or
linear motion.
3. Electric motors: They are used to actuate the parts of the robots like wheels, arms, fingers, legs,
sensors, camera, weapon systems etc. Different types of electric motors are used. The most often
used ones are AC motor, Brushed DC motor, Brushless DC motor, Geared DC motor, etc.
4. Pneumatic Air Muscles: They are devices that can contract and expand when air is pumped inside.
It can replicate the function of a human muscle. They contract almost 40% when the air is sucked
inside them.
5. Muscle wires: They are thin strands of wire made of shape memory alloys. They can contract by 5%
when electric current is passed through them.
6. Piezo Motors and Ultrasonic Motors: Basically, we use it for industrial robots.
7. Sensors: Generally used in task environments as it provides information of real-time knowledge.
8. Robot locomotion: Provides the types of movements to a robot. The different types are
(a) Legged (b) Wheeled (c) Combination of Legged and Wheeled Locomotion
(d) Tracked slip/skid

Unit 11  R ecent devel opments in physics 267

Unit-11.indd 267 7/31/2019 7:18:39 PM


INDUSTRIAL ROBOTS

Six main types of industrial robots Six-axis robots are ideal for
Cartesian 1 Arc Welding

* SCARA 2 Spot Welding

Cylindrical 3 Material Handling

Delta 4 Machine Tending


Polar 5
Other Applications
Vertically articulated 6

*Selective Compliance Assembly Robot Arm

Artificial Intelligence

Artificial
Intelligence

The aim of artificial intelligence is to


bring in human like behaviour in robots. It works on

1. Face recognition
2. Providing response to player’s actions in computer games
3. Taking decisions based on previous actions
4. To regulate the traffic by analyzing the density of traffic on roads.
5. Translate words from one language to another

268 Unit 11  R ecent devel opments in physics

Unit-11.indd 268 7/31/2019 7:18:40 PM


11.3.4 Applications

Outer space: Exploring stars, planets etc., investigation of the mineralogy


of the rocks and soils on Mars, analysis of elements found in rocks and soils.

Mars Rovers of NASA Twin Mars Rovers Mars Pathfinder Mission

Litter robot Welding Cutting Assembling Vacuum Cleaners


V

Lawn mowing
Packing Transport Surgery Weaponry

Laboratory Underwater Hospitals Agriculture Pool cleaning

The size of the nano robots is reduced to microscopic level to


perform a task in very small spaces. However, it is only in the
developmental stage. The future prospects of it are much expected
Nanorobots
in the medical field: Nano-robots in blood stream to perform small
surgical procedures, to fight against bacteria, repairing individual
cell in the body. It can travel into the body and once after the job is
performed it can find its way out. Chinese scientists have created
the world’s first autonomous DNA robots to combat cancer
tumours.

Unit 11  R ecent devel opments in physics 269

Unit-11.indd 269 7/31/2019 7:18:41 PM


Materials used to make robots
For robots, aluminum and steel are the most common metals. Aluminum is a softer
metal and is therefore easier to work with, but steel is several times stronger. In any
case, because of the inherent strength of metal, robot bodies are made using sheet, bar,
rod, channel, and other shapes.

11.3.5 Advantages of Robotics

1. The robots are much cheaper than humans.


2. Robots never get tired like humans. It can work for 24 x 7. Hence absenteeism
in work place can be reduced.
3. Robots are more precise and error free in performing the task.
4. Stronger and faster than humans.
5. Robots can work in extreme environmental conditions: extreme hot or cold,
space or underwater. In dangerous situations like bomb detection and bomb
deactivation.
6. In warfare, robots can save human lives.
7. Robots are significantly used in handling materials in chemical industries
especially in nuclear plants which can lead to health hazards in humans.

11.3.6 Disadvantages of Robotics


1. Robots have no sense of emotions or conscience.
2. They lack empathy and hence create an emotionless workplace.
3. If ultimately robots would do all the work, and the humans will just sit
and monitor them, health hazards will increase rapidly.
4. Unemployment problem will increase.
5. Robots can perform defined tasks and cannot handle unexpected situations
6. The robots are well programmed to do a job and if a small thing goes
wrong it ends up in a big loss to the company.
7. If a robot malfunctions, it takes time to identify the problem, rectify it,
and even reprogram if necessary. This process requires significant time.
8. Humans cannot be replaced by robots in decision making.
9. Till the robot reaches the level of human intelligence, the humans in work place will
exit.

270 Unit 11  R ecent devel opments in physics

Unit-11.indd 270 7/31/2019 7:18:41 PM


11. 4 Physics in medical diagnosis and therapy

Medical science very much revolves around physics principles. Medical instru-
mentation has widened the life span due to the technology integrated diagnosis
and treatment of most of the diseases. This modernisation in all fields is possible
due to efficient application of fundamental physics.

11.4.1 The development in medical field has been proportional to


the evolution of physics as indicated below
(Not for examination)
Invention in physics Technique
Year Image
(Inventors) used in medicine

X-rays Radiology-Xray
1 1895
(Wilhelm Conrad-Röntgen) imaging

Theory of Radioactivity
2 1896 and Radioisotope
(Antonie Henri Becquerrel,
1898 imaging
Pierre Curie and Marie Cuire)

Artifical Radioactivity
3 1934 (Joliot and Irene Curie) Scintigraphy

4 1950 Ultrasound Ecography

X-ray computed Computed


5 1979 tomography (Cormack and Tomography (CT)
Hounsfield)

Nuclear Magnetic
6 1952 Magnetic Resonance
Resonance (NMR) (Felix
Imaging (MRI)
Bloch and Edward Purcell)

Unit 11  R ecent devel opments in physics 271

Unit-11.indd 271 7/31/2019 7:18:41 PM


Invention in physics Technique
Year (Inventors) Image
used in medicine

Artifical Radioactivity Positron Emission


7 1934
(Joliot and Irene Curie) Tomography

8 1940’s Optical fibre Endoscopy,


Biomedical sensors

9 1960 LASER Surgical instrument


and diagonsis tool

Nanomedicine
10 1959 Nanotechnology
Drug delivary

11 2005 Dual Source Computed Computed


Tomography (DSCT) Tomography (CT)

Magnetic Resonance
12 2006 Magnetic resonance (MR)
Imaging (MRI)

272 Unit 11  R ecent devel opments in physics

Unit-11.indd 272 7/31/2019 7:18:41 PM


7/31/2019 7:18:42 PM
273
11.4. 2 The recent advancement in medical technology includes

STEP STEP STEP STEP


01 02 03 04
2

8
4

6
brain sensors
Precision
medicine

Artificial

inhalers
LOREM IPSUM LOREM IPSUM LOREM IPSUM LOREM IPSUM

Wireless
organs

Smart
Lorem ipsum dolor sit amet, consectetuer Lorem ipsum dolor sit amet, consectetuer Lorem ipsum dolor sit amet, consectetuer Lorem ipsum dolor sit amet, consectetuer
adipiscing elit, sed diam nonummy nibh adipiscing elit, sed diam nonummy nibh adipiscing elit, sed diam nonummy nibh adipiscing elit, sed diam nonummy nibh
euismod tincidunt ut laoreet dolore magna euismod tincidunt ut laoreet dolore magna euismod tincidunt ut laoreet dolore magna euismod tincidunt ut laoreet dolore magna
aliquam erat volutpat. Ut wisi enim ad aliquam erat volutpat. Ut wisi enim ad aliquam erat volutpat. Ut wisi enim ad aliquam erat volutpat. Ut wisi enim ad
minim veniam, quis nostrud exerci tation minim veniam, quis nostrud exerci tation minim veniam, quis nostrud exerci tation minim veniam, quis nostrud exerci tation
ullamcorper suscipit lobortis nisl ut aliquip ullamcorper suscipit lobortis nisl ut aliquip ullamcorper suscipit lobortis nisl ut aliquip ullamcorper suscipit lobortis nisl ut aliquip
ex ea Lorem ipsum dolor sit amet, ex ea Lorem ipsum dolor sit amet, ex ea Lorem ipsum dolor sit amet, ex ea Lorem ipsum dolor sit amet,
consectetuer adipiscing elit, sed diam consectetuer adipiscing elit, sed diam consectetuer adipiscing elit, sed diam consectetuer adipiscing elit, sed diam
nonummy nibh euismod tincidunt ut nonummy nibh euismod tincidunt ut nonummy nibh euismod tincidunt ut nonummy nibh euismod tincidunt ut
laoreet dolore magna aliquam erat laoreet dolore magna aliquam erat laoreet dolore magna aliquam erat laoreet dolore magna aliquam erat
volutpat. Ut wisi enim ad minim veniam, volutpat. Ut wisi enim ad minim veniam, volutpat. Ut wisi enim ad minim veniam, volutpat. Ut wisi enim ad minim veniam,
quis nostrud exerci tation ullamcorper quis nostrud exerci tation ullamcorper quis nostrud exerci tation ullamcorper quis nostrud exerci tation ullamcorper
suscipit lobortis nisl ut aliquip ex ea suscipit lobortis nisl ut aliquip ex ea suscipit lobortis nisl ut aliquip ex ea suscipit lobortis nisl ut aliquip ex ea
Lorem ipsum dolor sit amet, consectetuer Lorem ipsum dolor sit amet, consectetuer Lorem ipsum dolor sit amet, consectetuer Lorem ipsum dolor sit amet, consectetuer
READ MORE READ MORE READ MORE READ MORE

Unit 11  R ecent devel opments in physics


Virtual reality

EROM DAER EROM DAER EROM DAER EROM DAER

3-D printing
wearables
reutetcesnoc ,tema tis rolod muspi meroL reutetcesnoc ,tema tis rolod muspi meroL reutetcesnoc ,tema tis rolod muspi meroL reutetcesnoc ,tema tis rolod muspi meroL
ae xe piuqila tu lsin sitrobol tipicsus ae xe piuqila tu lsin sitrobol tipicsus ae xe piuqila tu lsin sitrobol tipicsus ae xe piuqila tu lsin sitrobol tipicsus

surgery
Robotic
reprocmallu noitat icrexe durtson siuq reprocmallu noitat icrexe durtson siuq reprocmallu noitat icrexe durtson siuq reprocmallu noitat icrexe durtson siuq

Health
,mainev minim da mine isiw tU .taptulov ,mainev minim da mine isiw tU .taptulov ,mainev minim da mine isiw tU .taptulov ,mainev minim da mine isiw tU .taptulov
tare mauqila angam erolod teeroal tare mauqila angam erolod teeroal tare mauqila angam erolod teeroal tare mauqila angam erolod teeroal
tu tnudicnit domsiue hbin ymmunon tu tnudicnit domsiue hbin ymmunon tu tnudicnit domsiue hbin ymmunon tu tnudicnit domsiue hbin ymmunon
maid des ,tile gnicsipida reutetcesnoc maid des ,tile gnicsipida reutetcesnoc maid des ,tile gnicsipida reutetcesnoc maid des ,tile gnicsipida reutetcesnoc
,tema tis rolod muspi meroL ae xe ,tema tis rolod muspi meroL ae xe ,tema tis rolod muspi meroL ae xe ,tema tis rolod muspi meroL ae xe
piuqila tu lsin sitrobol tipicsus reprocmallu piuqila tu lsin sitrobol tipicsus reprocmallu piuqila tu lsin sitrobol tipicsus reprocmallu piuqila tu lsin sitrobol tipicsus reprocmallu
noitat icrexe durtson siuq ,mainev minim noitat icrexe durtson siuq ,mainev minim noitat icrexe durtson siuq ,mainev minim noitat icrexe durtson siuq ,mainev minim
da mine isiw tU .taptulov tare mauqila da mine isiw tU .taptulov tare mauqila da mine isiw tU .taptulov tare mauqila da mine isiw tU .taptulov tare mauqila
angam erolod teeroal tu tnudicnit domsiue angam erolod teeroal tu tnudicnit domsiue angam erolod teeroal tu tnudicnit domsiue angam erolod teeroal tu tnudicnit domsiue
hbin ymmunon maid des ,tile gnicsipida hbin ymmunon maid des ,tile gnicsipida hbin ymmunon maid des ,tile gnicsipida hbin ymmunon maid des ,tile gnicsipida
reutetcesnoc ,tema tis rolod muspi meroL reutetcesnoc ,tema tis rolod muspi meroL reutetcesnoc ,tema tis rolod muspi meroL reutetcesnoc ,tema tis rolod muspi meroL
MUSPI MEROL MUSPI MEROL MUSPI MEROL MUSPI MEROL
10 20 30 40

7
3
PETS PETS PETS PETS

Unit-11.indd 273
The innovation in medical diagnosis has taken leaps and bounds due to the integration
of technology and basic physics. A few of such advancements are discussed.

1. Virtual reality

Medical virtual reality is effectively used to


stop the brain from processing pain and
cure soreness in the hospitalized patients.
Virtual reality has enhanced surgeries by
the use of 3D models by surgeons to plan
operations. It helps in the treatment of
Autism, Memory loss, and Mental illness.

2 2. Precision medicine
Precision medicine is an emerging approach for disease treatment and
prevention that takes into account individual variability in genes,
environment, and lifestyle for each person. In this medical model it is possible
to customise healthcare, with medical decisions, treatments, practices, or
products which are tailored to the individual patient.
PATIENT A MUTATION A DRUG A

PATIENT B MUTATION B DRUG B

274 Unit 11  R ecent devel opments in physics

Unit-11.indd 274 7/31/2019 7:18:42 PM


3 3. Health wearables

A health wearable is a device used for tracking a wearer's vital signs or


health and fitness related data, location, etc. Medical wearables with
artificial intelligence and big data provide an added value to healthcare with
a focus on diagnosis, treatment, patient monitoring and prevention.

NOTE
Big Data: Extremely large
data sets that may be analysed
computationally to reveal
patterns, trends, and
associations, especially
relating to human behaviour
and interactions.

4 4. Artificial organs

An artificial organ is an engineered device or tissue that is


implanted or integrated into a human. It is possible to interface it
with living tissue or to replace a natural organ. It duplicates or
augments a specific function or functions of human organs so that
the patient may return to a normal life as soon as possible.

Unit 11  R ecent devel opments in physics 275

Unit-11.indd 275 7/31/2019 7:18:42 PM


5 5. 3D printing

Advanced 3D printer systems and materials assist physicians in a


range of operations in the medical field from audiology, dentistry,
orthopedics and other applications.

6 6. Wireless brain sensors

Wireless brain sensors monitor intracranial pressure and tempera-


ture and then are absorbed by the body. Hence there is no need for
surgery to remove these devices.

276 Unit 11  R ecent devel opments in physics

Unit-11.indd 276 7/31/2019 7:18:42 PM


7 7. Robotic surgery

Robotic surgery is a type of surgical procedure that is done using robotic


systems. Robotically-assisted surgery helps to overcome the limitations of
pre-existing minimally-invasive surgical procedures and to enhance the
capabilities of surgeons performing open surgery.

8 8. Smart inhalers

Inhalers are the main treatment option for asthma. Smart inhal-
ers are designed with health systems and patients in mind so that
they can offer maximum benefit. Smart inhalers use bluetooth
technology to detect inhaler use, remind patients when to take
their medication and gather data to help guide care.

Unit 11  R ecent devel opments in physics 277

Unit-11.indd 277 7/31/2019 7:18:42 PM


Other recent developments in physics

Particle Physics
Particle physics deals with the theory of fundamental particles of nature and it is one of
the active research areas in physics. Initially it was thought that atom is the fundamental
entity of matter. In 1930s, it was established that atoms are made up of electrons, protons
and neutrons.

Molecule Atom Nucleus Nucleon


Composed of two or Electrons revolving Composed of Protons and
more atoms around the atomic protons neutrons
nucleus and neutrons comprising the
located in the nucleus of an atom
(Example) Electron
center of an atom
Molecule of water Neutron
Oxygen atom
Neutron

Quark
O H
H
Proton Quark
Hydrogen atoms
Proton
Nucleus

In the 1960s, quarks were discovered and it was understood that proton and neutron are
made up of quarks. In the meantime, the particle physics research gained momentum and
has grown exponentially both in theoretical and experimental perspective. Later it was
found that the quarks interact through gluons. It is the field which received more number
of noble prizes. Recently in the year 2013, famous ‘Higgs particles’ also known as “God”
particles were discovered and for this, Peter Higgs and Englert received noble prize in
physics. It is the ‘Higgs particle’ which gives mass to many particles like protons, neutrons
etc.

278 Unit 11  R ecent devel opments in physics

Unit-11.indd 278 7/31/2019 7:18:43 PM


Cosmology
Cosmology is the branch that involves the origin and evolution of the universe.
It deals with formation of stars, galaxy etc. In the year 2015, the existence of
“gravitational waves” was discovered and noble prize was awarded for this discovery
in the year 2017.

Gravitational waves are the disturbances in the curvature of space-time and it travels
with speed of light. Any accelerated charge emits electromagnetic wave. Similarly any
accelerated mass emits gravitational waves but these waves are very weak even for
masses like earth. The strongest source of gravitational waves are black holes. The
discovery of gravitational waves made it possible to study the structure of black holes
since it is the strongest source of gravitational waves. In fact, the recent discoveries of
gravitational waves are emitted by two black holes when they merge to a single black
hole. In fact, Albert Einstein theoretically proposed the existence of ‘gravitational
waves’ in the year 1915. After 100 years, it is experimentally proved that his predictions
are correct.

Gravitational wave

Black holes are end stage of stars which are highly dense massive object. Its mass
ranges from 20 times mass of the sun to 1 million times mass of the sun. It has very
strong gravitational force such that no particle or even light can escape from it. The
existence of black holes is studied when the stars orbiting the black hole behave
differently from the other stars. Every galaxy has black hole at its center. Sagittarius A*
is the black hole at the center of the Milky Way galaxy.

Unit 11  R ecent devel opments in physics 279

Unit-11.indd 279 7/31/2019 7:18:43 PM


Black hole sagittarus A*
The famous physicist Stephen Hawking worked in the field of black holes.

Actual photograph of a super massive black hole M87*

Super computers and eight telescopes stationed on five continents (EVENT HORIZON
TELESCOPE) were used to develope a huge data to accomplish this. It has once again
confirmed the Einstein’s theory of general relativity.

280 Unit 11  R ecent devel opments in physics

Unit-11.indd 280 7/31/2019 7:18:43 PM


Quantum information theory
Not for examination
It is another fast developing research area which deals with improving the information storage
using quantum computers. The present computers store information in the form of ‘bits’ but
quantum computers store information in the form of ‘qubits’. ‘qubit’ refers to quantum bit and it is
the basic unit of quantum information. Classical bit implies either 0 or 1. But qubit not only
includes 0 or 1 and also linear superposition of 0 and 1. This technology reduces the calculating
time exponentially. This research field has very promising application in future.

Information Qubit
is physical

+ α +β
Head ('0')
Nature
or is quantum

=
Quantum
Tail ('0') information
processing

Many striking innovations and discoveries originate from scientific fictions.


Robots are also no exception to this. The word robotics was derived from the
word robot. It was introduced in the play ‘Rossum Universal Robots’ by the Zech
writer Karel Capek in 1920. The word robot comes from the Slavic word rabota,
which means labour or work. The play begins in a factory that makes artificial people
called robots. They looked like creatures that can be mistaken for humans (picture
shown). These characters were very similar to the modern ideas of androids.

(A scene from the play Rossum Universal Robots, showing three robots)

Unit 11  R ecent devel opments in physics 281

Unit-11.indd 281 7/31/2019 7:18:43 PM


SUMMARY

„ Salient physics principles (covered in the higher secondary physics) are the foundation
for technology break through.
„ Physics is the basic building block for Science, Engineering, Technology and
Medicine. Nano science is the science of objects with typical sizes of 1–100 nm.
„ Nano means one-billionth of a metre that is 10–9 m.
„ Nanotechnology is a technology involving the design, production, characterization,
and applications of nano structural materials.
„ If the particle of a solid is of size less than 100 nm, it is said to be a ‘nano solid’.
„ When the particle size exceeds 100 nm, it forms a ‘bulk solid’.
„ Nano form of the material shows strikingly different properties when compared to
its bulk counterpart.
„ Quantum confinement effects and surface effects are the two important phenomena
that govern nano properties.
„ Nanoscience and technology is the interdisciplinary area covering its applications in
various fields.
„ Nano scale structures existed in nature long before scientists began studying them in
laboratories.
„ There are two ways of preparing the nanomaterials, top down and bottom up
approaches.
„ Nanotechnology applications cover various fields.
„ The major concern with nano application is that the nanoparticles have the dimensions
same as that of the biological molecules such as proteins.
„ Nano particles can easily get absorbed onto the surface of living organisms and they
might enter the tissues and fluids of the body.
„ The adsorbing nature depends on the surface of the nanoparticle.
„ It is possible to deliver a drug directly to a specific cell in the body by designing the
surface of a nanoparticle.
„ Nanoparticles of a few nanometers size may reach well inside biomolecules, which is
not possible for larger nanoparticles.
„ Nanoparticles can also cross cell membranes.
„ The inhaled nanoparticles reach the blood and that may also reach other sites such as
the liver, heart or blood cells.
„ Robotics is an integrated study of mechanical engineering, electronic engineering,
computer engineering, and science.
„ Robot is a mechanical device designed with electronic circuitry and programmed to
perform a specific task.
„ The robotic system mainly consists of sensors, power supplies, control systems,
manipulators and necessary software.

282 Unit 11  R ecent devel opments in physics

Unit-11.indd 282 7/31/2019 7:18:43 PM


„ The key components of a robot are Power conversion unit, Actuators, Electric motors,
Pneumatic Air Muscles, Muscle wires, Piezo Motors and Ultrasonic Motors, Sensors,
and Robot locomotion.
„ Six main types of industrial robots are Cartesian, SCARA, Cylindrical, Delta, Polar
and Vertically articulated robot.
„ Six-axis robots are ideal for Arc Welding, Spot Welding, Material Handling, Machine
Tending.
„ Five major fields of robotics: Human-robot interface, Mobility, Manipulation,
Programming and Sensors.
„ The aim of artificial intelligence is to bring in human like behavior in robots.
„ Artificial intelligence works on face recognition, providing response to players’
actions in computer games, taking decisions based on previous actions, regulating
the traffic by analyzing the density of traffic on roads and translate words from one
language to another.
„ Materials used to make robots: aluminum and steel are the most common metals. 
„ Aluminum is a softer metal and is therefore easier to work with.
„ Steel is several times stronger.
„ Due to the inherent strength of metal, robot bodies can be made using sheet, bar,
rod, channel, and other shapes.
„ Robots have many advantages in various applications but also have several
disadvantages.
„ In outer space robots are used for exploring stars, planets etc., investigation of the
mineralogy of the rocks and soils on Mars, analysis of elements found in rocks and soils.
„ Household robots are used as vacuum cleaners, floor cleaners, gutter cleaners, lawn
mowing, pool cleaning, and to open and close doors.
„ Industrial Robots are used for welding, cutting, robotic water jet cutting, robotic laser
cutting, lifting, sorting, bending, manufacturing, assembling, packing, transport,
handling hazardous materials like nuclear waste, weaponry, laboratory research,
mass production of consumer and industrial goods.
„ Nano-robots are being developed to be in the blood stream to perform small surgical
procedures, to fight against bacteria, repairing individual cell in the body.
„ The development in medical field has been proportional to the evolution of physics.
„ The recent medical technology includes virtual reality, precision medicine, health wearables,
artificial organs, 3D printing, wireless brain sensors, robotic surgery, smart inhalers.
„ Particle physics deals with fundamental particles of nature. Protons and neutrons are
made of quarks.
„ Cosmology is the branch that involves the origin and evolution of the universe.
„ Accelerated mass emits gravitational waves which are very weak.
„ Black holes are the strongest source of gravitational waves.

Unit 11  R ecent devel opments in physics 283

Unit-11.indd 283 7/31/2019 7:18:43 PM


CONCEPT MAP

Recent developments in Physics

A few areas as examples

Science Technology Medicine

Nanoscience Particle
& Cosmology Robotics Medical diagnosis
physics and therapy
Nanotechnology

Black holes Gravitational waves


Inventions in
physics & Recent medical
Nanoscience, Nanotechnology, Developments in technology
Nanoparticles Medical science

Interdisciplinary-
nanotechnology Components

Nano in nature
Types

Early beginning &


development
Applications
Nano in laboratories

Materials used to
Applications of make Robots
nanotechnology

Harmful effects of Advantages & Disadvantages


nanoparticles

284 Unit 11  R ecent devel opments in physics

Unit-11.indd 284 7/31/2019 7:18:43 PM


E V AL U A T I O N

I Multiple Choice Questions 6. The materials used in Robotics are


1. The particle size of ZnO material is a) Aluminium and silver
30 nm. Based on the dimension it is b) Silver and gold
classified as c) Copper and gold
a) Bulk material d) Steel and aluminum
b) Nanomaterial 7. The alloys used for muscle wires in
c) Soft material Robots are
d) Magnetic material a) Shape memory alloys
2. Which one of the following is the b) Gold copper alloys
natural nanomaterial. c) Gold silver alloys
a) Peacock feather d) Two dimensional alloys
b) Peacock beak 8. The technology used for stopping the
c) Grain of sand brain from processing pain is
d) Skin of the Whale a) Precision medicine
3. The blue print for making ultra durable b) Wireless brain sensor
synthetic material is mimicked from c) Virtual reality
a) Lotus leaf d) Radiology
b) Morpho butterfly 9. The particle which gives mass to
c) Parrot fish protons and neutrons are
d) Peacock feather a) Higgs particle
4. The method of making nanomaterial b) Einstein particle
by assembling the atoms is called c) Nanoparticle
a) Top down approach d) Bulk particle
b) Bottom up approach 10. The gravitational waves were theoretically
c) Cross down approach proposed by
d) Diagonal approach a) Conrad Rontgen
5. “Sky wax” is an application of nano b) Marie Curie
product in the field of c) Albert Einstein
a) Medicine d) Edward Purcell
b) Textile
c) Sports Answers
d) Automotive industry 1) b 2) a 3) c 4) b 5) c
6) d 7) a 8) c 9) a 10) c

Unit 11 R ecent devel opments in physics 285

Unit-11.indd 285 7/31/2019 7:18:43 PM


II Short answers III Long Answers
1. Distinguish between Nanoscience and 1. Discuss the applications of
Nanotechnology. Nanomaterials in various fields.
2. What is the difference between Nano 2. What are the possible harmful effects
materials and Bulk materials? of usage of Nanoparticles? Why?
3. Give any two examples for “Nano” in 3. Discuss the functions of key
nature. components in Robots?
4. Mention any two advantages and 4. Elaborate any two types of Robots with
disadvantages of Robotics. relevant examples.
5. Why steel is preferred in making 5. Comment on the recent advancement
Robots? in medical diagnosis and therapy.
6. What are black holes?
7. What are sub atomic particles?

BOOKS FOR REFERENCE

1. Pradeep, T. Nano Essential Under­standing Nano Science and Nano­technology, McGraw


Hill Education, India 2007.
2. Rita John, Solid State Physics, McGraw Hill Education, India 2016
3. Asim K Das, Mahua Das, An Introduction to Nano Science and Nano technology, CBS
Publishers and Distributors Pvt Ltd, India 2017.
4. Jerrold T. Bushberg, J.Anthony Seibert, The Essential Physics of Medical Imaging, Wolters
Kluwer, Lippin Cott Willams & Wilkins 2012
5. Brian R Martin, Particle Physics, Kindle edition, 2011
6. B S Murty, P Shankar, Baldev Raj, B B Rath, James Murday, Textbook of Nanoscience and
Nanotechnology, Springer, Universities Press, 2013

286 Unit 11  R ecent devel opments in physics

Unit-11.indd 286 7/31/2019 7:18:43 PM


ICT CORNER
Recent developments in physics

In this activity you will be able to (i) observe the changes in the
nuclear spins of the hydrogen nuclei of your water molecules due Topic: MRI scan
to the external magnetic field (ii)find out the resonance frequency
that promote a resultant photon.

STEPS:
• Go to ‘https://phet.colorado.edu/en/simulation/legacy/mri’ page and download simplified MRI java
file. Or go to Google → Phet → simulation → Physics → simplified MRI and download the java file.
• Open simplified MRI java file. Select simplified MRI tab.
• Observe the nuclear spins of the hydrogen nuclei present in the water molecules in brain (blue is the
hydrogen nuclei). Are they aligned in same direction? What happens when you change the external
magnetic field? Are they aligned in the same direction under external field? Discuss the reason.
• Now adjust the frequency bar. For a particular frequency, hydrogen nuclei emit radio waves from left
to right and find out the frequency when the nuclei start broadcasting radio waves. This is resonance
frequency.
• Add a tumour. Adjust the resonance frequency slightly to produce the strongest signal from the
tumour. Record the tumour resonance frequency. Is there a shift?
• With the help of shift in resonance frequency, tumour inside the brain can be calculated.

Step1 Step2

Step3 Step4

Note:
Install Java application if it is not in your browser.
URL:
https://phet.colorado.edu/en/simulation/legacy/mri
* Pictures are indicative only.
* If browser requires, allow Flash Player or Java Script to load the page.

Unit 11 R ecent devel opments in physics 287

Unit-11.indd 287 7/31/2019 7:18:44 PM


GLOSSARY
கைலச்ெசாற்கள்

1. Acceptance angle - ஏற்புக்காணம்


2. Acceptor energy level - ஏற்பு அணு ஆற்ைல மடடைம்
3. Activity - தையலபாடு
4. Amplitude modulation - வீச்சுப் பண்்பற்ைம்
5. Analyser - பகுப்பான
6. Anti fouling coating - கலைபடியா ்மற்பூச்சு
7. Anti particle - எதிர்த்துகள
8. Apparent depth - ்ைாற்ை ஆைம்
9. Astigmatism - ஒரு ைைப்பார்லவ
10. Attenuation - வலுவிைப்பு
11. Automotive industry - வாகனத் தைாழில
12. Auto-immune disease - ்நாய் எதிர்ப்பு ைகதிககு எதிரான ்நாய்
13. Band gap energy - படலடை இலடைதவளி ஆற்ைல
14. Barrier potential - அரண் மினனழுத்ைம்
15. Baseband signal - அடிககற்லை லைலக
16. Biasing - ைார்பளித்ைல
17. Binding energy - பிலணப்பாற்ைல
18. Bipolar junction transistor - இருமுலனச் ைந்தி டிரானசிஸ்டைர்
19. Boolean Algebra - பூலியன இயற்கணிைம்
20. Brain tumor - மூலைககடடி
21. Bright fringe - தபாலிவுப்படலடை
22. Broadcasting station - ஒலிபரப்பும் நிலையம்
23. Carrier concentration - ஊர்தி தைறிவு
24. Carrier signal - ஊர்தி லைலக
25. Central bright fringe - லமயப்தபாலிவுப்படலடை
26. Characteristic x-rays - சிைப்பு x கதிர்கள
27. Chemical Vapour - ்வதிவிலன நீராவி படிவு (அ)
Deposition (CVD) ்வதி ஆவி படிகமாககல
28. Classical electrodynamics - தைவ்வியல மினனியககவியல
29. Collector-base junction - ஏற்பான - அடிவாய் ைந்தி
30. Computed Tomography - கணிணி வலரவி
31. Concave lens - குழிதைனஸ்
32. Concave mirror - குழி அடி
33. Continuous x-rays - தைாடைர் x கதிர்கள
34. Convex lens - குவிதைனஸ்
288

Glossary.indd 288 7/31/2019 7:16:57 PM


35. Convex mirror - குவி அடி
36. Critical angle - மாறுநிலைக்கோணம்
37. Cut-off region - முறிவுப் பகுதி
38. Dark fringe - கரும்பட்டை
39. Decay mode - சிதைவுப் பாணி
40. Depletion region - இயக்கமில்லா பகுதி
41. Diffraction - விளிம்புவிளைவு
42. Diffusion current - விரவல் மின்னோட்டம்
43. Digital and analog signal - இலக்கமுறை மற்றும் த�ொடர் சைகை
44. Discrete - பிரிநிலை
45. Dispersion - நிறப்பிரிகை
46. Distance of closest approach - அணுகும் மீச்சிறு த�ொலைவு
47. Donor energy level - க�ொடை அணு ஆற்றல் மட்டம்
48. Doping - மாசூட்டல்
49. Drift current - இழுப்பு மின்னோட்டம்
50. Droplet - திவலை
51. Duality - இருமைப்பண்பு
52. Dynamic resistance - மாறு மின்தடை
53. Electron-hole recombination - எலக்ட்ரான் – துளை மறு இணைவு
54. Electron current - எலக்ட்ரான் மின்னோட்டம்
55. Electron emission - எலக்ட்ரான் உமிழ்வு
56. Electrostatic lens - நிலை மின்புல லென்ஸ்
57. Emitter-base junction - உமிழ்ப்பான் - அடிவாய் சந்தி
58. Emitter current - உமிழ்ப்பான் மின்னோட்டம்
59. Energy band diagram - ஆற்றல் பட்டை வரைபடம்
60. Endoscopy - அக உள்நோக்கி
61. Excitation energy - கிளர்வு ஆற்றல்
62. Extrinsic - புறவியலான
63. Feedback circuit - பின்னூட்டச் சுற்று
64. Fiber optic communication - ஒளி இழைத் தகவல் த�ொடர்பு
65. Field emission - புல உமிழ்வு
66. Focal length - குவியத்தொலைவு
67. Forward current gain - முன்னோக்கு மின்னோட்டப்பெருக்கம்
68. Frequency modulation - அதிர்வெண் பண்பேற்றம்
69. Fringe width - பட்டை அகலம்
70. Fuel cell - எரிப�ொருள் மின்கலன்
71. Glass slab - கண்ணாடிப்பட்டகம்
72. Global Positioning System - உலகளாவிய நிலை அறிவும் அமைப்பு
73. Ground State / excited state - தரை நிலை / கிளர்ச்சி நிலை
74. Ground wave propagation - தரை அலை பரவல்
75. Hole current - துளை மின்னோட்டம்
76. Hypermetropia - தூரப்பார்வை
77. Impact parameter - ம�ோதல் காரணி
78. Impurity atoms - மாசு அணுக்கள்

GLOSSARY 289

Glossary.indd 289 7/31/2019 7:16:57 PM


79. Interference - குறுக்கீட்டு விளைவு
80. Intrinsic - உள்ளார்ந்த
81. Knee voltage - பயன்தொடக்க மின்னழுத்தம்
82. Light emitting diode - ஒளி உமிழ் டைய�ோடு
83. Line of sight communication - நேர்க்கோட்டுப்பார்வை தகவல் த�ொடர்பு
84. Load current - பளு மின்னோட்டம்
85. Load resistance - பளு மின்தடை
86. Logic gates - தர்க்க வாயில்கள் (லாஜிக் கேட்டுகள்)
87. Looming - நிழல் த�ோற்றம்
88. Magnetic lens - காந்தப் புலலென்ஸ்
89. Magnification - உருப்பெருக்கம்
90. Majority charge carriers - பெரும்பான்மை மின்னூட்ட ஊர்திகள்
91. Marginal rays - ஓரக்கதிர்கள்
92. Matter waves - பருப்பொருள் அலைகள்
93. Maximum secondary voltage - பெரும துணை மின்னழுத்தம்
94. Minority charge carriers - சிறுபான்மை மின்னூட்ட ஊர்திகள்
95. Mirage - கானல் நீர்
96. Mobile communication - செல் பேசி தகவல் த�ொடர்பு
97. Moderator - தணிப்பான்
98. Myopia - கிட்டப்பார்வை
99. Near point focusing - அண்மைக் குவி நிலை
100. Negative space charge region - எதிர்மின்திரள் பகுதி
101. Normal focusing - இயல்பு குவி நிலை
102. Nuclear fission - அணுக்கரு பிளவு
103. Nuclear fusion - அணுக்கரு இணைவு
104. Optical fiber - ஒளிஇழை
105. Optoelectronic devices - ஒளியியல் மின்னணு சாதனங்கள்
106. Paraxial rays - அண்மை அச்சுக்கதிர்கள்
107. Peak inverse voltage - பெரும புரட்டு மின்னழுத்தம்
108. Phase - கட்டம்
109. Phase modulation - கட்டப் பண்பேற்றம்
110. Photo conductive cell - ஒளிமின் கடத்து மின்கலம்
111. Photoelectric emission - ஒளிமின் உமிழ்வு
112. Photoelectrons - ஒளி எலக்ட்ரான்கள்
113. Photo emissive cell - ஒளிமின் உமிழ்வு மின்கலம்
114. Photosensitive material - ஒளி உணர் ப�ொருள்
115. Photosensitive materials - ஒளி நுண் உணர்வு ப�ொருட்கள்
116. Photo voltaic cell - ஒளி வ�ோல்டா மின்கலம்
117. Polariser - தளவிளைவாக்கி
118. Polarization - தளவிளைவு
119. Positive and negative logic - நேர் மற்றும் எதிர் தர்க்கம்
120. Positive space charge region - நேர் மின்திரள் பகுதி
121. Potential barrier - மின்னழுத்த அரண்
122. Power of lens - லென்சின் திறன்

290 GLOSSARY

Glossary.indd 290 7/31/2019 7:16:57 PM


123. Presbyopia - வெள்ளெழுத்து
124. Prism - முப்பட்டகம்
125. Prosthetics - செயற்கைமூட்டு
126. Pulsating output - துடிப்பு வெளியீடு
127. Quantization - குவாண்டமாக்கல்
128. Radioactive recombination - கதிர்வீச்சு மறுஇணைவு
129. Radio isotope imaging - கதிர்வீச்சு ஜச�ோட�ோப்பு பிம்பம்
130. Radiology - கதிரியக்கச் சிகிச்சை
131. Radiology X-ray imaging - கதிரியக்க ஊடுகதிர் பிம்பம்
132. Rectification - திருத்துதல்
133. Rectifier efficiency - திருத்தியின் பயனுறுதிறன்
134. Reflection - ஒளி எதிர�ொளிப்பு
135. Refraction - ஒளிவிலகல்
136. Refractive Index - ஒளிவிலகல் எண்
137. Repeater - மறு ஒலிபரப்பி
138. Resolution - பிரித்தறிதல்
139. Resolving Power - பிரிதிறன்
140. Reverse saturation current - பின்னோக்கு செறிவு மின்னோட்டம்
141. Robot - இயந்திர மனிதன்
142. Robotic surgery - இயந்திரமனித அறுவைசிகிச்சை
143. Scanning Tunneling - துளைக்கும் வரிக்கண்ணோட்ட
Microscope (STM) நுண்ணோக்கி
144. Scattering - ஒளிச்சிதறல்
145. Secondary emission - இரண்டாம் நிலை உமிழ்வு
146. Sensor - உணர்வி
147. Skip distance - தாவுத் த�ொலைவு
148. Skip zone - தாவு மண்டலம்
149. Sky wave propagation - வான் அலை பரவல்
150. Space wave propagation - வெளி அலை பரவல்
151. Specific Charge - மின்னூட்ட எண்
152. Stopping potential - நிறுத்து மின்னழுத்தம்
153. Surface barrier - பரப்பு அரண்
154. Thermionic emission - வெப்பஅயனி உமிழ்வு
155. Threshold frequency - பயன்தொடக்க அதிர்வெண்
156. Total Internal reflection - முழு அக எதிர�ொளிப்பு
157. Transmitting antenna - பரப்பும் விண்ணலைக்கம்பி
158. Wavefront - அலைமுகப்பு
159. Wireless brain sensor - கம்பியில்லா மூளை உணர்வி
160. Work function - வெளியேற்று ஆற்றல்

Solved examples Competitive Exam corner


GLOSSARY 291

Glossary.indd 291 7/31/2019 7:16:57 PM


State Council of Educational Research and Training, Chennai-6
Higher Secondary Second Year Physics volume 2
List of Subject Experts in Textbook Writing

Domain Expert & Mentor Dr. S. S. Naina Mohammed SCERT Subject Coordinators
Prof. Dr. Rita John Assistant Professor, Mrs. P.Nantha
Professor and Head PG and Research Department of Physics, Senior Lecturer
Department of Theoretical physics, Government Arts College SCERT, Chennai – 06.
University of madras, Chennai. Udumalpet, Tiruppur District.
ICT Coordinator
Reviewers Dr. P. Elangovan Mr.G.Bergin
Dr. V. N. Mani Assistant Professor,
PG Assistant (Physics)
Principal Scientist F, Head (C-MET) PG and Research Department of Physics,
GBHSS, Sayalkudi,
Department of Electronics & Pachaiyappa’s College,
Ramanathapuram District.
Information Technology, Hyderabad, Chennai.
Government of India. QR CODE Management Team
Dr. B. Benita Merlin
R. Jaganathan
Prof. P. Ravindran Assistant Professor,
SGT, PUMS -  Ganesapuram,
Department of Physics, Alpha Arts and Science College,
Polur , Thiruvannamalai.
School of Basic and Applied Sciences, Chennai
Central University of Tamil Nadu, A. Devi Jesintha
Thiruvarur. Tamil Translators
B.T. Asst., GHS, N.M. Kovil,
Mr.E.Elangovan Thirupathur, Vellore
Dr. Rajeev shesha Joshi Head Master,
Assistant Professor, Government Hr Sec School V. Padmavathi, B.T,
School of Physical Sciences, Ramanayakanpettai, B.T. Asst., GHS, Vettriyur, Thirumanur,
Central University of Karnataka. Ariyalur Dist.
Vellore District.
Content Writers Art and Design Team
Dr.K.Vasudevan
Prof. Dr. Rita John PGT, Government ADW.Hr.Sec.School
Layout & QC
Professor and Head Kalangani
Department of Theoretical physics, V. Sridhar
Namakkal District.
University of madras, Chennai. B. Yogesh
Jerald Wilson
Mr.V.Balamurugan
Dr. P. Balamurugan PG Assistant in Physics,
M. Asker Ali
Assistant Professor, Government Hr.Sec.School,
PG and Research Department of Physics, Illustration
Kadambattur,
Government Arts College for Men K. Sasi Kumar
Tiruvallur District
Nandanam, Chennai. S. Durga Devi
V. Vinothkumar
Mr. S. Ravishankar
Dr. R. Sugaraj Samuel PG Assistant in Physics,
Assistant Professor,
Cover Design
S.R.M. Hr. Sec. School,
Kathir Arumugam
PG and Research Department of Physics, Ambattur,
The New College, Tiruvallur District Co-ordination
Royapettah, Chennai. Ramesh Munisamy
Mr. C. Joseph Prabagar
This book has been printed on 80 G.S.M.
Assistant Professor, Elegant Maplitho paper.
PG and Research Department of Physics, Printed by offset at:
Loyola College, Chennai

292

PH XII Std Acknowledgement.indd 292 7/31/2019 7:16:09 PM


NOTES

293

Note.indd 293 7/31/2019 7:17:42 PM


NOTES

294

Note.indd 294 7/31/2019 7:17:42 PM


NOTES

295

Note.indd 295 7/31/2019 7:17:42 PM


NOTES

296

Note.indd 296 7/31/2019 7:17:42 PM

Das könnte Ihnen auch gefallen